Sie sind auf Seite 1von 576

Table of Contents

Table of Contents ___________________________________________________________________ 0


CHAPTER 1: FINANCIAL REPORTING STANDARDS __________________________________ 2
1.1. Conceptual Framework for the Preparation & Presentation of Financial Statements ............. 2
1.2. NFRS 3 Business Combination ............................................................................................... 6
1.3. NAS 1 Presentation Of Financial Statements .......................................................................... 6
1.4. NAS 2 Inventories ................................................................................................................... 7
1.5. NAS 3 Statement of Cash Flows ........................................................................................... 12
1.6. NAS 8 Accounting Policy, Change in Accounting Estimates & Errors ................................ 13
1.7. NAS 10 Events After The Reporting Period ......................................................................... 16
1.8. NAS 16 Property, Plant & Equipment (PPE) ........................................................................ 20
1.9. NAS 17 Leases ...................................................................................................................... 28
1.10. NAS 18 Revenue ................................................................................................................... 33
1.11. NAS 20 Accounting for Government Grant & Disclosure of Government Assistance ........ 40
1.12. NAS 21 The Effects of Changes in Foreign Exchange Rates................................................ 43
1.13. NAS 37 Provisions, Contingent Liabilities & Contingent Assets ......................................... 52
1.14. NAS 11: Construction Contracts ........................................................................................... 55
1.15. NAS 12: Income Taxes .......................................................................................................... 55
1.16. NAS 23 Borrowing Costs ...................................................................................................... 59
1.17. NAS 24: Related Party Disclosure......................................................................................... 63
1.18. NAS 33 Earnings Per Share ................................................................................................... 65
1.19. NAS 36: Impairment of Assets .............................................................................................. 65
1.20. NAS 38 Intangible Assets ...................................................................................................... 68
1.21. NFRS 2 Share Based Payments ............................................................................................. 70
1.22. Miscellaneous ........................................................................................................................ 70
CHAPTER 2: ACCOUNTING FOR SPECIAL TRANSACTIONS __________________________ 79
2.1 Hire Purchase transaction .......................................................................................................... 79
A. Theoretical Questions ....................................................................................................... 79
B. Practical Questions ........................................................................................................... 80
2.2 Contract Accounting ............................................................................................................... 110
A. Theoretical Question ...................................................................................................... 110
B. Practical Questions ......................................................................................................... 112
2.3 Investment Accounting ........................................................................................................... 120
A. Theory Questions ............................................................................................................ 120
B. Practical Questions ......................................................................................................... 120
2.4 Branch & Departmental Accounts .......................................................................................... 131
A. Theoretical Questions (Departmental Accounts) ......................................................... 131
B. Theoretical Questions (Branch Accounts) .................................................................... 132
C. Practical Questions (Departmental Accounts) ............................................................. 133
D. Practical Questions (Branch Accounts) ........................................................................ 145
2.5 Insurance Claim – Loss of Stock & Profit .............................................................................. 158
A. Theoretical Questions ..................................................................................................... 158
B. Practical Questions ......................................................................................................... 159
CHAPTER 3: COMPANY ACCOUNTING ____________________________________________ 199
CAP II Paper 1: Advanced Accounting

3.1 Accounting for Shares & Debentures .................................................................................. 199


A. Theoretical Questions ..................................................................................................... 199
B. Practical Questions ......................................................................................................... 202
3.2 Preparation & Presentation of Financial Statements ........................................................... 227
A. Theoretical Questions ....................................................................................................... 227
B. Practical Questions ........................................................................................................... 227
3.3 Winding up of Companies ................................................................................................... 278
C. Theoretical Questions ..................................................................................................... 278
D. Practical Questions ......................................................................................................... 279
3.4.1 Accounting for Acquisition, Absorptions, Amalgamation and Merger ............................... 291
A. Practical Questions ......................................................................................................... 291
3.4.2 Reconstruction ..................................................................................................................... 344
A. Theoretical Questions ..................................................................................................... 344
B. Practical Questions ......................................................................................................... 344
CHAPTER 4 PARTNERSHIP ACCOUNTS ___________________________________________ 361
A. Theoretical Questions ..................................................................................................... 361
B. Practical Questions ......................................................................................................... 364
CHAPTER 5 PREPARATION & PRESENTATION of FINANCIAL STATEMENTS of SPECIAL
ORGANIZATIONS _______________________________________________________________ 449
5.1 Banking & Financial Institutions ......................................................................................... 449
A. Theoretical Questions ..................................................................................................... 449
B. Practical Questions ......................................................................................................... 453
5.2 Cooperatives ........................................................................................................................ 481
A. Theoretical Questions ..................................................................................................... 481
5.3 Insurance Companies ........................................................................................................... 482
A. Theory Questions ............................................................................................................ 482
B. Practical Questions ......................................................................................................... 487
5.4 Agriculture Accounting ....................................................................................................... 497
A. Theory Questions ............................................................................................................ 497
CHAPTER 6 INCOMPLETE RECORDS ______________________________________________ 500
A. Theory Questions ............................................................................................................ 500
B. Practical Questions ......................................................................................................... 501
CHAPTER 8 ANALYSIS & INTERPRETATION OF FINANCIAL STATEMENTS ___________ 520
A. Theory Questions ............................................................................................................ 520
B. Practical Questions ......................................................................................................... 522
CHAPTER 7 GOVERNMENT ACCOUNTING ________________________________________ 554
A. Theory Questions ............................................................................................................ 554
CHAPTER 8 ACCOUNTING IN COMPUTERIZED SYSTEM ____________________________ 556
A. Theoretical Questions ..................................................................................................... 556
Chapter 9 Miscellaneous ___________________________________________________________ 558
A. Theoretical Question ...................................................................................................... 558
B. Practical Questions ......................................................................................................... 563

© The Institute of Chartered Accountants of Nepal 1


CAP II Paper 1: Advanced Accounting

CHAPTER 1: FINANCIAL REPORTING STANDARDS

1.1. Conceptual Framework for the Preparation & Presentation of Financial


Statements

1. Materiality and prudence (CAP Dec. 2012 Q6e-2.5 Marks)

Answer
Materiality:
Information is material if its omission or misstatement could influence the economic decisions
of users taken on the basis of the financial statements. It depends on the size of the item or error
judged in the particular circumstances of its omission or misstatement. Often separate line item
or sub-item is decided bases on materiality. National level law may specify materiality limit
for separate disclosure of an item.

Prudence:
Prudence is the inclusion of a degree of caution in the exercise of the judgments needed in the
making the estimates required under conditions of uncertainty, such that assets or income are
not overstated and liabilities or expenses are not understated. The exercise of prudence does
not allow, for example, the creation of hidden reserves or excessive provisions.

2. Going Concern
(CAP Dec. 2011 Q6a-2.5 marks; Inter Jun. 2003 Q9d -3 Marks, Inter Dec. 2003 Q 6- 4
Marks)

Answer
The financial statements are normally prepared on the assumption that an enterprise is a going
concern and will continue in operation for the foreseeable future. Hence, it is assumed that the
enterprise has neither the intention nor the need to liquidate or curtail materially the scale of its
operations; if such an intention or need exists, the financial statements may have to be prepared
on a different basis and, if so, the basis used is disclosed.

3. Underlying assumption on Preparation and presentation of Financial Statement


Answer (CAP Dec. 2017 Q6a-3 Marks)
Accrual Basis
In order to make their objectives financial statements are prepared on the accrual basis of
accounting. Under this basis, the effects of transaction and other events are recognized when
they occur (and not as cash or equivalent is received or paid) and they are recorded in the
accounting records and reported in financial statements of the periods to which they relate.
Going Concern
The financial statements are normally prepared on the assumption that an entity is a going
concern and will continue in operation for foreseeable future. Hence, it is assumed that the
entity has neither the intention nor the need to liquidate or curtail materially the scale of

© The Institute of Chartered Accountants of Nepal 2


CAP II Paper 1: Advanced Accounting

operations; if such intention or need exists, the financial statements may have to be prepared
on a different basis.

4. Elements of Financial Statements (CAP Dec. 2018 Q6e-3 Marks)

Answer
The framework classifies items of financial statements can be classified in five broad
groups depending on their economic characteristics: Asset, Liability, Equity, Income and
Expense.
Assets Resource controlled by the entity as a result of past events from which
future economic benefits are expected to flow to the entity
Liability Present obligation of the entity arising from past events, the
settlement of which is expected to result in an outflow of a resource
embodying economic benefits.
Equity Residual interest in the assets of an entity after deducting all its
liabilities.
Income Increase in resources embodying economic benefits during the
accounting period in the form of inflows or enhancement of assets or
decreases in liabilities that result in increase in equity other than those
relating to contributions from equity participants
Expenses Decrease in economic benefits during the accounting period in the
form of outflows or depletions of assets or incurrence of liabilities
that result in decrease in equity other than those relating to
distributions to equity participants.

5. Measurement of the Elements of Financial Statements


(Inter Dec. 2011 Q6d-5 Marks; Inter Dec 04, Q5b)
Answer
Measurement is the process of determining the monetary amount at which the elements of the
Financial Statements are to be recognized and carried in the balance sheet and income
statement.
The elements of financial statements measure the financial position and performance of an
entity. The elements directly related to the measurement of financial position are assets,
liabilities and equity. And, the elements directly related to the measurement of performance in
the Statement or profit or loss are income and expenses.
An item that meets the definition of an element should be recognized if:
• it is probable that any future economic benefit associated with the item will flow to or
from the entity, and
• the item has a cost or value that can be measured with reliability.

After deciding that an item is to be recognized in the financial statements, the following
measurement bases are used in reporting:

© The Institute of Chartered Accountants of Nepal 3


CAP II Paper 1: Advanced Accounting

• Assets: Initial recognition of assets is done at cost with subsequent measurement made at
fair value
• Liabilities: Measured at the fair value of consideration required to settle the obligation
• Income: Measured at the fair value of consideration received or receivable
• Expense: Measured at the fair value of economic benefit out-flown or expected to outflow
• Equity: Residual interest. No specific recognition requirements

6. Qualitative characteristics of financial statements


(Inter Jun. 2010 Q6c – 5 Marks; CAP Jun. 2014 Q6a- 2.5 Marks)
Answer
These are the attributes of financial statements that make the information provided in them
useful to their users. Followings are the four major qualitative characteristics of financial
statements:
• Understandability:
For this, information is provided in the financial statements in such a way that a user of
reasonable knowledge of business, economic activities and accounting can study them with
reasonable diligence.
• Relevance:
Information has the quality of relevance when it influences the economic decisions of users by
helping them evaluate past, present or future events or confirming, or correcting, their past
evaluations.
• Reliability:
Information has the attribute of reliability when it is free from material error and bias. It can be
depended upon by users to represent faithfully that which it either purports to represent or could
reasonably be expected to represent.
• Comparability:
Users must be able to compare the financial statements of an entity through time in order to
identify trends in its financial position and performance. It needs disclosure of accounting
policies, presentation of past information and compliance of relevant accounting standards.

7. Explain the concept of Capital Maintenance


(CAP Jun. 2010 Q3b (ii) -3 Marks; Inter Jun. 2004 Q2a -4 Marks; Inter Dec. 2005 Q 6a-5
Marks)
Answer
Broadly, there are two major concept of capital maintenance:
First one is the concept of physical capital maintenance where profit means increase in
operating or production capacity of an enterprise at the end of a period compared to that at the
beginning of the period.
In second concept, profit is only earned if the monetary value of the net assets at the end of a
period exceeds the monetary value at the beginning of the period.
Both the concepts exclude any contributions from and distributions to the owners during the
period.

© The Institute of Chartered Accountants of Nepal 4


CAP II Paper 1: Advanced Accounting

Underlying principle of capital maintenance is that profit should be only recognized, after
ensuring that capital is intact as compared to the beginning of the period and it’s not eroded as
compared to the starting point of the period under consideration.

8. Fundamental Accounting Assumptions (CAP Jun. 2016 Q6e-3 Marks)

Answer
Accounting has its own assumption to make it relevant and for the purpose of facility. The
basic assumptions underlying accounts are as follows:
1. Going concern: The enterprise is normally viewed as a going concern, i.e. as continuing
operations for the foreseeable future. It is assumed that the enterprise has neither the
intention nor the necessity of liquidation.
2. Consistency: It is assumed that accounting policies are consistent from one period to
another. This adds the virtue of comparability to accounting data. If comparability is
lost, the relevance of accounting data for users’ judgment and decision making is gone.
3. Accrual: Revenues and costs are accrued, that is, recognized as they are earned or
incurred (and not as money is received or paid) and recorded in the financial statements
of the periods to which they relate. This assumption is the core of accrual accounting
system.
4. Disclosure requirements If the fundamental accounting assumption, viz. going
Concern, Consistency, and Accrual are followed in financial statements, specific
disclosure is not required. If a fundamental accounting assumption is not followed, the
fact should be disclosed.

9. Recognition of expenses (CAP Jun. 2014 Q6b- 2.5 Marks)

Answer
Expenses, as per the conceptual framework is referred to as a decrease in outflow of resources
embodying economic benefits or increase in liability other than those relating to distributions
to the equity participants. As per the recognition criteria, expenses in the financial statements
are recognized when:
• it is probable that any future economic benefit associated with the item will flow
from the entity, and
• the cost or value that can be measured with reliability

10. Matching Concept


(CAP Jun. 2019 Q6b-3 Marks)
Answer
The matching concept is an essential part of accrual accounting, these two are often
used interchangeably. Like accrual concept, the matching concepts also result from
periodicity concept. The matching concept requires that the expenses for an
accounting period should be matched against related incomes. The matching concept
states that the revenue and the expenses incurred to earned the revenues must belong
to the same accounting period. So, once the revenue is realized, the next step is to
allocate it to the relevant accounting period.

© The Institute of Chartered Accountants of Nepal 5


CAP II Paper 1: Advanced Accounting

Significance
• It guides how the expenses should be matched with revenue for determining
exact profit or loss for a particular period.
• It is very helpful for the investors/ shareholders to know the exact amount of
profit or loss of the business.
• It requires proper allocation of costs into appropriate period so that relevant
income and expenses are matched.

11. What is entity concept? (CAP Jun. 2011 Q3b (i) – 2 Marks)

Answer
Business consists of person and resources. Person representing the business is separate and
distinct from the business enterprises. Accounting system deals with the economic activities
of the business not of owner. In Entity Concept, preparation of Balance Sheet of the business
does not consider the personal assets and liability of the owner of the business.

1.2. NFRS 3 Business Combination

Mostly covered under Merger & Acquisition.

1.3. NAS 1 Presentation Of Financial Statements

1. Indicate any three areas in respect of which different accounting policies may be adopted by
different enterprises. Also indicate the requirements with regard to disclosure of accounting
policies as per the relevant NAS.
(CAP Dec. 2010 Q5c-5 Marks; Inter Jun. 2002 Q 6a-7 Marks)

Answer
Areas in which different accounting policies may be adopted: The following are three areas in
which different accounting policies may be adopted by different enterprises:
(i) Methods of depreciation
(ii) Cost formula for inventories
(iii) Subsequent measurement of Property, plant & equipment.
Disclosure requirements of accounting policies: The disclosure requirements as prescribed in
Accounting Standard 1 (NAS 1) ‘Presentation of Financial Statements’ are as follows:
An entity shall disclose its significant accounting policies comprising:
(a) the measurement basis (or bases) used in preparing the financial statements; and
(b) the other accounting policies used that are relevant to an understanding of the financial
statements.

2. Components of Financial Statements


(Inter Jun. 2010 Q6a- 5 Marks; CAP Jun. 2012 Q6c– 2.5 Marks)

© The Institute of Chartered Accountants of Nepal 6


CAP II Paper 1: Advanced Accounting

Answer
As per NAS 1, Presentation of Financial Statements, a complete set of financial statements
comprise:
(a) a statement of financial position as at the end of the period;
(b) a statement of profit or loss and other comprehensive income for the period;
(c) a statement of changes in equity for the period;
(d) a statement of cash flows for the period;
(e) notes, comprising significant accounting policies and other explanatory information; and
(f) a statement of financial position as at the beginning of the preceding period when an entity
applies an accounting policy retrospectively or makes a retrospective restatement of items in its
financial statements, or when it reclassifies items in its financial statements

3. Other Comprehensive Incomes’ as per NFRS. (CAP Jun. 2018 Q6c-3 Marks)

Answer
Other comprehensive income comprises items of income and expenses (including
reclassification adjustments) that are not recognized in profit and loss as required or permitted
by other NFRSs.
The components of other comprehensive income include;
1. Changes in revaluation surplus
2. Re-measurements of defined benefit plans
3. Gains and losses arising from translating the financial statements of a foreign operation
4. Gains and losses from investments in equity instruments measured at fair value through
other comprehensive income
5. The effective portion of gains and losses on hedging instruments in a cash flow hedge
For particular liabilities designed as at fair value through profit or loss, the amount of the
change in the fair value that is attributable to changes in the liability’s credit risk.

1.4. NAS 2 Inventories

1. In a production process, normal waste is 5% of input. 5,000 MT of input were put in process
resulting in wastage of 300 MT. Cost per MT of input is Rs. 1,000. The entire quantity of waste
is on stock at the year end. State with reference to Accounting Standard, how will you value
the inventories in this case? (CAP Dec. 2016 5b-5 Marks)

Answer
As per of NAS- 2,” Inventories”, abnormal amount of wasted materials, labor and other
production costs are excluded from cost of inventories and such costs are recognized as
expenses in the period in which they are incurred.
In this case, normal waste is 250 MT and abnormal waste is 50 MT. The cost of 250 MT will
be included in determining the cost of inventories (finished goods) at the year end. The cost of
abnormal waste (50MT x 1,052.63 = Rs 52,632) will be charged to the profit and loss statement.
Cost per MT (Normal Quantity of 4,750 MT) = 50,00,000 / 4,750 = Rs 1,052.63

© The Institute of Chartered Accountants of Nepal 7


CAP II Paper 1: Advanced Accounting

Total value of inventory = 4,700 x Rs 1,052.63 = Rs 49,47,368.

2. Rahul Trading gives the following information relating to items forming part of inventory as
on 32-3-2075. His factory produces Product X using Raw material A.
i) 600 units of Raw material A (Produce @ Rs. 120). Replacement cost of raw material A as
on 32-3-2075 is Rs. 90 per unit.
ii) 500 units of partly finished goods in the process of producing X and cost incurred till date
Rs. 260 per unit. These units can be finished next year by incurring additional cost of Rs.
60 per unit.
iii) 1500 units of finished product X and total cost incurred Rs. 320 per unit. Expected selling
price of Product X is Rs. 300 per unit.
Determine how each item of inventory will be valued as on 32-3-2075. Also calculate the value
of total inventory as on 32-3-2075.
(CAP Dec. 2018 Q5c-5 Marks)
Answer
As per NAS 2 ‘Inventories’ are valued at lower of cost and net realizable value. Materials and
other supplies held for use in the production of inventories are not written down below cost if
the finished products in which they will be incorporated are expected to be sold at cost or above
cost. However, when there has been a decline in the price of materials and it is estimated that
the cost of the finished products will exceed net realizable value, the materials are written down
to net realizable value. In such circumstances, the replacement cost of the materials may be the
best available measure of their net realizable value. In the given case, selling price of product
X is Rs. 300 and total cost per unit for production is Rs.320.
Hence the valuation will be done as under:
i) 600 units of raw material will be written down to replacement cost as market value of
finished product is less than its cost, hence value at Rs. 90 per unit.
ii) 500 units of partly finished goods will be valued at 240 per unit i.e. lower of cost Rs.
320 (Rs. 260+ additional cost Rs. 60) or Net estimated selling price Rs.
240(Estimated selling price Rs. 300 per unit less additional cost of Rs. 60)
iii) 1500 units of finished product X will be valued at NRV of Rs. 300 per unit since it is
lower than cost Rs. 320 of product X.

Valuation of Total Inventory as on 32-03-2075


Units Cost NRV/ Value=units x cost
(Rs.) Replacement or NRV whichever
cost is less (Rs.)
Raw material A 600 120 90 54,000
Partly finished goods 500 260 240 1,20,000
Finished goof X 1,500 320 300 4,50,000
Value of inventory 6,24,000

© The Institute of Chartered Accountants of Nepal 8


CAP II Paper 1: Advanced Accounting

3. In Fiscal Year 2064/065, M/s SD Ltd. changed its inventory valuation policy from FIFO
method to Simple Average method and financial statements were prepared accordingly. Give
your view on the above changes with reference to NAS - 02.
(Inter Dec. 2008 Q2b-5 Marks)
Answer
As per the Nepal Accounting Standard (NAS) - 2 'Inventories' - cost of inventories should be
ascertained by using specific identification method. It also allows first in first out (FIFO) or
weighted average method for interchangeable items. Hence, entity should adopt any formula
from the above. There is no option of adopting simple average as a cost determining formula.
So, M/s SD Ltd. can't adopt simple average method by replacing FIFO method.

4. Following are data of Robinson


➢ Market Value of Stock as on Shrawan 01, 2066: Rs. 300,000 (Market Price
being cost plus 20% thereof).
➢ Purchases for the year: Rs. 1,450,000 (Out of which goods worth Rs.150,000 was delivered on
Shrawan 08, 2067)
➢ Sales for the year: Rs. 1,480,000.
➢ Goods costing Rs. 20,000 was taken by the proprietor for personal use.
➢ Goods costing Rs. 10,000 was used for firm’s office stationery.
➢ Normally, the firm sells goods on cost plus 40%
Determine the value of stock to be taken as on Ashadh end, 2067
(CAP Jun. 2011 Q3b ii- 8 Marks)
Answer
Robinson
Statement showing Value of Stock
as on Ashadh end, 2067
Opening Stock (as on Shrawan 01, 2066) 250,000
Add:
Purchases during the Year 1,450,000
Less: Goods in Transit (150,000)
Less: Goods taken by proprietor for personal use (20,000)
Less: Goods used for office's stationery (10,000) 1,270,000
Less:
Cost of Goods Sold:
Sales for the Year 1,480,000
Less: Profit on Sales (422,857)
1,057,143
Stock in Hand (as on Ashadh end, 2067) 462,857
Items delivered after the year end do not form part of the inventory as there is no
control over the asset currently.
Working Notes:
1. Calculation of Opening Stock:
Market Value of Stock: 300,000

© The Institute of Chartered Accountants of Nepal 9


CAP II Paper 1: Advanced Accounting

Profit Margin (on Cost) 20%


Cost Price of Opening Stock: =300,000 = 250,000
(100+20)
2. Calculation of Profit on
Sales:
Sales 1,480,000
Profit Margin (on Cost) 40%
Profit Amount =1,480,000 X 40 = 422,857 1,057,143
(100+40)

5. The company deals in three products, A, B & C, which are neither similar nor interchangeable.
At the time of closing of its account for the year 2066/067, the Historical and Net Realizable
Value of the items of closing stock are determined as follows:

Items Historical Cost Net Realizable Value


(Rs. in Lakhs) ( Rs. in Lakhs)
A 40 28
B 32 32
C 16 24
Compute the value of closing stock?
(CAP Jun. 2011 Q5a- 5 Marks; Inter Dec. 2006 Q1a-4 Marks)
Answer
As per NAS 2, Inventories, inventory should be valued at the lower of cost and net realizable
value. Inventories should be written down to net realizable value on an item-by-item basis in
the given case:
Historical Net Realizable Valuation of Closing
Items Cost Value Stock
(Rs. in Lakhs) (Rs. in Lakhs) (Rs. in Lakhs)
A 40.00 28.00 28.00
B 32.00 32.00 32.00
C 16.00 24.00 16.00
76.00

Hence, closing stock will be valued at Rs. 76 lakhs

6. Everest Ltd. incurred Rs. 20,00,000 as fixed production overhead per year. It normally
produces 1,00,000 units in a year. In 2010-11 however its production has been only 40,000
units. At the year-ended 16th July 2011, the closing stock was 10,000 units. The cost of unit is
below:

Material = Rs. 500 per unit


Labor = Rs. 250 per unit

© The Institute of Chartered Accountants of Nepal 10


CAP II Paper 1: Advanced Accounting

Fixed Production overhead = Rs. 20,00,000 per annum


Fixed Administration = Rs. 10,00,000 per annum
Calculate the value of closing stock. (CAP Jun. 2012 Q5b– 5 Marks)

Answer
In accordance with NAS 2, “Inventories”, the costs of conversion include a systematic
allocation of fixed and variable production overheads that are incurred in converting materials
into finished goods. The allocation of fixed production overheads for the purpose of their
inclusion in the cost of conversion is based on the normal capacity of the production facilities.
Thus, cost per unit of finished goods can be computed as follows:

Particulars Rs. /unit


Material cost 500
Labor cost 250
Fixed Production Overhead [Rs. 2,000,000 / 100,000 units] 20
Total Cost of conversion 770
Thus, the value of 10,000 units of finished goods on stock at the year-end will be Rs. 7,700,000
(10,000 units X Rs. 770 per unit).

7. During the year 2063/64 a medium size manufacturing company wrote down its inventories to
net realizable value by Rs. 500,000. Is separate disclosure necessary?
(Inter Jun. 2007 Q6i -5 Marks)
Answer:
NAS 2 specifically requires the disclosures of the amount of any write down of inventory
recognized as an expense in the period in accordance with the measurement principle of
“lower of cost or NRV”. Therefore, the entity is required to make disclosure with regards
to write down.

8. A limited company has been including interest in the valuation of closing stock. In 2063/64 the
management of the company decided to follow NAS 02 and accordingly interest has been
excluded from the valuation of closing stock. This has resulted in a decrease in profits by Rs.
300,000. Is disclosure necessary? If so, draft the same. (Inter Jun. 2007 Q 6ii -5 Marks)

Answer:
Under the provisions of NAS 23 Borrowing costs, borrowing costs (interest) that are
directly attributable to the acquisition, construction or production of a qualifying asset
(asset that necessarily take a substantial period of time to get ready for its intended use or
sale) shall be capitalized as part of the cost of that asset.

In the given case, if the asset to which the interest costs were allocated met the definition
of qualifying asset, the accounting treatment done by the company is justified. However, if
the asset did not meet the definition of qualifying asset, the accounting treatment done by
the company should be considered as prior period error; in which case the company should
opt for retrospective restatement of figures in line with the provisions of NAS 08,

© The Institute of Chartered Accountants of Nepal 11


CAP II Paper 1: Advanced Accounting

Accounting policies, changes in accounting estimates & errors. Disclosure in which case
would be:
“the company had been including interest cost in valuing its closing inventories which is
not allowed under the provisions of NAS 23. The prior period error has been corrected by
restating the figure of retained earnings of earliest prior period presented by NRs. XXXX.”

9. Definition of inventories as per Nepal Accounting Standard 2.


(Inter Jun. 2005 Q 6b -4 Marks)

Answer
Inventories are assets
• Held for sale in the ordinary course of business
• In process of production for such sale
• In form of materials or supplies to be consumed in production process or in
rendering services.

1.5. NAS 3 Statement of Cash Flows

1. How will you disclose following items while preparing Cash Flow Statement of Thapathali
Ltd. as per Nepal accounting standard for the year ended 32nd Ashadh 2075?
(i) 10% Debentures issued as on 1.4.2074 Rs. 110,000
As on 32.3.2075 Rs. 77,000
(ii) Debentures were redeemed at 5% premium at the end of the year. Premium was charged
to profit/loss for the year.
(iii) Unpaid interest on debentures as on 1.4.2074 Rs. 275
As on 32.3.2075 Rs. 1,175
(iv) Debtors of Rs. 36,000 were written off against the provision for doubtful debts account
during the year.
(v) Investment in 10% Bonds As on 1.4.2074 Rs. 350,000
As on 32.3.2075 Rs. 350,000
(vi) Accrued interest on investments as on 32.3.2075 Rs. 10,500
(CAP Dec. 2018 Q3b-5 Marks)
Answer
Cash flow statement of Thapathali Ltd. for the year ended 32nd Ashadh 2075

A Cash flow from operating activities


Net profit as per Statement of Profit or Loss --------
Add: Premium on redemption of debentures 1,650
Add: interest on 10% debentures 11,000
Less: interest on 10% investments (35,000)
B Cash flow from investing activities
Interest on investments (35,000 – 10,500) 24,500
C Cash flow from financing activities

© The Institute of Chartered Accountants of Nepal 12


CAP II Paper 1: Advanced Accounting

Interest on debentures paid [11,000 – (1,175-275)] (10,100)


Redemption of debentures [(110,000-77000) at 5% premium] (34,650)
Writing of debtors against provision for bad debt will not be included in cash flow statement

2. Cash and cash equivalents (Inter Jun. 2012 Q6b – 5 Marks)

Answer
Cash equivalents are short-term, highly liquid investments that are readily convertible to
known amounts of cash and which are subject to an insignificant risk of changes in value.
Cash equivalents are held for the purpose of meeting short-term cash commitments rather than
for investment or other purposes. For an investment to qualify as a cash equivalent it must be
readily convertible to a known amount of cash and be subject to an insignificant risk of
changes in value. Therefore, an investment normally qualifies as a cash equivalent only when
it has a short maturity of, say, three months or less from the date of acquisition. Equity
investments are excluded from cash equivalents unless they are, in substance, cash
equivalents, for example in the case of preferred shares acquired within a short period of their
maturity and with a specified redemption date

3. Write short notes on Cash Flow Statement (Inter Jun. 2003 Q 9c- 3 Marks)

Answer
A Cash Flow Statement, as the name signifies, is a statement showing inflows and outflows of
cash and cash equivalents during a certain period of time. It summarizes the inflows and
outflows relating to the following three activities:
1. Operating activities: These include activities relating to the purchase and sale of goods
and service by the company in the normal course of business operations.
2. Investing activities: These deal with the acquisition and disposal of long-term assets
including long-term investments.
3. Financing activities: They reflect the cash effect of all transactions relating to long-term
liabilities and capital of the enterprise.

Where the amount of significant cash and cash equivalent balances held by an enterprise are
kept not available for use by the enterprise, the same should be disclosed separately together
with a commentary by management. A cash flow statement starts with the opening balance of
cash and cash equivalents at the start of the accounting period and is reconciled with the closing
balance of cash and cash equivalents at the end of the period.

1.6. NAS 8 Accounting Policy, Change in Accounting Estimates & Errors

1. While preparing its final accounts for the year ended 31st March 2012, Sky Limited created a
provision for bad and doubtful debts at 2% on trade debtors. A few weeks later the company
found that payments from some of the major debtors were not forthcoming. Consequently, the
company decided to increase the provision by 10% on the debtors as on 31st March, 2012 as

© The Institute of Chartered Accountants of Nepal 13


CAP II Paper 1: Advanced Accounting

the accounts were still open awaiting approval of the Board of Directors. Is this to be considered
as an extraordinary item or prior period item? The company wants to treat the expenditure as
deferred revenue expenditure. Give your comments for the financial year ending on 31-03-
2012 in the context of relevant NAS.
(CAP Dec. 2012 Q5a -5 Marks; Jun. 2006 Q 1b – 4 Marks; Dec. 2006, Q 1d)
Answer
As per NAS 01, Presentation of Financial Statements, an entity shall not present any items of
income or expenses as extra-ordinary therefore classification of the change in provision amount
as extraordinary is not allowable.

In the given case, Sky Limited created a provision for bad and doubtful debts at 2 % on trade
debtors while preparing its final accounts for the year ended 31st March, 2012. As the company
had initially provided provision @ 2%, it can be deduced that the increase in provision is due
to some events that took place after the reporting period and subsequently, the company
decided to increase the provision by 10%. Therefore, the company should assess whether this
is an estimation error or this is a result of events that took place after the reporting period. If it
is concluded that the increase is due to an estimation error, adjustments shall be made to the
financial statements. If it is concluded that the increase is due to some events that took after the
reporting period, the company is advised not to adjust the amount of provision but rather
disclose the fact in the notes if the amount involved is significant. Moreover, the company is
also advised to assess recoverability of individual debtor rather than increasing the amount of
provision by certain percentage.

No items of expenses or income are allowed to be deferred unless allowed by a standard.


Moreover, estimated loss on debtors does not meet the definition of “Assets” under the
conceptual framework itself therefore the company cannot carry this amount as deferred
expenditure but rather should expense it off.

2. Mention any four areas in which different accounting policies may be adopted by different
enterprises.

Answer
• Methods of depreciation e.g., WDV method, SLM method
• Conversion or translation of foreign currency items, e.g. average rate, TT buying rate
• Valuation of inventories, e.g., FIFO, Weighted average method
• Subsequent measurement of Property, plant & equipment

3. When Can a Company change its accounting policy?


(Inter Jun. 2010 Q5a- 3Marks, CAP Jun. 2018 Q6d-3 Marks)

Answer
A change in accounting policy is allowed, under NAS 08, only if the change:

© The Institute of Chartered Accountants of Nepal 14


CAP II Paper 1: Advanced Accounting

i) is required by a NFRS or
ii) results in the financial statements providing reliable and more relevant information
about the effects of transaction, other events or condition on the entity’s financial position,
financial performance or cash flows.

An entity is further required by NAS 08 to disclose the nature of change, reason of change, the
financial impact for current & prior periods presented and financial impact on the opening
balance of retained earnings as a result of such change.

4. Accounting estimate. (CAP Jun. 2011 6c- 3 Marks; Inter Dec. 2011 Q6a-5 Marks)

Answer
Accounting estimate is an approximation of a monetary amount in the absence of a precise
means of measurement. This term is used for an amount measured at fair value where there is
estimation uncertainty, as well as for other amounts that require estimation.
Because of the uncertainties inherent in business activities, some financial statement items can
only be estimated. Further, the specific characteristics of an asset, liability or component of
equity, or the basis of or method of measurement prescribed by the financial reporting
framework, may give rise to the need to estimate a financial statement item. These are called
accounting estimates. Accounting estimates are required to enhance the timeliness & relevancy
of the financial statements without trading off on reliability. The current financial
Examples of estimation in some fields are:
(i) Estimation of useful life of depreciable assets.
(ii) Fair value estimation
(iii) Determining the stage of completion of construction contracts

5. Z Ltd. manufactures a commodity product at two different locations A and B. It values closing
stock consistently as follows:
Location A
Raw materials: At cost, arrived at on FIFO basis.
Work in progress: At Raw Materials cost plus a proportionate share of variable factory
overheads.
Finished goods: At Raw Materials cost plus variable factory overheads.
Location B
Raw materials: At cost, arrived at on LIFO basis.
Work in progress: At Raw Materials cost plus a proportionate share of variable factory
overheads.
Finished goods: At Raw Materials cost plus variable factory overheads.
The overheads considered above are direct wages and benefits of workers, fuel costs and spares
consumed.
Comment on the above accounting policy of Z Ltd. (Inter. Jun. 2006, Q1c – 4 Marks)

Answer:

© The Institute of Chartered Accountants of Nepal 15


CAP II Paper 1: Advanced Accounting

As per the provisions of NAS 02, cost of items of inventory shall be identified using specific
identification. However, cost formula such as FIFO or weighted average may be used for
interchangeable items. LIFO is not an allowed alternative under NAS 2 therefore method of
valuation of inventory at Location A is not in accordance with the requirements of NAS 2.
Under NAS 08, same accounting policies shall be used for similar transactions therefore the
consistency in valuation of work in progress and finished goods at both the location is a
requirement. Moreover, the entity should also consider measuring inventories at lower of cost
or NRV.

1.7. NAS 10 Events After The Reporting Period

1. Distinguish between adjusting and non-adjusting events under relevant Nepal Accounting
Standard.
(CAP Dec. 2009 5a-5 Marks, CAP Jun. 2010 6d- 2.5 Marks, Inter Dec. 2009 Q6b-5 Marks)
Answer:
NAS 10 , Events after the reporting period defines “Events after the reporting period” as
follows:

Events after the reporting period are those events, favorable and unfavorable, that occur
between the end of the reporting period and the date when the financial statements are
authorised for issue. Two types of events can be identified:
(a) those that provide evidence of conditions that existed at the end of the reporting period
(adjusting events after the reporting period); and
(b) those that are indicative of conditions that arose after the reporting period (non-adjusting
events after the reporting period).

Examples of adjusting events include:


• The settlement after the reporting period of a court case that confirms that the entity
had a present obligation at the balance sheet date.
• Receipt of information after the reporting date, about impairment of asset (e.g.
bankruptcy of a customer or sale of inventories below cost) at the end of the reporting
period.
• The determination after the reporting period of the cost of assets purchased or the
proceeds from assets sold, before the end of the reporting period.
• The determination of the amount of profit sharing or bonus payments.
• The discovery of fraud or errors that show that the financial statements are incorrect.

The difference between adjusting & non adjusting events being that:
a. An entity shall adjust the amounts recognised in its financial statements to reflect adjusting
events after the reporting period.
b. An entity shall not adjust the amounts recognised in its financial statements to reflect non-
adjusting events after the reporting period

© The Institute of Chartered Accountants of Nepal 16


CAP II Paper 1: Advanced Accounting

However, if non-adjusting events after the reporting period are material, non-disclosure could
influence the economic decisions that users make on the basis of the financial statements.
Accordingly, an entity shall disclose the following for each material category of non-
adjusting event
after the reporting period:
(a) the nature of the event; and
(b) an estimate of its financial effect, or a statement that such an estimate cannot be made.

2. ABC Limited closed its accounting year on 30.06.2011 and the accounts for that period were
considered and approved by the board of directors on 20th August, 2011. The company was
engaged in laying pipeline for an oil company, deep beneath the earth. While doing the boring
work on 01.09.2011 it had met a rocky surface for which it was estimated that there would be
extra cost to the tune of Rs. 100 lakhs. Further, the court had given its verdict against the
company for the liability of Rs. 50 lakhs on 02.09.2011 shown as contingent liabilities on
accounts. You are required to state with reasons, how it would be dealt with in the financial
statements. (CAP Jun. 2012 Q5a– 5 Marks)

Answer:
NAS 10 , Events after the reporting period defines “Events after the reporting period” as
follows:
Events after the reporting period are those events, favourable and unfavourable, that occur
between the end of the reporting period and the date when the financial statements are
authorised for issue. Two types of events can be identified:
(a) those that provide evidence of conditions that existed at the end of the reporting period
(adjusting events after the reporting period); and
(b) those that are indicative of conditions that arose after the reporting period (non-adjusting
events after the reporting period).

In the first case the incidence, which was expected to push cost by a significant amount, became
evident after the date of authorization of issue of the financial statements (i.e. after approval by
the BOD) & therefore no adjustments should be made with regards to the item in the financial
statements.

In the second case also, the settlement of the court case happened after the financial statements
had been authorized for issue and therefore are not covered by NAS 10. It is assumed that the
disclosure as contingent liability by considering all the circumstances available at that time.
Therefore, the settlement of the case will be account for as an expense and liability in the
financial statements for the next reporting period.

3. State with reasons, how the following events would be dealt with in the financial statements of
National Trading Ltd. for the year ended 31stAshadh, 2072:
(i) An agreement to sell a land for Rs. 30 lakh to another company was entered into on
1stAshadh, 2072. The value of land is shown at Rs. 20 lakhs in the Balance sheet as on
31stAshadh, 2071. However, the sale Deed was registered on 15thShrawan, 2072.

© The Institute of Chartered Accountants of Nepal 17


CAP II Paper 1: Advanced Accounting

(ii) The negotiation with another company for acquisition of its business was started on
2ndMagh, 2071. National Trading Ltd. invested Rs. 40 lakhs on 12thAshoj, 2072. (CAP Jun.
2016 5c -5 Marks; CAP Jun. 2013 Q5c– 5 Marks; Inter Dec. 2006 Q 1e-4 Marks)

Answer
(i) According to NAS 10 ''Event after the reporting period'', assets and liabilities should be
adjusted for events occurring after the reporting period that provide additional evidence of
conditions existing at the end of the reporting period. In the given case, sale of immovable
property was carried out before the closure of the books of accounts. The agreement to sell was
affected on 1stAshadh, 2072 and registration of the sale deed happened on 15thShrawan, 2072,
provides evidence of sale transaction that took place prior to 31st Ashadh. Therefore,
adjustment to assets for sale of land is necessary in the financial statements of National Trading
Ltd. for the year ended 31stAshadh, 2072.
(ii) Events after the reporting period are those events, favorable and unfavorable, that occur
between the end of the reporting period and the date when the financial statements are
authorized for issue. Two types of events can be identified:
(a) those that provide evidence of conditions that existed at the end of the reporting period
(adjusting events after the reporting period); and
(b) those that are indicative of conditions that arose after the reporting period (non-adjusting
events after the reporting period).
In the given case, though the negotiations took place during the Financial Year, the actual
transaction took place during the next financial year. Therefore, no adjustments need to be
made in the financial statements for the year ended 31st Ashadh 2072. However, as per the
provisions of NAS 10, significant events after the reporting period which can influence
economic decision of users should be disclosed in the notes to the financial statements. In this
case, National Trading Ltd. should make disclosure in its financial statements in accordance
with the disclosure requirements of NFRS 3 Business Combination

4. A major fire has damaged assets in a factory of X Co. Ltd. On 8.4.2005, 8 days after the year
end closing of accounts. The loss is estimated to be Rs. 16 crores (after estimating the
recoverable amount of Rs. 24 crores from the Insurance Company).
If the company had no Insurance cover, the loss due to fire would be Rs. 40 crores.
Explain how the loss should be treated in the Final Accounts of the year ended 31.3.2005.
(Inter. Jun. 2006 Q1a-4 Marks)
Answer:
The present event does not relate to conditions existing at the end of the reporting period
therefore the loss of assets by fire of X Ltd. is a non-adjusting event. Hence, no specific
adjustment is required in the financial statements for the year ending on 31.3.2005.
However, as per the disclosure requirements to NAS 10, if non-adjusting events after the
reporting period are material, non-disclosure could influence the economic decisions that users
make on the basis of the financial statements. Accordingly, an entity shall disclose the
following for each material category of non-adjusting event after the reporting period:
(a) the nature of the event; and
(b) an estimate of its financial effect, or a statement that such an estimate cannot be made.

© The Institute of Chartered Accountants of Nepal 18


CAP II Paper 1: Advanced Accounting

Therefore, X Ltd. should make disclosure about the event and the expected loss from the event
in the notes.

5. Kathmandu Ltd. is a company that prepares accounts in accordance with Nepal Financial
Reporting Standards (NFRS). A meeting of the Directors of Kathmandu Ltd. is scheduled to
discuss the following matters with a view to finalizing the accounts for the year ending 32nd
Ashadh 2075:
i) A fire occurred in one of the warehouses of Kathmandu Ltd. on 3rd Shrawan 2075,
destroying inventory which had a cost price of Rs. 100,000 and a net realizable value of
Rs. 150,000.
ii) On 9th Shrawan 2075, Kathmandu Ltd. received information that one of their largest
customers had gone bankrupt. At 32nd Ashadh 2075, this customer owed Kathmandu Ltd.
Rs. 235,000. It is anticipated that Kathmandu Ltd. can only receive 10 paisa for every Rs.
1 they were owed.
iii) In Shrawan 2075, Kathmandu Ltd. sold inventory which had been in one of their
warehouses for the past two years, for Rs. 75,000. This had been included in the financial
statements, for the year ended 32nd Ashadh 2075, at its cost price of Rs. 105,000.
iv) On 30th Ashadh 2075, an employee of Kathmandu Ltd. fell and injured her arm at work.
This employee has commenced legal action. The solicitors for Kathmandu Ltd. informed
the company on 10th Ashwin 2075, that it is probable they will be found liable and have
to pay this employee Rs. 33,000.
Required: Advise the board on the accounting treatment of these issues.
(CAP Jun. 2019 Q5a-5 Marks)
Answer:

i) This is a non-adjusting event as it occurred after the reporting date. If it is material it


should be disclosed in the financial statements, but it should not be recognized in the
financial statements.
ii) This is an adjusting event as it provides evidence of conditions that existed at the
reporting date. The company should recognize this in the accounts by debiting bad
debts Rs. 211,500 (90/100*235,000) and crediting trade receivables Rs. 211,500.
iii) This is also an adjusting event as it confirms the net realizable value of the goods that
were in stock at 32nd Ashadh 2075. The company will have to recognize this by
reducing the value of closing inventory in the financial statements.
iv) This is an adjusting event. As the injury took place prior to the year end it has now
been confirmed that the company will probably have to pay out Rs. 33,000 in
compensation. The company will have to recognize a provision of Rs. 33,000 in the
financial statements by debiting expense of Rs. 33,000 and crediting provision in the
statement of financial position for Rs. 33,000.

© The Institute of Chartered Accountants of Nepal 19


CAP II Paper 1: Advanced Accounting

1.8. NAS 16 Property, Plant & Equipment (PPE)

1. During the current year 2069/70, M/S Harish Power made the following expenditure relating
to its Plant & Machinery:
Particulars Amount (Rs.)
General Repairs 400,000
Repairing of Electric Motors 100,000
Partial Replacement of Parts of Machinery 50,000
Substantial improvements to the electrical wiring system
which will increase efficiency of the plant & machinery 1,000,000
Explain with reference to relevant NAS; how the above expenses should be treated?
(CAP Dec. 2013 Q5c-5 Marks; Inter Dec. 2006, Q 1c-4 Marks)
Answer
As per NAS 16 Property, Plant and Equipment, both initial & subsequent expenditure incurred
on an item of PPE (add to, replace part of or servicing cost) are recognized as included in the
cost of an item of PPE if, and only if:
a. It is probable that economic benefit associated with the item will flow to the entity &
b. The cost of the item can be measured reliably.

Also, the standard states that cost of day-to-day servicing of an item of PPE are recognized
in the profit or loss when incurred.
Based on these principles the treatment of expenditure incurred by Harish Power shall be
as follows:
Particulars Capitalized Charged to P/L
General Repairs 400,000
Repairing of Electric Motors 100,000
Partial Replacement of Parts of Machinery * 50,000
Substantial improvements to the electrical wiring system 1,000,000
which will increase efficiency of the plant & machinery
* With respect to partial replacement of parts of the machinery, the company shall
derecognize the carrying amount of replaced parts and include current replacement cost .

2. An electricity company decided to replace some parts of its plant by an improved plant. The
plant to be replaced was built in 2043 for Rs. 4,200,000. It is estimated that it would cost Rs.
7,800,000 to build a new plant of the same size and capacity. The cost of the new plant as per
the improved design was Rs. 12,600,000 and in addition, material belonging to the old plant
valued at Rs. 456,000 was used in the construction of the new plant. The balance of the plant
was sold for Rs. 360,000.
Compute the amount to be written off to and the amount to be capitalized. Also prepare Plant
Account and Replacement Account.
(CAP Dec. 2014 3a-7 Marks; CAP Jun. 2015 Q3a- 7 Marks)

© The Institute of Chartered Accountants of Nepal 20


CAP II Paper 1: Advanced Accounting

Answer
The question is missing information with regards to accumulated depreciation till date. For
solving, it has been assumed that the accumulated depreciation on the old plant till date is
Rs. 30,00,000. Since we are given the cost of the plant to be replaced, the information
provided with regards to the cost breakdown and increment is of no significance in solving
the question.

Old Plant A/C


Dr. Cr.
Particulars Amount Particulars Amount
To Balance b/d 4,200,000 By Accumulated depreciation 30,00,000
By New Plant A/C 4,56,000
By Bank A/C 3,60,000
By Loss on replacement 384,000
(charged to P/L for the period)
44,00,000 44,00,000

New Plant A/c


Dr. Cr.
Particulars Amount Particulars Amount
To Material/Labor/OH 12,600,000 By Balance c/d 13,056,000
To Old Plant A/c 456,000
13,056,000 13,056,000

3. Revaluation of PPE (Inter Jun. 2003 Q 9e - 3 Marks)

Answer
As NAS 16 Property, Plant & Equipment:
On upward Revaluation:
If an asset's carrying amount is increased as a result of a revaluation:
• the increase shall be credited directly to equity under the heading of Revaluation
Surplus.
• However, the increase shall be recognized in profit or loss to the extent that it
reverses a revaluation decrease of the same asset previously recognized in profit or
loss.
Hence, in case of revaluation of property, plant & equipment to Rs. 2,500,000; Rs.
500,000 shall be credited directly to equity under Revaluation Surplus.

On downward Revaluation:
If an asset's carrying amount is decreased as a result of a revaluation:
• the decrease shall be recognized in profit or loss.

© The Institute of Chartered Accountants of Nepal 21


CAP II Paper 1: Advanced Accounting

• However, the decrease shall be debited directly to equity under heading of


revaluation surplus to the extent of any credit balance existing in the revaluation
surplus in respect of that asset.

When an asset is revalued, the entire class of concerned asset should be revalued. Revaluation
should be reviewed on a periodic basis so that the revalued amount stays attune to the market
value.

4. The Written Down Value of Property, Plant & Equipment of Prudence International Pvt. Ltd.
as on Ashadh end, 2072 was Rs. 2,000,000. The company decided to revalue its Property, Plant
& Equipment on Ashadh end, 2072. This is the first instance when the company has gone for
any revaluation of assets.
With reference to NAS; explain the financial impact on account of revaluation of Property,
Plant & Equipment on (i) Reserves & Surplus and (ii) Profit & Loss if:
i) Property, Plant & Equipment is revalued at Rs. 2,500,000.
ii) Property, Plant & Equipment is revalued at Rs. 1,600,000.
(CAP Dec. 2015 Q-5B)
Answer
As NAS 16 Property, Plant & Equipment:
On upward Revaluation:
If an asset's carrying amount is increased as a result of a revaluation:
• the increase shall be credited directly to equity under the heading of Revaluation
Surplus.
• However, the increase shall be recognized in profit or loss to the extent that it
reverses a revaluation decrease of the same asset previously recognized in profit or
loss.
Hence, in case of revaluation of property, plant & equipment to Rs. 2,500,000; Rs.
500,000 shall be credited directly to equity under Revaluation Surplus.

On downward Revaluation:
If an asset's carrying amount is decreased as a result of a revaluation:
• the decrease shall be recognized in profit or loss.
• However, the decrease shall be debited directly to equity under heading of
revaluation surplus to the extent of any credit balance existing in the revaluation
surplus in respect of that asset.
Hence, in case of revaluation of property, plant & equipment to Rs. 1,600,000; Rs.
400,000 shall be recognized as expense in Profit or Loss for the period.

5. Tinkune Ltd.’s head office building is the only building it owns. Using professional valuers, it
revalued the building on 1st Shrawan 2074, at Rs. 21,00,000. Tinkune Ltd. has adopted a
revaluation policy for buildings from this valuation date and has decided that the original useful
life of buildings has not changed as a result of the revaluation. The building was acquired on
1st Shrawan 2064. The cost of the building on acquisition was Rs. 25,00,000 and the

© The Institute of Chartered Accountants of Nepal 22


CAP II Paper 1: Advanced Accounting

accumulated depreciation to the Ashadh end, 2074 amounted to Rs. 5,00,000. The depreciation
up to 1st Shrawan 2074 was depreciated evenly since acquisition. The professional valuer
believes that the residual value on the building would be Rs. 6,00,000 at the end of its useful
life.
Required:
Calculate the depreciation amount of the building for the year ended 32nd Ashadh 2075 based
on the information provided in the above scenario. (CAP Dec. 2018 Q5a-5 Marks)

Answer
The depreciation amount is as follow:
To calculate the new depreciation amount, we use the following depreciation formula.
Revalued cost of asset-residual value 21,00,000-6,00,000
Expected useful life of asset 40 Years

Depreciation per year 37,500

Working Note 1
Building-original cost 25,00,000
Building- Accumulated Depreciation 5,00,000
Accumulated Depreciation/Cost= 20%
Building has been depreciated by 20 % over 10 years, so annual rate of depreciation has been
2 % i.e. 20%/10 years, as asset has been depreciated evenly since acquisition. Therefore, the
original useful life is 50 years and the remaining useful life is 40 years.

6. Nepal Electricity Authority decides to replace one of its old plants with a modern one with a
larger capacity. The plant when installed in 2040 B.S., cost was Rs. 44,00,000, the components
of materials, labor and overheads being in the ratio 5:3:2. It is ascertained that the costs of
materials and labor have gone up by 40% and 80% respectively. The proportion of overheads
to total costs is expected to remain the same as before.
The cost of the new plant as per improved design is Rs. 60,00,000 and in addition, material
recovered from the old plant of a value of Rs. 2,40,000 has been used in the construction of
the new plant. The old plant was scrapped and sold for Rs. 7,50,000.
The accounts of the company are maintained under the double accounts system. Indicate how
much would be capitalized and the amount that would be charged to revenue. Show the ledger
accounts. (Inter Dec. 2009 Q4-10 Marks; CAP Jun. 2010 3a (i) – 7 Marks)

Answer
The question is missing information with regards to accumulated depreciation till date. For
solving, it has been assumed that the accumulated depreciation on the old plant till date is
Rs. 30,00,000. Since we are given the cost of the plant to be replaced, the information
provided with regards to the cost breakdown and increment is of no significance in solving
the question.

© The Institute of Chartered Accountants of Nepal 23


CAP II Paper 1: Advanced Accounting

Old Plant A/C


Dr. Cr.
Particulars Amount Particulars Amount
To Balance b/d 44,00,000 By Accumulated depreciation 30,00,000
By New Plant A/C 2,40,000
By Bank A/C 7,50,000
By Loss on replacement 4,10,000
(charged to P/L for the period)
44,00,000 44,00,000

New Plant A/c


Dr. Cr.
Particulars Amount Particulars Amount
To Material/Labor/OH 60,00,000 By Balance c/d 62,40,000
To Old Plant A/c 2,40,000
62,40,000 62,40,000

7. In Fiscal Year 2061/062, M/S PD Ltd. re-valued its Plant and Machinery upward by Rs. 57,000
by crediting as income in Income Statement. In the fiscal year 2063/64, the assets under Plant
and Machinery are re-valued downwards by Rs. 25,000 by changing as expense to income
statement. Give your view on the accuracy of above accounting treatments with reference to
NAS-16.
(Inter June. 2008 Q4b)
Answer
According to NAS – 16 'Property, Plant and Equipment and Depreciation" – when an asset's
carrying amount is increased as a result of a revaluation, the increase should be recognized in
other comprehensive income and accumulated in equity under the heading of revaluation
surplus. However, the increase shall be recognized in profit or loss to the extent that it reverses
a revaluation decrease of the same asset previously recognized in profit or loss.

When an asset's carrying amount is decreased as a result of revaluation, the decrease should be
recognized as an expense. However, the decrease shall be recognized in other comprehensive
income to the extent of any credit balance existing in the revaluation surplus in respect of that
asset. The decrease recognized in other comprehensive income reduces the amount
accumulated in equity under the heading of revaluation surplus.

In the given case the plant and machinery were revalued upward by Rs. 57,000 in fiscal year
2061/62 and the surplus on revaluation is credited to income statement. The credit of
revaluation surplus to income statement is against NAS-16, which should be credited to
revaluation surplus. Similarly, the downward revaluation of plant and machinery in fiscal year
2063/64 should be deducted from 'revaluation surplus', created in fiscal year 2061/62.
Impact on depreciation has been ignored due to lack of information

© The Institute of Chartered Accountants of Nepal 24


CAP II Paper 1: Advanced Accounting

8. Nepal Electricity Company laid down a Main at a cost of Rs. 8,000,000. Some years later the
company laid down an auxiliary Main for one-fourth of the old Main at a cost of Rs.3,000,000.
It also replaced the rest of the length of the old Main at a cost of Rs.9,000,000 the cost of
material and labor having gone up by 15%. Sale of old materials realized Rs. 200,000. Old
materials valued at Rs. 200,000 were used in renewal and those valued at Rs. 300,000 were
used in auxiliary Main.
Pass Journal Entries for recording the above transactions in the books of Nepal Electricity
Company. Show your workings (Inter Jun. 2011 Q5a- 10 Marks)

Answer
The question is missing information with regards to accumulated depreciation till date. For
solving, it has been assumed that the accumulated depreciation on the old plant till date is Rs.
6,000,000. Since we are given the cost of the plant to be replaced, the information with regards
to current cost of replacement and cost increment is of no significance in solving the question.

Journal Entries in the books of Nepal Electricity Company


Dr Cr
Particulars
Rs. Rs.
Auxiliary Main Account 3,000,000
To Bank Account 3,000,000
(Being cost incurred on laying auxiliary)
Accumulated Depreciation Account 6,000,000
Bank A/C 200,000
Auxiliary Main A/C 300,000
New Main A/c 200,000
Loss on replacement (P/L) 1,300,000
To Old Main A/C 8,000,000
(Being accumulated depreciation on scrapped asset
netted, scrap sold and part used on new and auxiliary
main recorded)
New Main Account 9,000,000
To Bank Account 9,000,000
(Being additional cost of New Main capitalized)
Note: It has been assumed that the total cost of 3,000,000 & 9,000,000 is excluding cost
of material used from the old main. Students may make an assumption that the cost
is inclusive, in which case the solution will slightly vary.

9. Palpa Power company decided to replace some parts of its plant by an improved plant. The
plant to be replaced was built in 2065 for Rs.70,00,000. It is estimated that it would cost
Rs.130,00,000 to build a new plant of the same size and capacity. The cost of the new plant as
per the improved design was Rs.2,10,00,000 and in addition, material belonging to the old plant
valued at Rs.7,60,000 was used in the construction of the new plant. The balance of the plant

© The Institute of Chartered Accountants of Nepal 25


CAP II Paper 1: Advanced Accounting

was sold for Rs.6,00,000. Compute the amount to be written off to revenue and the amount to
be capitalized. Also prepare Plant account and Replacement account.
(CAP Jun. 2011 Q4c- 5 Marks)

Answer
The question is missing information with regards to accumulated depreciation till date. For
solving, it has been assumed that the accumulated depreciation on the old plant till date is Rs.
6,000,000. Since we are given the cost of the plant to be replaced, the information with regards
to current cost of replacement is of no significance in solving the question.

Old Plant A/C


Dr. Cr.
Particulars Amount Particulars Amount
To Balance b/d 70,00,000 By Accumulated depreciation 60,00,000
By gain on replacement 3,60,000 By New Plant A/C 7,60,000
(charged to P/L for the period)
By Bank A/C 6,00,000

73,60,000 73,60,000

New Plant A/c


Dr. Cr.
Particulars Amount Particulars Amount
To Bank 2,10,00,000 By Balance c/d 2,77,60,000
To Old Plant A/c 7,60,000
2,77,60,000 62,40,000

10. Discuss the treatment of upward and downward revaluation of assets as per NAS 16.
(CAP Jun. 2014 Q5b- 5 Marks)
Answer
As per NAS 16 if an asset’s carrying amount is increased as a result of a revaluation, the
increase shall be recognized in other comprehensive income and accumulated in equity under
the heading of revaluation surplus. However, the increase shall be recognized in profit or loss
to the extent that it reverses a revaluation decrease of the same asset previously recognized in
profit or loss.

If an asset’s carrying amount is decreased as a result of a revaluation, the decrease shall be


recognized in profit or loss. However, the decrease shall be recognized in other comprehensive
income to the extent of any credit balance existing in the revaluation surplus in respect of that
asset. The decrease recognized in other comprehensive income reduces the amount
accumulated in equity under the heading of revaluation surplus.

© The Institute of Chartered Accountants of Nepal 26


CAP II Paper 1: Advanced Accounting

11. M/s. Laghu Udyog Limited has been charging depreciation on an item of plant and machinery
on straight line basis. The machine was purchased on 1-4-2070 at Rs. 3,25,000. It is expected
to have a total useful life of 5 years from the date of purchase and residual value of Rs. 25,000.
Calculate the book value of the machine as on 1-4-2072 and the total depreciation charged till
31-3-2072 under SLM. The company wants to change the method of depreciation and charge
depreciation @ 20% on WDV from 2072-73. Is it valid to change the method of depreciation?
Explain the treatment required to be done in the books of accounts in the context of Accounting
Standards.
Ascertain the amount of depreciation to be charged for 2072-73 and the net book value of the
machine as on 31-3-2073 after giving effect of the above change.
(CAP Jun. 2017 Q5b-5 Marks)
Answer
As per NAS 16 ‘ Property, Plant & Equipment’, the depreciation method applied to an asset
shall be reviewed at least at each financial year end and, if there has been a significant change
in the expected pattern of consumption of the future economic benefits embodied in the asset,
the method shall be changed to reflect the changed pattern. Such a change shall be accounted
for as a change in an accounting estimate in accordance with NAS 08.

As per NAS 08 ‘Accounting Policies, Changes in Accounting Estimates & Errors’, changes in
accounting estimates shall be adjusted prospectively that means the effect of a change in an
accounting estimate shall be included in the determination of net profit or loss in:
(a) The period of the change, if the change affects the period only; or
(b) The period of the change and future periods, if the change affects both.

In the given case, the company can change the method of depreciation from year 2072-73 if
the conditions set aside in above paragraph have been fulfilled.
Depreciation for year 2072-73 and net book value of Machine as on Rs.
31.3.73 after effect of the change
Book value of Machinery as on 01.04.2072 2,05,000
Current year depreciation as per new method (WDV) (2,05,000 X 20%) 41,000
Net Book value as on 31.03.2073 (2,05,000–41,000) 1,64,000
Working Note:
Book Value of Machinery and Depreciation under SLM as on 01-04-2072
Rs.
Cost of Machine purchased on 01.04.2070 3,25,000
Less: Residual Value 25,000
Depreciable amount 3,00,000
Useful life of Machine 5 Years
Depreciation for 2 Years (Rs.3,00,000x2/5) 1,20,000
Book value as on 01.04.2072 2,05,000

12. The depreciation of machinery under two different methods is as given below:
Year SLM (lacs) WDV (lacs)

© The Institute of Chartered Accountants of Nepal 27


CAP II Paper 1: Advanced Accounting

1 7.8 21.38
2 7.8 15.80
3 7.8 11.68
4 7.8 8.64
Total 31.2 57.50

What would be the amount of resultant surplus/ deficiency if the company decides to switch
over from WDV method to SLM method after first four years? Also state how you will treat
the same in the Accounts. (Inter Jun. 2009 Q2b- 4 Marks)

Answer
Method of depreciation as per the provisions of NAS 16 and NAS 8 are merely accounting
estimates and therefore changes to depreciation method is accounted for prospectively, i.e. the
carrying number of NRs. 57.50 lakhs are taken as depreciable amount for the rest of the useful
life of the asset.

13. Alex Ltd. intends to set up a solar plant. Alex Ltd. has acquired a dilapidated factory, having
an area of 7,500 acres at a cost of Rs. 70,000 per acre. Alex Ltd. has incurred Rs. 5,000,000 on
demolishing the old factory building thereon. A sum of Rs. 4,357,500 (including 13% VAT)
was realized from sale of material salvaged from the site. Alex Ltd. also incurred Registration
Charges of 5% of Land Value, paid legal and consultancy charges Rs. 500,000 for land
acquisition and incurred Rs. 200,000 on Title Guarantee Insurance. Compute the value of land
acquired to be booked in books of the company.
(CAP Jun. 2019 Q5b-5 Marks)

Answer
Particular Rs in lakhs
Purchase price @ Rs 70,000 per acre for 7,500 acres 5,250.00
Stamp duty & registration charges @5% 262.50
Legal fees 5.00
Title guarantee insurance 2.00
Demolition expenses
Less: Sale of salvaged materials (net of VAT)
(43,57,500× 100/113) = (38.56) 11.44
Value of land 5,530.94

1.9. NAS 17 Leases

1. Write short notes on Minimum Rent and Short workings


(Inter Jun. 2001, Q 6a 4 Marks)
Answer:
Minimum Rent also known as Dead Rent is the minimum amount which the lessee has to pay
for each period even if the mine has not been worked at all or when the output for such period
is below a certain quantity provided for in the agreement.

© The Institute of Chartered Accountants of Nepal 28


CAP II Paper 1: Advanced Accounting

Short workings arise when the minimum rent paid is in excess of that amount which would
have been payable on the basis of actual output.

Such short working may be recouped in the future years that means the actual royalty due
will be reduced to the extent of the unrecouped year/years which had shortcomings in the
past.

2. Basic criteria for classification of leases into Operating Lease and Finance Lease.
(June 2007, Q 5c)
Answer:
Leases are classified based on the extent to which risks and rewards incident to ownership of
a leased asset lie with the Lessor or the Lessee.
Risks include the possibilities of losses from idle capacity or technological obsolescence and
of variations in return due to changing economic conditions.
Rewards may be represented by the expectation of profitable operation over the economic life
of the asset and of gain from appreciation in value or realization of residual value.

A lease is called a Finance Lease if it transfers substantially all the risks and rewards incident
to ownership. Title may or may not eventually be transferred. A lease is called an Operating
Lease if it does not transfer substantially all the risks and rewards incident to ownership.

3. Sale and lease back transaction


(CAP Dec. 2010 6a-2 Marks; CAP Dec. 2017 Q6c-3 Marks; CAP Jun. 2009 6e- 4 Marks)
Answer
As per NAS 17 on ‘Leases’, a sale and leaseback transaction involve the sale of an asset by the
vendor and the leasing back of same asset. The lease payments and the sale price are usually
interdependent, as they are negotiated as a package. The accounting treatment of a sale and
lease back transaction depends upon the type of lease involved.
If a sale and leaseback transaction results in a finance lease, any excess of sale proceeds over
the carrying amount shall not be immediately recognized as income by a seller-lessee. Instead
it shall be deferred and amortized over the lease term.
If sale and leaseback transaction results in an operating lease, and it is clear that the transaction
is established at fair value, any profit or loss should be recognized immediately. If the sale price
is below fair value any profit or loss should be recognized immediately except that, if the loss
is compensated by future lease payments at below market price, it should be deferred and
amortized in proportion to the lease payments over the period for which the asset is expected
to be used. If the sale price is above fair value, the excess over fair value should be deferred
and amortized over the period for which the asset is expected to be used.

4. Lease (CAP Dec. 2013 6d-2.5 Marks)

Answer

© The Institute of Chartered Accountants of Nepal 29


CAP II Paper 1: Advanced Accounting

A lease is an agreement whereby the lessor conveys to the lessee in return for a payment or
series of payments the right to use an asset for an agreed period of time. A lease, based on the
features of the agreement can be classified as follows:
Finance Lease: A finance lease is a lease that transfers substantially all the risks and rewards
incidental to ownership of an asset. Title may or may not eventually be transferred. There are
certain conditions that are to be satisfied under NAS 17 for a lease to be classified as finance
lease.
Operating Lease: An operating lease is a lease other than a finance lease

5. A machine having expected useful life of 6 years is leased for 4 years. Both the cost and fair
value of the machinery are Rs. 17,00,000. The amount will be paid in 4 equal installments and
at the termination of lease, lessor will get back the machinery. The unguaranteed residual value
at the end of the 4th year is Rs. 1,70,000. The IRR of investment is 10%. The present value of
annuity factor of Rs. 1 due at the end of 4th year at 10% IRR is 3.169. The present value of Rs.
1 due at the end of 4th year at 10% rate of interest is 0.683.
State with reason on the basis of your calculation, whether the lease constitutes finance lease
or not. (CAP Dec. 2018 Q5c-5 Marks)

Answer
As per NAS 17 on "Leases", one of the situations that individually or in combination would
normally lead to a lease being classified as a finance lease is that if at the inception of the
lease the present value of the minimum lease payment amounts to at least substantially all of
the fair value of the leased asset.
Determination of nature of lease Rs.
Fair value of asset is Rs. 17,00,000 and unguaranteed residual value is Rs.1,70,000
Present value of residual value at the end of 4th =1,70,000*0.683 =1,16,110
year
Present value of lease payment recoverable = 17,00,000–1,16,110 =15,83,890
The percentage of present value of lease
payment to fair value of the asset is = (15,83,890/17,00,000) *100% =93.17%

Since the present value of minimum lease payment substantially cover the major portion of
fair value of leased assets and life of the asset, the lease transaction meets the definition of
finance lease as per NAS -17. Hence, it constitutes a finance lease.

6. Finance lease (Inter Jun. 2012 Q6d-5 Marks; CAP Jun. 2017 6e-3 Marks)
Answer:
A finance lease is lease that transfers substantially all the risks and rewards incidents to
ownership of an asset. Title may or may not eventually transferred.
Whether a lease is a finance lease or an operating lease depends on the substance of the
transaction rather than the form of the contract.1 Examples of situations that individually or
in combination would normally lead to a lease being classified as a finance lease are:

© The Institute of Chartered Accountants of Nepal 30


CAP II Paper 1: Advanced Accounting

a) the lease transfers ownership of the asset to the lessee by the end of the lease term;
b) the lessee has the option to purchase the asset at a price that is expected to be sufficiently
lower than the fair value at the date the option becomes exercisable for it to be reasonably
certain, at the inception of the lease, that the option will be exercised;
c) the lease term is for the major part of the economic life of the asset even if title is not
transferred;
d) at the inception of the lease the present value of the minimum lease payments amounts to
at least substantially all of the fair value of the leased asset; and
e) the leased assets are of such a specialized nature that only the lessee can use them without
major modifications.
Indicators of situations that individually or in combination could also lead to a lease being
classified as a finance lease are:
a) if the lessee can cancel the lease, the lessor’s losses associated with the cancellation are
borne by the lessee;
b) gains or losses from the fluctuation in the fair value of the residual accrue to the lessee
(for example, in the form of a rent rebate equaling most of the sales proceeds at the end of
the lease); and
c) the lessee has the ability to continue the lease for a secondary period at a rent that is
substantially lower than market rent.

7. Following figures represent expected production of a machinery taken on 5 years’ operating


lease:
Years Lease Amount (Rs) Production (MT)
1 15,000 13,000
2 35,000 21,000
3 50,000 34,000
4 47,000 32,500
5 44,500 30,000
191,500 130,500
Required:
Pass necessary journal entries in the books of lessee for initial two years.
(CAP Jun. 2010 Q4b- 5 Marks)
Answer
a) Lease rental to be recognized as expense in proportionate to units of production.
Here, Total lease rental for 5 years= Rs.191,500
Total Production for 5 years=130,500

Hence, Lease rental to be charged in 1st year=Rs. 13,000/130,500 X 191,500= Rs.19,077


Lease rental to be charged in 2nd year=Rs. 21,000/130,500 X 191,500=Rs.30,816

Journal Entries in the books of lessee for first and second year:
First Year
a) Lease Rent Dr 15,000
To Bank Cr. 15,000

© The Institute of Chartered Accountants of Nepal 31


CAP II Paper 1: Advanced Accounting

(Being Lease rent for the first year of lease paid)

b) Lease Rent Dr 4,077


To Lease Rent Adjustment Ac Cr 4,077
(Being adjustment of lease rent for the year)

Second Year
a) Lease Rent Dr. 35,000
To Bank Cr. 35,000
(Being Payment of Lease rent for the year)

b) Lease Rent Adjustment Dr. 4,184


Lease Rent Expenses Dr. 30,816
To Lease Rent Cr. 35,000
(Being recognition of lease rent as expense for the year)

8. Initial recognition of finance lease in the financial statement of Lessee.


(CAP Jun. 2014 Q6d- 2.5 Marks)
Answer
At the commencement of the lease term, lessees shall recognize finance lease as assets and
liabilities at amount equal to the fair value of the lease property or, if lower, the present value
of the minimum lease payments, each determined at the inception of the lease. The discount
rate to be used in calculating the present value of the minimum lease payments is the interest
rate implicit in the lease, if practicable to determine; if not, the lessee’s incremental borrowing
rate shall be used. Any initial direct cost of the lessee is added to the amount as the asset.

9. M/s. Zipee Traders sold a car to a customer on installment sales basis.


The sales price of the car is Rs. 6,000,000.
The customer was required to pay following:
Shrawan Down Payment (At the
01, 2072 time of delivery) 3,000,000
Shrawan (includes Interest of
01, 2073 At the end of Year 1 1,860,000 Rs. 360,000)
Shrawan (includes Interest of
01, 2074 At the end of Year 2 1,680,000 Rs. 180,000)
Total Amount 6,540,000
M/s. Zipee Traders booked the entire Rs. 6,540,000 as sales on Shrawan 01, 2072. With
reference to the applicable NAS, explain whether the accounting treatment of M/s. Zipee
Traders is justifiable or not. (CAP Jun. 2016 Q5a- 5 Marks)

Answer

© The Institute of Chartered Accountants of Nepal 32


CAP II Paper 1: Advanced Accounting

Under the provisions of NAS 17 Leases, Manufacturer or dealer lessors shall recognize selling
profit or loss in the period, in accordance with the policy followed by the entity for outright
sales. The lessor, under finance lease is required to recognize receivable at an amount equal to
Net investment in the lease and finance income should be recognized shall be based on a pattern
reflecting constant periodic return on the net investment in the lease.

The recognition of entire amount of Rs. 6,540,000 as sales is not in line with the
aforementioned provisions. Zipee Traders should recognize sales to the extent of Rs. 6,000,000
at the time of the sales whereas the balance of 540,000 should be recognized as finance income
over the period of the lease.

10. Sagun Ltd. took a factory premises on lease on 01.04.2073 for Rs. 1, 00,000 per month. The
lease is operating lease. During Ashadh, 2074, Sagun Ltd. relocates its operation to a new
factory building. The lease of the old factory premises continues to live up to 31.12.2076. The
lease cannot be cancelled and cannot be sub-let to another user. The auditor insists that lease
rent of balance 33 months up to 31.12.2076 should be provided in the accounts for the year
ending 31.03.2074. Sagun Ltd. seeks your advice. (CAP Jun. 2017 Q5a-5 Marks)

Answer
In accordance with the provisions of NAS 37 ‘Provisions, Contingent Liabilities and
Contingent Assets’, if an enterprise has a contract that is onerous, the present obligation
under the contract should be recognized and measured as a provision. An onerous contract is a
contract in which the unavoidable cost of meeting the obligations under the contract exceed the
economic benefit expected to be received under it.
In the given case, the operating lease contract has become onerous as the economic benefit of
lease contract for next 33 months up to 31.12.2076 will be nil. However, the lessee, Sagun Ltd.,
has to pay lease rent of Rs.3, 300,000 (i.e. Rs.100, 000 p.m. for next 33 months). Therefore,
provision on account of Rs.3, 300,000 is to be provided in the accounts for the year ending
31.03.2074.

Hence auditor’s contention to provide for the lease rent of balance 33 months up to 31.12.2076
in the accounts for the year ending 31.03.2074 is correct.

1.10. NAS 18 Revenue

1. X Limited has recognized Rs. 10 lakhs, on accrual basis, income from dividend on units of
mutual funds of the face value of Rs. 50 lakhs held by it as at the end of the financial year 31st
March, 2006. The dividends on mutual funds were declared at the rate of 20% on 15th June,
2006. The dividend was proposed on 10th April, 2006 by the declaring company. Whether the
treatment is as per the relevant Accounting Standard? (Inter Dec 2006, Q1b-4 Marks)

Answer

© The Institute of Chartered Accountants of Nepal 33


CAP II Paper 1: Advanced Accounting

NAS-18 Revenue, states that dividends should be recognized when the shareholders' right to
receive payment is establishment. In the given case, the dividend is proposed on 10th April
2006, while it is declared on 15th June 2006. Hence, the right to receive payment is established
on 15th June 2006. As per the above-mentioned paragraph, income from dividend on units of
mutual funds should be recognized by X Ltd. in the financial year ended on 31st March 2007.

2. A National Finance Company Ltd. recognized Rs. 5 lakhs, on accrual basis, income from
dividend during the year 2071/72, on shares of the face value of Rs. 25 lakhs held by it in
Everest Bank Ltd. as at 31st Ashadh, 2072. Everest Bank Ltd. proposed dividend @ 20% on
25th Shrawan, 2072. However, dividend was declared on 30th Ashoj 2072. Financial Statement
of the National Finance Company was approved on 15th Kartik, 2072 by the Board of Directors
of the company. Please state with reference to relevant Nepal Accounting Standard whether
the treatment accorded by National Finance Company Ltd. is in order.
(CAP Dec. 2015 Q-5c)

Answer
Para 30 of NAS 18 "Revenue" states that dividends shall be recognized when the shareholder's
right to receive payment is established. Thus, an investment in shares dividend income is not
recognized in the statement of Profit or Loss (or other comprehensive income as per the
categorization adopted for the investment) until the right to receive dividend is established.
In the given case, the dividend is proposed on 25th Shrawan, 2072, while it was declared on
30th Ashoj 2072. Hence, the right to receive dividend is established on 30th Ashoj 2072 only.
Therefore, on applying the provisions stated in the standard, income from dividend on shares
should be recognized by National Finance Company Ltd. in the financial year 2072/73 only.

3. Rajesh Suppliers purchased goods on credit from Prakash Hardwares for Rs. 1.5 crores for
export. The export order was cancelled. Rajesh Suppliers decided to sell the same goods in the
local market with a price discount. Prakash Hardwares was requested to offer a price discount
of 15 %. The Chief Accountant of Prakash Hardwares wants to adjust the sales figures to the
extent of the discount requested by Rajesh Suppliers. Discuss whether the treatment is justified.
(CAP Dec. 2013 Q5a-5 Marks)
Answer
Prakash Hardwares has sold goods to Rajesh Suppliers on credit worth Rs. 1.5 Crores and the
sale was completed in all respects. Rajesh Supplier’s decision to sell the same in the domestic
market at a discount does not affect the amount recorded as sales by the Prakash Hardwares.
The discount of 15% offered by Prakash Hardwares after request of Rajesh Suppliers is in the
nature of cash discount and not trade discount. Therefore, Prakash Hardware should record this
as discount provided rather than altering the amount of sales.

4. Vikas Electronics is a manufacturer of LED television sets since past 2 years. It deals in both
wholesale trade via its dealers and retail trade via its showroom located at 15 different places
across the country. Vikas electronics has just received an order from a newly established hotel
for supply of 50 pieces of 32 inches LED sets and 100 pieces of 42 inches LED sets. Vikas
electronics entered into the agreement 3 months ago and all LED sets had already been supplied

© The Institute of Chartered Accountants of Nepal 34


CAP II Paper 1: Advanced Accounting

to the hotel. However, the hotel management has found serious flaws on 25 pieces of 32 inches
LED sets and 30 pieces of 42 inches LED sets. Since the warranty period of the television sets
had not expired, the hotel management has decided to return the damaged sets to Vikas
electronics. Vikas electronics seeks your advice as to how to recognize the revenue for this
transaction. Advice the management.
(CAP Dec. 2015 5a)
Answer
Revenue from the sale of goods shall be recognized when all the following conditions
have been satisfied:
(a) the entity has transferred to the buyer the significant risks and rewards of ownership
of the goods;
(b) the entity retains neither continuing managerial involvement to the degree usually
associated with ownership nor effective control over the goods sold;
(c) the amount of revenue can be measured reliably;
(d) it is probable that the economic benefits associated with the transaction will flow to
the entity; and
(e) the costs incurred or to be incurred in respect of the transaction can be measured
reliably.

The customer has returned goods on account of damages during the warranty period. It appears
that the goods were sold under warranty ad not sale on approval basis. Therefore, liability under
warranty does not provide enough ground to implicate that the risk and reward of ownership
had not been transferred to the customer. Therefore, Vikas Electronics should recognize
revenue on the entire transaction. However, it should provide for adequate provision for
liability under warranty and the recent return of goods returned by the customer should be taken
into consideration in estimation of the warranty provision.

5. Shree Ganesh Ltd., a manufacturing company produces durable consumer goods with an
annual turnover of Rs. 100 crores. The company receives orders from its commission agents
all over the country, but goods are dispatched directly to the customers. The documents
including transport bills are sent through the bank for collection. At the end of the 6th year, it
is found that documents covering the dispatch of goods worth Rs. 10 crores were still lying
with the banks not cleared by the customers even though the normal collection period of 15
days from the date of dispatch has expired. Should revenue be recognized in the above case?
(CAP Dec. 2016 5a-5 Marks)
Answer
According to NAS - 18, revenue from the sale of goods shall be recognized when
• the seller of goods has transferred to the buyer the significant risks and rewards
of ownership of the goods;
• the seller retains neither continuing managerial involvement to the degree
usually associated with ownership nor effective control over the goods sold;
• the amount of revenue can be measured reliably.

© The Institute of Chartered Accountants of Nepal 35


CAP II Paper 1: Advanced Accounting

• It is probable that the economic benefits associated with the transaction will
flow to the entity; and
• The costs incurred or to be incurred in respect of the transaction can be
measured reliably.
Though the transport bills were sent through bank for collection, the seller neither retains risk
and reward of ownership not has any managerial involvement. The fact that the amount is lying
unpaid after the expiry of normal credit period, does not necessarily implicate that amounts
will not be collected from the debtors. Hence the company is advised to recognize revenue
pertaining to these transactions. However, the fact that collection period has expired indicates
impairment of trade receivables and adequate impairment (provisions) shall be made in this
regard with respect to the debtors.

6. Nepa Roadways has taken a transit insurance policy. Suddenly in the year 2073-2074 the
percentage of accident has gone up to 12% and the company wants to recognize insurance
claim as revenue in 2073-2074 in accordance with relevant Accounting Standard. Do you
agree? (CAP Dec. 2017 Q5b-5 Marks)

Answer
NAS 18 on “Revenue” defines revenue as “gross inflow of economic benefits during the period
arising in the course of the ordinary activities of an entity when those inflows result in increases
in equity, other than increases relating to contributions from equity participants”
To recognize revenue NAS 18 requires that revenue arises from ordinary activities and that it
can be measured reliably and it is probable that the economic benefits associated with the
transaction will flow to the entity.
In the given case, Nepa Roadways wants to recognize insurance claim because it has increased
over the previous year. However, the claim is not to be in the course of ordinary activity of the
company and therefore Nepa Roadways is not advised to recognize the Insurance claim as
revenue. It may however be required to separately disclose the amount of income generated
from transit insurance claim based on materiality.

7. How do you recognize income arising from following events:


i) Instalment sales, under which the consideration is receivable in instalments.
ii) Admission fees.
iii) Installation fees.
(Inter. Dec. 2009 Q2b-3 Marks)
Answer
Instalment sales, under which the consideration is receivable in instalments
The amount of revenue that would be recognized on outright sales transaction is recorded as
revenue as per NAS 17 Leases. The balance of amount received against recognized sales is
recognized as finance income on a systematic basis over the lease term.

Admission fees

© The Institute of Chartered Accountants of Nepal 36


CAP II Paper 1: Advanced Accounting

Revenue from artistic performances, banquets and other special events is recognized when the
event takes place. When a subscription to a number of events is sold, the fee is allocated to
each event on a basis which reflects the extent to which services are performed at each event.

Installation fees
Installation fees are recognized as revenue by reference to the stage of completion of the
installation, unless they are incidental to the sale of a product in which case, they are recognized
when revenue from the sale of goods is recognized.

8. On 25th September, 2010, Planet Advertising Limited obtained advertisement rights for World
Cup Hockey Tournament to be held in December, 2010 and January 2011 for Rs. 520 Lakhs.
They furnish the following information:
i) The company obtained the advertisements for 70% of available time for Rs. 700 Lakhs by
30th September, 2010.
ii) For the balance time they got bookings in October, 2010 Rs. 240 Lakhs.
iii) All the advertisers paid the full amount at the time of booking the advertisements.
iv) 40% of the advertisements appeared before the public in December 2010 and balance 60%
appeared in the month of January 2011.
v) Planet Advertising Limited follows Accounting year January-December.
You are required to calculate the amount of profit / loss to be recognized for the year 2010 and
2011 as per Nepal Accounting Standard- Revenue. (Inter Dec. 2011 Q5a-5 Marks)

Answer
As per paragraph NAS – 18 'Revenue' when the outcome of a transaction involving the
rendering of services can be estimated reliably, revenue associated with the transaction should
be recognized by reference to the stage of completion.

In the given problem, 40% of the advertisement appeared before the public in December, 2010
and balance 60% appeared in January, 2011.
Total profit will be computed as follows:
Rs. in Lakhs
th
Advertisement for 70% of available time obtained by 30 Sep. 2010 700
Advertisement for 30% of available time obtained by Oct. 2010 240
Total 940
Less: Cost of advertising rights 520
Profit 420
The profit amounting to Rs. 420 lakhs should be apportioned in the ratio of 40 : 60 for
the year 2010 and 2011. Thus, the company should recognize Rs. 168 lakhs (40%) in
the year 2010 and rest Rs. 252 lakhs (60%) in the year 2011.

9. What are the conditions that have to be satisfied for recognition of revenue from sale of
goods? (CAP Jun. 2011 Q5b- 5 Marks; Inter Jun. 2003 Q9f- 3 Marks)
Answer

© The Institute of Chartered Accountants of Nepal 37


CAP II Paper 1: Advanced Accounting

As per NAS 18, Revenue from Sale of goods shall be recognized when all the following
conditions have been satisfied:
i. The entity has transferred to the buyer the significant risks and rewards of ownership of
goods;
ii. The entity retains neither continuing managerial involvement to the degree usually
associated with ownership nor effective control over the goods sold;
iii. The amount of revenue can be measured reliably;
iv. It is probable that the economic benefits associated with the transaction will flow to the
entity; and
v. The cost incurred or to be incurred in respect of the transaction can be measured reliably.

10. Rajesh Brothers sells their goods to their approved customers on “Sale or Return” basis treating
all such transactions as actual sales at the time of dispatch. They sent on 15th December goods
costing Rs. 10,000 to Rama Stores at 20% profit on sale and passed the goods through Sales
Day Book. How would you adjust the transaction on 31st December, if Rama Store’s consent
is pending? (CAP Jun. 2012 Q4a– 5 Marks)

Answer
Rs.
Cost of goods sent to customers 10,000
Add: 20% Profit on sales i.e. 25% on cost. 2,500
Selling price of goods sent to customer 12,500

As stated in the question, Rajesh Brothers sells their goods to approved customers on "Sale or
Return" basis treating all such transactions as actual sales at the time of dispatch. So following
journal entry must have been passed at the time the goods were sent to Rama Stores:
Dr Cr
Date Particulars
Rs. Rs.
th
15 December Rama Stores Dr 12,500
To Sales A/c 12,500
st
Since Rama Store's consent has not been received till 31 December, following adjustment
entry needs to be passed on 31st December:
Dr Cr
Date Particulars
Rs. Rs.
st
31 December (a) For cancellation of sale:
Sales A/c Dr 12,500
To Rama Stores 12,500
(Being reversal of sales for goods lying with
Rama Stores earlier treated as sales)
(b) For recording stock with customer at
cost:
Stock with customers Dr 10,000
To Inventory A/c 10,000

© The Institute of Chartered Accountants of Nepal 38


CAP II Paper 1: Advanced Accounting

(Being goods with Rama Stores recorded as


stock with customer)

11. The following information are related with Purple Nepal Ltd.
i) Goods of Rs. 60,000 were sold on 20-3-2074 but at the request of the buyer these were
delivered on 10-4-2074.
ii) On 15-3-2074 goods of Rs. 1,50,000 were sent on consignment basis of which 20% of the
goods unsold are lying with the consignee as on 31-3-2074.
iii) Rs. 1,20,000 worth of goods were sold on approval basis on 1-12-2073. The period of
approval was 3 months after which they were considered sold. Buyer sent approval for
75% goods up to 31-1-2074 and no approval or disapproval received for the remaining
goods till 31-3-2074.
iv) Apart from the above, the company has made cash sales of Rs. 7,80,000 (gross). Trade
discount of 5% was allowed on the cash sales.
You are required to advise the accountant of Purple Nepal Ltd. with valid reasons, the amount
to be recognized as revenue in above cases in the context of NAS -18 and also determine the
total revenue to be recognized for the year ending 31-3-2074.
(CAP Jun. 2018 Q5a-5 Marks)
Answer
As per NAS 18, Revenue from Sale of goods shall be recognized when all the following
conditions have been satisfied:
i. The entity has transferred to the buyer the significant risks and rewards of ownership of
goods;
ii. The entity retains neither continuing managerial involvement to the degree usually
associated with ownership nor effective control over the goods sold;
iii. The amount of revenue can be measured reliably;
iv. It is probable that the economic benefits associated with the transaction will flow to the
entity; and
v. The cost incurred or to be incurred in respect of the transaction can be measured reliably.

In case (i):
The sale is complete but delivery has been postponed at buyer's request. Purple Nepal Ltd.
should recognize the entire sale of Rs. 60,000 for the year ended 31st Ashadh, 2074.

In case (ii):
20% goods lying unsold with consignee should be treated as closing inventory and sales should
be recognized for Rs. 1,20,000 (80% of Rs. 1.50,000). In case of consignment sale revenue
should not be recognized until the goods are sold to a third party.

In case (iii):
In case of goods sold on approval basis, revenue should not be recognized until the goods have
been formally accepted by the buyer or the buyer has done an act adopting the transaction or
the time period for rejection has elapsed or where no time has been fixed, a reasonable time

© The Institute of Chartered Accountants of Nepal 39


CAP II Paper 1: Advanced Accounting

has elapsed. Therefore in case (iii) revenue should be recognized for the total sales amounting
Rs.1,20,000 as the time period for rejecting the goods had expired.

In case (iv):
Trade discounts given should be deducted in determining revenue. Thus Rs. 39,000 should be
deducted from the amount of turnover of Rs.7,80,000 for the purpose of recognition of revenue.
Thus, revenue should be Rs. 7,41,000.

Thus total revenue amounting Rs. 10,41,000 (60,000+1,20,000+1,20,000+7,41,000) will be


recognized for the year ended 31st Ashadh, 2074 in the books of Purple Nepal Ltd.

12. On 1st Shrawan 2071, Dillibazar Furniture sold some furniture to his regular customer M/S
Zebra for Rs. 400,000 with three years interest-free credit. Dillibazar Furniture’s cost of capital
is 8%. You are required to advise how much revenue should be recognized in fiscal year ending
on Ashadh 2072, 2073 and 2074 in accordance with NAS 18?
(CAP Jun. 2019 Q5c-5 Marks)
Answer
On Shrawan 2071, sales revenue will be Rs. 317,532.90 i.e. (400,000×1/(1.08)3 . Also,
Dillibazar Furniture will recognize interest income of Rs. 25,402.63 i.e. 317,532.90 ×8% on
Ashadh 2072.

In 2073 Ashadh, interest income will be Rs. 27,434.84 i.e. (317,532.90+25,402.63)×8%.

In 2074 Ashadh, remaining amount out of Rs. 400,000 i.e. Rs. 29,629.63 will be
recognized as interest income.

1.11. NAS 20 Accounting for Government Grant & Disclosure of Government


Assistance

1. X Ltd. received a grant of Rs. 2 crores from the Government for the purpose of installation of
special machinery during F/Y 2062/063. The cost of Machinery was Rs. 20 crores and it had a
useful life of 9 years. During F/Y 2066/067, the grant has become refundable due to non-
fulfillment of certain conditions attached to it. Assuming the entire grant was deducted from
the cost of machinery in the year of acquisition, state with reasons, the accounting treatment to
be followed in the year 2066/067. (CAP Dec. 2010 Q5b-5 Marks)

Answer
As per NAS 20, the amount refundable in respect of a government grant related to assets is
recorded by increasing the book value of the asset by the refund amount. Also, the cumulative
additional depreciation that would have been recognized in profit or loss to date in the absence
of the grant shall be recognized immediately in profit or loss. The accounting treatment is given
as Journal Entry as follows:

© The Institute of Chartered Accountants of Nepal 40


CAP II Paper 1: Advanced Accounting

Machinery Dr. 2
Depreciation Dr. 0.89
Accumulated Depreciation 0.89
Grant Repayable 2

Working:
Grant Grant Not
Particulars
Received Received
Cost of Machinery 20 20
Less: Grant 2 -
Depreciable Amount 18 20
Annual Depreciation 18/9 = 2 20/9 = 2.22
Depreciation charged till 2065/66 8 8.89
Book Value 10 11.11

Note: The annual depreciation from FY 2065/66 onwards will be 2.22 crores.

2. GP Ltd. received a grant of Rs. 40 lakhs from the Government for the purpose of special
machinery during fiscal year 2064-065. The cost of machinery was Rs. 400 lakhs and had a
useful life of 10 years. During fiscal year 2068-069, the grant has become refundable due to
non-fulfillment of certain conditions attached to it. Assuming the entire grant was deducted
from the cost of machinery in the year of acquisition; state with reason, the accounting
treatment to be followed in the fiscal year 2068-069. (CAP Dec. 2012 4c-5 Marks)

Answer:
As per NAS 20, the amount refundable in respect of a government grant related to assets is
recorded by increasing the book value of the asset by the refund amount. Also, the cumulative
additional depreciation that would have been recognized in profit or loss to date in the absence
of the grant shall be recognized immediately in profit or loss. The accounting treatment is given
as Journal Entry as follows:

The computation of depreciation and expense can be given as:


Cost of Machinery Rs. 400 lacs
Less: Grant Received Rs. 40 lacs
Cost of Machinery Rs. 360 lacs
Useful life of Machinery 10 years
Depreciation per year as per straight line method (Assuming residual value to be zero Rs. 360
lacs/10 = Rs. 36 lacs
Total depreciation for 4 years (2064/65 to 2067/68) = 36×4 =Rs. 144 lacs

Cost of machinery without government grants Rs. 400 lacs


Useful life of Machinery 10 years
Depreciation that would have been recognized in absence of grant Rs. 400/10 = 40 Lacs

© The Institute of Chartered Accountants of Nepal 41


CAP II Paper 1: Advanced Accounting

Cumulative total depreciation that would have been recognized in absence of grant=40×4 =160
lacs
Expenses to be recognized in FY 2068/069 = 160 lacs – 144 lacs=16 lacs

Depreciation to be charge for another 6 years = (400 – 160)/6


Amount of depreciation as an expense for 2068/069 = 40 + 16 = 56 lacs
Depreciation for remaining 5 years = 40 lacs per year

3. A Ltd. received a grant of Rs 2 crores from the Government for purchase of a special purpose
machinery during 2062-063. The cost of machinery was Rs 38 crores and had a useful life of
9 years. During the current year 2065-066 the grant has become refundable due to non-
fulfillment of certain conditions attached to it. Assuming that the entire amount of grant was
deducted from the cost of machinery in the year of acquisition, state with reasons, the
accounting treatment to be followed in the rear 2065-066
(CAP Jun. 2011 Q5c- 5 Marks)
Answer
Value of asset (Without grant) = Rs.38 crore
Depreciation (Without Grant) = Rs.38 crore/9years
= 4.22 crore per year
As per NAS, the additional depreciation needs to be charge of expenses. Hence,
Already charge Depreciation
Book value (before refund of grant) = Rs 20 crore
(assuming SLM method and no residual value, annual depreciation will be Rs.(38-2)/9 = Rs. 4
crore, depreciation for 4 years will be Rs.16 crore
Additional Depreciation for 4 years = (4.22×4)-(4×4)
(change as expenses for current year = 16.88-16=0.88 crore
Revises book Value = 38-16.88 =21.12 crore, Depreciation for next 5 year =4.22

4. Discuss accounting treatment of repayment of Government Grants as per NAS 20.


(CAP Jun. 2013 Q5b–5 Marks)
Answer
A government grant that becomes repayable shall be accounted for as a change in accounting
estimate. Repayment of a grant related:
(a) to income shall be applied first against any unamortized deferred credit recognized in
respect of the grant. To the extent that the repayment exceeds any such deferred credit, or
when no deferred credit exists, the repayment shall be recognized immediately in profit or
loss.
(b) (b)to an asset shall be recognized by increasing the carrying amount of the asset or
reducing the deferred income balance by the amount repayable. The cumulative
additional depreciation that would have been recognized in profit or loss to date in the
absence of the grant shall be recognized immediately in profit or loss.

5. Presentation of ‘Government Grant related to Assets’ in Financial Statement.


(CAP Jun. 2016 Q6a - 3 Marks)

© The Institute of Chartered Accountants of Nepal 42


CAP II Paper 1: Advanced Accounting

Answer
Government grants related to assets, including non-monetary grants at fair value, shall be
presented in the statement of financial position either by setting up the grant as deferred income
or by deducting the grant in arriving at the carrying amount of the asset.
One method sets up the grant as deferred income which is recognized as income on
a systematic and rational basis over the useful life of the asset.
The other method deducts the grant in arriving at the carrying amount of the asset.
The grant is recognized as income over the life of a depreciable asset by way of a
reduced depreciation charge.

1.12. NAS 21 The Effects of Changes in Foreign Exchange Rates

1. Carlin Co. has head office at New York (USA) and branch at Kathmandu, Nepal. Nepal
branch is an integral foreign operation of Carlin & Co. Nepal branch furnishes you with its
trial balance as on 31st March, 2006 and the additional information given there after:
Debit Credit
Particulars NRs NRs
Stock on 1st April,2005 300,000.00 -
Purchases and Sales 800,000.00 1,200,000.00
Sundry Debtors and Creditors 400,000.00 300,000.00
Bills of exchange 120,000.00 240,000.00
Wages and Salaries 560,000.00 -
Rent, rates and taxes 360,000.00 -
Sundry charges 160,000.00 -
Computers 240,000.00 -
Bank balance 420,000.00 -
New York office a/c - 1,620,000.00
Total 3,360,000.00 3,360,000.00

Additional Information
a) Computers were acquired from a remittance of US $6,000 received from New York
head office and paid to the supplier. Depreciate computers at 60% for the year.
b) Unsold stock of Nepal branch was worth NRs. 420,000 on 31st March ,2006.
c) The rates of exchange may be taken as follows:
# on 1.4.2005 @ Rs 40 per US$
# on 31.3.2006 @ Rs42 per US $
# Average exchange rate for the year @ Rs 41 per US $
#conversion in $ shall be made up to two decimal accuracy

You are asked to prepare in US dollars the revenue statement for the year ended 31st March,
2006 and the Statement of Financial Position as on that date of Nepal branch as would appear
in the books of New York head office of Carlin & Co

© The Institute of Chartered Accountants of Nepal 43


CAP II Paper 1: Advanced Accounting

You are informed that Nepal branch account showed a debit balance of US $ 39,609.18 on
31.3.2006 in New York books and there were no items pending reconciliation.
(CAP Jun. 2009 Q2- 16 Marks)
Answer
CARLIN & CO. Ltd.
Statement of Profit or Loss
Of Kathmandu, Nepal Branch
For the year ended 31st March, 2006

Particulars Amount (NRs.) Amount (USD)


Sales 1,200,000.00 29268.29
COGS
OS 300,000.00 7500
Purchases 800,000.00 19512.2
Closing Stock -420,000 680,000.00 -10000 17,012.20
Wages and Salaries 560,000.00 13658.54
Rent, rates and taxes 360,000.00 8780.49
Sundry charges 160,000.00 3902.44
Depreciation 144000 3512.2
Net Profit/Loss -704,000.00 -17,597.58

Statement of Other Comprehensive Income


Particulars Amount
Net Profit as per SOPL (17,597.58)
Translation Gain 835.66
Total Comprehensive Income (16,761.92)

CARLIN & CO. Ltd.


Kathmandu, Nepal Branch
Statement of Financial Position as on 31st March 2006
Assets Amount (NRs.) Amount (USD)
Bank 420,000.00 10,000
Sundry Debtors 400,000.00 9,523.81
Bills Receivable 120,000.00 2,857.14
Inventory 420,000.00 10,000
Computers 240,000.00
Acc. Dep -144000 96,000.00
Total 1,456,000.00 34,666.66

Equity & Liabilities


Head Office 1,620,000.00 38,571.43
Accumulated Loss -704,000.00 -17,597.58

© The Institute of Chartered Accountants of Nepal 44


CAP II Paper 1: Advanced Accounting

Translation Reserve 0.00 835.66 -16,761.92


Sundry Creditors 300,000.00 7,142.86
Bills Payable 240,000.00 5,714.29
Total 1,456,000.00 34,666.66

Notes: As per the requirements of NAS 21, foreign operation has been translated as follows:
- Items of P/L translated on average rate (except closing stock which is translated on
closing rate)
- Items of SFP translated on closing rate
- Equity has been translated using closing rate. As NAS 21 does not clearly state
requirement of translation of equity, translation at closing rate is the best alternative as
all assets/liabilities represented by equity are translated at closing rate.
- Translation gain has been recognized directly in equity under Translation Reserve. The
transfer should be made through Statement of Other Comprehensive Income.

2. Following Figure relates to a Company:


Exchange Rate per $
Goods purchased on 1.1.2065 of $ 10,000 Rs. 75
Exchange rate as on 31.03.2065 Rs. 74
Date of actual payment 07.07.2065 Rs. 73
Ascertain the loss/gain for financial year 2064-65 and 2065-66. Also give the treatment as per
NAS-21.
(Inter Jun. 2010 Q5b- 5 Marks)
Answers:
As per NAS 11 on 'The Effects of Changes in Foreign Exchange Rates' all foreign currency
transactions should be recorded applying the exchange rate on the date of transactions. Thus,
goods purchased on 1.1.2065 and corresponding creditor would be recorded at Rs.750000
(i.e.$10,000* Rs. 75)

According to the standard, at the reporting date, all monetary items should be reported using
the closing rate. Thus, creditor US $ 10000 on 31.03.2065 will be reported at Rs.740,000 (i.e.$
10,000*Rs.74) and exchange profit of Rs.10000(i.e.750,000 – 740,000) should be recognized
in Profit or Loss for the year 2064-65.

On 07.07.2065, creditor of $10,000 is paid at the rate of Rs. 73. Exchange difference on
settlement of the account should also be recognized in Profit or Loss Account .Therefore,
Rs.10000 (i.e.740000- 730000) is recognized in Profit or Loss for the year 2065/66

3. Megha Ltd., Kathmandu, have a branch in Sydney, Australia. At the end of 31st March, 2011,
the following ledger balances have been extracted from the books of the Kathmandu Office
and the Sydney Office:

© The Institute of Chartered Accountants of Nepal 45


CAP II Paper 1: Advanced Accounting

Kathmandu Sydney
(Rs. in thousands) ( Australian dollars in
thousands)
Debit Credit Debit Credit
Share Capital 2000
Reserves and Surplus 1000
Land 500
Buildings (Cost) 1000
Buildings Acc. Depreciation 200
Plant & Machinery ( Cost) 2500 200
Plant & Machinery Acc. Depreciation 600 130
Debtors and Creditors 280 200 60 30
Stock (1.4.2010) 100 20
Branch Stock Reserve 4
Cash & Bank Balances 10 10
Purchase and Sales 240 520 20 123
Goods Sent to Branch 100 5
Managing Director's Salary 30
Wages & Salaries 75 45
Rent 12
Office Expenses 25 18
Commission Receipts 256 100
Branch / H.O. Current Account 120 7
4880 4880 390 390

The following information is also available:


a) Stock as at 31.03.2011 ;
Kathmandu Rs. 150000
Sydney A $ 3,125
b) Head office always sent goods to the branch at cost plus 25%.
c) Provision is to be made for doubtful debts at 5%.
d) Depreciation is to be provided on building at 10% and on plant and machinery at 20% on
written down values.
e) The managing director is entitled to 2% commission on net profits before provision for
doubtful debts.
f) Income- tax is to provide at 47.5%.

You are required:


To convert the branch trial balance into rupees, use the following rates of exchange:
Opening rate A $ = Rs. 20
Closing rate A $ = Rs. 24
Average rate A $ = Rs. 22
For fixed assets A $ = Rs. 18

© The Institute of Chartered Accountants of Nepal 46


CAP II Paper 1: Advanced Accounting

To prepare the Statement of Profit or Loss for the year ended 31st March, 2011 showing to the
extent possible head office result and branch results separately. (Balance Sheet not required.)
(Inter Jun. 2011 Q2- 15 Marks)
Answer
Megha Ltd
Trial Balance of Sydney Branch
Sydney Sydney
(Australian dollars Nepalese Rupees
Particulars Rate
in thousands) in Thousand
Debit Credit Debit Credit
Plant & Machinery ( Cost) 200 24 4800 0
Cost
Acc. Depreciation 144 24 0 3456
Debtors and Creditors 57 30 24 1368 720
Cash & Bank Balances 10 24 240 0
Sales 123 22 0 2706
COGS 0 0
Stock (1.4.2010) 20 20 400 0
Purchases 20 22 440 0
Goods Sent to Branch 5 100 0
CS -3.125 24 -75 0
Wages & Salaries 45 22 990 0
Rent 12 22 264 0
Office Expenses 18 22 396 0
Commission Receipts 100 22 0 2200
Branch / H.O. Current Account 7 24 0 168
Depreciation (for the year) 14 22 308 0
Provision for doubtful debts 3 22 66 0
Translation Gain 122
Closing Inventory 3.125 24 75 0
Total 404 404 9372 9372

Statement of Profit or Loss


For the Year ended 31.03.2011
Branch
Particulars HO Total
$ Rate Rs
Sales (plus branch stock res) 524 123 22 2,706 3,230
Goods Sent to Branch 100 - 100
COGS
OS 100 20 20 400 500
Purchases 240 20 22 440 680
Goods from HO - 5 - 100 100
CS (150) (3) 24 (75) (225)

© The Institute of Chartered Accountants of Nepal 47


CAP II Paper 1: Advanced Accounting

Gross Profit 434 81 - 1,841 2,275


Commission 256 100 22 2,200 2,456
Managing Director's Salary (30) - 22 - (30)
Wages & Salaries (75) (45) 22 (990) (1,065)
Rent (12) 22 (264) (264)
Office Expenses (25) (18) 22 (396) (421)
Provision for debtors (14) (3) 22 (66) (80)
Depreciation (460) (14) 22 (308) (768)
86 89 2,017 2,103
MD’s Commission 43
Profit Before Tax 2,060
Current Tax Expense 978.5
Net Profit 1,081.5
Translation Gain 122
Total Comprehensive Income 1,203.5
* Note: Gain is not recognized in P/L but shown under separate component of equity through
transfer from statement of other comprehensive Income. Separate SOCI has not been prepared
as question does not have the requirement

Rate given for "Fixed Assets" is not relevant.


Working Notes:-

Computation of tax liability and MD’s bonus


Profit as per above 2,103
Less: Unrealized profit on stock (15)
Add: Provision for doubtful debts 80
Bonus Eligible Figure 2168
Bonus @2% 43
Profit Before tax 2,060

Calculation of Depreciation : H.O Branch


A. Building – cost 1000
Less: Depreciation Provision 200
800
Depreciation @ 10% 80
B. Plant & Machinery , cost 2500 3600
Less: Depreciation Provision 600 2340
1900 1260
Depreciation @ 20% 380 252
Total depreciation (A.+B) 460 252

Calculation of Branch Stock Reserve :


Closing Stock 75

© The Institute of Chartered Accountants of Nepal 48


CAP II Paper 1: Advanced Accounting

Reserve on closing stock (75×1/5) 15

4. A Ltd. Purchased non-current assets costing Rs. 850 Lakhs on 1.1.2069. This was financed by
foreign currency loan (U.S. Dollars) payable in three equal instalments. Exchange rates were
$1=Rs. 85 and Rs. 88 as on 1.1.2069 and 31.3.2069. First instalment was paid on 31.3.2069.
You are required to state, how these financial transactions would be accounted for?
(CAP Jun. 2013 Q4c –5 Marks)
Answer:
As per NAS-21: The Effects to Changes in Foreign Exchange Rates, exchange
differences arising on the settlement of monetary items or on reporting an enterprise
monetary items at rates different from those at which they were initially recorded during
the period, or reported in previous financial Statements, should be recognized as income
or expense in the period they arise. Thus, exchange differences arising on repayment of
liabilities incurred for the purpose of acquiring fixed assets are recognized as income or
expense.

Calculation of Exchange Difference:


Foreign Currency Loan = Rs 850 Lacs / Rs 85 = $10 Lacs
Exchange Difference = $ 10 Lacs X (88-85) = Rs 30 Lacs
Loss due to exchange difference amounting to Rs 30 Lacs should be charged to Profit &
Loss for the year ended 31.03.2069.

5. A Company purchased non-current costing Rs. 50,88 lakhs on 1st Shrawan 2069 and the same
was fully financed by foreign currency loan in U.S. Dollars, repayable in four equal annual
installments. Exchange rate at the time of purchase was 1 US Dollar Rs. 84.80. The first
installment was paid on 31 Ashadh 2070 when 1 US Dollar 90.80. The entire loss on exchange
was included in cost of goods sold of normal business operations. The Company provides
depreciation on their assets at 20% on WDV basis. Show the correct accounting treatment with
reference to relevant accounting standards. (CAP Jun. 2014 Q4c- 5 Marks)

Answer
As per NAS 21 ‘The Effects of changes in Foreign Exchange Rates’, foreign currency non-
monetary items which are carried in terms of historical cost denominated in a foreign currency
should be reported using the exchange rate at the date of the transaction and exchange
differences arising on the settlement of monetary items or on reporting an enterprise’s monetary
items at rates different from those at which they were initially recorded during the period, or
reported in previous financial statements, should be recognized as income or as expense in the
period in which they arise. Foreign currency loss will be computed as Rs. 360 lakhs {$60 lakhs
[5088 Lakhs/84.80] x (Rs. 90.80-Rs. 84.80)}. The entire loss on exchange difference of Rs.
360 lakhs should be recognized as an expense for the year ended 31st Ashadh 2070 and should
not be included in the cost of goods sold. Depreciation on assets amounting Rs. 1,017.60 lakhs
(20% of 5088 lakhs) should be provided for in the financial statements for the year ended 31st
Ashadh 2070.

© The Institute of Chartered Accountants of Nepal 49


CAP II Paper 1: Advanced Accounting

6. Gorkha Company Ltd. imported raw materials worth USD 9,000 on 24th Jestha, 2074, when
the exchange rate was Rs.104 per USD. The transaction was recorded in the books at the above
mentioned rate. The payment of the transaction was made on 10th Shrawan, 2074, when the
exchange rate was Rs.108 per USD. At the year-end 31st Ashadh, 2074, the rate of exchange
was Rs.109 per USD.

The Account Officer of the company passed an entry on 31st Ashadh, 2074 adjusting the cost
of the raw material consumed for the difference between Rs.108 and Rs.104 per USD. Discuss
whether this treatment is justified as per the provision of NAS-21.
(CAP Jun. 2018 Q5c-5 Marks)
Answer
As per NAS-21, “The effects of changes in foreign exchange rates”,
(i) Initial recognition of a foreign currency transaction is done in the functional currency, by
applying the spot exchange rate between the functional currency and foreign currency at
the date of the transaction.
(ii) At the end of each reporting period, foreign currency monetary items shall be translated
using the closing rate.
(iii)Exchange difference arising on settlement of monetary items or on translating monetary
items at rates different from those at which they were translated on initial recognition
during the period or in previous financial statement shall be recognized in profit and loss
in the period in which they arise.
In the given case, at the date of transaction the raw material purchased and its creditors will be
recorded at USD 9,000×Rs.104= Rs. 936, 000.
At the end of the reporting period, such transaction is reported at closing rate of exchange,
hence it will be valued at the closing rate i.e. Rs.109 per USD (USD 9,000×Rs.109=Rs.981,
000)
The difference of exchange rate between the closing date and transaction date is Rs.5 per USD
(i.e. Rs. 109-Rs.104). The difference of Rs.45,000 (USD 9,000×5) will be shown as an
exchange loss in the profit and loss for the year ended 31st Ashadh, 2074 and will not be
adjusted against the cost of raw materials.

At the settlement date, the company would recognize or provide in the profit and loss an
exchange gain of Rs. 9,000 (i.e. at the rate Rs. 1 per USD, the difference of exchange rate
between the reporting date and the date of settlement, i.e. Rs.109 and Rs.108 per USD).
Hence the accounting treatment adopted by the Account officer is not as per NAS 21.

7. Briefly explain the objective and scope of Accounting for The effects of changes in Foreign
Exchange rates – NAS 21. (Inter Dec. 2001, Q 6b-4 Marks)
Answer:
The objective & Scope of NAS 21 has been provided in the standard as follows:
a. Objective
An entity may carry on foreign activities in two ways. It may have transactions in foreign
currencies or it may have foreign operations. In addition, an entity may present its financial

© The Institute of Chartered Accountants of Nepal 50


CAP II Paper 1: Advanced Accounting

statements in a foreign currency. The objective of this Standard is to prescribe how to


include foreign currency transactions and foreign operations in the financial statements of
an entity and how to translate financial statements into a presentation currency. The
principal issues are which exchange rate(s) to use and how to report the effects of changes
in exchange rates in the financial statements
b. Scope
NAS 21 shall be applied:
i. in accounting for transactions and balances in foreign currencies, except for those
derivative transactions and balances that are within the scope of IFRS 9 Financial
Instruments;
ii. in translating the results and financial position of foreign
iii. operations that are included in the financial statements of the entity by consolidation or
the equity method; and
iv. in translating an entity’s results and financial position into a presentation currency.

© The Institute of Chartered Accountants of Nepal 51


CAP II Paper 1: Advanced Accounting

1.13. NAS 37 Provisions, Contingent Liabilities & Contingent Assets

1. Accounting treatment of contingency gains. (CAP Dec. 2009, 6a-2.5 Marks)


Answer
Contingency gains are not recognized in financial statement since their recognition may result
in the recognition of revenue which may never be realized. However, when the realization of
a gain is virtually certain, then such a gain is not a contingency and accounting for the same
is appropriate.

2. Recognition of contingent assets and liabilities


(CAP Dec. 2015 Q6d; CAP Jun. 2013 Q6d– 2.Marks)
An entity shall not recognize a contingent liability.
1. A contingent liability is disclosed, unless the possibility of an outflow of resources
embodying economic benefits is remote.
2. Where an entity is jointly and severally liable for an obligation, the part of the
obligation that is expected to be met by other parties is treated as a contingent
liability. The entity recognizes a provision for the part of the obligation for which an
outflow of resources embodying economic benefits is probable, except in the
extremely rare circumstances where no reliable estimate can be made.
3. Contingent liabilities may develop in a way not initially expected. Therefore, they
are assessed continually to determine whether an outflow of resources embodying
economic benefits has become probable. If it becomes probable that an outflow of
future economic benefits will be required for an item previously dealt with as a
contingent liability, a provision is recognized in the financial statements of the period
in which the change in probability occurs (except in the extremely rare circumstances
where no reliable estimate can be made).
An entity shall not recognize a contingent asset.
1. Contingent assets usually arise from unplanned or other unexpected events that give
rise to the possibility of an inflow of economic benefits to the entity. An example is
a claim that an entity is pursuing through legal processes, where the outcome is
uncertain.
2. Contingent assets are not recognized in financial statements since this may result in
the recognition of income that may never be realized. However, when the realization
of income is virtually certain, then the related asset is not a contingent asset and its
recognition is appropriate.
3. A contingent asset is disclosed, where an inflow of economic benefits is probable.
4. Contingent assets are assessed continually to ensure that developments are
appropriately reflected in the financial statements. If it has become virtually certain
that an inflow of economic benefits will arise, the asset and the related income are
recognized in the financial statements of the period in which the change occurs. If
an inflow of economic benefits has become probable, an entity discloses the
contingent asset (see paragraph 89).

© The Institute of Chartered Accountants of Nepal 52


CAP II Paper 1: Advanced Accounting

3. Difference between provisions and contingent liabilities. (Jun. 2006, Q 7d- 3 Marks)
Answer:
Provision is a liability of uncertain timing and amount. A provision shall be recognized when:
(i) An entity has a present obligation (legal or constructive) as a result of a past event
(ii) It is probable that any outflow of resources embodying economic benefits will be
required to settle the obligation, and
(iii) A reliable estimate can be made of the amount of obligation

A contingent liability is:


(i) A possible obligation that arises from the past events and whose existence will be
conformed only by the occurrence of or non-occurrence of one or more uncertain future
events not wholly within the control of the entity, or
(ii) A present obligation that arises from past events but is not recognized because:
a. It is not probable that an outflow of resources embodying economic benefits will be
required to settle the obligation; or
b. The amount of the obligation cannot be measured with sufficient reliability.

4. Difference between Provisions and other liabilities. (CAP Jun. 2014 Q6c- 2.5 Marks)
Answer
Provision can be distinguished from other liabilities such as trade payable and accruals because
there is uncertainty about the timing or amount of future expenditure required in settlement.
By contrast:
Trade payables are liabilities to pay for goods or services that have been received or supplied
and have been invoiced or formally agreed with the supplier; and
Accruals are liabilities to pay for goods or services that have been received or supplied but not
have been paid, invoiced or formally agreed with the supplier, including amounts due to
employees. Although it is sometimes necessary to estimate the amount or timing of accruals,
the uncertainty is generally much less than for provision.
Accruals are often reported as part of trade and other payables, whereas provisions are reported
separately.

5. Alpha Ltd. has entered into a sale contract of Rs. 7 crores with Gamma Ltd. during 2072-73
financial years. The profit on this transaction is Rs. 1 crore. The delivery of goods to take place
during the first month of 2073-74 financial years. In case of failure of Alpha Ltd. to deliver
within the schedule, a compensation of Rs. 2 crores is to be paid to Gamma Ltd. Alpha Ltd.
planned to manufacture the goods during the last month of 2072-73 financial year. As on the
reporting date (31.3.2073), the goods were not manufactured and it was unlikely that Alpha
Ltd. will be in a position to meet the contractual obligation. You are required to advise Alpha
Ltd. on requirement of provision for contingency in the financial statements for the year ended
31st Ashadh, 2073, in line with provisions of Accounting Standards?
(CAP Jun. 2017 Q5c-5 Marks)
Answer
NAS 37 “Provisions, Contingent Liabilities and Contingent Assets” provides that a
provision shall be recognized when:

© The Institute of Chartered Accountants of Nepal 53


CAP II Paper 1: Advanced Accounting

(a) an entity has a present obligation (legal or constructive) as a result of a past event;
(b) it is probable that an outflow of resources embodying economic benefits will be
required to settle the obligation; and
(c) a reliable estimate can be made of the amount of the obligation.

If these conditions are not met, no provision shall be recognized. when an enterprise has a
present obligation, as a result of past events, that probably requires an outflow of resources
and a reliable estimate can be made of the amount of obligation, a provision should be
recognized.

In the given case, Alpha Ltd. has present obligation to deliver the goods as a result of
contract entered with the customer and it is more likely than not that Alpha Ltd. will be
required to settle the obligation on account of penalties. Therefore, Alpha Ltd. should
provide for the provision amounting Rs. 2 crores as per NAS 37.

6. M/s Dalima Ltd. is in a dispute with the competitor company. The dispute is regarding the
alleged infringement of copyrights. The competitor has filed a suit in the court seeking damages
of Rs. 325 lakhs.
Directors are of the view that the claim can be successfully resisted by the company.
How the matter be dealt in the financial statements of the company in the light of NAS 37.
Explain in brief giving reasons for your answer. (CAP Jun. 2018 Q5b-5 Marks)

Answer
As per NAS 37 "Provisions, Contingent liabilities and Contingent assets" a provision should
be recognized when
a) an entity has a present obligation as a result of a past event.
b) it is probable that the outflow of resources embodying economic benefits will be required
to settle the obligation and
c) a reliable estimate can be made of the amount of obligation.
If these conditions are not met, no provision should be recognized.

In the given situation, since the directors of the company are of the opinion that the claim can
be successfully resisted by the company, therefore there will be no outflow of resources. Hence
no provision is required. The company can disclose the same as contingent liability by way of
following note.

Litigation is in the process against the company relating to dispute with the competitor who
alleged that the company has infringed copyrights and seeking damages of Rs. 325 lakhs.
However the director are of the opinion that the claim can be successfully resisted by the
company.

7. Contingent Liability (Jun 07 Q 5e)


Answer
A possible obligation that arises from past events and the existence of which will be

© The Institute of Chartered Accountants of Nepal 54


CAP II Paper 1: Advanced Accounting

confirmed only by the occurrence or non-occurrence of one or more uncertain future events
not wholly within the control of the enterprise, or.
A present obligation that arises from past events but it is not recognized because
(a) It is not probable that an outflow of resources embodying economic benefits will be
required to settle the obligation, or
(b) A reliable estimate of the amount of the obligation cannot be made.
A contingent liability should not be recognized, but only disclosed. Also contingent liability
should be periodically reviewed by the enterprise.

8. Difference between Provisions and Reserves (Dec 07 Q 6c)

Answer:

A provision is a liability of uncertain timing or amount whereas he portion of earnings, receipts


or other surplus of an enterprise whether capital or revenues, apportioned by the management
for a general or specific purpose is called reserve. Reserves are primarily of two types: capital
reserves and revenue reserves. Provision is the charge against the profit whereas reserve is the
appropriation out of profits.

1.14. NAS 11: Construction Contracts


All questions have been included under Accounting for special transaction : Contract
Accounting

1.15. NAS 12: Income Taxes


1. Briefly describe the disclosure requirements for "Deferred Tax Assets" and "Deferred Tax
Liabilities" as per relevant Nepal Accounting Standard. (CAP Dec. 2009 2b ii-5 Marks)

Answer
An enterprise should offset deferred tax assets and deferred tax liabilities if:
➢ the enterprise has a legally enforceable right to set off assets against liabilities representing
current tax, and
➢ the deferred tax assets and the deferred tax liabilities relate to taxes on income levied by
the same governing taxation laws.
Deferred tax assets and liabilities should be distinguished from assets and liabilities
representing current tax for the period. Deferred tax assets and liabilities should be disclosed
under a separate heading in the balance sheet of the enterprise, separately from current assets
and current liabilities.
The break-up of deferred tax assets and deferred tax liabilities into major components of the
respective balances should be disclosed in the notes to accounts.
The nature of the evidence supporting the recognition of deferred tax assets should be
disclosed, if an enterprise has unabsorbed depreciation or carry forward of losses under tax
laws.

© The Institute of Chartered Accountants of Nepal 55


CAP II Paper 1: Advanced Accounting

2. Deferred Tax Liability (CAP Dec. 2013 6c-2.5 Marks; CAP Dec. 2015 Q6b)
Answer
According to NAS 12, A deferred tax liability shall be recognized for all taxable temporary
differences, unless the deferred tax liability arises from:
(a) the initial recognition of an asset or liability in a transaction which:
(i) is not a business combination; and
(ii) at the time of the transaction, affects neither accounting profit nor taxable profit (tax
loss).
It is inherent in the recognition of an asset that its carrying amount will be recovered in the
form of economic benefits that flow to the entity in future periods. When the carrying amount
of the asset exceeds its tax base, the amount of taxable economic benefits will exceed the
amount that will be allowed as a deduction for tax purposes. This difference is a taxable
temporary difference and the obligation to pay the resulting income taxes in future periods is a
deferred tax liability. As the entity recovers the carrying amount of the asset, the taxable
temporary difference will reverse and the entity will have taxable profit. This makes it probable
that economic benefits will flow from the entity in the form of tax payments.

3. Prudent prepares financial statement to 31st December each Year, on 1st January 20x0 the entity
purchased a non-current asset for Rs 1.6 million that had an anticipated useful life of four Years.
This asset qualified for immediate tax relief of 100% of the cost of that asset.

For the year ended 31st December 20x0; the draft account showed a profit before tax of Rs. 2
million. The directors anticipate that this level of profit will be maintained for the foreseeable
future.

Prudent pays tax at a rate of 30%. Apart from the difference caused by the purchase of no current
asset in 20x0, there are no other difference between accounting profit and taxable profit or the
tax base and carrying amount of net asset. Current year tax for the year is Rs 240,000 and for
the years 20x1 to 20x3 are Rs. 720,000 for each year.
You are required to compute the pre, and post-tax profits for the Prudent for each of the four
Years ending 31st December 20x0 to 20x3 assuming deferred tax is recognized.
(CAP Dec. 2014 5a-5 Marks; CAP Jun. 2015 Q5a- 5 Marks)
Answer
Particulars 20x0 (Rs.) 20x1 (Rs.) 20x2 (Rs.) 20x3 (Rs.)
Carrying Amount 1,200.00 800.00 400.00 -
Tax Base - - - -
Temporary difference 1,200.00 800.00 400.00 -
Closing deferred tax
Liability@30% 360.00 240.00 120.00 -
Opening deferred tax
Liability - (360.00) (240.00) (120.00)
Charge/(credit) to
income 360.00 (120.00) (120.00) (120.00)

© The Institute of Chartered Accountants of Nepal 56


CAP II Paper 1: Advanced Accounting

Income statement 20x0 (Rs.) 20x1 (Rs.) 20x2 (Rs.) 20x3 (Rs.)
Profit Before tax 2,000.00 2,000.00 2,000.00 2,000.00
Current Tax (240.00) (720.00) (720.00) (720.00)
Deferred Tax (360.00) 120.00 120.00 120.00
Profit after Tax 1,400.00 1,400.00 1,400.00 1,400.00

4. Timing differences and permanent differences as per NAS -12 (CAP Dec. 2015 6a)

Answer
Timing differences and Permanent differences as per NAS -12.
Temporary differences are differences between the carrying amount of an asset or liability in
the balance sheet and its tax base. Temporary differences may be either:
(a) Taxable temporary differences, which are temporary differences that will result in taxable
amounts in determining taxable profit (tax loss) of future periods when the carrying amount
of the asset or liability is recovered or settled; or
(b) Deductible temporary differences, which are temporary differences that will result in
amounts that are deductible in determining taxable profit (tax loss) of future periods
when the carrying amount of the asset or liability is recovered or settled.

The tax base of an asset or liability is the amount attributed to that asset or liability for
tax purposes.

Permanent differences are those which arise in one period and do not reverse
subsequently. For e.g. an income exempt from tax or an expense that is not allowable
as a deduction for tax purposes.
Timing differences are those which arise in one period and are capable of reversal in
one or more subsequent periods. For e.g. Depreciation, Provision for expenses etc.

5. Tax base of an asset (Inter Jun. 2010 Q6b- 5 Marks)

Answer
The tax base of an asset is the amount that will be deductible for tax purposes against any taxable
economic benefits that will flow to an entity when it recovers the carrying amount of the asset.
If those economic benefits will not be taxable, the tax base of the asset is equal to its carrying
amount. For example, a plant purchased for Rs.10,000 has depreciation of Rs.4,000 deducted in
past. It is intended to use in future for generating more revenue. Its tax base is Rs.6,000 which
is its carrying amount.

6. NAS 12 “Income Taxes” uses the concept of temporary differences. Temporary differences are
the difference between the carrying value of an asset and its tax base. The standard distinguishes
between “taxable temporary differences” and “deductible temporary differences”.
Required:

© The Institute of Chartered Accountants of Nepal 57


CAP II Paper 1: Advanced Accounting

Explain the distinction between taxable and deductible temporary differences.


(CAP Jun. 2010 5c- 5 Marks)
Answer
Temporary differences are differences between the carrying amount of an asset or liability in
the balance sheet and its tax base. The tax base of an asset or liability is the amount attributed
to that asset or liability for tax purposes.

Temporary differences may be either taxable temporary differences or deductible temporary


differences. Taxable temporary differences are temporary differences that will result in taxable
amounts in determining taxable profit (tax loss) of future periods when the carrying amount of
the asset or liability is recovered or settled. For instance if a tax authority taxes interest income
on a cash basis then any income accrued at the balance sheet date will not be included in the
current tax charge but will be taxed in the future when the cash is received.
Deductible temporary differences are the temporary differences that will result in amounts that
are deductible in determining taxable profit (tax loss) of future periods when the carrying
amount of the asset or liability is recovered or settled. For instance, if a tax authority taxes
interest expense on a cash basis then any expense accrued at the balance sheet date will not be
included in the current tax charge but will be deductible in the future when it is paid.

© The Institute of Chartered Accountants of Nepal 58


CAP II Paper 1: Advanced Accounting

1.16. NAS 23 Borrowing Costs

1. A company obtained term loan during the year ended 31st March, 2012 to an extent of Rs. 650
lakhs for modernization and development of its factory. Building worth Rs. 120 lakhs were
completed and plant and machinery worth Rs. 350 lakhs were installed by 31st March, 2012. A
sum of Rs. 70 lakhs has been advanced for assets the installation of which is expected in the
following year. Rs. 110 lakhs has been utilized for working capital requirements. Interest paid
on the loan of Rs. 650 lakhs during the fiscal year 2011-012 amounted to Rs. 58.50 lakhs. How
should the interest amount be treated in the account of the company? Give your comments for
the financial year ending on 31-03-2012 in the context of relevant NAS.

Answer
According to NAS 23 (“Borrowing Cost”), interest on borrowed funds, which is directly
related to the acquisition, construction or production of qualifying asset should be capitalized.
As factory building, Plant and Machinery are qualifying asses as per NAS 23 interest paid on
the loan being borrowing cost should be capitalized and included in the gross book value of
these assets. The interest pertaining to the money spent on the working capital should be
charged off as expense. In the given case, the interest amount of Rs. 58.50 lakhs shall be
treated as follows:

Proportion Rs. In
lakhs
To be added to the cost of building 120  58.50
10.80
650
To be added to the cost of Plant and Machinery 350  58.50
31.5
650
To be charged off as expense (Advance) 70  58.50
6.3
650
To be charged off as expenses 110  58.50
9.9
650
Total
58.50

2. Conditions to be satisfied to capitalize the borrowing costs


(CAP Dec. 2010 6d-2 Marks; CAP Dec. 2012 6d -2.5 Marks; CAP Dec. 2015 Q6 A)
Answer
Capitalization of borrowing costs as part of the cost of a qualifying asset should commence
only when all the following conditions are satisfied:

© The Institute of Chartered Accountants of Nepal 59


CAP II Paper 1: Advanced Accounting

1. The expenditure is being incurred for the Acquisition, construction or production of a


qualifying asset;
2. Borrowing costs are being incurred; and
3. Activities that are necessary to prepare the asset for its intended use or sale, (including
any technical or administrative work prior to the commencement of physical
construction but excluding such activities during which no production or development
take place) are in progress.

3. Discuss the treatment of capitalization of borrowing cost as per NAS 23


(CAP Dec. 2014 5b-5 Marks; CAP Jun. 2015 Q5b- 5 Marks)
Answer
Borrowing costs that are directly attributable to the acquisition, construction or production of
a qualifying asset shall be capitalized as part of the cost of asset. Borrowing cost are interest
and other cost incurred by an entity in connection with the borrowing of funds.
A qualifying asset is an asset that necessarily takes a substantial period of time to get ready for
its intended use or sale.
Commencement of capitalization
The capitalization of borrowing cost as part of qualifying asset shall commence when:
Expenditures for the asset are being incurred;
Borrowing costs are being incurred; and
Activities that are necessary to prepare the asset for its intended use or sale are in progress.
Suspension of capitalization
Capitalization of borrowing cost shall be suspended during extended periods in which active
development is interrupted.
Cessation of capitalization
Capitalization of borrowing cost shall cease when substantially all the activities necessary to
prepare qualifying asset for its intended use or sale are complete.

4. Define Borrowing cost & qualifying assets as per NAS 8? When borrowing cost is capitalized?
(CAP Jun. 2012 Q5c– 5 Marks)
Answer
Borrowing costs are interest and other cost incurred by an entity in connections with
borrowing of funds.
Qualifying asset is an asset that necessarily takes a substantial period of time to get ready
for its intended use or sale.
Borrowing cost is capitalized if
I. Expenditure for the asset are being incurred
II. Borrowing costs are being incurred ;and
III. Activities that are necessary to prepare the assets for its intended use or sale
are in progress.

5. M/s Biotic Company Limited obtained a loan for Rs. 14 crores on Shrawan 15, 2069 from
Nepal Bank Limited, to be utilized as under:
Construction of Factory building Rs. 2.5 crores

© The Institute of Chartered Accountants of Nepal 60


CAP II Paper 1: Advanced Accounting

Purchase of Plant and Equipment Rs. 2.0 crores


Working Capital Rs'. 1.5 crores
Advance for purchase of trucks Rs. 1.0 crore
In Ashadh 2070, construction of the factory building was completed and Plant and Equipments,
which was ready for its intended use, was installed. Delivery of trucks was received in the next
FY. Total interest of Rs. 9,100,000 was charged by the bank for the financial year ending
31.3.2070.
Show the treatment of interest under NAS 23 “Borrowing Cost” and also explain the nature of
assets. (CAP Jun. 2014 Q4b- 5 Marks)

Answer:
(a) Treatment of Interest (Borrowing cost) as per NAS 23 ‘Borrowing Cost’
Interest to be Interest to be
S.No. Particulars Nature
capitalized expensed
Rs. Rs.
(i) Construction of Factory Qualifying 91,00,000 x 25/70
Building (Refer Note 1) Assets =Rs.32,50,000
(ii) Purchase of Plant and 91,00,000 x 20/70
Equipment =Rs.26,00,000
Non
(iii) Working Capital 91,00,000 x 15/70
Qualifying -
=Rs.19,50,000
Assets
(iv) Advance for purchase of 91,00,000 x 10/70
Trucks =Rs.13,00,000
Total Rs. 32,50,000 Rs. 58,50,000

Notes:
1. It is assumed that construction of a factory building was completed on 31st Ashadh, 2070
2. It is assumed that the Plant and Equipment was ready for its intended use at the time of
its acquisition.
As per NAS 23, assets have been defined as ‘qualifying asset’ and non-qualifying assets.
(i) Qualifying assets is an asset that necessarily takes a substantial period of time to get
ready for its intended used or sale; whereas
(ii) Non-qualifying asset is an asset other than qualifying asset.
The Loan at one place is 14 crore but in detail is 7 crore. Hence, there are 2 options for
calculation of borrowing rate:
i) As given in Answer
91 𝑙𝑎𝑐𝑠
ii) = 6.5%
14 𝑐𝑟𝑜𝑟𝑒

6. The notes to accounts of X Ltd. for the year 2063/64 include the following:
Interest on bridge loan from banks and financial institutions and on debentures specifically
obtained for the company’s Fertilizer Project amounting to Rs. 18,080,000 has been
capitalized during the year, which includes approximately Rs. 17,033,465 capitalized in

© The Institute of Chartered Accountants of Nepal 61


CAP II Paper 1: Advanced Accounting

respect of the utilization of loan and debenture money for the said purpose.” Is the treatment
correct? Briefly comment.
(June 2007, Q 6iii)
Answer:

NAS 23 on “Borrowing Costs” states that borrowing cost incurred with respect to
acquisition, of qualifying asset should be included as part of the cost of the asset.

In the given case, X ltd capitalized the finance cost of Rs. 1,80,80,000 with the cost of the
Fertilizer Project out of which Approximately Rs. 17,033,465 has been capitalized. The
disclosure requirements of NAS 23 requires an entity to disclose:
a. the amount of borrowing costs capitalized during the period; and
b. the capitalization rate used to determine the amount of borrowing costs eligible for
capitalization.

Therefore, X ltd. is suggested to make disclosure in line with the requirements of NAS 23.
Also, the entity shall consider commencement, suspension & cessation requirements of the
standard in capitalizing the borrowing costs.
(due to lack of clarity in the question, we are not able to assess whether the capitalized
portion is eligible for capitalization or not)

© The Institute of Chartered Accountants of Nepal 62


CAP II Paper 1: Advanced Accounting

1.17. NAS 24: Related Party Disclosure


1. Related party (CAP Dec. 2009 5d-5 Marks)

Answer
Parties are considered to be related if at any time during reporting period one party has ability
to control over other party or exercise significant influence over the other party in making
financial and or operating decision.

2. Briefly describe the disclosure requirements of related party transactions as per Nepal
Accounting Standard 24 (CAP Jun. 2009 6b – 4 Marks; CAP Dec. 2014 6c-2.5 Marks)

Answer
NAS 24 requires disclosure of related party transactions and outstanding balances in the
separate financial statements of a parent, venture or investor company or entity. The
following information shall be disclosed in the financial statements as per this Accoun ting
Standard.
i) Relationship between parents and subsidiaries shall be disclosed irrespective of
whether there have been transactions between those related parties.
ii) An entity shall disclose key management personnel compensation.
iii) If there have been transactions between related parties, an entity shall disclose the
nature of the related party relationship as well as information about the transactions
and outstanding balances necessary for an understanding of the potential effect of
the relationship on the financial statements.
Items of the similar nature may be disclosed in aggregate except when separate disclosure
is necessary for an understanding of the effects of related party transactions on the financial
statements of the entity.

3. Krishna Infrastructures Limited (KIL) is engaged in carrying out the construction activities of
hydropower projects and sanitation projects in Joint venture with other private construction
companies. Recently, KIL entered into an agreement under its leadership and management to
lay the pipelines in Hetauda Industrial Area with Premier constructions Pvt. Ltd., Raktakali
Nirman Sewa and Pukar Nirman and Constructions Limited. KIL wants to close the books of
accounts as on 31st Ashadh 2070. The management of the company seeks your suggestion as
to whether Premier constructions Pvt. Ltd., Raktakali Nirman Sewa and Pukar Nirman and
Constructions Limited are to be disclosed as Related Party in the financial statements or not.
Advice the management taking into account relevant pronouncements of Nepal Accounting
Standard.
(CAP Jun. 2014 Q5c- 5 Marks)
Answer:
According to Nepal Accounting Standard relating to Related Party, Related party, a party is
related to an entity if:
(a) directly, or indirectly through one or more intermediaries, the party:

© The Institute of Chartered Accountants of Nepal 63


CAP II Paper 1: Advanced Accounting

(i) controls, is controlled by, or is under common control with, the entity (this includes
parents, subsidiaries and fellow subsidiaries);
(ii) has an interest in the entity that gives it significant influence over the entity; or
(iii) has joint control over the entity;
In this case, KIL entered into an agreement to lay the pipelines in Hetauda Industrial Area with
Premier constructions Pvt. Ltd., Raktakali Nirman Sewa and Pukar Nirman and Constructions
Limited. KIL has the leadership as well as management to control the project and the other 3
entities are just the intermediaries of KIL. In case of any other projects there may be different
types of intermediaries.
However, it should be noted that the intermediaries are not such that these three entities are the
subsidiaries of KIL. In considering each possible related party relationship, attention is directed
to the substance of the relationship and not merely the legal form.
In the context of this Standard, the parties need not necessarily be related only because they
share joint control over a joint venture. Thus, there may not necessarily be related party
relationship between KIL and other three entities.

4. Related party transaction (CAP Jun. 2015 Q6c- 2.5 Marks)


A business deal or arrangement between two parties who are joined by a special relationship
prior to the deal. For example, a business transaction between a major shareholder and the
corporation would be deemed a related-party transaction. As per the Provision of Nepal
Accounting Standards 23, any such transaction should be properly disclosed in the Financial
Statement of the Organization.

© The Institute of Chartered Accountants of Nepal 64


CAP II Paper 1: Advanced Accounting

1.18. NAS 33 Earnings Per Share

1. Goverdhan Ltd. has equity capital of Rs. 2,000,000 consisting of fully paid equity shares of Rs.
10 each. The net profit for the year ended 31.03.2069 was Rs. 3,000,000. It has also issued
18,000, 10% convertible debentures of Rs. 50 each. Each debenture is convertible into 5 equity
shares. The tax rate applicable is 30%.
Compute the diluted earnings.(CAP Jun. 2013 Q4b– 3 Marks)

Answer
Particulars Amount
Interest on Debentures @10% for the year 90,000
(18,000 debentures x Rs50x10%)
Less Tax on Interest @ 30% -27,000
63,000
Add: Net Profit for the year ended 31.3.2069 30,00,000
Diluted Earning 30,63,000

1.19. NAS 36: Impairment of Assets

1. Explain about corporate assets (CAP Dec. 2009-3 Marks)

Answer
Corporate assets are the assets other than goodwill that contribute to the future cash flow
of both cash generating unit under review and other cash generating units.

2. NAS 18 "Impairment of Assets" requires an entity to write down the value of its assets, or
group of assets, whenever the recoverable amount of an asset is less than its carrying value.

Required:
Define “recoverable amount” and state why an asset should be written down to this value if it
is below its carrying value. (CAP Jun. 2010 5a- 5 Marks)

Answer
NAS 36 “Impairment of Assets” defines recoverable amount as the higher of an asset’s fair
value less costs to sell and its value in use.
It further defines the fair value less costs to sell as the amount obtainable from the sale of an
asset (in an arm’s length transaction between knowledgeable, willing parties) less costs of
disposal.
Also, it defines the value in use as the present value of the future cash flows expected to be
derived from an asset (or cash-generating unit).
The recoverable amount of an asset represents the amount of economic benefits that the asset
will generate for an entity. If the carrying value of an asset exceeds its recoverable amount this
means that the asset will not generate sufficient economic benefits to meet its carrying value,

© The Institute of Chartered Accountants of Nepal 65


CAP II Paper 1: Advanced Accounting

the asset should therefore be written down to the value that is recoverable by either continuing
to use the asset or by selling it.

3. Brindawan Ltd. has an item of plant that is carried in the balance sheet at a revalued amount of
Rs. 16,200,000. The plant manufactures a product which, until recently, was the only product
of its type in the market place. A competitor is now manufacturing a similar product, and
Brindawan Ltd.’s market share has consequently fallen. Brindawan Ltd. has re-assessed the
expected cash flows, to be generated from using the plant over the remaining four years of its
life, to be as follows:
Rs.
2067/68 7,000,000
2068/69 4,000,000
2069/70 3,500,000
2070/71 2,000,000
Brindawan Ltd. has been offered Rs. 15,000,000 for the plant by an overseas company.
Brindawan Ltd. would be responsible for any shipping and conversion costs that are estimated
to be Rs. 1,000,000.

The depreciated historic cost of the asset is Rs 15,200,000.


Brindawan Ltd.’s cost of capital is 8%. The present value of Re 1 receivable at the end of each
year, based on discount rates of 8% can be taken as:

End of year 1 0·93


2 0·86
3 0·80
4 0·74
Required:
Calculate the impairment loss in respect of the plant and state how the loss should be accounted
for in Brindawan Ltd.’s financial statements for the year ended 31st Ashadh, 2067.
(CAP Jun. 2010 5b- 6 Marks)
Answer

Computation of fair value less cost to sell:


Selling price 15,000,000
Shipping and conversion costs (1,000,000)
Net proceeds 1,4000,000

Computation of Value in use


Year Cashflow DF Rs
2067/68 7,000,000 0.93 6,510,000
2068/69 4,000,000 0.86 3,440,000
2069/70 3,500,000 0.8 2,800,000
2070/71 2,000,000 0.74 1,480,000

© The Institute of Chartered Accountants of Nepal 66


CAP II Paper 1: Advanced Accounting

14,230,000

Impairment Loss ( Rs) = Carrying Amount – Value in Use


= 16,200,000 – 14,230,000 = Rs. 1,970,000

The recoverable amount of the asset is the greater amount, i.e. its value in use of Rs 1,4230,000.
The asset is carried at a revalued amount of Rs 16,200,000, this means that an impairment loss
of Rs 1,970,000 has occurred.

As the asset is carried at a revalued amount and the depreciated historic cost of the asset is Rs
15,200,000 there will be a revaluation surplus of Rs 970,000 (Rs.15,2500,000 –
Rs.14,230,000)in respect of the asset, this surplus would initially be utilized for the write down.
The remaining impairment loss of Rs. 1,000,000 will be charged to the profit and loss for the
period.

© The Institute of Chartered Accountants of Nepal 67


CAP II Paper 1: Advanced Accounting

1.20. NAS 38 Intangible Assets

4. A pharma company spent Rs. 135 lakhs during the accounting year ended 31st March, 2012 on
a research project to develop a drug to treat “AIDS”. Experts are of the view that it may take
four years to establish whether the drug will be effective or not and even if found effective it
may take two to three more years to produce the medicine, which can be marketed. The
company wants to treat the expenditure as deferred revenue expenditure. Give your comments
for the fiscal year ending on 31-03-2012 in the context of relevant NAS.

Answer
As per NAS 27 on intangible assets, the research cost is to be expenses as and when incurred.

The development expenses, cost of internal project also to be expenses as incurred unless they
meet asset recognition criteria, before recognizing these costs as assets the following points
should be demonstrated:
• Technical feasibility of the product
• Availability of product for use or sale
• Identification of cost incurred
• Probability of external market or
• The realistic expectation that there will be sufficient revenues to cover cost.

In the given case, the above conditions not having been fulfilled (Nothing is stated about future
revenue or benefits), the sum of Rs. 135 lakhs should be charged as an expense in the
accounting year ended 31st March, 2012.

5. A company with a turnover of Rs. 250 crores and an annual advertising budget of Rs. 2 crore
has taken up the marketing of a new product. It was estimated that the company would have a
turnover of Rs. 25 crores from the new product. The company has charged the total expenditure
of Rs. 2 crore incurred on extensive special initial advertisement campaign for the new product
as expense. Is the procedure adopted by the company correct?
(CAP Dec. 2013 Q5b-5 Marks)
Answer
For recognition of intangible items as intangible asset, it should meet the definition of an asset
as per NFRS. As the advertisement are already incurred with no control over it and no future
economic benefit associated to it that can be measured reliably, the amount shall be recognized
as an expense. Therefore, the accounting treatment followed by the company is correct.

6. R has developed a software program during the year to 30th November 2014. The cost of
developing the software was Rs. 50 million. The software is used by the rest of the group and
sold to third parties. Net revenue of Rs. 40 million is expected from sale of the software .Which
has quickly become market leader in this field. The software is expected to generate revenue
for four years, after which an upgraded version will be developed

© The Institute of Chartered Accountants of Nepal 68


CAP II Paper 1: Advanced Accounting

Should R recognize the internally developed software as an Intangible asset?


(CAP Dec. 2014 Q5c-5 Marks, CAP Jun. 2015 Q5c- 5 Marks)
Answer
Internally generated intangibles, other than goodwill, is required to be recognized as an
intangible asset under the provisions of NAS 38.
The software developed by R is capable of generating economic benefit through both use
within the group and through sale to external parties which establish that the asset is identifiable
and within the control of the entity. Therefore, R Ltd. should recognize the internally generated
software as an asset. The cost of the asset (50 million) is assumed to be recovered through sale
and use. R ltd. would, however, be required to assess impairment on the intangible on a periodic
basis. Further, being an asset having definite useful life, the intangible shall be amortized over
its useful life of 4 years.

© The Institute of Chartered Accountants of Nepal 69


CAP II Paper 1: Advanced Accounting

1.21. NFRS 2 Share Based Payments

1. Employee stock option plan and its importance (CAP Jun. 2017 Q6a-3 Marks)

Answer
It is a plan under which the enterprise grants employee stock options. Employee stock option
is a contract that gives the employees of the enterprise the right, but not the obligation, for a
specified period of time to purchase or subscribe the shares of the company at a fixed or
determinable price.
The importance of these plans is as follows:
1) Employee stock option plans encourage employees to have higher participation in the
company.
2) Stock options provide an opportunity to employees to contribute in the growth of the
company.
3) Stock option creates long term wealth in the hands of the employees.
4) They are important means to attract, retain and motivate the best available talent for the
company.
5) It creates a sense of ownership between the company and its employees.

1.22. Miscellaneous

1. What are the issues, with which accounting standards deal? (CAP Dec. 2015 Q3b)

Answer
Accounting Standards deal with the issues of
i) Recognition of events and transactions in the financial statements,
ii) Measurement of these transactions and events,
iii) Presentation of these transactions and events in the financial statements in
a manner that is meaningful and understandable to the reader, and
iv) Disclosure requirements which should be there to enable the public at large
and the stakeholders and the potential investors in particular, to get an insight
into what these financial statements are trying to reflect and thereby
facilitating them to take prudent and informed business decisions.

2. Money Measurement Concept (CAP Dec. 2013 6a-2.5 Marks)

Answer
The concept of Money measurement is an important aspect of accounting. It is an important
convention in accounting which explains that any transaction or event that can be measured in
terms of money can only be recorded in accounting. Accounting is disclosure of all the business
activities in an organized way that includes the figures. Anything that cannot be measured in
terms of money cannot be shown in accounts. E.g. even though human resources are an

© The Institute of Chartered Accountants of Nepal 70


CAP II Paper 1: Advanced Accounting

important asset to any entity, these cannot be recorded in the books of accounts as they cannot
be reasonably measured in monetary terms.

3. Contingency reserve (CAP Dec. 2012 6b-2.5 Marks)

Answer
The Contingency reserves are sum set aside to cover anticipated future liabilities or reduction
in assets value. This reserve is required when the company believes the value of its assets likely
decrease or it has incurred liabilities and it is able to reasonable estimate the amounts loss.
Contingency reserves are net up by deducting the appropriate sum from income. Contingencies
include:
• Potentially uncollectable money owed to company.
• Potential obligation under product warranties or related to products defects judgment
for pending threaten litigation.
• Likely loss due to fire and other hazards.
The Contingency reserve must be disclosed in financial statement when required and may be
utilized for the following purposes:
a. Expenses or loss of profits arising out of accidents, strikes or circumstances which the
management could not have prevented.
b. Expenses on replacement or removal of plant or works (other than normal maintenance
or renewals).
c. Statutory obligation for payment of any compensation, if there is no special provision
for such compensation.

4. Realization concept (CAP Dec. 2015 6C

Answer
Realization concept in accounting, also known as revenue recognition principle, refers to the
application of accruals concept towards the recognition of revenue (income). Under this
principle, revenue is recognized by the seller when it is earned irrespective of whether cash
from the transaction has been received or not. In case of sale of goods, revenue must be
recognized when the seller transfers the risks and rewards associated with the ownership of the
goods to the buyer. This is generally deemed to occur when the goods are actually transferred
to the buyer. Where goods are sold on credit terms, revenue is recognized along with a
corresponding receivable which is subsequently settled upon the receipt of the due amount
from the customer. In case of the rendering of services, revenue is recognized on the basis of
stage of completion of the services specified in the contract. Any receipts from the customer
in excess or short of the revenue recognized in accordance with the stage of completion are
accounted for as prepaid income or accrued income as appropriate.

5. Window dressing (CAP Dec. 2015 Q6 E

© The Institute of Chartered Accountants of Nepal 71


CAP II Paper 1: Advanced Accounting

Answer
Window dressing is actions taken to improve the appearance of a company's financial
statements. Window dressing is particularly common when a business has a large number
of shareholders, so that management can give the appearance of a well-run company to
investors who probably do not have much day-to-day contact with the business. It may also
be used when a company wants to impress a lender in order to qualify for a loan. If a business
is closely held, the owners are usually better informed about company results, so there is no
reason for anyone to apply window dressing to the financial statements.

Examples of window dressing are:


▪ Cash. Postpone paying suppliers, so that the period-end cash balance appears higher than
it should be.
▪ Accounts receivable. Record an unusually low bad debt expense, so that the accounts
receivable (and therefore the current ratio) figure looks better than is really the case.
▪ Fixed assets. Sell off those fixed assets with large amounts of accumulated depreciation
associated with them, so the net book value of the remaining assets appears to indicate a
relatively new cluster of assets.
▪ Revenue. Offer customers an early shipment discount, thereby accelerating revenues
from a future period into the current period.
▪ Depreciation. Switch from accelerated to straight-line depreciation in order to reduce the
amount of depreciation charged to expense in the current period. The mid-month
convention can also be used to further delay expense recognition.
▪ Expenses. Withhold supplier expenses, so that they are recorded in a later period.

6. Fair Value Accounting (CAP Dec. 2017 Q6e -3 Marks)

Answer
Fair Value is the amount for which an asset could be exchanged or a liability settled between
knowledgeable willing parties in an arm's length transaction. Fair value is measured using the
price in the principal market for the asset or liability (i.e. the market with the greatest volume
and level of activity for the asset or liability) or, in the absence of a principal market, the most
advantageous market for the asset or liability. Detailed guidance shall be required for
measuring the fair value of liabilities, including a description of the compensation that market
participants would demand to take on an obligation.

7. Financial Instrument (CAP Dec. 2018 Q6b -3 Marks)

Answer
Financial instrument is any contract that gives rise to a financial asset to one entity and a
financial liability or equity instrument to another entity. Hence, financial instruments include
financial assets, financial liability and equity instrument. This means that financial assets of
one entity shall be financial liabilities or equity instruments of another entity and financial
liabilities or equity instrument of one entity shall be financial assets of another entity. For
example, bond, debenture or bank loan is financial liabilities of entity issuing such bond or

© The Institute of Chartered Accountants of Nepal 72


CAP II Paper 1: Advanced Accounting

debenture or raising loan and it is financial assets for holder of debenture or bond holder or
provider of loan. Similarly, share capital is equity instrument for share issuing entity and it
is financial assets of holder of equity.

8. Receipt and Expenditure Account (CAP Dec. 2018 Q6c -3 Marks)

Answer
Receipt and Expenditure Account also can be taken as part of Financial Statements. Some
non-profit making organization like professional firms, educational institutes etc. prefers to
prepare Receipts and Expenditure account instead of Income and Expenditure account as part
of Financial Statements. Such an account includes all expenses on accrual basis but incomes
are recorded on cash basis. In other words, to find out the result, all outstanding expenses are
taken into account but the incomes that are outstanding are not considered. The main reason
behind this kind of practice is that professionals consider it imprudent and risky to recognize
the outstanding incomes.

9. Write the difference between Cash Flow and Funds Flow Statement
(Inter Dec. 2008 Q5b -5 Marks)
Answer
(i) Cash flow statement deals with the change in cash position between two points of time.
Fund flow statement deals with the changes in working capital position.
(ii) Cash flow statement contains opening as well as closing balances of cash and cash
equivalents. The fund flow statement does not contain any such opening and closing
balance.
(iii) Cash flow statement records only inflow and outflow of cash. Fund flow statement
records sources and application of funds.
(iv) Fund flow statement can be prepared from the cash flow statement under indirect
method. However, a cash flow statement cannot be prepared from fund flow statement.
(v) A statement of changes in working capital is usually prepared along with fund flow
statement. No such statement is prepared along with the cash flow statement.

10. Off Balance Sheet Items:


(Inter Dec. 2008 Q6b-4 Marks; CAP Jun. 2015 Q6d- 2.5 Marks; CAP Dec. 2014 6d-2.5
Marks)
Answer
Off Balance Sheet Items are obligations that are contingent liabilities and thus do not appear
on the balance sheet.
Off Balance Sheet item could involve a lease or a separate subsidiary or a contingent liability
such as a letter of credit. It also involves loan commitments, futures, forwards and other
derivatives except such derivatives pertaining to equity securities, ESOP, or phantom stock,
which usually must be held as reserves in the Long-Term Debt section of a Balance Sheet.
In general, off-balance sheet items includes direct credit substitutes in which a financial
institution substitutes its own credit for a third party, including standby letters of credit,
irrevocable letters of credit that guarantee repayment of commercial paper or tax-exempt

© The Institute of Chartered Accountants of Nepal 73


CAP II Paper 1: Advanced Accounting

securities, sale and re-purchase agreements, interest rate swaps, interest rate options and
currency options and so on.

11. True and fair view of financial statements (Inter Dec. 2009 Q6a-5 Marks)

Answer
A financial statement can be said as presenting true and fair view of the business when it is
prepared with proper application of relevant and appropriate accounting standards. Further, it
should also contain the principal qualitative characteristics; viz. understandability, relevance,
reliability and comparability. Such true and fair presentation of financial statements are
expected to meet the objective of providing information about the financial position,
performance and changes in financial position of an entity that is useful to a wide range of users
in making economic decision.

12. Capital and Revenue Receipt (Inter Dec. 2009 Q6c-5 Marks)

Answer
Subscription by shareholders towards share capital of a Company or for Purchasing its
debentures are considered by the Company as capital receipts. By the same criterion,
contributions by partners or proprietors to capital of their business are capital receipts. Profits
to a business on sale of fixed Assets (not those on the sale of goods in which it trades) also are
capital receipts since these are distinguishable from revenue receipts, e.g., those from sale of
merchandise , rent on property , interest on investment , professional fee for services rendered,
etc. It will be evident that capital receipts emanate out of a fund already held or arise on
conversion of an asset , whereas revenue receipts flow from personal exertion, use of a capital
asset or from sale or transfer of floating assets .

13. Accounting Convention of Conservatism. (Inter Dec. 2010 Q6 c-5 Marks)

Answer
Accounting convention of conservatism states that the accountants should not anticipate income
and should provide for all possible losses. The underlying principle is that revenues should only
be recognized when there is reasonable certainty about their realization. At the same time
provision must be made for all possible liabilities, whether the amount is known with certainty
or is based on estimates. Faced with the choice between two methods of valuing an asset, the
method which leads to lesser value must be selected. To illustrate, inventories are recorded at
the cost or market value, whichever is less or if there is a possibility that a debt may not be
realized, a specific amount is set aside from profits as a provision for doubtful debts.

14. Deferred Revenue Expenditure.


(Inter Dec. 2010 Q6 d-5 Marks; Inter Jun. 2009 Q6a- 5 Marks)
Answer

© The Institute of Chartered Accountants of Nepal 74


CAP II Paper 1: Advanced Accounting

The term deferred revenue expenditure was traditionally used in the case of pre-operating
expenses or in the case of expenditures which were unusually huge to affect the financial
performance or in the case of expenditure whose benefit was assumed to be available for more
than a single accounting period. Such expenditures were treated as asset and were written off
over a period of several years based upon management’s discretion. For example, advertisement
cost, cost of staff training etc. However, with an introduction of NAS 27 Intangible Assets,
only those assets which meet the identifiability criterion as prescribed in the standard is allowed
to be recognized as intangible assets. Accordingly all other expenditures that do not meet the
criteria are charged as expenditure irrespective of their volume and nature.

15. Prior Period Items (Inter Jun. 2010 Q6d- 5 Marks)

Answer
The term "Prior Period Item" refers only to income or expenses which arise in the current period
as a result of errors or omissions in the preparation of the financial statements of one or more
prior periods. The term does not include other adjustments necessitated by circumstances,
which though related to prior periods, are determined in the current period, e.g. arrears payable
to workers as a result of revision of wages with retrospective effect during the current period.
Errors in the preparation of the financial statements of one or more prior periods may be
discovered in the period. Errors may occur as a result of mathematical mistakes, mistakes in
applying accounting policies, misinterpretation of facts or oversight. Such items are generally
infrequent in nature and can be distinguished from changes in accounting estimates.
Prior Period Items are normally included in the determination of net profit or loss for the current
period. An alternative approach is to show such items in the statement of profit and loss after
determination of current period net profit or loss.

16. Substance over form (Inter Jun. 2009 Q6 c- 5 Marks)

Answer
If information is to express faithfully the transactions and other events that it purports
to represent, it is necessary that they are accounted for and presented in accordance
with their substance and economic reality and not merely their legal form. The
substance of transactions or other events is not always consistent with that which is
apparent from their legal of contrived form. For example, an entity may dispose of an
asset to another party in such a way that the documentation purports to pass legal
ownership to that party; nevertheless, agreements may exist that ensure that the entity
continue to enjoy the future economic benefits embodied in the asset. In such
circumstances, the reporting of a sale would not represent faithfully the transaction
entered into.

17. Following is the information of two business organizations belonging to same industry for
2065/066;
Particulars Birat Industry Kirat industry

© The Institute of Chartered Accountants of Nepal 75


CAP II Paper 1: Advanced Accounting

Rs Rs

Opening balance 25,000 15,000


Cash flow from operating activities 8,500 24,000
Cash from investing activities;
Sale of fixed assets 35,000 6,000
Cash from financing activities:
Amount borrowed 40,000 5,000
Closing Balance 1,08,500 50,000
Birat claims that it’s cash generating activity is better than that of Kirat. Do you
agree ? Comment on the basis of cash generating ability of both organization .
(CAP Jun. 2011 4b- 5 Marks)

Answer
Kirat organization has a better capacity to generate cash. Claim of Birat organization is not
appropriate. The evaluation is based on the quantum of cash flow from operating activities
rather than on investing or financing activities. Birat organization is generating funds from
either sale of goods or out of borrowing and quantum of funds so generated is not an
appropriate indicator for evaluation.

18. Matching Concept (CAP Jun. 2012 Q6a – 4 Marks)


Answer
Matching concept requires periodic revenue should be matched with that period’s expense.
While reporting financial performance of the entity only that portion of expense should be
recognized as expense which is attributable for earning revenue and remaining portion of the
expenses should be carried forward for the next accounting period so as to derive actual
performance of the entity during that period. That is the reason why prepaid expenses are not
recognized in periodic performance though outflow is incurred during that particular period.
Provisioning of the expense follow same rule. Depreciation is charged on the ground of same
underlying principle.

19. Adjusting entries and correcting entries (CAP Jun. 2017 6b-3 Marks)
Generally, adjusting entries are required every accounting period so that a company's
financial statements reflect the accrual method of accounting.
It is typical for the adjusting entries to be dated as of the last day of the accounting
period and to include an income statement account and a balance sheet account.

Adjusting entries are necessary to:


I. accrue expenses and losses and the related liabilities.
II. accrue revenues and gains and the related assets.
III. defer expenses and the related assets.
IV. defer revenues and the related liabilities.

© The Institute of Chartered Accountants of Nepal 76


CAP II Paper 1: Advanced Accounting

V. record depreciation expense or bad debts expense and the change in the related contra
asset account.

A correcting entry is needed only if an error is discovered in an account. Correcting


entries can involve any combination of income statement and balance sheet accounts.
Correcting entries are recorded if:
I. an erroneous amount was used in a previously posted entry.
II. an entry was recorded in the wrong account.

20. Financial Reporting Standard (Inter Dec 03, Q 6(iii))


Answer:
International Accounting Standard Board publishes its standard in a series of pronouncements
called International Financial Reporting Standards (IFRS). Upon its inception the IASB
adopted the International Accounting Standards issued by International Accounting Standard
Committee. The term International Financial Reporting Standards (IFRS) includes IFRS's,
IFRIC Interpretation, IASs and SIC Interpretations.

21. Entity Concept (Dec 03, Q6(iv))


Answer:
Business consists of person and resources. Person representing the business is separate and
distinct from the business enterprises. Accounting system deals with the economic activities of
the business not of owner. In this concept, Preparation of Balance Sheet of the business does
not consider the personal assets and liability of the owner of the business.

22. Advantages and disadvantages of setting of Accounting Standards.


(Inter Jun. 2006 Q 7a--3 Marks)

Answer:
The Accounting Standards seek to describe the accounting principles, the valuation
techniques and the methods of applying the accounting principles in the preparation and
presentation of financial statements so that they may give a true and fair view. The ostensible
purpose of the standard setting bodies is to promote the dissemination of timely and useful
financial information to investors and certain other parties having an interest in companies’
economic performance. The setting of accounting standards has the following advantages:
i. Standards reduce to a reasonable extent or eliminate altogether confusing variations in the
accounting treatments used to prepare financial statements.
ii. There are certain areas where important information are not statutorily required to be
disclosed. Standards may call for disclosure beyond that required by law.
iii. The application of accounting standards would, to a limited extent, facilitate comparison of
financial statements of companies situated in different parts of the world and also of
different companies situated in the same country. However, it should be noted in this
respect that differences in the institutions, traditions and legal systems from one country
give rise to differences in accounting standards practiced in different countries.

© The Institute of Chartered Accountants of Nepal 77


CAP II Paper 1: Advanced Accounting

However, there are some disadvantages of setting of accounting standards:


i. Alternative solutions to certain accounting problems may each have arguments to
recommend them. Therefore, the choice between different alternative accounting
treatments may become difficult.
ii. There may be a trend towards rigidity and away from flexibility in applying the
accounting standards.
iii. Accounting standards cannot override the statute. The standards are required to be framed
within the ambit of prevailing statutes.

© The Institute of Chartered Accountants of Nepal 78


CAP II Paper 1: Advanced Accounting

CHAPTER 2: ACCOUNTING FOR SPECIAL TRANSACTIONS

2.1 Hire Purchase transaction

Hire purchase debtors and stock method, hire purchase trading account method are not
acceptable accounting models under NFRS. NAS 17 relates Hire Purchase Transactions
as Finance lease which should be accounted for accordingly. Questions, where
information allows, have been solved as per the requirements of NAS 17.

A. Theoretical Questions

1. List out the differences between Hire-purchase system and Installment Purchase
system.
(Inter Dec. 2001, Q 6c-4 Marks)
Answer:
Hire Purchases System Installment Purchase System
(a) Ownership in goods will pass to the Ownership in the goods will pass to the
buyer only on payment of last buyer immediately at the time of sale
installment.
(b) In case if the buyer fails to pay any The seller cannot recover the goods. He can
installment the seller can recover the only sue for recovery of price and damages.
goods back from buyer.
(c) Default in payment by the buyer the The money paid by the buyer, will be taken
seller can forfeit all moneys paid by as a payment towards part of the selling
the buyers so far. price and the seller can sue only for the
balance.

2. What is the basic principle involved in accounting for hire purchase transactions in
the books of seller?
(Inter Jun. 2007 Q 2a)
Answer
The basic principle involved in accounting for hire purchase transactions in the books of the
seller is the spreading of recognition of revenue over the period of the transaction i.e. as per
the terms of hire purchase agreement.

3. Features of Installment Sale Transaction (Inter Dec. 2011 Q6c-5 Marks)


Answer
The essential features of Instalment Sale Transaction are as follows:
1. The ownership in the goods passes to the buyer as soon as the transaction is completed.
2. The seller has right only to sue for the unpaid balance. He has no right to repossess the
goods for default in payment of instalments.

© The Institute of Chartered Accountants of Nepal 79


CAP II Paper 1: Advanced Accounting

3. The buyer has no right to terminate the agreement by returning the goods. Therefore, he
cannot escape his liability for payment of future instalments.
4. The buyer, as the owner of the goods, has the right of disposing of the goods in any manner
he likes.
5. Any loss of goods should be borne by the buyer a risk lies with the ownership.

B. Practical Questions

1. Nepal Transport Ltd. purchased three trucks costing Rs. 6,25,000 each from Kathmandu Auto
Ltd. on 1st January 2000 on hire purchase system. The terms were:
Payment on delivery Rs. 2,50,000 for each truck and balance of the principal amount by three
equal installments plus interest at the rate of 15% per annum to be paid at the end of each year.
Nepal Transport's rate of depreciation on trucks is 25% on diminishing balance method. Nepal
Transport paid the installment due on 31st December 2000 and 31st December 2001 but could
not pay the final installment. Kathmandu Auto Ltd. repossessed these two trucks adjusting
values against the amount due. The repossession was done on 1st January 2003 on the basis of
40% depreciation on diminishing balance method.
i. Prepare ledgers in the books of Nepal Transport Ltd. showing the transactions up to the
date of repossession.
ii. Show the disclosure of the balance arising from the above in the Financial Position of
Nepal Transport Ltd. as on 31st December 2002.
(Inter Dec 03 Q5)
Answer:
Trucks Account
Dr. Cr.
Date Particulars Rs. Date Particulars Rs.
1.1.2000 Kathmandu 18,75,000 31.12.2000 By Balance c/d 18,75,000
Auto Ltd.
18,75,000 18,75,000
1.1.2001 To Balance 18,75,000 31.12.2001 By Balance c/d 18,75,000
c/d
18,75,000 18,75,000
1.1.2002 To Balance 18,75,000 31.12.2002 By Balance c/d 18,75,000
b/d
10,54,687 18,75,000
1.1.2003 To Balance 18,75,000 1.1.2003 By Accumulated Dep. 722,656
b/d
1.1.2003 By Kathmandu Auto 270,000
Ltd.
1.1.2003 By Loss on 257,344
repossession
1.1.2003 By Balance c/d 625,000
18,75,000 18,75,000

© The Institute of Chartered Accountants of Nepal 80


CAP II Paper 1: Advanced Accounting

Accumulated Depreciation A/C


Date Particulars Rs. Date Particulars Rs.
31.12.2000 To Balance 4,68,750 31.12.2000 By Depreciation 4,68,750
c/d A/c
4,68,750 4,68,750
31.12.2001 To Balance 820,313 01.01.2001 By Balance b/d 4,68,750
c/d
31.12.2001" By Depreciation 3,51,563
A/c
820,313 820,313
31.12.2002 To Balance 1,083,985 01.01.2002 By Balance b/d 820,313
b/d
31.12.2002 By Depreciation 263,672
A/c
1,083,985 1,083,985
1.1.2003 To Truck A/c 722,656 1.1.2003 By Balance c/d 1,083,985
1.1.2003 To Bal b/d 361,329
1,083,985 1,083,985

Kathmandu Auto Ltd. A/c


1.1.2000 To Bank A/c 750,000 1.1.2000 By Trucks 1,875,000
Account
To Bank A/c 543,750 31.12.2000 By Interest 168,750
(375,000+168,750)
To Balance c/d 750,000
2,043,750 2,043,750

31.12.2001 To Bank A/c 487,500 1.1.2001 By Balance b/d 750,000


(375000+112500) 31.12.2001 By Interest 112,500
To Balance c/d 375,000
862,500 862,500
31.12.2002 To Balance c/d 4,31,250 1.1.2002 By Balance c/d 3,75,000
31.12.2002 By Interest 56,250
4,31,250 4,31,250
1.1.2003 To Truck A/c 2,70,000 1.1.2003 By Balance b/d 4,31,250
To Balance c/d 1,61,250
4,31,250 4,31,250

An Extract of Financial Position of Nepal Transport Ltd. as on 31.12.2002

© The Institute of Chartered Accountants of Nepal 81


CAP II Paper 1: Advanced Accounting

Other Liabilities Rs. Assets Rs.

Kathmandu Auto Ltd. 4,31,250 Trucks


Reserves (loss on Repo.) (257,344) Cost: 625,000
Accumulated Depreciation (361,329)
173,906 263,671

Working Notes:
i. Value of two truck repossessed on 1.1.2003 @ 40% depreciation:
2000 2001 2002
Cost / WDV in the beginning 1,250,000 750,000 450,000
Dep. @ 40% 500,000 300,000 180,000
Book Value 750,000 450,000 270,000
ii. Loss on return of two trucks:
Written down value of the trucks as on 1.1.2003 7,91,015
Less: Value of 1 truck retained (1/3) 2,63,672
Written down value of the repossessed trucks 5,27,343
Agreed value of repossessed trucks 2,70,000
Loss 2,57,343

2. A acquired, on 1st January, 2009 a machine under a Hire-Purchase agreement which provides
for 5 half-yearly installments of Rs. 6,000 each, the first installment being due on 1st July,
2009. Assuming that the applicable rate of interest is 10 percent per annum, calculate the cash
value of the machine.
(CAP Dec. 2009 Q2(b)(i) -5 Marks; CAP Jun. 2012 Q4c -5 Marks; CAP Jun. 2019 Q4b -5
Marks)
Answer
Alternative I
Given annual interest rate = 10%
Periodic interest rate = 10%/2 = 5%
Number of payments = 5
Periodic installment amount = 6,000

Now, Present Value Factor (annuity) @5% for 5 (n) periods = 4.3294
Therefore, cash value of the machinery = 4. 3294 × 6000 = 25,977
(If 2 decimal points considered, value of the machinery = 4.33 × 6000 = 25,980).

Alternative II
Principal Installment Principal
Installment # Interest @5%
Outstanding Amount Repayment
5 5,714 6,000 286 5,714
4 11,157 6,000 558 5,442
3 16,340 6,000 817 5,183

© The Institute of Chartered Accountants of Nepal 82


CAP II Paper 1: Advanced Accounting

2 21,276 6,000 1,064 4,936


1 25,977 6,000 1,299 4,701
The cash purchase price of machinery is Rs. 25,977.

3. The hire purchase department of Whirlpool Finance Ltd. sells television sets and room
coolers. This department was newly started in 2068. The relevant information is as follows:
Television Set Room Coolers
Cost (Rs.) 5,400 2,000
Cash Price (Rs.) 6,300 2,400
Cash down payment (Rs.) 900 400
Monthly installment (Rs.) 600 200
Number of installments 10 12
During the year, 100 television sets and 120 room coolers were sold on hire purchase basis.
Two television sets on which 3 installments only could be collected and 4 room coolers on
which 5 installments had been collected were repossessed. These were valued at Rs. 10,000
and after reconditioning at a cost of Rs. 1,000 were sold outright for Rs. 14,000. Other
installments collected and those due (customer still paying) were respectively as follows:
Television sets 270 and 20
Room coolers 400 and 30
Prepare Accounts on Stocks and Debtors System to reveal the profit of the Department.
(CAP Dec. 2011, Q2a-10 Marks; Inter Jun. 2009 Q3 - 15 Marks)

Answer
Whirlpool Limited
Hire Purchase Stock A/c
Particulars Rs. Particulars Rs.
To Goods sold on H.P. A/c 1,026,000 By H.P. Debtors A/c 405,600
By Goods Repossessed A/c –
Installments not due on repossessed
goods. 14,000
By Balance c/d – Installments not yet
due 606,400
1,026,000 1,026,000

Hire Purchase Debtors A/c


Particulars Rs. Particulars Rs.
To Hire Purchase Stock A/c 405,600 By Bank A/c 387,600
By Balance c/d 18,000
405,600 405,600

Goods Repossessed A/c


Particulars Rs. Particulars Rs.

© The Institute of Chartered Accountants of Nepal 83


CAP II Paper 1: Advanced Accounting

By H.P. Adjustment A/c – Bal.


To H.P. Stock A/c 14,000 4,000
fig.
By Balance c/d 10,000
14,000 14,000
To Balance b/d 10,000 By Bank – Sales 14,000
To Bank – Expenses 1,000
To H.P. Adjustment A/c – Profit 3,000
14,000 14,000

Goods sold on Hire Purchase A/c


Particulars Rs. Particulars Rs.
To Hire Purchase Adj. A/c –
246,000 By Hire Purchase Stock A/c 1,026,000
Loading
To Trading A/c 780,000
1,026,000 1,026,000
Hire Purchase Adjustment A/c
Particulars Rs. Particulars Rs.
By Goods Sold on H.P. –
To Goods Repossessed A/c - Loss 4,000 246,000
Loading
To Stock Reserve 144,971
By Goods Repossessed A/c -
To Profit 100,029 3,000
Profit
249,000 249,000

Working Notes:
(i) Hire Purchase Price is Rs. 6,900 for each Television Set and Rs. 2,800 for each Room
Cooler. Total cost and sales on this basis are as follows:
H.P. Price (Rs.) Cost Price (Rs.)
Television Sets – 100 690,000 540,000
Room Coolers – 120 336,000 240,000
Total 1,026,000 780,000

(ii) Cash Collected:


T.V.
Cooler
Set
Rs.
Rs.
Down payment (Rs. 900 × 100) & (Rs. 400 × 120) 90,000 48,000
Installments collected (Rs. 600 × 270) & (Rs. 400 × 200) 162,000 80,000
Amount collected on repossessed goods (Rs. 600 × 2 × 3) & (Rs. 200
× 5 × 4) 3,600 4,000
255,600 132,000
(iii) Installment not yet due:
Television Set: Total installments on 98 sets 980
Installments collected & due 290

© The Institute of Chartered Accountants of Nepal 84


CAP II Paper 1: Advanced Accounting

Balance 690
Amount of 690 installments @ Rs. 600 each [A] Rs. 414,000
Room Cooler: Total installments on 116 sets 1,392
Installments collected & due 430
Balance 962
Amount of 962 installments @ Rs. 200 each [B] Rs. 192,400
Total Amount [A] + [B] Rs. 606,400

(iv) Stock Reserve:


Television Set: [1,500 / 6,900 × 414,000] Rs. 90,000
Room Cooler: [800 / 2,800 ×192,400] Rs. 54,971
Total Rs. 144,971
(v) Installment not due on repossessed goods:
2 Television Sets 7 installments on each @ Rs. 600 Rs. 8,400
4 Room Coolers 7 installments on each @ Rs. 200 Rs. 5,600
Total Rs. 14,000
(vi) Installment due but not collected:
Television Sets: (20 × 600) Rs. 12,000
Room Cooler: (30 × 200) Rs. 6,000
Total Rs. 18,000

Note: Hire purchase transaction needs to be classified as finance lease as per the
provisions of NAS 17. As question specifies stock and debtors method itself, the original
solution has not been altered. However, accounting treatment required as per NAS 17 has
been provided hereunder:
1. Calculation of implicit rate in the lease
Month CF (TV) CF (Cooler)
0 (5,400.00) (2,000.00)
1 600.00 200.00
2 600.00 200.00
3 600.00 200.00
4 600.00 200.00
5 600.00 200.00
6 600.00 200.00
7 600.00 200.00
8 600.00 200.00
9 600.00 200.00
10 600.00 200.00
11 - 200.00
12 - 200.00
Implicit Rate
(%) 1.96% 2.92%

© The Institute of Chartered Accountants of Nepal 85


CAP II Paper 1: Advanced Accounting

2. Journal Entries
Hire Purchase Debtors Dr. 780,000
Bank Dr. 138,000
Sales Cr.
100 TV @ 6,300 630,000
120 Coolers @ 2,400 288,000
(Sales and debtors recognized)

Cost of Sales Dr. 780,000


Inventory Cr. 780,000
(Cost of sales charged to profit/loss for the period)

Bank Dr. xxxx


Debtors Cr. xxxx
Finance Income Cr. xxxx
(Not Determinable as details are not provided. Each
installment due would be accounted for by splitting
the installment amount to interest & principal
component)

Inventory Dr. 10,000


Loss on repossession Dr. 2,776
Debtors Cr. 12,776
(Being Goods Repossessed (WN. 1))

Inventory Dr. 1,000


Bank Cr. 1,000
(Cost of repair charged to inventory)

Bank Dr. 14,000


Sales Cr. 14,000
(Being repossessed items sold)

Cost of Sales Dr. 11,000


Inventory Cr. 11,000
(Cost of sales charged to profit/loss for the period)

WN1
TV
At the time of sales the following entry will be passed:
2 TVs 4 Coolers
Hire Purchase Debtors Dr. 10,800 8,000
Bank Dr. 1,800 1,600
Sales Cr. 12,600 9,600

© The Institute of Chartered Accountants of Nepal 86


CAP II Paper 1: Advanced Accounting

At the time of collection of each of the installments, the following entries will be passed:
TV I1 I2 I3
Hire Purchase Debtors Dr. 212 193 173
Finance Income Cr. (O/S amt X 1.96%) 212 193 173

Bank Dr. 1,200 1,200 1,200


Hire Purchase Debtors Cr. 1,200 1,200 1,200
Hire Purchase Debtors Balance for repossessed TV = 7,777

Cooler I1 I2 I3 I4 I5
Hire Purchase Debtors Dr. 234 217 200 183 165
Finance Income Cr. 234 217 200 183 165

Bank Dr. 800 800 800 800 800


Hire Purchase Debtors Cr. 800 800 800 800 800
Hire Purchase Debtors Balance for repossessed coolers = 4,999
Total HP Debtors for repossessed items = 12,776

4. Prischal sells goods on hire-purchase at cost plus 25%, prepare hire purchase trading
account from the information given below: (CAP Dec. 2012 2b-10 Marks)
Particulars Rs.
Stock with Customers on Hire -Purchase (Opening) 162,000
Stock in hand at shop (Opening) 324,000
Instalments Overdue (Opening) 150,000
Purchases during the year 1,080,000
Goods Repossessed (Instalments not due Rs. 36,000) 9,000
Stock at shop excluding repossessed goods (Closing) 360,000
Cash received during the year 1,035,000
Instalments Overdue (Closing) 162,000
The Vendor Spent Rs. 2,000 on goods repossessed and then sold it for Rs.
15,000.
Answer
Hire Purchase Trading Account
Particulars Rs. Particulars Rs.
To Opening Balance By Cash Received (Instalments) 1,035,000
Hire Purchase Debtors 150,000 By Stock Reserve (Opening ) W.N. 2 32,400
Hire Purchase Stock 162,000 By Goods sold on Hire Purchase
(Instalments Overdue) (Loading) (W.N.1) 261,000
By Cash Received (On sale of Re-
possessed goods) 15,000
To Goods sold on hire
Purchase (W.N.1) 1,305,000 By Closing Balance

© The Institute of Chartered Accountants of Nepal 87


CAP II Paper 1: Advanced Accounting

Hire Purchase Stock (Inst. 384,000


To Cash (Expense) 2,000 Overdue) (W.N.4)
To Stock reserve (W.N.5) 76,800 Hire Purchase Debtors 162,000
To Profit & Loss Account 193,600
1,889,400 1,889,400

Working Notes
1. Memorandum Stock at Shop Account
Particulars Rs Particulars Rs
To Balance B/d 324,000 By Goods Sold on Hire Purchase 1,044,000
Account
To Purchases(at Cost) 1,080,000 By Balance C/d 360,000
1,404,000 1,404,000
Goods Sold on Hire Purchase Account at Invoice Price (1044000 x 125%) 1,305,000
Loading Rs 1305000-1044000 261,000

2. Opening Reserve Account = 162,000x25/125=32,400

3. Hire Purchase Debtor Account


Particulars Rs Particulars Rs
To Balance B/d 150,000 By Cash Received 1,035,000
To Goods Sold on Hire 1,305,000 By Hire Purchase Stock Account (Bal 258,000
Purchase .Fig)
By Balance c/d 162,000
1,455,000 1,455,000

4. Hire Purchase Stock Account


Particulars Rs Particulars Rs
To Balance B/d 162,000 By Goods Repossessed 36,000
To Hire Purchase Debtors 258,000 By Balance C/d (Bal Fig) 384,000
420,000 420,000

5. Closing Stock Reserve = 384,000x25/125=76,800

Note: Alternative solution cannot be suggested as the question lacks fundamental


information required for the calculation based on NAS 17.

5. From the following particulars, calculate:


a. Agreed value of two plants taken back by the hire vendor.
b. Book value of plant left with the hire purchaser.
c. Profit or loss to hire purchaser on two plants taken back by the hire vendor.
d. Profit or loss on plants repossessed, when sold by the hire vendor.

© The Institute of Chartered Accountants of Nepal 88


CAP II Paper 1: Advanced Accounting

Particulars:
X purchased three plants from Y on hire purchase basis, the cash price of each plant being
Rs. 100,000.
The hire purchaser charged depreciation @ 20% on diminishing balance method.
Two plants were seized by the hire vendor when second installment was not paid at the end
of the second year. The hire vendor valued the two plants at cash price less 30% depreciation
annually charged at diminishing balance method.
The hire vendor spent Rs. 40,000 on overhauling of the plants and then sold them for a total
sum of Rs. 160,000.
(CAP Dec. 2013 Q4a -7 Marks; CAP Dec. 2016 4b-5 Marks)
Answer
a. Agreed value of two plants taken back by the hire vendor
Price of two plants = Rs. 100,000 x 2 200,000
Less: Depreciation for the first year @ 30% 60,000
Less: Depreciation for the second year Rs. 140,000 x 30/100 42,000
Agreed value of two plants taken by the hire vendor 98,000

b. Book value of the plant left with the hire purchaser


Cash purchase price of one plant 100,000
Less: Depreciation on Rs. 100,000 @ 20% for first year 20,000
Written down value of the plant at the end of first year 80,000
Less: Depreciation on Rs. 80,000 @ 20% for the second year 16,000
Book value of the plant left with the hire purchaser 64,000

c. Profit or loss to hire purchaser


Book Value of one plant (b) 64,000
Book Value of two plants 128,000
Agreed Value 98,000
Loss 30,000

d. Profit or loss on plants repossessed


Sale proceeds of plants repossessed 160,000
Less: Agreed Value 98,000
Less: Overhauling Charges 40,000
Profit on resale 22,000

6. On 2068.04.01, Samuels acquired a car on Hire Purchase System from Nelson & Co.,
agreeing to pay 4 annual installments of Rs 120,000 each payable at the end of each year.
There is no down payment. Interest is charged @ 20 % per annum and is included in the
annual installments.
Answer the followings:
Show the account of Nelson & Co. for all the years in the books of Samuels.
What amount Samuels would require to pay if he buys the car on cash?
CAP Dec. 2015 3a

© The Institute of Chartered Accountants of Nepal 89


CAP II Paper 1: Advanced Accounting

Solution
Principal Installment Interest Principal
Installment #
Outstanding Amount @ 20% Repayment
Ashadh end, 2072 100,000 120000 19,999.95 100,000
Ashadh end, 2071 183,333 120000 36,666.62 83,333
Ashadh end, 2070 252,778 120000 50,555.52 69,444
Ashadh end, 2069 310,648 120000 62,129.60 57,870
Samuels would require to pay Rs. 3,10,648 if he buys the car on cash.

Dr. Nelson &Co. Cr.


Ashadh To Bank 120,000 Ashadh By Car A/c 310,648
end, 2069 To Bal c/d 252,778 end, 2069 By Interest A/c 62,130
372,778 372,778
Ashadh To Bank 120,000 Ashadh By Bal b/d 252,778
end, 2070 To Bal c/d 183,333 end, 2070 By Interest A/c 50,556
303,333 303,333
Ashadh To Bank 120,000 Ashadh By Bal b/d 183,333
end, 2071 To Bal c/d 100,000 end, 2071 By Interest A/c 36,667
220,000 220,000
Ashadh To Bank 120,000 Ashadh By Bal b/d 100,000
end, 2072 end, 2072 By Interest A/c 20,000
120,000 120,000

7. Eastern Appliances Pvt. Ltd. commenced the business of selling washing machines both on
cash basis and by Installments. Installment Sales require a deposit of 20% of Cash Selling
Price with the balance payable in 18 Equal Monthly Installments. No additional charge is
made for this serve.
At the end of each financial year, company takes credit for the profit on Installment sales
only in respect of the proportion represented by Deposits and Installments actually received.
The following transactions took place during the first two years of operation:
(Amounts in Rs.)
Particulars Year 1 Year 2
Cash Sales 35,735 48,920
Installments Sales 321,615 440,280
New Washing Machines Purchased 265,360 287,345
Cash collections on Installments contracts :
Initial Deposit 64,323 88,056
Installments - Year 1 Sales 80,403 128,645
Year 2 Sales 0 110,070
Overdue Installments 16,080 22,015
th
Shop Stock at 16 July (at cost) (at close of financial 66,340 74,835
year)

© The Institute of Chartered Accountants of Nepal 90


CAP II Paper 1: Advanced Accounting

You are required to prepare (for two year):


Hire Purchase Trading Account and
General Trading Account
(Inter Dec. 2011 Q3-15 Marks)
Answer
1 Computation of HP Sales (Installments Due during the year)
Particulars Year 1 Year 2
Cash Collections on Installment Contracts :
Cash collected on Initial Deposit 64,323 88,056
Cash collected on Installments due on year 1 Sales 80,403 128,645
Cash collected on Installments due on year 2 sales - 110,070
Closing Installments due and Not Collected 16,080 22,015
160,806 348,786
Less: Opening Installments Due and Not Collected - (16,080)
HP Sales for the year – Credited to HP Trading Account 160,806 332,706

2 Computation of HP Stock (Installments Not Due)


Particulars Year 1 Years 2
Opening Stock - Installments Not Due as on 17st July - 160,809
HP Sales for the Period 321,615 440,280
Less: Installments Due for the period - HP Sales credited to HP
(160,806) (332,706)
Trading Account
Closing Stock – Installments Not Due as on 31st March at HP
160,809 268,383
price

3 Computation of Cost of HP Sales (stock Transferred)


Particular Year 1 Year 2
Opening Stock of Shop Stock - 66,340
Add: Purchase during the year 265,360 287,345
Less : Closing Stock of Shop Stock (66,340) (74,835)
Cost of Goods Sold during the year ( including HP Sales)
199,020 278,850
( A)
HP Sales (B) 321,615 440,280
Cash Sales © 35,735 48,920
Total Sales (D= B+C) 357,350 489,200
Ratio Of HP Sales (E= B ÷D) 90% 90%
Cost of HP Sales (Stock Tfd General Trading A/c [A X E] 179,118 250,965
Notes: In the absence of information, the Total Cost of Goods sold is apportioned between
Installment Sales and Ordinary Sales in the ratio of Sales.
4 Calculation of Cost of Closing HP Stock
Year Computation Rs

© The Institute of Chartered Accountants of Nepal 91


CAP II Paper 1: Advanced Accounting

1 = HP Stock At HP Price X ( Cost of Stock Transferred + Opening HP Stock 89,560


at cost)
(HP Sales + Opening HP Stock at HP Price)
= Rs. 160809 X [(Rs.179118+Rs.Nil)/ (Rs. 321615+ NIL)]
2 = HP Stock at HP Price X ( Cost of Stock Transferred + Opening HP Stock 152,043
at Cost )
(HP Sales + Opening HP Stock at HP Price)
= Rs. 268383 X (Rs. 250965+Rs.89560)/ (Rs.440280+Rs.160809)
5 HP Trading Account for the year ending…………..
Particulars Year 1 Year 2 Particulars Year 1 Year 2
To Opening HP Stock – 89560 BY HP Sales A/c – 160806 332706
Installments not due at (WN 1)
Cost (WN 4 )
To General Trading A/c 179118 250965 By Closing HP 89560 152043
– (Cost Of HP Sales Stock – Installments
)(WN 3) not due at Cost –
(WN 4)
By Profit and Loss A/c 71248 144224
– ( Hp Profit transferred
) bal. figure )
Total 250366 484749 Total 250366 484749

6 General Trading Account for the year ending……………….


Particular year 1 year 2 particular year 1 year 2
To Opening Stock - Shop 0 66340 By Cash Sales - given 35735 48920
Stock – given
To Purchases - given 265360 287345 By HP Trading A/c 179118 250965
(Cost of HP Sales )
(WN 3 )
To P&L A/c (Profit 15833 21035 By Closing Stock - 66340 74835
Transferred) given
Total 281193 374720 total 281193 374720

7 HP Debtors Account (installments due but not collected)

Particular year 1 year 3 particular year 1 year 2


To balance b/d 0 176889 By Bank A/c 0 0
To HP Sales A/C (WN 321615 440280 Initial Deposit - given 64323 88056
1)
Installments Collected 80403 238715
By Balance c/d ( bal. fig.) 176889 290398
Total 321615 617169 total 321615 617169

© The Institute of Chartered Accountants of Nepal 92


CAP II Paper 1: Advanced Accounting

Note: Alternative solution cannot be suggested as the question lacks fundamental


information required for the calculation based on NAS 17.

8. From the following prepare the hire purchase trading account of Bikash Traders who sells
goods on hire purchase basis at cost plus 25 %.
Rs.
Installments not due on 31-12-2005 30, 00,000
Installments due and collected during 2006 80, 00,000
Installments due but not collected during 2006 (Including Rs 100,000 for 5, 00,000
which goods were repossessed)
Installments not due on 31-12-2006 (Including Rs 200,000 for which goods 37, 00,000
were repossessed)
Installments collected on repossessed stock 1, 50,000
Bikash Traders valued repossessed stock at 60 % of original cost.
(Inter JUNE. 2008 4a)
Answer:

Hire Purchase Trading Account

Dr Cr.
To Goods with customers (At 2,400,000 By Hire Purchase sales 8,500,000
cost) (3,000,000*100/125)
To Shop Stock 7,200,000 By Goods with customers 2,800,000
(closing)
To Bad Debts 20,000
To Loss on Repossession 24,000
To Profit & Loss A/c (Transfer 1,656,000
of HP Profit)
Total Rs. 11,300,000 Total Rs. 11,300,000

Working Notes:
1 Hire Purchase Sales
Installments Due and collected 8,000,000
Add: Installments Due but not collected 500,000
8,500,000

2 Loss on repossessed stock


Installments collected 150,000
Installments due 100,000
Installments not due 200,000
HP price of repossessed stock 450,000
Cost of repossessed goods (450,000*100/125) 360,000

© The Institute of Chartered Accountants of Nepal 93


CAP II Paper 1: Advanced Accounting

Valuation of repossessed goods


(360,000*60/100) 216,000
Cost of installments due and not due (100,000
+ 200,000) * 100/125 240,000
Loss on repossessed goods 24,000

3 Goods taken from shop stock at cost


Hire purchase sales at cost
(8,500,000*100/125) 6,800,000
Stock with customers at cost
(3,500,000*100/125) 2,800,000
9,600,000
Stock with customers at cost (opening)
Less: (3,000,000*100/125) 2,400,000
Goods taken from shop stock 7,200,000

4 Bad Debts
Installments due but not collected (at HP
Price) 100,000
Installments not yet due (at cost)
(200,000*100/125) 160,000
260,000
Cost of Installments due and installments not
Less: yet due 240,000
Bad Debts 20,000

Note: Alternative solution cannot be suggested as the question lacks fundamental


information required for the calculation based on NAS 17.

9. Progressive Limited sold out goods on hire – purchase basis at profit of 25% on cost
price. Following details are furnished to you:
Rs.
Stock in godown:
On 01.04.2011 300,000
On 31.03.2012 250,000
Overdue instalments :
On 01.04.2011 20,000
On 31.03.2012 30,000
Goods with customers on hire – purchase on 01.04.2011 360,000
Purchases 646,000
Instalments received 600,000

Prepare in the books of Progressive Limited:


Hire – purchase stock account (double column);

© The Institute of Chartered Accountants of Nepal 94


CAP II Paper 1: Advanced Accounting

Shop stock account and;


Hire – purchase debtors account.
(Inter Jun. 2012 Q5a- 5 Marks)
Answer In the books of Progressive Limited
Hire – Purchase Stock Account
Date Particulars Invoice Cost Date Particulars Invoice Cost
1.4.'11 To Balance b/d 360,000 288,000 31.3.'12 By H.P. Debtors 610,000 610,000
31.3.'12 To Shop Stock 870,000 696,000 By Balance c/d 620,000 496,000
To Gross Profit 122,000 (Bal. figure)
1,230,000 1,106,000 1,230,000 1,106,000
1.4.'12 To Balance b/d 620,000 496,000

Shop Stock Account


Date Particulars Rs. Date Particulars Rs.
1.4.'11 To Balance b/d 300,000 31.3.'12 By H.P. Stock – Bal. fig. 696,000
To Purchases 646,000 By Balance c/d 250,000
946,000 946,000
1.4.'11 To Balance b/d 250,000

Hire – Purchase Debtors Account


Date Particulars Rs. Date Particulars Rs.
1.4.'11 To Balance b/d 20,000 31.3.'12 By Bank – Instalment recvd. 600,000
To H.P. Stock – Bal. fig. 610,000 By Balance c/d 30,000
630,000 630,000
1.4.'12 To Balance b/d 30,000

Note: Alternative solution based cannot be suggested as the question lacks fundamental
information required for the calculation based on NAS 17.

10. On Baishakh 01 2063, Rama & Rama took delivery of a car from Sita & Sita on hire purchase
system, Rs. 105,000 being paid on delivery and the balance in six annual installments of Rs.
60,000 each payable at the end of each year. The Cash Price of the car is Rs. 3,60,000. Calculate
the amount of interest for each year.
(CAP Jun. 2011 Q4A -5 Marks)
Answer
A. Calculation of Implicit rate in the lease
Period Cash Flows
0 (255,000.00)
1 60,000.00
2 60,000.00
3 60,000.00
4 60,000.00
5 60,000.00

© The Institute of Chartered Accountants of Nepal 95


CAP II Paper 1: Advanced Accounting

6 60,000.00

By Calculation, Implicit rate in the lease(IRR) = 10.84%


B. Computation of interest for each year
Interest
Installment Principal Installment Principal
@
# Outstanding Amount Repayment
10.84%
6 54,132 60,000 5,868 54,132
5 102,969 60,000 11,163 48,837
4 147,030 60,000 15,939 44,061
3 186,781 60,000 20,249 39,751
2 222,644 60,000 24,137 35,863
1 255,000 60,000 27,644 32,356

11. Digital traders started business on 1st Shrawan 2068 selling one model of digital cameras on
hire purchase. During the year to 31st Ashadh 2069, it purchased 2,000 cameras at a uniform
price of Rs. 9,000 and sold 1,900 cameras at a total selling price under hire purchase
agreements of Rs. 15,000 per camera, payable by an initial deposit of Rs. 4,500 and quarterly
installments of Rs. 1,050.
The following trial balance is extracted from the firm's books as at 31st Ashadh 2069:
Particulars Dr. (Rs.) Cr. (Rs.)
Capital 13,602,600
Drawings 4,000,000
Non-current Assets 950,000
Purchases 18,000,000
Cash collected from customers 12,540,000
Rent, rates and insurance 500,000
Salaries 2,700,000
General expenses 510,000
Balance at bank 742,600
Accounts payable 1,260,000
27,402,600 27,402,600
The personal accounts of customers are memorandum records (i.e. they are not part of the
double entry system). The firm prepares its financial statements on the basis of taking credit
for profit (including interest) in proportion to cash collected from customers.

Ignore depreciation of Non-current Assets, and


Prepare hire purchase income statement for the year ending 31st Ashadh 2069.
Prepare the Financial Position as at 31st Ashadh 2069.
(CAP Jun. 2013 Q2b– 10 Marks)
Answer
Digital Traders
Income Statement for the year ended 31st Ashadh 2069

© The Institute of Chartered Accountants of Nepal 96


CAP II Paper 1: Advanced Accounting

Particulars Rs. Rs.


Sales (1900 x Rs.15,000) 28,500,000
Less: Cost of sales
Purchases (2000 x Rs.9,000) 18,000,000
Less closing stock (100 x Rs.9,000) (900,000) (17,100,000)
Provision for unrealized profit & interest (WN 1) (6,384,000)
Gross Profit 5,016,000
Less: Other expenses
Rent, rates and insurance 500,000
Salaries 2,700,000
General expenses 510,000 (3,710,000)
Net profit 1,306,000

Digital Traders
Statement of Financial Position as at 31st Ashadh 2069
Capital & Liabilities Rs. Rs. Rs.
Capital 13,602,600
Add: Net profit for the year 1,306,000
Less: Drawing (4,000,000)
Total 10,908,600
Assets
Fixed Assets 950,000
Current Assets:
Closing stock 900,000
HP accounts receivable 15,960,000
Less: Provision for unrealized profit & interest (6,384,000) 9,576,000
Balance at bank 742,600
11,218,600
Less: Current Liabilities
Accounts payable (1,260,000)
9,958,600
Total 10,908,600

WN 1) Calculation of amount of unrealized profit


Rs.
Sales amount 28,500,000
Less: Cash collected from customers 12,540,000
Cash yet to be collected 15,960,000
Profit & interest per unit of camera = Rs. 15,000 –Rs. 9,000 = Rs.6,000

Unrealized Profit and interest = (15,960,000/28,500,000) x Rs. 6,000 x 1900


= Rs. 6,384,000

© The Institute of Chartered Accountants of Nepal 97


CAP II Paper 1: Advanced Accounting

Note: Alternative solution based cannot be suggested as the question lacks fundamental
information required for the calculation based on NAS 17.

12. X Limited sells products on hire purchase terms, the price being cost plus 33 1/3%. From the
following particulars for the year ended 31stAshadh 2071, prepare the necessary accounts on
stock-debtors system to reveal the profit earned:
2070 1st Shrawan
Stock out on hire at Hire purchase price 400,000
Stock in hand at the shop 50,000
Installments due (Customers still paying) 30,000
st
2071 31 Ashadh
Stock out on hire at Hire Purchase price 460,000
Stock in hand at the shop 70,000
Installments due (Customers still paying) 50,000
Cash received during the year 800,000
(CAP Jun. 2016 Q4a- 10 Marks)
Answer

Hire Purchase Debtors Account


Date Particulars Amount Date Particulars Amount
2070 2070
01/04 To balance b/d 30,000 01/04 By Cash 8,00,000
01/04 To Hire Purchase to By Balance c/d 50,000
to stock account –total 21/03
31/03/ number of 8.20,000 2071
2071 installments which
become due
(balancing figure 1)
8,50,000 8,50,000

Hire Purchase Stock Account


Date Particulars Amount Date Particulars Amount
2070 2070
01/04 To balance b/d 4,00,000 01/04 By Hire Purchase 8,20,000
01/04 To goods sold on hire to Debtors A/c (1)
to purchase (balancing figure 2) 8,80,000 31/03
31/03/ 2071
2071 31/03 4,60,000
By Balance c/d
12,80,000 12,80,000

Hire Purchase Adjustment Account


Date Particulars Amount Date Particulars Amount

© The Institute of Chartered Accountants of Nepal 98


CAP II Paper 1: Advanced Accounting

2071 2070
31/03 To Hire purchase 1,15,000 01/04 By Hire Purchase Stock 1,00,000
Stock Reserve A/c Reserve A/c (opening balance)
2071
31/03 To Profit and Loss A/c 2,05,000 31/03 By Goods sold on hire 2,20,000
(transfer of profit) purchase A/c (Loading 25% of
Rs.8,80,000)

3,20,000 3,20,000

Note: Alternative solution based cannot be suggested as the question lacks fundamental
information required for the calculation based on NAS 17.

13. Fantastic Home Ltd. commenced business on 1st January, 2016. The business is to sell Radio
Recorders and Televisions both in hire -purchase and cash basis. The information is given
below:-
Radio Recorder Television
Set
Cost Price (Rs.) 6,000 18,000
Sale Price (Cash) (Rs.) 7,800 19,500
Down payment for H.P. (Rs.) 600 3,000
Monthly instalments for H. P. (Rs.) 360 660
No. of instalments 20 25
The company purchased goods costing Rs. 24,00,000 in all and made cash sales totaling Rs.
15,00,000. Stock on hand was valued at Rs. 3,90,000. Hire Purchase transactions were as
follows:
Number Instalments Instalments-due
Sold Collected (customer’s Paying)
Radio Recorders 30 300 15
Television Sets 25 250 10
4 Radio Recorders and 3 Television sets on which only 10 monthly instalments were
collected were repossessed and were valued at Rs. 45,000. This is not included in the stock
mentioned above.
Prepare hire purchase trading account to show the profit or loss made by the hire vendor
company.
(CAP Jun. 2017 Q2b-10 Marks)
Answer
Hire Purchase Trading Account
Rs. Rs.

To Goods Sold on H.P. (1) 7,21,500 By Cash 3,66,000


By Goods repossessed 45,000
By Instalments due 12,000

© The Institute of Chartered Accountants of Nepal 99


CAP II Paper 1: Advanced Accounting

To Stock reserve on 36,600 By Goods sold an H.P. Loading 91,500


instalments not due (4)
By Instalments not yet
55,800 due-stock with customers c/d 29,9,400
Profit and Loss Account 813,900 (3) 813,900
(Profit)

Working Notes:
1. Cost Price:
Radio Recorders 1,80,000 (30 × 6,000) 234,000 (30 × 7,800)
Television sets 4,50,000 (25 × 18000) 487,500 (25 × 19,500)
Total 6,30,000 721,500
Loading (Margin) = 7,21,500 – 6,30,000 = Rs. 91,500

2. Cash Collected Radio Reorders Television sets


Rs. Rs
Down Payment 30 × 600 = 18,000 25 × 3000 = 75,000
Instalments 300 × 360=1,08,000 250 × 660 = 1,65,000
1,26,000 2,40,000
Total 126,000 + 240,000 = = 366,000

3. Instalments not due:

Radio Recorders:
Total Instalments on 26 sets = 26 × 20 520
Less: Instalments collected and due (315 – 40) 275
Not yet due 245
Amount 245 × 360 Rs. 88,200

Television Sets
Total Instalments on 22 sets = 22 × 25 550
Less: Collected and due (260-30) 230
Not yet due 320
Amount 320 × 660 = Rs. 2,11,200
Total Installments not yet due = Rs. 88,200 + Rs. 211,200 = Rs. 299,400

4. Stock Reserve
Radio Recorders Television Sets
Total Amount due 7,800 19,500
H.P.P.
Cost 6,000 18,000
1,800 1,500

© The Institute of Chartered Accountants of Nepal 100


CAP II Paper 1: Advanced Accounting

Reserve Required 1,800/7,800 X 88,200 = Rs. 1,500/19,500X 2,11,200 = Rs.


20,354 16,246
Total = Rs. 2,0354 + Rs. 16,246 = Rs. 36,600

Alternative Solution
No Hire Purchase Trading account is prepared under NFRS. Sales is recognized at
normal selling price and interest is recognized when each payment is due by computing
implicit rate in the lease.
In the question, the Hire purchase price and cash price are equal for both Radio
Recorders and Television sets (which would not be the case in actual life) and therefore
the implicit rate in the lease is 0%. Sales profit is recognized at the time of sales itself as
HP transaction is finance lease.

Accounting entries will be as follows:


Particulars Amount
Hire Purchase Debtors Dr. (30 × 20 ×360 + 25 × 25 × 660) 628,500
Bank Dr. (30 × 600 + 25 × 3000) 93,000
Cr. Sales (30 × 7,800 + 25 × 19,500 ) 721,500
(Being Hire Purchase Sales Recognized)

Cost of Sales Dr. 6,30,000


Inventory Cr. 6,30,000
(Being Hire Purchase Sales Recognized)

Bank Dr. 273,000


HP Debtors Cr. 273,000
(Being installments collected from debtors)
(300 × 360 + 250 × 660)
Inventory Dr. 45,000
HP Debtors Cr. 44,100
Gain on Repossession Cr. 900
(Goods Repossessed, valued at stated amount and HP Debtors for installments
not collected credited.)
(Recorders = 4 units × 10 unpaid installments × 360 per installment
(TV = 3 Units × 15 Unpaid Installments × 660 per installment)

14. On January 1, 2003, Manakamana Stores Ltd. Sold on hire purchase an electric appliance
washing machine, cash price of which is Rs. 36,000 and the rate of interest charged from the
hire-vendor Vijay is 10% p.a.
Show the journal entries and ledger entries for the following cases in the books of the hire
vendor:

© The Institute of Chartered Accountants of Nepal 101


CAP II Paper 1: Advanced Accounting

(i) Cash down is Rs. 11,750 and the balance is annually paid in 3 equal installments of Rs.
9,750 each;
(ii) Cash down is Rs. 11,750 and the balance is annually paid in 3 unequal installments of
Rs. 10,000, Rs. 9,800 and Rs. 9,400 respectively, at the end of the first, second and
third years.
(Inter Jun. 2003, Q 6b- 8 Marks)
Answer:
Case I:
Cash down Rs. 11,750 and the balance is annually paid in equal instalments of Rs. 9,750 each:
Journal Entries in Manakamna Stores Ltd.'s Books
Particulars January December 31
1
2003 2003 2004 2005
Vijay A/C Dr. 24,250 - - -
Cash A/c Dr. 11,750 - - -
To Sales A/c 36,000 - - -
Car sold on Hire purchase
recorded
Vijay A/C Dr. - 2,425 1,692 883
To Interest Income - 2,425 1,692 883
Interest due on Booked -

Cash A/c Dr. - 9,750 9,750 9,750


To Vijay A/c - 9,750 9,750 9,750
Receipt of year-end -
installment.

Ledger Accounts in Manakamna Stores Ltd 's Books


Vijay's A/c
1.1.03 To Hire Sales 36,000 1.1.03 By Cash 11,750
31.12.03 To Interest 2,425 31.12.03 By Cash 9,750
31.12.03 Balance c/d 16,925
38,425 38,425
1.1.04 To Balance c/d 16,925 31.12.04 By Cash 9,750
31.12.04 To Interest 1,692 31.12.04 Balance c/d 8,867
18,617 18,617
1.1.05 To Balance c/d 8,867 31.12.05 By Cash 9,750
31.12.05 To Interest (Bal. 883
Fig.)
9,750 9,750

Sales A/c
1.1.03 By Vijay 36,000

© The Institute of Chartered Accountants of Nepal 102


CAP II Paper 1: Advanced Accounting

Interest Account
31.12.03 By Vijay 2,425
31.12.04 By Vijay 1,692
31.12.05 By Vijay 883

Case II:
Initial cash down of Rs. 11,750 and the balance in 3 unequal installments.
Journal Entries in Manakamna Stores Ltd.'s Books
Particulars January December 31
1
2003 2003 2004 2005
Vijay A/C Dr. 24,250 - - -
Cash A/c Dr. 11,750 - - -
To Sales A/c 36,000 - - -
Car sold on Hire purchase
recorded
Vijay A/C Dr. - 2,425 1,667 858
To Interest Income - 2,425 1,667 858
Interest due on Booked -

Cash A/c Dr. - 10,000 9,800 9,400


To Vijay A/c - 10,000 9,800 9,400
Receipt of year-end -
installment.

Vijay's A/c
1.1.03 To Hire Sales 36,000 1.1.03 By Cash 11,750
31.12.03 To Interest 2,425 31.12.03 By Cash 10,000
31.12.03 Balance c/d 16,675
38,425 38,425
1.1.04 To Balance c/d 16,675 31.12.04 By Cash 9,800
31.12.04 To Interest 1,667 31.12.04 Balance c/d 8,542
18,342 18,342
1.1.05 To Balance c/d 8,542 31.12.05 By Cash 9,400
31.12.05 To Interest (Bal. 858
Fig.)
9,400 9,400

Interest Account
2,425 31.12.03 By Vijay 2,425
1,667 31.12.04 By Vijay 1,667
858 31.12.05 By Vijay 858

© The Institute of Chartered Accountants of Nepal 103


CAP II Paper 1: Advanced Accounting

15. The following information relating to Mysales P. Ltd. are given to you. Prepare the Hire
Purchase Trading Account and the General Trading Account for the year ended on March 31,
2004.
Total purchases for the year amounted to Rs. 90,000. General sales totaled Rs. 85,000 and stock
at the close amounted to Rs. 15,000. Two of the items were sold on hire purchase terms, of
which the details are given here:
Cost Down No. of further Total installments falling
payment installments due in 2003-04 (excluding
per unit per unit down payments)
Item 1: 30 X Rs. 150 Rs. 30 6 of Rs. 30 50
Item 2: 20 X Rs. 400 Rs. 80 8 of Rs. 80 30
The installments due during the year were totally received.
(Inter Jun. 2004 Q 2b -14 Marks)

Answer:
The question is outdated as both Hire purchase trading account or General Trading
account does not exist. Original answers have not been altered as the answer is correct as
per these methods.
In the books of Mysales Private Limited
Hire Purchase Trading Account
Dr. Cr.
Date Particulars Rs. Date Particulars Rs.
31.3.2004 To Goods sold on H.P. 20,700 31.3.2004 By Goods sold on
A/c (Note 1) H.P. A/c (Note 4) 8,200
To Stock Reserve A/c By Cash A/c (Note 6,400
(Note 7) 5,736 5) 14,300
To Profit & Loss A/c 2,464 By Balance C/d
(Hire Purchase
Stock - Note 6)
28,900 28,900

General Trading Account for the year ended on 31st March, 2004

Particulars Rs. Particulars Rs.


To Purchase A/c 90,000 By Sales A/c 85,000
To Gross Profit C/d 22,500 By Goods sold on Hire Purchase A/c
(Note 8) 12,500
By Closing Stock 15,000
112,500 112,500

Working Notes:
1. Details of sales 2003-04
Item No. 1: (6  30 + Rs. 30)  30 = Rs. 6,300

© The Institute of Chartered Accountants of Nepal 104


CAP II Paper 1: Advanced Accounting

Item No. 2 : (8  80 + Rs. 80)  20 = Rs. 14,400


= Rs. 20,700
2. Profit on Item No. 1: (Rs. 6,300 - 30  Rs. 150) = Rs. 1,800. Therefore, profit
included in sales = Rs. 1,800/Rs. 6,300 = 2/7
3. Profit on Item No. 2: (Rs. 14,400 - 20  Rs. 400) = Rs. 6,400. Therefore, profit
included in sales = Rs. 6,400/Rs. 14,400 = 4/9
4. Total Profit = Rs. 1,800 + Rs. 6,400 = Rs. 8,200
5. Cash Received:
Item No. 1: Cash down (30  Rs. 30) = 900
Installments (50  Rs. 30) = 1500 2,400
Item No. 2: Cash down (20  Rs. 80) = 1600
Installments (30  Rs. 80) = 2400 4,000
6,400
6. Installments not yet due:
Item No. 1: (30  6-50)  Rs. 30 = 3900
Item No. 2: (20  8 - 30)  Rs. 80 = 10,400 14,300
7. Stock Reserve:
Item No. 1: 2/7 of Rs. 3,900 = 1114
Item No. 2: 4/9 of Rs. 10,400 = 4622 5,736
8. Goods sold on H.P Account
Date Particulars Rs. Date Particulars Rs.
2003- To HP Trading A/c 8,200 2003- By Hire Purchase
04 To general Trading A/c 12,500 04 Trading A/c 20,700
(Bal.)
20,700 20,700

16. Mr. India sells goods on hire purchase basis. He fixes hire purchase price by adding
50 % to the cost of the goods. The following are the figures relating to his hire purchase
business for the year ending on 31st March 2002.

01.04.2001 31.03.2002
Rs. Rs.
Hire Purchase Stock 12,000 ?
Hire Purchase Debtors. 300 ?
Shop Stock 10,000 15,000

Goods purchase during the year Rs. 65,400. Cash received from customer during the year Rs.
92,400, Total amount of installment that fell due during the year Rs. 92,700. One customer to
whom goods had been sold for Rs. 1,200 paid only 5 installments of Rs. 100 each. On his
failure to pay the monthly installment of Rs. 100 each on 4th March 2002. the goods were
repossessed on 27th March 2002 after due legal notice.
Required : Prepare the Hire Purchase trading account.

© The Institute of Chartered Accountants of Nepal 105


CAP II Paper 1: Advanced Accounting

(Inter Dec. 2004 Q 3a)

Answer:
The question is outdated as both Hire purchase trading account or General Trading
account does not exist. Original answers have not been altered as the answer is correct as
per these methods.
Hire Purchase Trading Account
Dr. Cr
To opening balances By Hire Purchase Stock Reserve 4,000
Hire purchase stock 12,000 [Rs.12,000 x 50/150]
Hire purchase Debtors 300 By Bank A/c 92,400
To Good Sold on Hire Purchase 90,600 By Goods sold on hire purchase
A/c [Rs. 90,600 x 50/150] 30,200
To Hire purchase Stock Reserve By Goods Repossessed A/c 467
A\c [9,300 x 50/150] 3,100 [At Revaluated figure}]
To Profit t/f to General P& L By Closing Balances :
A/c 30,867 Hire purchase stock 9,300
Hire purchase debtors 500

1,36,867 1,36,867

Working Notes:

(i) Shop Stock Account


Dr. Cr.
Particulars Rs Particulars Rs
To Balance b\d 10,000 By Goods Sold on Hire
To Purchases 65,400 Purchases A/c 60,400
By Balance c/d 15,000
75,400 [Excluding Goods Repossessed] 75,400

(ii) Goods sold on Hire Purchase Account


Dr. Cr.
Particulars Rs Particulars Rs

To shop stock A/c 60,400 By Hire Purchase Trading A/c 90,600


To Hire Purchase
Trading A\c 30,200

90,600 90,600

© The Institute of Chartered Accountants of Nepal 106


CAP II Paper 1: Advanced Accounting

(iii) Memorandum Hire Purchase Stock Account


Dr. Cr.
Particulars Rs Particulars Rs
To Balance b/d 12,000 By Hire Purchase Debtors 92,700
To Goods sold on By Goods Repossessed A/c 600
Hire purchase 90,600 By Balance c/d (b.f) 9,300

1,02,600 1,02,600

(iv) Memorandum Hire Purchase Debtors Account


Dr. Cr.
Particulars Rs Particulars Rs
To balance b/d 300 By Bank A/c 92,400
To Hire Purchase Stock A\c 92,700 By Goods Repossessed A/c 100
By Balance c/d (b.f) 500

93,000 93,000

Working Notes:
Calculation of the value of Goods Repossessed Value of Goods Repossessed.
= Cost Price x Unpaid Amount (Whether due or not)
H.P Price
= Rs 800 x Rs.700 = Rs.467
Rs.1,200

17. Kathmandu Television Emporium sells goods both on cash and hire- purchase basis
and records hire-purchase transactions on ‘stock and Debtors’ system and closes its books on
December 31, every year.
On 1.4.2004, it sold a color TV set and DVD set to Rajkumar, other particulars are as follow:
-
Items TV Set DVD
Cost Price Rs.9, 000 Rs.16,000
Down Payment Rs.2, 000 Rs.4,000
Number of Installments Payable 10 8
Amount in each installment Rs.1, 000 Rs.2,000
Mode of Payment Monthly Once in two months
First Installment Due on 1-5-2004 1-6-2004
Rajkumar paid all the installments due except for those due on December 1, 2004. it was
decided that Kathmandu Television Emporium will take back DVD at an agreed price of Rs.11,
000 and excess amount, if any, will be adjusted against the installment due on TV set.
DVD repossessed was sold for Rs.12,000 after repair , charges for repairing was Rs.500 prepare
necessary ledger accounts to record the above transactions and find out the profit, if any.

© The Institute of Chartered Accountants of Nepal 107


CAP II Paper 1: Advanced Accounting

(June 2005, Q 3a)


Answer:
Assumed that Cash Sales price of TV Set is Rs. 10,000 and DVD is 18,000
Dr. Rajkumar's A/C Cr.
1.04.2004 To Sales 22,000 1.05.2004 By Cash 1,000
To Interest
1.05.2004 Income 342 1.06.2004 By Cash 3,000
To Interest
1.06.2004 Income 743 1.07.2004 By Cash 1,000
To Interest
1.07.2004 Income 285 1.08.2004 By Cash 3,000
To Interest
1.08.2004 Income 635 1.09.2004 By Cash 1,000
To Interest
1.09.2004 Income 222 1.10.2004 By Cash 3,000
To Interest
1.10.2004 Income 520 1.11.2004 By Cash 1,000
To Interest By
1.11.2004 Income 154 1.12.2004 Inventory 11,000
To Interest
1.12.2004 Income 398 1.12.2004 By Bal c/d 1,301
Total 25,301 Total 25,301

Dr. Inventory A/C Cr.


By Cost of
1.04.2004 To Bal b/d XXX 1.04.2004 Sales 25,000
To Rajkumar's By Cost of
1.12.2004 A/C 11,000 1.12.2004 Sales 11,500
1.12.2004 To Cash A/C 500 1.12.2004 By Bal c/d XXX

Dr. Cost of Sales A/C Cr.


1.04.2004 To Inventory 25,000
1.12.2004 To Inventory 11,500

Dr. Interest Income A/C Cr.


1.05.2004 By Rajkumar's A/c 342
1.06.2004 By Rajkumar's A/c 743
1.07.2004 By Rajkumar's A/c 285
1.08.2004 By Rajkumar's A/c 635
1.09.2004 By Rajkumar's A/c 222
1.10.2004 By Rajkumar's A/c 520
1.11.2004 By Rajkumar's A/c 154

© The Institute of Chartered Accountants of Nepal 108


CAP II Paper 1: Advanced Accounting

1.12.2004 By Rajkumar's A/c 398


Working Notes:
1. Computation of implicit rate in the lease
Installment
# TV Set DVD
0 (8,000) (14,000)
1 1,000 2,000
2 1,000 2,000
3 1,000 2,000
4 1,000 2,000
5 1,000 2,000
6 1,000 2,000
7 1,000 2,000
8 1,000 2,000
9 1,000 -
10 1,000 -
4.28% 3.07%
2. Amortization Schedule
TV Set
Installment Installment Principal Installment Installment Principal
Interest Interest
# Amount Repayment # Amount Repayment
10 1,000 41 959 5 1,000 222 778
9 1,000 80 920 4 1,000 254 746
8 1,000 118 882 3 1,000 285 715
7 1,000 154 846 2 1,000 314 686
6 1,000 189 811 1 1,000 342 658

DVD Player
Installment Installment Principal Installment Installment Principal
Interest Interest
# Amount Repayment # Amount Repayment
8 2,000 60 1,940 4 2,000 280 1,720
7 2,000 117 1,883 3 2,000 332 1,668
6 2,000 173 1,827 2 2,000 381 1,619
5 2,000 228 1,772 1 2,000 429 1,571

Journal Entries
1.04.2004 TV Set DVD Total
Rajkumar A/C Dr 8,000 14,000 22,000
Cash Dr. 2000 4000 6,000
Sales Cr. 10,000 18,000 28,000
(Being Sales Recognized)
Cost of Sales A/C Dr. 9,000 16000 25,000

© The Institute of Chartered Accountants of Nepal 109


CAP II Paper 1: Advanced Accounting

Inventory Cr. 9,000 16,000 25,000


(Being Inventory sold recognized as cost)

May
Particulars 1 June 1 July 1 Aug 1 Sep. 1 Oct. 1 Nov. 1 Dec. 1
Rajkumar A/C Dr 342 743 285 635 222 520 154 398
Interest Income Cr. 342 743 285 635 222 520 154 398
(Being Interest Recognized)
Cash Dr. 1,000 3000 1000 3000 1000 3000 1000 -
Rajkumar A/C Cr. 1,000 3,000 1,000 3,000 1,000 3,000 1,000 -
(Being Payments received from Rajkumar)
Inventory A/C Dr. 11,000
Rajkumar A/C Cr. 11,000
(Being Goods Repossessed at agreed value)
Inventory A/C Dr. 500
Cash A/C Cr. 500
(Being Expenses incurred on repair)
Cash A/C Dr. 12,000
Sales A/C Cr. 12,000
(Being Repossessed items sold)
Cost of Sales Dr. 11,500
Inventory Cr 11,500
(Being Inventory sold recognized as cost)

2.2 Contract Accounting


(Including NAS 13)

A. Theoretical Question

1. Escalation Clause in Contracts (CAP Jun. 2011 Q6b)


This is a clause provided in the contracts to cover up any changes in the price of the contract
due to changes in the price of raw materials and labor or change in utilization of factor of
production. The object of this clause is to safeguard the interest on both sides against
unfavorable change in prices. The basis, on which the factor prices are based, is laid down in
the contract. For e.g.: in a contract with transport undertaking, it may be provided that the price
per ton km will increase or decrease for each rise or fall in price of diesel by 10% of the
prevailing prices.

2. Calculation of Profit from the Notional profit


(CAP Jun. 2018 Q6e-3 Marks)
If the contract account shows a loss, such loss must be fully provided. However, when the
contract account discloses a profit, all such profit cannot be deemed as profit since, in a

© The Institute of Chartered Accountants of Nepal 110


CAP II Paper 1: Advanced Accounting

subsequent accounting period there may be escalation of costs and such other contingencies.
Therefore, only a proportion of the notional profit is deemed to be profit.
As per the provision of NAS 11, profit to be recognized during a particular year will be
calculated as follows:
Total Contract Revenue xxx
Total Estimated Cost xxx
Total Estimated or Notional Profit xxx

Profit to be recognized during a particular year= Total Estimated Profit × Stage of


completion- Profit recognized up to previous year
Where, Stage of completion can be calculated by any of the following method
i. the proportion of contract costs incurred for work performed to date bear to the estimated
total contract costs;
ii. surveys of work performed; or
iii. completion of a physical proportion of the contract work

3. How could you measure the contract revenue with reference to NAS 13? Explain briefly.
(Inter Dec. 2006 Q 3b-5 Marks; CAP Jun. 2010 Q6b – 2.5 Marks)
As per NAS-13 Contract revenue should comprise of:
(a) the initial amount of revenue agreed in the contract; and
(b) Variations in contract work, claims and incentive payments:
(i) to the extent that it is probable that they will result in revenue; and
(ii) they are capable of being reliably measured.
Contract revenue is measured at the fair value of the consideration received or receivable.
The measurement of contract revenue is affected by a variety of uncertainties that depend
on the outcome of future events and often need to be revised as events occur and
uncertainties are resolved. Therefore, the amount of contract revenue may increase or
decrease from one period to the next. For example:
(a) a contractor and a customer may agree variations or claims that increase or decrease
contract revenue in a period subsequent to that in which the contract was initially
agreed;
(b) the amount of revenue agreed in a fixed price contract may increase as a result of cost
escalation clause;
(c) the amount of revenue may decrease as a result of penalties arising from delays caused
by the contractor in the completion of the contract; or
(d) when a fixed price contract involves a fixed price per unit of output, contract revenue
increases or decreases as the number of units is increased or decreased.
Similarly, a variation is an instruction by the customer for a change in the scope of the work
to be performed under the contract. A variation may lead to an increase or a decrease in
contract revenue. Examples of variations are changes in the specifications or design of the
asset and changes in the duration of the contract. A variation is included in contract revenue
when:
(a) it is possible that the customer will approve the variation and the amount of revenue
arising from the variation; and

© The Institute of Chartered Accountants of Nepal 111


CAP II Paper 1: Advanced Accounting

(b) the amount of revenue can be reliably measured.


Furthermore, incentive payments are additional amounts paid to the contractor if specified
performance standards are met or exceeded. For example, a contract may allow for an
incentive payment to the contractor for early completion of the contract. Incentive
payments are included in contract revenue when the contract is sufficiently advanced and
near to completion that it is probable that the specified performance standards will be met
or exceeded; and the amount of the incentive payment can be measured reliably.

4. X, a construction contractor, undertakes the construction of an industrial complex. He has


separate proposals raised for each unit to be constructed in the industrial complex. Since each
unit was subject to separate negotiation, he was able to identify the costs and revenues
attributable to each unit. Should X treat construction of each unit as a separate construction
contract? (CAP Dec. 2010 Q5a-5 Marks)

Answer
The provision of Nepal Accounting Standard on “Construction Contracts” Para 8 and 9
explains the situations where accounting segmentation and combination of construction
contracts can be applied. As per para 8 of the standard, when separate proposals have been
submitted, and separate negotiations have been concluded, costs and revenues identified
separately, the contract for construction of a number of assets shall be treated as separate
construction contracts. Therefore, X has to treat construction of each unit as a separate
construction contract.

B. Practical Questions
Though specifically required by some of the questions, Contract Account does not form
part of double entry accounting system. Nevertheless, control accounts may be useful to
confirm the overall accuracy of accounting entries relating to construction contracts &
therefore, the requirements have been changed to Contract (Control) Account wherever
the question requires preparation of Contract Account.

1. Mr. Gee started work on 1st January 2011 on a contract for Rs. 500,000. On 31st March 2011
when he prepared his financial statements, the following information relating to the contract
was extracted from his books of account:
Particulars Rs.
Materials issued from stores and sent to site 160,000
Wages Paid 101,200
st
Wages outstanding on 31 March 2011 37,520
st
New machine purchased and sent to site on 1 January 2011 148,000
Direct charges paid 7,500
st
Direct charges outstanding on 31 March 2011 600
Establishment charges apportioned to contract 6,400

© The Institute of Chartered Accountants of Nepal 112


CAP II Paper 1: Advanced Accounting

On 31st March 2011 materials lying unused at site were valued at Rs. 21,620. Machines were
depreciated @ 20% per annum. Value of work certified by 31st March 2011 was Rs. 350,000
while the cost of work done but not yet certified as on that date was Rs. 18,000. On the basis
of architect’s certificate Mr. Gee had received a total sum of Rs. 280,000 from the client till
31st March 2011.
You are requested to prepare contract account in Mr. Gee’s ledger with appropriate working
notes.
(CAP Dec. 2012 Q 3b (i) -7 Marks; CAP Dec. 2013 Q4b-8 Marks; CAP Dec. 2016 Q4a-10
Marks)

Answer
Contract Control Account
Particulars Rs. Particulars Rs.
Revenue Recognized 350,000 Cost Recognized 281,000
Cost Incurred 299,000 Contract Work in Progress 18,000
Trade receivable (Billed less
received; Assumed billed amount = 20,000
300,000)
Amount received 280,000
Amount due from customer 50,000
Total 649,000 Total 649,000

Mr. Gee
Statement of Profit or Loss Extracts
Contract Revenue (WN 2) 350,000
Contract Cost (WN 2) 281,000
Profit 69,000

Mr. Gee
Statement of Financial Position Extracts
Current Assets
Trade Receivables 20,000
Amount Due from Customer 50,000
Contract WIP 18,000
Material at site 21,620
Reserves
Retained Earnings (profit for the period) 69,000
Current Liabilities
Outstanding Expenses (37,520+ 600) 38,120

Working Notes:
i) Calculation of written down value of machines on 31st March 2011:
Cost of Machine on 1st January 2011 148,000

© The Institute of Chartered Accountants of Nepal 113


CAP II Paper 1: Advanced Accounting

Depreciation on NPR 148,000 @20% p.a. for 3 months (NPR 148,000 x 7,400
20/100 x 3/12)

ii) Revenue & Costs to be recognized (Notes)


The question is silent about any of the methods for computation of stage of completion and
therefore the value of work certified is considered in computation of the stage of completion.
Percentage of completion = Work Certified ÷ Contract Value = 350,000 ÷ 500,000 = 70%
Material (Less: Closing Material at site) 138,380
Wages (Add outstanding) 138,720
Depreciation on machine 7,400
Direct Charges (Add outstanding) 8,100
Establishment Charges (assumed attributable) 6,400
Total Cost Incurred 299,000
Less: Contract Work in progress (Uncertified works) 18,000
Cost Recognized 281,000
Revenue Recognized = Contract Price × Stage of Completion 350,000

2. Kalika Constructions undertook the construction of a highway on 01.04.2073. The contract


was to be completed in 2 years. The contract price was estimated at Rs. 150 crores. Up to
31.03.2074 the company incurred Rs. 120 crores on the construction. The engineers involved
in the project estimated that a further Rs. 45 crores would be incurred for completing the work.

What amount should recognize as revenue for the year 2073-2074 as per the provision of Nepal
Accounting Standard 11 “Construction Contract"? Show the extract of the Statement of Profit
or Loss of Kalika Constructions.
(CAP Dec. 2017 Q5c

Statement showing the amount of Revenue as per NAS 11


Particulars Rs. in Crores
Cost of construction incurred up to 31.03.2074 120
Add: Estimated future Cost 45
Total estimated cost of construction 165
Degree of completion (120/165×100) 72.73%
Revenue recognized (A = 72.73% of 150) 109 ( approx.)
Cost Incurred (B = A-C) 124
Loss (100% of expected loss) (15)

Statement of profit or loss (Extract)


Particulars Rs. in crores
Contract Revenue 109
Contract Costs 124
Gross Loss (15)

© The Institute of Chartered Accountants of Nepal 114


CAP II Paper 1: Advanced Accounting

3. The following was the expenditure on a contract for Rs. 6,00,000 commenced in February,
2011:
Materials Rs. 1,20,000; Wages Rs. 1,64,400; Plant Rs. 20,000; Business Charges Rs. 8,600.
Cash received on account by 31st December, 2011 amounted to Rs. 2,40,000 being 80% of
work certified; the value of materials on hand at 31st December, 2011 was Rs. 10,000.
Prepare the Contract Control Account for 2011 showing the amount of profit to be recognized
for the year. Plant is to be depreciated at 10%.
(CAP Jun. 2012 Q4b-5 Marks)
Answer
Contract Control Account
Particulars Rs. Particulars Rs.
Revenue Recognized 300,000 Cost Recognized 285,000
Cost Incurred 285,000 Contract Work in Progress -
Trade receivable (Billed less received; -
Assumed billed amount = 240,000)
Amount received 240,000
Amount due from customer (Yet to be
60,000
billed)
Total 585,000 Total 585,000

Working Note:
Material (Less: Closing Material at site) 110,000
Wages 164,400
Depreciation on machine 2,000
Direct Charges 8,600
Total Cost Incurred 285,000
Less: Contract Work in progress (Uncertified works) -
Cost Recognized 285,000
Revenue Recognized = Contract Price × Stage of Completion 300,000
Stage of completion = Value of works certified ÷ Value of the contract
= 240,000÷0.80÷600,000 = 50%

4. An amount of Rs. 3,465,000 was incurred on a contract work up to 31.03.2070. Certificates


have been received to date to the value of Rs. 4,200,000 against which Rs. 3,780,000 has been
received in cash. The cost of work done but not certified amounting to Rs. 78,750. It is
estimated that by spending an additional amount of Rs. 210,000 (including provision for
contingencies) the work can be completed in all respects in another two months. The agreed
contract price of work is Rs. 4,375,000. Compute the estimate of the revenue and cost to be
recognized in the statement of profit or loss as per NAS 11.
(CAP Jun. 2014 Q5a-5 Marks)
Answer:
According NAS 11 ‘Construction Contract’, when the outcomes of a construction contract
can be estimated reliably, contract revenue and contract costs associated with the construction

© The Institute of Chartered Accountants of Nepal 115


CAP II Paper 1: Advanced Accounting

contract should be recognized as revenue and expenses respectively by reference to state of


completion of the contract activity at the reporting date. Amount of cost and revenue and the
corresponding profit to be recognized is shown as follows:

Particulars Rs.
Expenditure incurred up to 31.3.2070 (Assumed that this cost includes cost
of uncertified works as well) (A) 3,465,000
Estimated additional expenses (including contingency) (B) 210,000
Total Estimated Cost (C = A + B) 3,675,000
Stage of completion (D = C ÷A) 94.29%
Contract Price (E) 4,375,000
Revenue to be recognized (F = E ×D) 4,125,000
Cost to be recognized (G = C ×D) 3,465,000
Profit for the period (F – G) 660,000

5. Orchid Ltd. undertook a contract for Rs. 500,000 on 1st Magh 2069. On 30th Paush 2070, when
the accounts were closed, the following details about the contract were gathered:
Rs.
Materials purchased 1,00,000
Wages paid 45,000
General expenses 10,000
Plant purchased 50,000
Material on hand 30.9.2070 25,000
Wages accrued 30.9.2070 5,000
Work certified 2,00,000
Cash received 1,50,000
Work uncertified 15,000
Depreciation on plant 5,000
The above contract contained an escalation clause which read as follows:
“In the event of price of materials and rates of wages increased by more than 5%, the contract
price will be increased accordingly by 25% of the rise in the cost of materials and wages beyond
5% in each case.”

It was found that since the date of signing the agreement, the price of materials and wages rates
increased by 25%. The value of the work certified does not take into account the effect of the
above clause. Prepare contract control account.
(CAP Jun. 2017 Q4a -12 Marks)
Answer
Particulars Rs. Particulars Rs.
Revenue Recognized 200,000 Cost Recognized 125,000
Cost Incurred 140,000 Contract Work in Progress 15,000

© The Institute of Chartered Accountants of Nepal 116


CAP II Paper 1: Advanced Accounting

Trade receivable (Billed less


-
received; assumed billed = received)
Amount received 150,000
Amount due from customer
50,000
(Revenue less billed)
Total 340,000 Total 340,000

Working Notes:
1. Escalation charges:
(a) Materials
Total Increase = 75,000×25/125 = 15,000
Limit (Up to 5%) = 75,000×5/125 = 3,000
Excess Cost Increment = 12,000
(b) Wages
Total Increase = 50,000×25/125 = 10,000
Limit (Up to 5%) = 50,000×5/125 = 2,000
Excess Cost Increment = 8,000
(c) Total Increase (a) +(b) = 20,000
(d) Increase in Contract Price = 20,000×25/100 = Rs. 5,000

2. Computation of Revenue to be recognized


Contract Price = Initial Price + Escalation = 500,000 + 5,000 = 505,000
Stage of completion = Work Certified ÷ Contract Price = 200,000 ÷ 505,000 = 39.60%
Amount of revenue to be recognized = % of completion × Contract price = 39.60% of
505,000 = 200,000

3. Computation of costs incurred


Materials 75,000
Wages 50,000
General Expenses 10,000
Depreciation 5,000
Total Cost Incurred 140,000
Less: Contract WIP (Uncertified Works) 15,000
Cost Recognized 125,000

6. A contractor has entered into a contract to construct a building. The costs on the contract
worked out to Rs. 10,50,000. What would be his profit or loss on the contract?
i) If the contract is for a fixed price of Rs. 10,00,000
ii) For a price of cost plus 10%.
(CAP Jun. 2017 Q4b-3 Marks)
Answer

© The Institute of Chartered Accountants of Nepal 117


CAP II Paper 1: Advanced Accounting

i) In this case as the revenue is Rs. 10,00,000 and the cost are Rs. 10,50,00 the loss on
contract is Rs. 50,000. The loss is to be recognized immediately even if the contract is in
its executory phase.
ii) In this case, the contractor would receive
Rs. 10,50,000 plus 10% of Rs. 10,50,000 = Rs. 11,55,000
Less: Cost incurred 10,50,000
Profit on the contract 1,05,000

7. You are required to prepare the Contract Control Account for the year ended 31 March 2004,
and show the calculation of cost & revenue to be recognized in the Statement of Profit or Loss
for that year.

On 1st July 2003 MN Ltd. commenced work on a contract, which was to be completed by 30th
September 2004 at an agreed price of Rs.520,000.
M N Limited’s financial year ended on 31 March 2004 and on that day expenditure on the
contract totaled Rs.263,000 made up as under :-
Rs.
Plant 30,000
Materials 1,24,000
Wages 95,000
Sundry Expenses 5,000
Head Office Charges 9,000
2,63,000
Cash totaling Rs. 1,95,000 had been received by 31 March 2004 representing 75% of the work
certified as completed on that date, but in addition, work costing Rs.30,000 had been completed
but not certified.
A sum of Rs.9,000 had been obtained on the sale of materials, which had cost Rs.8,000 but
which had been found unsuitable. On 31st March 2004 stock of unused materials on site had
cost Rs.10,000 and the plant was valued at Rs.20,000. to complete the contract by 30 September
2004 it was estimated that:-
i. The following additional expenditure would be incurred:
Wages Rs.64, 000
Materials Rs.74, 400
Sundry Expenses Rs. 9, 000
ii. Further plant costing Rs.25,000 would be required;
iii. The residual value of all plants used on the contract up to 30 September 2004 would be
Rs.15, 000.
iv. Head office charges to the contract would be at the same annual rate plus 10%

It was estimated that the contract would be completed on time but a contingency provision of
Rs.15, 000 should be made.
(Inter Dec. 2005 Q3-15 Marks)

© The Institute of Chartered Accountants of Nepal 118


CAP II Paper 1: Advanced Accounting

Answer

Particulars Rs. Particulars Rs.


Revenue Recognized 281,567 Cost Recognized 205,000
Cost Incurred 235,000 Contract Work in Progress 30,000
Trade receivable (Billed less received;
260,000
Assumed total work certified is billed)
Amount due from customer (Revenue
21,567
less billed)
Total 516,567 Total 516,567

Statement of profit or loss (Extract)


Particulars Amount
Contract Revenue 281,567
Contract Costs 205,000
Gross Profit (Loss) 76,567

Working Notes:
1. Costs Incurred
Depreciation on plant 10,000
Materials 116000
Wages 95,000
Sundry Expenses 5,000
Head Office Charges 9,000
Total (A) 235,000

2. Estimated Total Cost


Plant Depreciation (30,000+25,000-15,000) 40,000
Material (124,000-8,000+74,400) 190,400
Wages (95,000+64,000) 159,000
Sundry Expenses (5,000+9,000) 14,000
Head Office Charges:
For 9 months up to 31 March 2004 9,000
For 6 months up to 30 Sept.2004 6,000
15,000
Add: 10% of Rs. 6,000 600 15,600
Contingent Costs 15,000
Total Cost to complete (B) 434,000

3. Stage of completion = A ÷ B = 54.15%


Revenue to be recognized = 520,000 × 54.15% = 281,567

© The Institute of Chartered Accountants of Nepal 119


CAP II Paper 1: Advanced Accounting

2.3 Investment Accounting

A. Theory Questions
None

B. Practical Questions

1. On 1st April, 2008 Anand held 20,000 fully paid equity shares of Rs.10 each in P Ltd.
appearing in Anand's book at Rs.3,05,000. On 1st June, 2008 he acquire Dec. 5,000 more
equity shares in the company cum dividend at an all-inclusive cost of Rs.17 per share. On
30th June, 2008 P Ltd. announced a bonus issue at the rate of one fully paid up equity share
of Rs.10 for every five shares held. Anand received the bonus share on 4th August, 2008.
P Ltd. also made a right issue, the terms being as follows:
i) The issue would entitle the shareholders to subscribe to one equity share of Rs.10 in
the company for every three shares held as on 10th August, 2008; the new shares would
be issued at a premium of Rs.5 per share, the whole amount being payable by 30th
September, 2008.
ii) The shareholders would be entitled to renounce their entitlements either wholly or in
part to outsiders.
Anand exercised his option under the issue for 50% of his entitlements and sold the balance
of his rights to another person at Rs.1.50 per share.
P Ltd. declared a dividend at the rate of 20% for the year ended 31st March, 2008. Anand
received the dividend on 3rd October 2008.
On 1st December, 2008 Anand sold 15,000 equity shares and received a net sum of
Rs.2,62,500.
From the information given above, you are required to prepare Investment Account in
Anand's ledger for the year ended 31st March, 2009. Use average cost method.
(CAP Dec. 2009 Q2a-10 Marks)
Answer
NFRS in its entirety is not applicable to individuals and therefore, the requirements of NAS
32, 39 & NFRS 9 have not been considered in solving this problem

© The Institute of Chartered Accountants of Nepal 120


CAP II Paper 1: Advanced Accounting

Investment Account – Equity Shares in P Ltd.


Income Principal Income Principal
Date Particulars No. Date Particulars No.
Rs. Rs. Rs. Rs.
01.04.'08 To Bal. b/fd 20,000 3,05,000 ----- By Bank -Sale of rights 7,500
01.06.'08 To Bank 5,000 85,000 03.10.'08 By Bank – Dividend for 40,000 10,000
2007/08
04.08.08 To Bonus Share 5,000
30.09.'08 To Bank – Purchase of Rights Share 5,000 75,000 01.12.'08 By Bank - Sale 15,000 2,62,500
01.12.'08 To P/L A/c – Profit on sale 70,715
31.03.'09 To P/L A/c 40,000 31.03.'09 By Bal.c/d 20,000 2,55,715
35,000 40,000 5,35,715 35,000 40,000 5,35,715
01.04.'09 To Bal. b/d 20,000 2,55,715
Working notes:
(i) Calculation of profit on sale of 15,000 shares:
Particulars Rs. Rs.
Amount paid for 35,000 shares: 4,65,000
(Rs.3,05,000 + Rs.85,000 + Rs.75,000)
Less: Amount received on sale of rights 7,500
Dividend for 2007/08 10,000 17,500
Cost of 35,000 shares 4,47,500
Sales proceeds of 15,000 shares 2,62,500
Less: Cost Rs.(4,47,500×15,000/35,000) 1,91,785
Profit 70,715

© The Institute of Chartered Accountants of Nepal 121


CAP II Paper 1: Advanced Accounting

2. The following transactions by Mr. Nishchal took place during the year ended 31st March,
2012:

1st April Purchased Rs. 12,00,000 8 % bonds @ Rs. 80.5 cum-interest.


Interest is payable on 1st May.
12th April Purchased Rs. 1,00,000 equity shares of Rs. 10 each in X Ltd.
for Rs. 40,00,000.
1st May Received half year’s interest on 8% bonds.
15th May X Ltd. made a bonus issue of three equity shares for every two
held. Mr. Nishchal sold 1,25,000 bonus shares @ Rs. 20 each.
1st October Sold Rs. 3,00,000 8% bonds @ Rs. 81 ex-interest.
1st November Received half year’s bond interest.
1st December Received. 18% dividend on equity shares in X Ltd.

Prepare the relevant investment accounts in the books of Mr. Nishchal for the year ended in
31st March, 2012.
(CAP Jun. 2012 Q2a -10 Marks)
Answer
NFRS in its entirety is not applicable to individuals and therefore, the requirements of NAS
32, 39 & NFRS 9 have not been considered in solving this problem.

In the books of Mr. Nischal


8% Bond Account (Rs,’000)
Nominal Nominal
Date Particulars Income Cost Date Particulars Income Cost
Value Value
By Bank
1/4/11 To Bank [1] 1,200 40 926 1/5/11 - 48 -
[2]
By Bank
31/3/12 To P/L: 1/10/11 300 10 243
[3]
By Bank
Int. income - 84 - 1/11/11 - 36 -
[4]
Profit on sale - - 11.5 31/3/12 By Bal. c/d 900 30 694.5
1,200 124 937.5 1,200 124 937.5

Equity Shares in X Ltd.


Nomin
Incom Nomina Incom
Date Particulars al Cost Date Particulars Cost
e l Value e
Value
12/4/1 4,00 15/5/1
To Bank 1,000 - By Bank 1,250 - 2,500
1 0 1
15/5/1 1/12/1 By Bank
To Bonus [7] 1,500 - - - 225 -
1 1 [8]

© The Institute of Chartered Accountants of Nepal 122


CAP II Paper 1: Advanced Accounting

31/3/1 31/3/1 By Bal.


To P/L: 1,250 - 2,000
2 2 c/d
Dividend - 225 -
Profit on sale - - 500
4,50
2,500 225 2,500 225 4,500
0

Working Notes:
1. Cost & interest income of 8% Bond:
Rs.
Cum interest purchase cost [Rs. 1,200,000 / 100 × 80.50] 966,000
Less: Interest for 5 months [Rs. 1,200,000 × 8% × 5 / 12] 40,000
Ex-interest purchase cost 926,000
2. Half yearly interest [Rs. 1,200,000 × 8% × 6 / 12] = Rs. 48,000
3. Sale value of Bond [Rs. 300,000 / 100 × 81] = Rs. 243,000.
Interest for 5 months [Rs. 300,000 × 8% × 5 / 12] = Rs. 10,000
Profit on sale of Bond [Rs. 243,000 – (Rs. 926,000 / 1,200,000 × 300,000)] = Rs. 11,500
4. Half yearly interest [Rs. 900,000 × 8% × 6 / 12] = Rs. 36,000
5. Accrued interest on Bond for 5 months as on 31.03.2012 [Rs. 900,000 × 8% × 5 / 12]
= Rs. 30,000
6. Closing balance of 8% Bond as on 31.03.2012 [Rs. 926,000 / 1,200,000 × 900,000] =
Rs. 694,500
7. Bonus issue of equity shares of X Ltd. [100,000 / 2 × 3] = 150,000 shares of Rs.10
each.
8. Dividend on equity shares [Rs. 1,000,000 + Rs. 1,500,000 – Rs. 1,250,000] × 18% =
Rs. 225,000
9. Profit on sale of equity shares:
Rs.
Sales value [125,000 × Rs. 20] 2,500,000
Less: Cost [Rs. 4,000,000 / 250,000 × 125,000] 2,000,000
Profit 500,000
10. Closing balance of equity shares of X Ltd. [Rs. 4,000,000 / 250,000 × 125,000] = Rs.
2,000,000

3. On 1st Shrawan 2069, Mr. Pradhan has 100,000 equity shares of ABC Bank Ltd. at a book
value of Rs. 150 per share (face value Rs. 100 each). Further information have been provided
to you.
i) On 20th Ashwin 2069, he purchased another 20,000 shares of ABC Bank Ltd at Rs. 160
per share.
ii) 1st Mangsir 2069, ABC Bank Ltd. issued one equity bonus share for every six shares held
by the shareholders.

© The Institute of Chartered Accountants of Nepal 123


CAP II Paper 1: Advanced Accounting

iii) 29th Magh, 2069, the directors of ABC Bank Ltd. announced a right issue which entitles
the holders to subscribe three shares for every seven shares at Rs. 150 per share.
Shareholders can transfer their rights in full or in part.

Mr. Pradhan sold 1/3rd entitlement to Mr. Sharma for a consideration of Rs. 20 per share and
subscribe the rest on 5th Fagun 2069.
You are required to prepare Investment A/c in the books of Mr. Pradhan for the year ending
31st Ashadh 2070.
(CAP Jun. 2014 4a-5 Marks)
Answer
NFRS in its entirety is not applicable to individuals and therefore, the requirements of NAS
32, 39 & NFRS 9 have not been considered in solving this problem.

In the books of Mr. Pradhan


Investment Account
(Equity share in ABC Bank Ltd.)
No. of Amount No. of Amount
Date Particulars Shares Rs. Date Particulars Shares Rs.
By Bank A/c
(sale of right)
1.4.069 To Balance B/d 100000 15000000 5.11.069 (W.N.3) 400000
By Balance C/d
20.6.069 To Bank A/c 20000 3200000 31.32070 (Bal.Fig) 180000 23800000
To Bonus issue
1.8.069 (W.N.1) 20000 0
To Bank
A/c(Right
shares)
5.11.069 (W.N. 4) 40000 6000000 ______ ________
180000 24200000 180000 24200000

Working Notes
1) Bonus shares = (100000+20000)/6 20000
Right shares = (100000+20000+20000)/7
2) x3 60000
3) Sale of right = 60000 x 1/3 x Rs. 20 400000
4) Right subscribed = 40000 shares x Rs. 150 6000000

4. Retirement Benefit Plan of ABC Bank Ltd. purchased preference shares of XYZ Bank Ltd.
face value of Rs. 1,000,000 at Rs. 1,037,000 on 1st Shrawan, 2069. The coupon rate of the
preference shares is 12% and maturity date of the shares is 31st Ashadh, 2074. You are required
to prepare the investment account in the books of retirement benefit plan up to the year ended
31st Ashadh, 2074. The effective interest rate of the investment is 11%.

© The Institute of Chartered Accountants of Nepal 124


CAP II Paper 1: Advanced Accounting

(CAP Jun. 2018 Q4a-10 Marks)


Answer
Retirement Benefit Plan of ABC Bank Ltd.
Investment Account

Date Particulars Amount Date Particulars Amount


1,04.2069 To Bank A/c 10,37,000 31.03.2070 By Bank A/c 5,930
31.03.2070 By Balance C/d 10,31,070

Total 10,37,000 Total 10,37,000


1,04.2070 To Balance B/d 10,31,070 31.03.2071 By Bank A/c 6,582
31.03.2071 By Balance C/d 10,24,488

Total 10,31,070 Total 10,31,070


1,04.2071 To Balance B/d 10,24,488 31.03.2072 By Bank A/c 7,306
31.03.2072 By Balance C/d 10,17,181

Total 10,24,488 Total 10,24,488


1,04.2072 To Balance B/d 10,17,181 31.03.2073 By Bank A/c 8,110
31.03.2073 By Balance C/d 10,09,071

Total 10,17,181 Total 10,17,181


1,04.2073 To Balance B/d 10,09,071 31.03.2074 By Bank A/c 9,071
31.03.2074 By Bank A/c 10,00,000

Total 10,09,071 Total 10,09,071

Amortization Schedule
Year (A) Beginning (B) Dividend (C) Reported (D) (E) End-of-
of period and maturity dividend Amortization period
amortized cost cash inflow income of Premium amortized
[=(A)×11%] [=(C) – (B)] cost[=(A)+(D)]
2069- 10,37,000 1,20,000 1,14,070 5,930 10,31,070
70
2070- 10,31,070 1,20,000 1,13,418 6,582 10,24,488
71
2071- 10,24,488 1,20,000 1,12,694 7,306 10,17,181
72
2072- 10,17,181 1,20,000 1,11,890 8,110 10,09,071
73
2073- 10,09,071 11,20,000 1,10,929 9,071 10,00,000
74

© The Institute of Chartered Accountants of Nepal 125


CAP II Paper 1: Advanced Accounting

5. During the year 2002 Neo-Bull Investment Ltd. Purchased and sold investments as per the
following details:
31st March, 2002: Purchased 5,000, 5% Debenture of Rs. 100 each of Bear Ltd. at Rs. 97,
brokerage and stamp duty amounting to Rs. 12,800. Interest is payable on 1st July and 1st
January.
1st May, 2002: Purchased 5,000, 6% Cumulative Preference Shares of Rs. 100 each in Bull
Ltd. at Rs. 95, brokerage and stamp duty being Rs. 12,100. Dividends are payable on 30th June
and 31st December.
1st July, 2002: Sold Rs. 3,00,000 Debentures of Bear Ltd. at Rs. 99 less brokerage etc. Rs 1800.
1st October, 2002: Purchased further 2,000, 6% cumulative preference shares of Rs. 100 each
of Bull Ltd. at Rs 90, brokerage and stamp duty being Rs. 4,300
Write up the Ledger accounts of these investments for the year 2002 in the books of the
investing company.
(June 2003, Q 3)
Answer
5% Rs. 100 Debentures of Bear Ltd.
(Interest dates: 1st Jan. & 1st July)

Nominal Interest Capital Nominal Interest Capital


2002 Rs. Rs. Rs. 2002 Rs. Rs. Rs.
March Bank 5,00,000 6,250 4,91,550 July 1 Bank 12,500
31
Dec. 31 Profit & 11,250 270 July 1 Bank 3,00,000 2,95,200
Loss A/c
Dec. 1 Accrued 2,00,000 5,000 1,96,620
Interest
Balance
5,00,000 17,500 4,91,820 5,00,000 17,500 4,91,820

6% Rs. Cumulative Preference Shares in Bull Ltd.


(Dividend dates: 30th June and 31st December)

Nominal Dividend Capital Nominal Dividend Capital


2002 Rs. Rs. Rs. 2002 Rs. Rs. Rs.
May 31 Bank 5,00,000 10,000 4,77,100 June 30 Bank 15,000
Oct. 31 Bank 2,00,000 3,000 1,81,300 Dec. 31 Bank 21,000
Dec. 31 Profit Balance 7,00,000 6,58,400
& Loss 23,000
A/c
7,00,000 36,000 6,58,400 7,00,000 36,000 6,58,400
Note (1): In the absence of any information as to the ruling price at December 31, 2002, the
investments have been carried forward at cost.

© The Institute of Chartered Accountants of Nepal 126


CAP II Paper 1: Advanced Accounting

6. On 1st May 2006, Tea Ltd. Purchased 13.5% Convertible Debentures in Coffee Ltd. of face
value of Rs. 5,00,000 @ 105 ex-interest. Interest on the debentures is payable each year on 31st
March and 30th September. The accounting year adopted by Tea Ltd. is calendar year. The
following other transactions were entered into in 2006 by Tea Ltd. in regard to these
debentures:
Aug. 1 Purchased Rs. 2,50,000 Debentures @ 107 cum interest.
Oct. 1 Sale of Rs. 2,00,000 Debentures @ 103 cum interest.
Dec. 31 Receipt of 10,000 Equity Shares in Coffee Ltd. of Rs. 10 each in
conversion of 20% of the Debentures held.
The market value of the Debentures and Equity Shares in Coffee Ltd. at the end of 2006 was
106 and Rs. 15 respectively.
Prepare the Debenture Investment Account in the books of Tea Ltd. on Average Cost basis.
((Inter Jun. 2007 Q3-15 Marks)
Answer
Books of Tea Ltd.
13.5% Convertible Debentures in Coffee Ltd. Account
(Interest payable 31st March, 30 September)

Date Nominal Interest Amount Date Nominal Interest Amount


2006 2006
Jan To Sep By Bank
1 Bank 5,00,000 5,625 5,25,000 30 (Interest for 6 50,625
months)
Aug To Oct By Bank
2,50,000 11,250 2,56,250 2,00,000 2,06,000
1 Bank 1
Dec To Oct By Loss on
31 Interest 52,313 1 sale A/C 2,333
Income
Dec To FV 8,483 Dec By equity
31 Gain * 31 shares In 1,15,000
Coffee Ltd
Dec By Bank
31 (Interest On
3,713
Converted
Debentures
Dec By Balance
4,40,000 14,850 466,400
31 c/d
7,50,000 69,188 7,89,733 7,50,000 69,188 7,89,733
* Financial assets other than those required to be measured at amortized cost is measured at
FV through PL and therefore, the equity shares obtained and balance of investment in
debentures have been measured at FV.

© The Institute of Chartered Accountants of Nepal 127


CAP II Paper 1: Advanced Accounting

Working Notes:
S.N. Particulars Rs. Rs.
(1) Interest paid on 1st May on purchase of Rs.5,00,000 Deb. 5,625
for 1 month 33,750
(2) Interest received on Sept.30 on Rs.5,00,000 16,875 50,625
on Rs.2,50,000 11,250
(3) Interest paid on 1st Aug. on 2,50,000 for 4 months
(4) Loss on Sale of Debentures
2,08,333
Cost (Rs.5,25,000 + Rs.2,56,250)  2,00,000
7,50,000 2,06,000
Selling price 2,333
Loss 3,713
(5) Interest on Rs. 1,10,000 Debentures for 3 months
(6) Cost of Debentures converted
1,14,583
(Rs. 5,25,000 + Rs.2,56,250)  1,10,000
7,50,000
(7) Cost of Balance Debentures Rs. 4,40,000
4,58,334
(Rs.5,25,000 + Rs.2,56,250)  4,40,000
7,50,000

7. On 1st Shrawan 2069; Mr. Akash purchased 5,000 equity shares of Rs. 100 each in BB Bank
Ltd. @ Rs. 120 each from a broker who charged 2% brokerage. On 31st Baishakh 2070; bonus
shares were declared in the ratio 1:2. Before and after the record of bonus shares, the shares
were quoted Rs. 175 per share and Rs. 90 per share respectively. On 31st Ashadh 2070, Mr.
Akash sold bonus shares to a broker who charged 2% brokerage.

Show the investment account in the books of Mr. Akash, who held the shares as current assets
and closing value should be made as per NAS.
10 (CAP Dec. 2013 2B
Answer:
NFRS in its entirety is not applicable to individuals and therefore, the requirements of NAS
32, 39 & NFRS 9 have not been considered in solving this problem

Investment in Equity shares of BB Bank Ltd


Account in the books of Mr. Akash For the Year Ended 31st Ashadh 2070
Date Particulars Nominal Cost Date Particulars Nominal Cost
1.4.2069 To Bank A/C 5,00,000 6,12,000 31.3.2070 By Bank 2,50,000 2,20,500
A/C
(W.N.1) (W.N.2)

© The Institute of Chartered Accountants of Nepal 128


CAP II Paper 1: Advanced Accounting

31.3.2070 To Bonus Shares 2,50,000 - 31.3.2070 By Balance 5,00,000 4,08,000


C/d
31.3.2070 To Profit & Loss - 16,500
Account
(W.N.3)
7,50,000 6,28,500 7,50,000 6,28,500

Working Notes:
1.Calculation of cost of equity shares purchased on 1.4.2069
5000 x Rs 120 + 2% of Rs 600,000 = 6120000

2. Calculation of profit proceeds of equity shares sold on 31.3.2070


2500 x Rs 90-2% of Rs 2,25,000 = Rs 2,20,500

3. Calculation of profit on sale of bonus shares on 31.3.2070


Sales Proceeds -Average Cost
220,500-204,000 i .e. (612000x250000/750000) = Rs 16,500

4. Valuation of equity shares on 31.3.2070


Cost = 612000 x 500000/750000 = Rs 408000
Market Value = Rs 5000 X Rs 90 = Rs 450000
Hence, Valuation is taken as Rs 408000

8. On 01.04.2071, Mr. M. Pandey purchased 5,000 equity shares of Rs 100 each in Jyoti Bikash
Bank Ltd. @ Rs 120 each from a broker, who charged 2 % brokerage. He incurred 50 paisa
per Rs 100 as cost of shares transfer stamps. On 31-01-2072 bonus was declared in the ratio of
1:2. Before and after the record date of bonus shares, the shares were quoted at Rs 175 per
share and Rs 90 per share respectively. On 31-03-2072, Mr. M. Pandey sold bonus shares to a
broker, who charged 2% brokerage.

Show the Investment Account in the books of Mr. M. Pandey, who held the shares as Current
Assets and closing value of investments shall be made at cost or market value whichever is
lower.
(CAP Dec. 2015 4A
Answer
NFRS in its entirety is not applicable to individuals and therefore, the requirements of NAS
32, 39 & NFRS 9 have not been considered in solving this problem

In the books of Mr. M. Pandey


Investment Account
For the year ended 31st Ashadh, 2072

Date Particulars Nominal Cost Date Particulars Nominal Cost

© The Institute of Chartered Accountants of Nepal 129


CAP II Paper 1: Advanced Accounting

1.4.2071 To Bank A/c 5,00,000 6,15,000 31.03.2072 By Bank A/c 2,50,000 2,20,500
31.1.2072 Bonus Shares 2,50,000 - 31.03.2072 By Balance 5,00,000 4,10,000
c/d
31.03.2072 To P/L A/c 15,500
7,50,000 6,30,500 7,50,000 6,30,500

Working Notes:

1. Cost of equity shares purchased on 1.04. 2071


= Cost +Brokerage + Cost of Transfer Stamps
= 5,000* Rs. 120+ 2% of Rs. 6,00,000+ ½ % of Rs. 6,00,000
= Rs. 6,15,000
2. Sale proceeds of equity share sold on 31.03. 2072
= Sales Price-Brokerage
=2,500* Rs. 90-2% of Rs. 2,25,000
=Rs. 2,20,500
3. Profit on sale of bonus shares on 31.03. 2072
=Sales proceeds – Average cost
Sales proceeds = Rs. 2,20,500
Average Cost= Rs. ( 6,15,000* 2,50,000/7,50,000)
= Rs. 2,05,000
Profit = Rs. 2,20,500- Rs. 2,05,000 = 15,500.
4. Valuation of equity shares on 31.03. 2072
Cost = Rs. (6,15,000* 5,00,000/7,50,000)= Rs. 4,10,000 i.e. Rs. 82 per share
Market Value = 5,000 shares * Rs. 90 = Rs 4,50,000
Closing stock of equity share has been valued at Rs. 4,10,000 i.e. cost being lower than
the market value.

© The Institute of Chartered Accountants of Nepal 130


CAP II Paper 1: Advanced Accounting

2.4 Branch & Departmental Accounts

A. Theoretical Questions (Departmental Accounts)

1. Explain inter-departmental transfers in the context of departmental accounts.


(Inter Jun. 2003 Q 6a- 5 Marks)
Answer:
The basic objective of departmentalization is to make each department a responsibility center.
The departments are also profit centers. When one responsibility center renders service or
supplies goods to another center, the price used is known as 'transfer price'. Such price may be
the market price if available. The transfer price may be adjusted for cash discount and selling
costs that do not arise in internal transfers to make it more accurate and equitable to both the
buying and the selling centers. Alternatively, one might take cost plus certain margin as transfer
price. This is generally set as a standard to ensure that the selling departments do not transfer
their inefficiencies as a part of the cost in transfer price.

Thus, it can be concluded that transfer price is a good mechanism to record inter-departmental
transfers. Among others, this helps the management in setting up profit centers and fixing
responsibility on departmental managers. This also ultimately helps the management in
efficiency and productivity evaluation.

2. Basis of allocation of common expenditure among departments


(Inter Jun. 2004 Q 6b (iii) -4 Marks; Inter Dec. 2005 Q 6b-5 Marks; Inter Dec. 2006 Q4b-5
Marks)

Answer:
The following are the basis of allocation of common expenditure among departments:
1. Expenses that are specifically incurred for each department are charged directly to the
department e.g., direct salary of workers in the department.
2. Common expenses, the benefit of which is shared by all the departments and which are
capable of precise allocation (e.g. lightning, rent etc.) are distributed among the
departments concerned on some equitable basis that is considered suitable in the
circumstances of the case. For example, rent is distributed on the basis of floor area of
each department.
3. However, common expenses, which are not capable of accurate measurement, are dealt
with as follows:
a. On the basis of sales: selling expenses like commission, discount and debts.
b. Equal allocation: administrative and other expenses such as salaries of managers,
directors, common advertisement expenses, depreciation etc., which are shared
by all departments. Some organizations take an alternative route of not allocating
these two departments and charging them directly to the Profit and Loss.

© The Institute of Chartered Accountants of Nepal 131


CAP II Paper 1: Advanced Accounting

3. Distinguish between Branch account and Departmental account.


(Inter Jun. 2011 Q5- 10 Marks)
Answer
Branch Account Departmental Account
Accounting records Separate accounting for branch No separate accounting for
department.
Allocation of common No question of allocation of common Common expenditures are
expenditure expenditure allocated as per the
predetermined basis.
Reconciliation of Reconciliation of transaction between No requirement of
transaction head office and branch should be done. reconciliation of transaction.
Conversion of foreign At the time of finalization of account, No such problem.
branch figure. the conversion of foreign branch figure
may create some problem.

B. Theoretical Questions (Branch Accounts)

1. Explain the concept for determining of profits or loss under ‘Debtors System’ of Branch
accounting. (Inter Jun. 2005 Q5b-4 Marks)

Answer
Branch account maintained under Debtors System of branch accounting is a Nominal account
and the profit or loss ascertained by applying the Balance Sheet approach of Income
determination. The difference between assets and outsiders’ liabilities always represents the
Net Worth and it can be ascertained either by deducting liabilities from assets or by showing
the assets and liabilities in the two sides of the same account. As per the Net Worth approach
of income determination, increase in net worth represents the profit and decrease in the same
represents the loss, however the effect of fresh capital introduced and capital withdrawn shall
be given to special attention to ascertain the correct profit and loss. Recording opening and
closing assets and liabilities in the two sides of the Branch Account represents opening capital
(net worth) and Closing capital (net worth) of the head office in the branch. If we consider
goods sent to the branches and other amount remitted as additional capital injected to the branch
and receipts form debtors, cash received from branch and goods return as capital withdrawn
from the branch, the branch accounts exhibit the opening capital, closing capital, further capital
introduced and capital withdrawn. By comparing the capital of any two given particular period,
we can determine the profit or loss made by the branch during the period.

2. Foreign Branches (Inter Jun. 2007 Q5d-5 Marks)

Answer:

© The Institute of Chartered Accountants of Nepal 132


CAP II Paper 1: Advanced Accounting

Foreign branches generally maintain independent and complete record of business transacted
by them in currency of the country in which they operate. Thus problems of incorporating
balances of foreign branches relate mainly to translation of foreign currency into reporting
currency..

C. Practical Questions (Departmental Accounts)

1. Nepal Readymade Centre has two departments, clothes and readymade garments. The
readymade garments are made by the firm itself out of the clothes supplied by the cloth
department at its usual selling price. The following figures of the year 2002 are available:

Cloth Readymade
Department Department

Opening stock on 01.01.2002 Rs. 3,00,000 Rs. 50,000


Purchases Rs. 20,00,000 Rs. 15,000
Sales Rs. 22,00,000 Rs. 4,50,000
Transfer to Readymade dept. Rs. 3,00,000 –
Expenses - Manufacturing – Rs. 60,000
Expenses - Selling Rs. 20,000 Rs. 6,000
Closing stock on 31.12.2002 Rs. 2,00,000 Rs. 60,000

The stock in the readymade department may be considered as consisting of 75% clothes and
25% other expenses. The cloth department earned gross profit at the rate of 15% in the year
2002. General Expenses of the business as a whole came to Rs. 1,01,000. Prepare
Departmental Trading and Profit and Loss Accounts for the year 2002.
(Inter Dec. 2003 Q2a-8 Marks)
Answer:

Departmental Trading & Profit and Loss Account (Memorandum)


for the year ended on 31.12.2002

Cloth Readymade Cloth Readymade


(Rs.) (Rs.) (Rs.) (Rs.)

To Opening Stock 3,00,000 50,000 By Sales 22,00,000 4,50,000


To Purchase 20,00,000 15,000 By Readymade 3,00,000 –
To Cloth – 3,00,000 By Closing 2,00,000 60,000
Stock
To Manufacturing Exp. – 60,000
To Gross Profit 4,00,000 85,000 ________ _______
27,00,000 5,10,000 27,00,000 5,10,000

© The Institute of Chartered Accountants of Nepal 133


CAP II Paper 1: Advanced Accounting

To Selling Expenses 20,000 6,000 By Gross Profit 4,00,000 85,000


To Net Profit transferred
to General P/L Account
3,80,000 79,000 ______ ______
4,00,000 85,000 4,00,000 85,000

2. Mercantile Ltd. prepares separate Departmental Profit and Loss Accounts. The nature of their
operation required frequent supply of articles/services from one department to another. The
Company consisted of three departments – A , B and C. It had been decided that A department
will charge for services supplied to other departments the cost thereof plus 10% thereon.
Likewise B department was to charge the other departments cost plus 20% thereof in respect
of supplies made to them. The C department supplies to the other departments were charged at
the prevailing rates applicable to outsiders. The accounts for the year ended on 31st March 2005
had been closed without taking into account the interdepartmental debits and credits. From the
following figures, show the net variation in the departmental Profit and Loss Accounts as a
result of such adjustments:
Rs.
(1) Cost of A Department’s services extended to :
B 8,400
C 4,500

(2) Cost of supplies made by B Department to:


A 29,800
C 5,400

(3) Value of supplies made by C Department to:


A 400
B 5,600

In addition, the following are the charges to be made for interchange of staff from one
department to another for temporary periods during the year:

B’s staff shifted to A department 4,400


A’s staff shifted to B department 1,100
(Inter Jun. 2006 Q5 -10 Marks)
Answer:
Memorandum Profit & Loss (Adjustment) Account
Dr. Cr.
Particulars A Rs. B Rs. C Rs. Particulars A Rs. B Rs. C Rs.
To Services - 9,240 4,950 By Services
from A
To Supplies 35,760 - 6,480 From A:
from B To B 9,240 -

© The Institute of Chartered Accountants of Nepal 134


CAP II Paper 1: Advanced Accounting

To C 4,950 14,190
To Supplies 400 5,600 - From B:
from C To A 35,760 -
To C 6,480 - 42,240
To Charge in 4,400 1,100 - By Supplies
respect of staff
borrowed From C:
To A 400 - - 6,000
To B 5,600

To Increase in - 30,700 - By Recoveries in 1,100 4,400 -


deptt. Profit (or respect of staff
decrease in
deptt. loss)
By Decrease in
deptt. Profit (or
increase in deptt.
loss) 25,270 - 5,430

40,560 46,640 11,430 40,560 46,640 11,430

Note: 10% has been added to costs of A deptt. services to find out transfer price for B and
C. 20% has been added to costs of supplies of B deptt. to find out transfer price for A and C
dept

3. X Ltd has two departments, Department A and Department B. From the following particulars
prepare the departmental and Consolidated-trading account in the columnar form for the year
ended 32 Ashadh 2063.
Department A Department B
Rs. Rs.
Opening Stock (At cost) 20,000 12,000
Purchases 92,000 68,000
Sales 140,000 112,000
Wages 12,000 8,000
Carriage 2,000 2,000

Closing Stock
Purchased goods 4,500 6,000
Finished Goods 24,000 14,000

© The Institute of Chartered Accountants of Nepal 135


CAP II Paper 1: Advanced Accounting

Purchased goods transferred


By B to A 10,000
By A to B 8,000

Finished goods transferred


By A to B 35,000
By B to A 40,000

Return of Finished goods


By A to B 10,000
By B to A 7,000

The purchased goods have been transferred mutually at their respective departmental purchase
cost and finished goods at departmental market price and that 20 % of the finished goods
(Closing) at each department represents the finished goods received from the other department.
(Inter Dec. 2007 Q5a-10 Marks)

Answer
a)
X Ltd.
Departmental and Consolidated Trading Account (Memorandum)
for the year ending 32 Ashadh 2063
Dr. Cr.

Dept A Dept B Consolidated Dept A Dept B Consolidated


To Opening By
Stock 20,000 12,000 32,000 Sales 140,000 112,000 252,000
To Purchases 92,000 68,000 160,000 By Purchased
goods
To Wages 12,000 8,000 20,000 transferred 8,000 10,000
To Carriage 2,000 2,000 4,000 By Finished
To Purchased goods
goods transferred 35,000 40,000
transferred 10,000 8,000 By Return of
To Finished Finished
goods goods 10,000 7,000
transferred 40,000 35,000 By Closing Stock
To Return of - Purchased
Finished goods 4,500 6,000 10,500
goods 7,000 10,000 Finished goods 24,000 14,000 38,000
To Stock
Reserve 2,196
To Gross Profit 38,500 46,000 82,304
221,500 189,000 300,500 221,500 189,000 300,500

© The Institute of Chartered Accountants of Nepal 136


CAP II Paper 1: Advanced Accounting

Working Notes:
Dept A (Rs.) Dept B (Rs.)
Closing stock out of transfer 4,800 2,800
(14,000 * 20
(24,000 * 20 %) %)
Sale 140,000 112,000
Add Transfer 35,000 40,000
175,000 152,000
Less Returns 7,000 10,000
Net Sales plus transfer 168,000 142,000
Rate of gross profit 38,500 22.917 % 46,000 32 .394 %
168,000 142,000

Unrealized Profit =4800*32.394 % =2800*22.917%


= 1,555 = 642

4. Spice Ltd. has three departments I, J and K. The following information is provided for the year
ended 31st Ashadh, 2065:
I J K
Rs. Rs. Rs.
Opening stock 5,000 8,000 19,000
Opening reserve for unrealized profit ― 2,000 3,000
Materials consumed 16,000 20,000 ―
Direct labor 9,000 10,000 ―
Closing stock 5,000 20,000 5,000
Sales ― ― 80,000
Area occupied (sq. mtr.) 2,500 1,500 1,000
No. of employees 30 20 10

Stocks of each department are valued at cost to the department concerned. Stocks of I are
transferred to J at cost plus 20% and stocks of J are transferred to K at a gross profit of
20% on sales. Other common expenses are salaries and staff welfare Rs. 18,000 and rent
Rs. 6,000.

Prepare Departmental Trading, Profit and Loss Account for the year ending 31st Ashadh,
2065.
(CAP Dec. 2009 Q 4a-10 Marks)
Answer:

a)
Departmental Trading and Profit and Loss Account (Memorandum)
for the year ended 31st Ashadh, 2065
I J K Total I J K Total

© The Institute of Chartered Accountants of Nepal 137


CAP II Paper 1: Advanced Accounting

Rs. Rs. Rs. Rs. Rs. Rs. Rs. Rs.


To Opening By Sales 80,000 80,000
stock 5,000 8,000 19,000 32,000
To Material By Inter-
consumed 16,000 20,000 36,000 dept.
To Direct 9,000 10,000 19,000 transfer 30,000 60,000 90,000
labor By Closing
stock 5,000 20,000 5,000 30,000
To Inter-dept. 30,000 60,000 90,000
transfer
To Gross
profit 5,000 12,000 6,000 23,000 ______ ______ ______ _______
35,000 80,000 85,000 2,00,000 35,000 80,000 85,000 2,00,000
To Salaries 9,000 6,000 3,000 18,000 By Gross 5,000 12,000 6,000 23,000
and staff By profit b/d 7,000 7,000
welfare Net loss
To Rent 3,000 1,800 1,200 6,000
To Net profit ______ 4,200 1,800 6,000 _____ _____ _____ _____
12,000 12,000 6,000 30,000 12,000 12,000 6,000 30,000
To Net loss (I) 7,000 By Stock 5,000
To Stock reserve
reserve b/d (J +
(J+K) K)
(Refer 3,000 By Net profit 6,000
W.N.) (J + K)
To Balance 1,000 _____
transferred
to Profit
and loss
account
11,000 11,000
Working Note:
Calculation of unrealized profit on closing stock
Stock reserve of J department Rs.
Cost 30,000
Transfer from I department 30,000
60,000
Stock of J department 20,000
Rs.30,000
Proportion of stock of I department = Rs.20,000 × = Rs.10,000
Rs.60,000
20
Stock reserve =Rs.10,000× = Rs.1667 (approx.)
120
Stock reserve of K department Rs.
Stock transferred from J department 5,000
Less: Profit (stock reserve) 5,000  20% 1,000
Cost to J department 4,000
Rs.30,000
Proportion of stock of I department =Rs.4,000 × = Rs.2,000
Rs.60,000
20
Stock reserve = 𝑅𝑠.2,000 × = Rs.333 (approx.)
120
Total stock reserve = Rs.1,000 + Rs.333 = Rs.1,333

© The Institute of Chartered Accountants of Nepal 138


CAP II Paper 1: Advanced Accounting

5. Textile Ltd. prepares separate Departmental Profit and Loss Accounts. The nature of their
operation required frequent supply of articles/ services from one department to another. The
company consisted of three departments- A, B and C. It had been decided that A department
will charge, for services supplied to other departments, the cost thereof plus 10% thereon.
Likewise B department will charge the other departments cost plus 20% thereof in respect of
supplies made to them. The C department’s supplies to the other departments will be charged
at the prevailing rates applicable to outsiders. The accounts for the year ended on Ashadh 32,
2067 has been closed without taking into account the interdepartmental transactions.

Following are the departmental transactions:


Rs.
i) Cost of A Department’s service extended to :B 8,400
C 4,500
ii) Cost of supplies made by B Department to: A 29,800
C 5,400
iii) Value of supplies made by C Department to: A 400
B 5,600

In addition, the following are the charges to be made for interchange of staffs from one
department to another for temporary period during the year:
B’s staff shifted to A Department 4,400
A’s staff shifted to B Department 1,100
Required:
Show the net variation in the departmental Profit and Loss Accounts as a result of above
adjustments.
(Inter Dec. 2010 Q2a-6 Marks)

Answer:
a)
Memorandum Profit & Loss (Adjustment) Account

Particulars A Rs. B Rs. C Rs. Particulars A Rs. B Rs. C Rs.


To Services from A 9,240 4,950 By Services from A to B 9,240
To Supplies from B 35,760 6,480 By Services from A to C 4,950
To Supplies from C 400 5,600 By Services from B to A 35,760
To Charge in respect
of staff 4,400 1,100 By Services from B to C 6,480
To Increase in Dept.
Profit 30,700 By Services from C to A 400
(or Decrease in Dept.
Loss) By Services from C to B 5,600

© The Institute of Chartered Accountants of Nepal 139


CAP II Paper 1: Advanced Accounting

By Recovery in respect
of Staff 1,100 4,400
By Decrease in Dept.
Profit
(or Increase in Dept.
Loss) 25,270 5,430

40,560 46,640 11,430 40,560 46,640 11,430

Notes:

10% has been added to cost of A dept. services to find out transfer price for B and C. 20% has been added
to costs of supplies of B dept. to find out transfer price for A and C dept.

Working Note:

Statement showing transfer price

From Dept. To Dept. Cost/ Value (Rs.) Transfer Price (Rs.)


A B 8,400 9,260
A C 4,500 4,950
B A 29,800 35,760
B C 5,400 6,480
C A 400 400
C B 5,600 5,600

6. M/s Baboon & Co. has four departments A, B, C, D. Each department was being managed by
a departmental manager whose commission was 10% of the respective departmental profit,
subject to a minimum of Rs. 6,000 in each case. Inter departmental transfers took place at a
loaded price as follows.

From Department A to Department B: 10% above cost


From Department C to Department D: 20% above cost
From Department A to Department D: 20% above cost
From Department C to Department B: 20% above cost

For the year ended 31st Ashadh, 2074 the company had already prepared and closed the
Departmental Trading and Profit and Loss Account. Subsequently it was discovered that the
closing stock of departments had included inter departmentally transferred good at loaded
price instead of cost price.

From the following information, prepare a statement re-computing the departmental profit or

© The Institute of Chartered Accountants of Nepal 140


CAP II Paper 1: Advanced Accounting

loss.
(CAP Dec. 2017 Q4b-5 Marks)

Answer:
(Rs.)
Particulars Dept. A Dept. B Dept. C Dept. D
Final Profit/ Loss 38,000 50,400 72,000 1,08,000
(Loss) (Profit) (Profit) (Profit)
Inter departmental transfers 70,000 4,800
included at loaded price in the (22,000 from (3,600
departmental stock Dept. A and from Dept.
8,000 from C and
Dept. C) 1,200 from
Dept. A)

Statement of re-computation of department profit/Loss


Particulars Dept. A Dept. B Dept. C Dept. D
Final profit/Loss as computed earlier (38,000) 50,400 72,000 1,08,000
Add –Department manager’s commission 6,000 6,000 8,000 12,000
already deducted from profit ( W.N-1)
Profit before charging manager’s (32,000) 56,400 80,000 120,000
commission
Loss- Profit earned due to transfer of (2,200) - (8,600) -
goods at loaded price and included in
departmental unsold stock ( WN-2)
Correct departmental profit before (34,200) 56,400 71,400 120,000
charging manager’s commission
Manager’s Commission @10% of (6,000) (6,000) (7,140) (12,000)
departmental profit subject to a minimum
of Rs.6,000
Adjusted Profit/Loss (40,200) 50,400 64,260 108,000

Working Notes
Manager’s commission is payable @10 % of departmental profit before charging such
commission (subject to a minimum of Rs.6, 000). Hence manager’s commission already
deducted will be as follows.

Departments Profit /Loss after Manager’s Commission


charging
manager’s
commission

© The Institute of Chartered Accountants of Nepal 141


CAP II Paper 1: Advanced Accounting

A (38,000) 6,000
B 50,400 1/9 of Rs.50,400 or 6,000 whichever is
higher i.e. 6,000
C 72,000 1/9 of Rs.72,000=8,000
D 108,000 1/9 of Rs 1,08,000=12,000

2. Unrealized profit on unsold departmental stock:


Profit earned by Department A by transfer of stock Amount
Department B@110%= Rs.22,000*10/110=2,000
Department D @120%= Rs.1,200*20/120=200 2,200

Profit earned by the Department C by transferring stock to


Department D@120%= Rs.3,600*20/120=600
Department B@120%= Rs.48,000*20/120=8,000 8,600

7. M/s Manakamna Limited has three department stores A, B & C. From the particulars given
below the values of stock as on 31st December 2015 and the departmental trading result for the
year ended 31st December 2015.
(a)
A B C
(Rs.) (Rs.) (Rs.)
st
Stock as on 1 January 2015 24,000 36,000 12,000
Purchases 146,000 124,000 48,000
Actual sales 172,500 159,400 74,600
G P on normal selling prices 20% 25% 33.33%
(b) During the year certain items were sold at a discount and these discounts
were reflected in the values of sales shown above. The items sold at
discount were:
Particulars A B C
(Rs.) (Rs.) (Rs.)
Sales at normal price 10,000 3,000 1,000
Sales at actual prices 7,500 2,400 600

(CAP Jun. 2016 Q3b - 5 Marks)


Answer
Memorandum Departmental Trading Account
For the year ending 31st December 2015
A B C A B C
Rs. Rs. Rs. Rs. Rs. Rs.

© The Institute of Chartered Accountants of Nepal 142


CAP II Paper 1: Advanced Accounting

To Opening 24,000 36,000 12,000 By Sales 172,500 159,400 74,600


Stock 146,000 124,000 48,000 By Closing 30,000 40,000 10,000
To 32,500 39,400 24,600 Stock
Purchases
To Gross
Profit
202,500 199,400 84,600 202,500 199,400 84,600

Working Note:
(i) Ascertainment of cost of sales A B C
Rs. Rs. Rs.
Sales at actual price 172,500 159,400 74,600
Less: Discounted sales price (7,500) (2,400) (600)
165,000 157,000 74,000
Add: Sales at normal selling price 10,000 3,000 1,000
Total Sales at normal price 175,000 160,000 75,000

Profit on normal prices 20% 25% 33.33%

8. During the fiscal year ending 15th July 2008, following purchases were made by a business
house having three departments at a total cost of Rs 100,000:
Department A 1,000 units
Department B 2,000 units
Department C 2,400 units
Stocks on 16th July 2007 were as follows:
Department A 120 units
Department B 80 units
Department C 152 units

The sales made during the fiscal year 2007/08 were as follows:
Department A 1,020 units @ Rs 20 each
Department B 1,920 units @ Rs 22.50 each
Department C 2,496 units @ Rs 25 each

The rate of gross profit is the same in each case.


Prepare Departmental Trading Account for the fiscal year 2007/08.
(Inter Dec. 2008 Q5a-10 Marks)

© The Institute of Chartered Accountants of Nepal 143


CAP II Paper 1: Advanced Accounting

Answer
Memorandum Departmental Trading Account
for the fiscal year 2007/08

Particulars A B C Total Particulars A B C Total


To Opening 1,920 1,440 3,040 6,400 By Sales 20,400 43,200 62,400 126,000
stock
To Purchase 16,000 36,000 48,000 100,000 By Closing 1,600 2,880 1,120 5,600
stock (WN
1)
To Gross 4,080 8,640 12,480 25.200
Profit
Total 22,000 46,080 63,520 131,600 Total 22,000 46,080 63,520 131,600

Working Notes 1) Calculation of closing stock.

A B C
Particulars Units Units Units
Opening Stock 120 80 152
Purchases 1000 2000 2400
Total 1120 2080 2552
Less: Sales 1020 1920 2496
Closing Stock 100 160 56
Cost per unit (WN2) 16 18 20
Value of closing stock 1600 2880 1120

Working Notes 2) Calculation of rate of gross profit

Assuming all the purchases are sold, the sale proceeds would be:
A 1,000 units @ Rs 20 20,000
B 2,000 units @ Rs 22.5 45,000
C 2,400 units @ Rs 25 60,000
125,000
Total cost is Rs 100,000; hence gross profit would be Rs 25,000 or 20% of the selling price.

For each article the profit and cost:

Profit (Rs) Cost( Rs)


A 1/5 of Rs 20 4 16
B 1/5 of Rs 22.5 4.5 18
C 1/5 of Rs 25 5 20

© The Institute of Chartered Accountants of Nepal 144


CAP II Paper 1: Advanced Accounting

D. Practical Questions (Branch Accounts)


Note: Foreign Branch dealt under NAS 21
1. A Kathmandu based Shop opens a new branch in Biratnagar which trades independently of the
head office. The transactions at the branch for the year ended 31st March 2013 are as under:
(Rs.)
Goods supplied by head office 200,000
Purchases from outsiders:-
Credit 155,500
Cash 30,000 185,500
Sales:
Credit 250,500
Cash 46,000 296,500
Cash received from customers 304,500
Cash paid to creditors 142,500
Expenses paid to creditors 89,500
Furniture purchased by the branch on credit 35,000
Cash received from head office initially 40,000
Remittances to head office 110,000
Prepare the branch final accounts after taking the following into consideration:
i) Write off depreciation on furniture at 5% per annum for full year.
ii) A remittance of Rs. 20,000 from the branch to the head office is in transit.
iii) The values its closing stock at Rs. 120,000
(CAP Dec. 2014 Q2b-10 Marks; CAP Jun. 2015 Q2b- 10 Marks)

Answer:

In Branch Book
Trading and Profit & Loss Account for the year ended on 31st March 2013
Dr. Cr.
Particulars Amount Particulars Amount
Rs. Rs.
To Goods supplied by head 200,000 By Sales:
office Credit
To Purchases: 250,500 296,500
Credit 155,000 185,500 Cash 120,000
Cash 30,000 31,000 46,000
To Gross Profit c/d By Closing Stock
Total 416,500 Total 416,500
To Expenses 89,500 BY Gross Profit b/d 31,000
To Depreciation on Furniture 1,750 BY Head Office- transfer of net 60,250
loss
Total 91,250 Total 91,250

© The Institute of Chartered Accountants of Nepal 145


CAP II Paper 1: Advanced Accounting

Balance Sheet as on 31st March 2013


Liabilities Amount Assets Amount
Rs Rs
Sundry Trade Creditors Closing Stock 120,000
(155,500-142,500) 13,000 Cash in Transit 20,000
Creditor for Furniture 35,000 Cash in hand 18,500
Advances from Trade Debtors
(304,500-250,500) 54,000
Head Office Account 56,500
Total 158,500 Total 158,500

Cash Book
Liabilities Amount Assets Amount
Rs Rs
To Head Office (Initial 40,000 By Cash Purchases 30,000
Receipt) 46,000 By Trade Creditors 142,500
To Cash Sales 304,500 By Expenses 89,500
To Trade Debtors By Head Office Remittances 110,000
By Balance c/d 18,500
Total 390,500 Total 390,500

2. Following are the trial balances as on 31st March 2008 of PD Ltd.


(Amount in '000)
Head Office Branch
Capital 20,000 -
Retained Earnings 1.4.2007 2,715 -
Profit for the year 8,415 3,484
Creditors on Open Account 5,930 -
Creditors for expenses 390 -
Goods from Head Office - 7,000
Cash Received from Branch 9,987 -
Head Office current account 1.4.2007 - 7,298
Total 47,437 17,782

Premises 8,000 4,000


Furniture & Fittings 800 400
Stock in Trade 9,000 1,800
Trade Debtors 8,000 650
Branch current account 1.4.2007 7,298 -
Goods sent to branch 7,200 -
Cash remitted to Head Office - 10,287

© The Institute of Chartered Accountants of Nepal 146


CAP II Paper 1: Advanced Accounting

Cash at Bank 7,139 645


Total 47,437 17,782

From the above prepare the combined balance sheet of PD Ltd. as on 31st March 2008 and
show the branch current account as it would appear in the head office books.
(Inter Dec. 2008 Q2a-10 Marks)

Answer:
Balance sheet of PD Ltd. as on 31st March, 2008
Liabilities Amount Assets Amount
(Rs in '000') (Rs in '000')
Capital 20,000 Premises Rs
Retained Earnings Head Office 8,000
Last Year 2,715 Branch 4,000 12,000
Current Year 11,899 14,614 Fixtures
Trade Creditors 5,930 Head Office 800
Creditors for Expenses 390 Branch 400 1,200
Stock
Head Office 9,000
Branch 1,800
In Transit (WN.1) 200 11,000
Debtors
Head Office 8,000
Branch 650 8,650
Bank
Head Office 7,139
Branch 645 7,784
Cash in Transit (WN. 2) 300
40,934 40,934

Branch Current Account


Particulars Amount Particulars Amount
(Rs in '000') (Rs. in '000')
To Balance b/d 7,298 By Cash 9,987
To Goods sent to branch 7,200 By Stock in Transit 200
To Profit & Loss account 3,484 By Cash in Transit 300
By Balance c/d 7,495
17,982 17,982

The branch account balance of Rs. 7,495 calculated above is represented by:
Premises 4,000
Fixtures 400
Stock 1,800

© The Institute of Chartered Accountants of Nepal 147


CAP II Paper 1: Advanced Accounting

Debtors 650
Bank 645
Total 7,495

Working Notes:

1. Computation of Value of Stock-in-transit:


Rs in '000
Goods sent to branch as per Head office books 7,200
Less: Goods received from head office as per Branch books 7,000
Goods in transit 200

2. Computation of Value of Cash-in-transit:


Rs in '000'
Cash remitted to Head office as per branch records 10,287
Cash received from branch as per Head Office records 9,987
Cash-in-transit 300

3. Abhinav Enterprises has a retail branch at Pokhara. Goods are sold to the customers at Cost
plus 100%. The wholesale price is cost plus 80%. Goods are invoiced to Pokhara at wholesale
price. From the following particulars, find out the profit made at Head office and Pokhara for
the year ended 31st Ashadh 2070 :

Head Office Pokhara


Rs. Rs.
st
Stock as on 1 Shrawan 2069 225,000 -
Purchases 2,150,000 -
Goods sent to branch at invoice price 954,000 -
Sales 2,853,000 950,000
st
Stock as on 31 Ashadh 2070 260,000 99,000
Sales at Head Office are made only on wholesale basis and that at Branch only to
consumers. Stock at branch is valued at invoice price.
(CAP Jun. 2014 Q3a- 3 Marks)
Answer:

In the books of Abhinav Enterprises.


Memorandum Trading and Profit and Loss Account
Head Pokhara Head Pokhara
Office Office
To opening stock 225,000 - By Sales 28,53,000 9,50,000
To Purchases 21,50,000 - 9,54,000 -

© The Institute of Chartered Accountants of Nepal 148


CAP II Paper 1: Advanced Accounting

To Goods received from - 9,54,000 By Goods sent to 2,60,000 99,000


HO branch
To Profit and Loss A/c 16,92,000 95,000 By Closing stock
(Gross Profit)

40,67,000 10,49,000 40,67,000 10,49,000


To Branch stock reserve 44,000 By Gross Profit c/d 16,92,000 95,000
(99,000X80/180)
To Net Profit
(Before expenses) 16,48,000 95,000

4. Pashupati Stores Pvt Ltd with its head office at Kathmandu, invoiced goods to its branch at
Gajuri at 20% less than the list price which is cost plus 100%, with instruction that cash sales
were to be made at invoice price and credit sales at list price.
From the following particulars available from the branch, prepare Branch Stock Account,
Branch Adjustment A/c, Branch Profit and Loss A/c and Branch Debtors A/c for the year
ending 31st Chaitra, 2074.
(CAP Jun. 2018 Q3a-5 Marks)

Particulars Rs. Rs.


Stock on 1st Baishakh, 2074 ( invoice price) 6,000
Debtors on 1st Baishakh, 2074 5,000
Goods received from Head Office (invoice price) 66,000
Sales
Cash 23,000
Credit 50,000 73,000
Cash received from debtors 42,817
Expenses at branch 8,683
Debtors on 31st Chaitra, 2074 12,183
Stock on 31st Chaitra, 2074 (invoice price) 8,800
Remittance to Head Office 60,000

Answer:

a) In the Books of Pashupati Stores Pvt Ltd:


Dr Branch Stock Account Cr
Particulars Amount Particulars Amount
To Balance b/d 6,000 By Bank ( Cash Sales) 23,000
To Goods sent to Branch 66,000 By Branch Debtor 50,000

© The Institute of Chartered Accountants of Nepal 149


CAP II Paper 1: Advanced Accounting

To Branch Adjustment( W.N-1) 10,000 By Shortage in stock (W. N-2) 200


By Balance c/d 8,800
Total 82,000 Total 82,000

In the Books of Pashupati Stores Pvt Ltd


Dr Branch Adjustment Account Cr
Particulars Amount Particulars Amount
To Shortage in stock (W.N-2) 75 By Branch Stock(W.N-1) 10,000
To Stock Reserve (W.N-5) 3,300 By Stock Reserve (W.N-3) 2,250
To Gross Profit & Loss 33,625 By Goods sent to Branch (W.N-4) 24,750
Total 37,000 Total 37,000

In the Books of Pashupati Stores Pvt Ltd


Dr Branch Profit and Loss Account Cr
Particulars Amount Particulars Amount
To Branch Expenses 8,683 By Gross Profit 33,625
To Shortage in sales (cost) 125
To General Profit and Loss 24,817
Total 33,625 Total 33,625

In the Books of Pashupati Stores Pvt Ltd


Dr Branch Debtors Account Cr
Particulars Amount Particulars Amount
To Balance b/d 5,000 By Bank 42,817
To Branch Stock 50,000 By Balance c/d 12,183
Total 55,000 Total 55,000

Working Notes
1) Let the Cost price= X
List price = 100% of cost price
=100 +100= Rs.200
Invoice price = 20% less than List price
=200-(20% of 200)
=Rs.160
Cash sales are made at invoice price i.e.Rs.160 and credit sales are made at list price i.e.Rs.200
Excess amount charged on credit sales= Rs.200-Rs.160= Rs.40 i.e. 40/200*100%= 20 % of list
price.
Good sold on credit= Rs.50, 000
Amount charged in excess of the invoice price = 20% of list price
= 20% of Rs.50, 000
= Rs.10, 000
2) If the cost =Rs.100 then invoice price = Rs.160
Therefore loading on invoice price = 60/160*100%=37.5%

© The Institute of Chartered Accountants of Nepal 150


CAP II Paper 1: Advanced Accounting

Loading on Shortage =37.5% of Rs.200= Rs.75


= Rs.200-Rs.75= Rs.125
3) Loading of Opening stock
=37.5% of Rs.6, 000=Rs.2, 250
4) Loading on Goods sent to Branch
=37.5% of Rs.66, 000=Rs.24, 750
5) Loading on closing stock
=37.5% of Rs.8, 800=Rs.3, 300

5. M/s Brown & Co. have their Head Office at New Delhi and a Branch at Jodhpur. The goods
are sent to Branch at 20% less than the list price, which is cost plus 100%.
From the following particulars, ascertain the Profit made by the Branch as well as Head Office
on wholesale basis:
Head Office Branch Office
(Rs.) (Rs.)

Opening stock at cost price 40,000 –


Opening stock at Invoice price – 20,000
Purchases 4,00,000 –
Expenses 60,000 12,000
Goods destroyed in an accident at Invoice Price – 2,000
Sales at list price 3,40,000 1,60,000
Goods sent to Branch/Received by Branch at Invoice 1,60,000 1,60,000
price
Branch Manager's salary – 12,000
Head Office Manager's salary 30,000 –

You are asked to prepare Branch Stock Account, Branch Profit and Loss Account, Goods sent
to Branch Account and General Trading Profit and Loss Account of Head Office.
(Inter Dec. 2001 Q2-16 Marks)

Answer:

Branch Stock A/c

Rs. Rs.

To Balance B/d 20,000 By Cash Sales 1,60,000


To Goods Sent to Branch A/c 1,60,000 By P & L A/c (Goods destroyed) 2,000
To Gross Profit C/d 32,000 By Balance C/d (Note i) 50,000
2,12,000 2,12,000

Branch Profit and Loss A/c

Rs. Rs.

© The Institute of Chartered Accountants of Nepal 151


CAP II Paper 1: Advanced Accounting

To Goods destroyed 2,000 By Gross Profit B/d 32,000


To Branch expenses 12,000
To Branch Managers Salary 12,000
To Net Profit 6,000
32,000 32,000

Goods Sent to Branch A/c

Rs. Rs.

To Trading A/c 1,60,000 By Branch Stock A/c 1,60,000

1,60,000 1,60,000

Let the cost price = 100/- Rs.


List Price = 200/- Rs.
Invoice Price = 20% Less than the List price
= 200 - 40 = Rs. 160/-

Note i:
Closing Stock at Branch at Invoice Price
Closing Stock = (Opening stock + Goods from H.O. - Cost of Goods sold -
Stock Destroyed)
= 20000+160000 - 128000 - 2000 = Rs. 50000/-
160000 x 160/200 = 128000

Memorandum Trading, Profit and Loss A/c


Head Office

Rs. Rs.

To Opening Stock 40,000 By Sales 3,40,000


To Purchases 4,00,000 By Goods Sent to Branch A/c 1,60,000
To Gross Profit C/d 2,30,000 By Closing Stock (Note ii) 1,70,000
6,70,000 6,70,000

To Expenses 60,000 By Gross Profit B/d 2,30,000


To Manager's Salary 30,000 By Branch P & L A/c 6,000
To Stock Reserve 18,750 By Stock Reserve A/c (Note iii) 7,500
To Net Profit Transferred 1,34,750
2,43,500 2,43,500

Closing Stock at H.O. at Cost

© The Institute of Chartered Accountants of Nepal 152


CAP II Paper 1: Advanced Accounting

Closing Stock = (Opening Stock + Purchases - Cost of Goods sold - Cost of goods sent to
branch)

= 40,000 + 4,00,000 - 1,70,000 - 1,00,000 = Rs. 2,70,000/-


Cost of goods sold = 3,40,000  100/200 = Rs. 1,70,000/-
Cost of goods sent to Branch = 1,60,000  100/160 = Rs. 1,00,000/-
Opening & Closing Stock at Branch are appearing at Invoice price. The Head Office will
have to make adjustment for unrealized profit.

Stock Reserve for opening stock = 20,000  60/160 = Rs. 7,500/-


Stock Reserve for closing stock = 50,000  60/160 = Rs. 18,750/-

6. Lovely Limited with its head office in Kathmandu, opened on January 1, 2001, a branch at
Pokhara, where all sales were to be made on credit basis.
All goods required by the branch were supplied by the company from head office and invoiced
to the branch at cost plus 20%.
During the year ended December 31, 2001 the following transactions took place:
Goods sent to Pokhara Branch (at cost to HO) Rs. 22,000
Returns to HO (at cost to HO) 840
Sales as shown by Pokhara Branch report 19,670
Cash received from debtors and remitted to HO 15,190
Debtors balances at branch written off as bad 640
Loss of goods at branch through pilferage is estimated at 1 per cent of goods received.
The stock of goods held by the branch on December 31, 2001, amounted to Rs. 5,820 an invoice
price. You are required to record the entries in the appropriate accounts in the ledger of the
Head Office showing the balances as on December 31, 2001 and the branch gross and net
profits for the year ended as on that date. You may assume that there are no other charges.
(Inter Jun. 2002 Q4 -15 Marks)

Answer
In the books of Lovely Limited Pokhara Branch A/C

To Gods sent to Br. 26,400 By Goods sent to Br. 4,400


(Loading as Goods Sent)
1
26,400 x 6
To Goods Sent to Br. 168 By Goods sent to Br. 1,008
(Loading on Goods return) (Return to H.O.)

© The Institute of Chartered Accountants of Nepal 153


CAP II Paper 1: Advanced Accounting

1
(1008 x 6 )
To Stock Reserve 970 By Sales (Collection from Brs.) 15,190
1 
6 x 5820
 
To General P& L A/c (B/F) 2,720 By Balance C/d Debtors (WN 3,840
1)
Stock 5,820
--------- ---------
30,258 30,258
======= =======

Memorandum Trading & P/L A/C

To Goods sent to Br. 22,000 By Loss Through Pilferage 212


(1% of 22,000 - 840)
By Goods sent to Br. 840
By Sales 19,670
To Gross Profit 3,572 By Closing Stock 4,850
(5820 - 1/6 of 5820)
--------- ---------
25,572 25,572
======= =======
To Bad Debts 640 By Gross Profit 3,572
To Net Loss Through 212
Pilferage
To Net Profit (b/f) 2,720
--------- ---------
3,572 3,572
======= =======

Working Note 1:
Memorandum Debtors A/C
To Credit Sales 19,670 By Cash (Collection) 15,190
By Bad Debts 640
By Balance C/D (B/F) 3,840
--------- ---------
19,670 19,670
======= =======

7. On February 1, 2001 goods costing Rs. 33,000 were invoiced by Rama (India) Ltd. Calcutta to
its branch at Kanpur and charged up at a selling price designed to produce a gross profit of 25
per cent on the selling price. At the end of the month, the return from the kanpur branch showed

© The Institute of Chartered Accountants of Nepal 154


CAP II Paper 1: Advanced Accounting

that sales were Rs.30,000. Goods invoiced at Rs. 600 to kanpur branch had been returned to
Calcutta Head Office. The closing stock at kanpur branch was Rs. 12,300 at selling price.

Record the above transaction in kanpur Branch Stock Account, Kanpur Branch Adjustment
Account and Goods sent to kanpur Branch Account in the ledger and balance the said accounts
of February 28,2001.
(Inter Dec. 2004 Q3c)

Answer
Kanpur Branch Stock Account
Dr. Cr.
Particulars Rs Particulars Rs

To Goods Sent to Branch A\c 44,000 By Goods Sent to Branch A/c 600
(Returns)
By Bank / Debtors (sales) 30,000
By Branch Adjustment A/c
(shortage) 275
By Branch P&L A/c (Cost of
Shortage) 825
By Balance C/d 12300

44,000 44,000

Goods Sent to Kanpur Branch Account


Dr. Cr.
Particulars Rs Particulars Rs

To Kanpur Branch Stock A/c By Kanpur Branch Stock A\c 44,000


(Return) 600
To Branch Adjustment A/c
[Rs.43,000 x 25/100] 10,850
To Trading A/c (b.f) 32,550

44,000 44,000

Kanpur Branch Adjustment Account

Dr. Cr.
Particulars Rs Particulars Rs
To Branch Stock A/c 275 By Goods sent to Branch A/c
(Loading on shortage ) [(Rs.44,000-Rs.600)x25/100] 10,850
To Stock Reserve A/c 3,075

© The Institute of Chartered Accountants of Nepal 155


CAP II Paper 1: Advanced Accounting

[Rs. 12,300 x 25/100]


To Gross Profit c/d 7,500

10,850 10,850

Kantipur Branch Profit & Loss Account

Dr. Cr.
Particulars Rs Particulars Rs

To Branch Stock A/c By Branch Adjustment A/c 7,500


(Cost of goods Lost) 825 (Gross Profit)
To Net Profit transferred to
general P& L a/c 6,675

7,500 7,500

8. SIMRAN (Pvt.) Ltd. is a Kathmandu based trading company having its Branch Office in
Birgunj. As per the policy of the company, goods are sent from the Head Office to Branch at
1
cost plus 10%. Head Office makes a uniform gross profit of 33 % on selling price on all its
3
other sales. The Branch sells goods at a uniform gross profit to Branch of 25% on selling price.
The following transactions have taken place during the financial year ended on 16th July 2006.

a) Head Office purchases amounted to Rs. 1,46,40,000, purchase returns were Rs.
6,16,500 and discount allowed by suppliers amounted to Rs. 3,00,900.
b) Sales by Head Office amounted to Rs. 1,08,00,000, Goods sent to Branch were Rs.
54,45,000 (at invoice price) and discount allowed to customers amounted to Rs.
91,800.
c) Goods sent to Branch for Rs. 6,60,000 in July 2006 were not received at the Branch
until July end.
d) Branch purchased goods locally for Rs. 18,75,000, discount allowed by suppliers
amounted to Rs. 48,750.
e) Overhead expenses of Head Office were Rs. 28,02,600 and of Branch Rs. 8,04,750.
f) Sales by the Branch amounted to Rs. 72,00,000, discount allowed to customers
amounted to Rs. 56,400 and cost of goods lost in-transit was Rs. 80,100.
g) Branch Stock as on 16th July 2006 included Stock invoiced by Head Office at Rs. 11,
55, 000.
You are required to prepare memorandum Trading, Profit and Loss Account of the Head Office
and Branch for the year ended 16th July 2006 in columnar form.

© The Institute of Chartered Accountants of Nepal 156


CAP II Paper 1: Advanced Accounting

(Inter Dec. 2006 Q4a-15 Marks)


Answer:

Trading and Profit & Loss Account

Dr. for the year ended on 16th July 2006 Cr.

Particulars H.O. B.O. Total Particulars H.O. B.O. Rs.


(Rs.) (Rs.) (Rs.) (Rs.)
To Purchases By Sales 1,08,00,000 72,00,000 1,80,00,000
Less: Return 1,40,23,500 18,75,000 1,58,98,500 By Goods sent to
To Goods Received - 47,85,000 Branch less in transit 47,85,000 - -
To Gross Profit 40,35,000 18,00,000 58,35,000 By Goods lost in
transit 80,100 80,100
By Closing Stock 24,73,500 11,79,900 36,53,400
1,80,58,500 84,60,000 2,17,33,500 1,80,58,500 84,60,000 2,17,33,500

28,02,600 8,04,750 36,07,350 40,35,000 18,00,000 58,35,000


To Expenses 91,800 56,400 1,48,200 By Gross Profit 3,00,900 48,750 3,49,650
To Discount allowed By Discount received
To Goods lost in transit 80,100 80,100
To Stock Reserve 1,05,000 1,05,000
To Net Profit 13,36,500 9,07,500 22,44,000

43,35,900 18,48,750 61,84,650 43,35,900 18,48,750 61,84,650


Working Notes:

Rs.
(i) Closing Stock at Head Office:
Purchase less return 1,40,23,500
Cost of Sales: 1,08,00,000×2/3 72,00,000
Goods invoiced to Branch: 47,85,0000×100/110 43,50,000
24,73,500
(ii) Closing Stock at Branch:
Branch Purchase 18,75,000
Goods Received from H.O. 47,85,000
66,60,000
Less: Cost of Branch Sales 72,00,000×3/4 54,00,000
Goods lost in transit 80,100
11,79,000
(iii) Stock Reserve at Branch: 1/11 of 11,55,000 1,05,000
(iv) Goods in transit have been included in H.O. Stock.
(v) It has been presumed that goods lost in transit are those which were
sent on the Branch to customers.

© The Institute of Chartered Accountants of Nepal 157


CAP II Paper 1: Advanced Accounting

2.5 Insurance Claim – Loss of Stock & Profit

A. Theoretical Questions

1. Define the term ‘Indemnity Period’ and’ Standard Turnover’ in relation to calculation of
insurance claim. (CAP Dec. 2010 Q3b ii-3 Marks)
Answer
The period beginning with the occurrence of the Damage and ending not later than 12 months
thereafter during which the results of the business shall be affected in consequence of the
damage.

2. Standard Turnover
Answer
The turnover during the period in the twelve months immediately before the date of damage
which corresponds with the indemnity period to which such adjustments shall be made as may
be necessary to provide for the trend of the business and for variations in or special
circumstances affecting the business either before or after the damage.

3. Claim under Loss of profit policy – Elucidate. (Inter Dec. 2001 Q 6e-4 Marks)

Answer:
Fire results loss of profits to the business on account of its dislocation. Such a loss can be
covered by taking a loss of profit policy. The policy specifies both the period as well as the
amount it covers. While determining the amount of policy the insured should take into account
not only the amount of net profit he earn but also the amount of standing or fixed charges which
have been charged against the revenue for determining the amount of net profit. This policy
covers profit that would have been earned as if the reduction in sales (short sales) would not
have been there. i.e., Had the short sales would not have resulted what would have been earned
as normal profit will also be covered under this policy.

4. Average clause.
(Inter Jun. 2004 Q 6b ii - 4 Marks; Inter Dec. 2011 Q6b-5 Marks; CAP Jun. 2019 Q6e-3
Marks)
Answer:
Irrespective of full or partial insurance, a fire insurer meets in full, unless contracted otherwise,
the loss up to the policy amount. Thus, if fire destroys half of Rs. 50,000 stock, the insurer will
indemnify the entire loss unless the policy is below Rs. 25,000. In such a situation, both fully
and partially insured matters are indemnified alike in spite of their unequal premium
contributions (which vary with the policy amounts). This is rather inequitable. To mitigate the
same, a fire policy often provides an 'average clauses' whereby an insured becomes a coinsurer
in the event of under insurance. Both the insurer and the insured then bear the loss in proportion
to covered and uncovered sums. Thus, if Rs. 40,000 policy is taken for Rs. 50,000 stock, the
under insurance will be by Rs. 10,000 and the insurer and the insured will be the co-insurers

© The Institute of Chartered Accountants of Nepal 158


CAP II Paper 1: Advanced Accounting

for Rs. 40,000 and Rs. 10,000, respectively. If Rs. 25,000 stock is lost, the former will
indemnify only for Rs. 20,000 (40,000/50,000  25,000) and the balance Rs. 5,000 will be
borne by the insured himself.

B. Practical Questions

1. On 30/06/2068, an accidental fire destroyed a major part of the stocks in the shop of Davidson
Enterprises. Stocks costing Rs. 60,000 could be salvaged but not their stores ledgers. A fire
insurance policy was in force under which the sum insured was Rs. 700,000. From available
records, the following information was retrieved:
i) Total of sales invoices during the period 1/04/2068-30/06/2068 amounted to Rs.
6,040,000. An analysis showed that goods of the value of Rs. 600,000 had been
returned by the customers before the date of the fire.
ii) Opening stock as on 01/04/2068 was Rs. 440,000 including stocks of the value of Rs.
40,000 being lower of cost and net value subsequently realized.
iii) Purchases during the period 1/04/2068-30/06/2068 amounted to Rs. 4,200,000.
iv) Normal gross profit rate was 33.33% on sales.
v) A sum of Rs. 60,000 was incurred by way of firefighting expenses on the day of the
fire.
Prepare a statement showing the Insurance Claim Recoverable.
(CAP Dec 11 Q4a-5 Marks)
Answer
Calculation of Insurance Claim Recoverable
Stock Destroyed: Rs.
Stock on the date of fire – Working note 1,000,000
Less: Salvaged Stock 60,000
Stock Destroyed by fire 940,000
As the policy amount is less than the value of Closing Stock, average clause will apply.
Therefore, Amount of Claim = [Policy Amount × Loss / Stock on the date of fire]
= [Rs. 700,000 × 940,000 / 1,000,000]
= Rs. 658,000
As firefighting expenses have been incurred, these expenses may be admitted as part of the
loss. In that case loss will be Rs. 940,000 + Rs. 60,000 = Rs. 1,000,000.
Applying average clause, claim = [Rs. 700,000 × 1,000,000 / 1,000,000] = Rs. 700,000

Working note:
Memorandum Trading Account
for the three months period ended on 30th Aswin, 2068
Particulars Rs. Rs. Particulars Rs. Rs.
To Opening Stock 440,000 By Sales 6,040,000
Less: Abnormal
Less: Abnormal Item 40,000 400,000 40,000
Item
To Purchases 4,200,000 6,000,000

© The Institute of Chartered Accountants of Nepal 159


CAP II Paper 1: Advanced Accounting

To Gross Profit – 1/3rd of Sales 1,800,000 Less: Return 600,000 5,400,000


By Closing Stock – Bal. figure 1,000,000
6,400,000 6,400,000

2. The final accounts of Jyoti Limited ended on 31st Ashadh 2069 showed sales at Rs. 7,400,000
with a net profit of Rs. 612,000 after debiting standing charges of Rs. 128,000. On 1st Shrawan
2069 the company took a loss of profit insurance policy for Rs. 819,000 under which the period
of indemnity was four months with an average clause. On 1st Mangsir 2069 a fire broke out in
the company. Consequently, sales were affected during five months. The comparative sales
figures were as under:

Months Year 2068 (Rs.) Year 2069 (Rs.)


Mangsir 600,000 100,000
Poush 580,000 200,000
Magh 630,000 350,000
Fagun 610,000 470,000
Chaitra 650,000 590,000

The sales for twelve months ended on 30th Kartik 2069 were Rs. 7,800,000. On 1st Shrawan
2069 the old plant was replaced and a modern plant was installed to increase the profitability
by 2% on the sales of the financial year 2069/70.
You are required to prepare a statement showing the calculation of insurance claim
(CAP Dec. 2013 3c-7 Marks)
Answer
Statement Showing Calculation of Insurance Claim
Sales from 1st Mangsir 2068 to 30th Fagun 2068 Rs.
(600,000+580,000+630,000+610,000) 2,420,000
st th
Less: Sales from 1 Mangsir 2069 to 30 Fagun 2069
(100,000+200,000+350,000+470,000) 1,120,000
Short Sales 1,300,000
Gross Profit Ratio= Net profit+ Insured standing charges X 100
Annual sales
= 612,000+128,000 X 100
7,400,000
=10%
Add: Increase in profitability on sales = 2%
Effective Gross Profit Ratio = 12%
Loss of profit = Short sales x Effective Gross Profit Ratio
= 1,300,000 X 12/100= 156,000
Sales for twelve months preceding the date of fire = 7,800,000
Insurable amount of loss of profit= 7,800,000 X 12/100= 936,000
But the policy has been taken for NPR 819,000 only.

© The Institute of Chartered Accountants of Nepal 160


CAP II Paper 1: Advanced Accounting

Hence, average clause is applicable:


Amount of claim =( Loss of profit X Sum insured) / Insurable value
= (156,000 X 819,000)/ 936,000 = 136,500.

3. A Ltd. prepares accounts on 30th September each year, but on 31st December 2013 fire
destroyed the greater part of its stock .Following information was collected from books of
account:

Stock as 1.10 .2013 Rs. 29,700,000


Purchase from 1.10.2013 to 31.12.2013 Rs. 75,000,000
Wages from 1.10.2013 to 31.12.2013 Rs. 33,000,000
Sales from 1.10.2013 to 31.12.2013 Rs. 140,000,000

The rate of gross profit is 33.33% on cost .Stock to the value of Rs. 3,000,000
was salvaged .Insurance policy was for Rs. 25,000,000.
Additional Information:
i) Stock in the beginning was calculated at 10% less than cost.
ii) A plant was installed by firm’s own workers. They were paid Rs. 500,000
which was included in the wages.
iii) Purchase includes the purchase of plant of Rs. 5,000,000.
You are required to calculate the claim for the loss of stock. (CAP Dec. 2014 3c-7 Marks)
Computation of claim for loss of stock
Stock on the date of fire (31.12.2013) Rs. 30,500,000
Less salvage stock Rs. 3,000,000
Loss of Stock Rs. 27,500,000
Amount of claim = insured value/cost of stock on date of fire *loss of stock
= 25,000,000/30,500,000*27,500,000
= 22,541,000
Working Note
Memorandum Trading Account for period from 1.10.2013 to 31.12.2013
Particulars Rs. Particulars Rs.
To opening stock(29700000*100/90) 33,000,000 By sales 140,000,000
To Purchase 75,000,000 By Closing Stock 30,500,000
Less cost of plant 5,000,000 70,000,000 (balancing figure)
To wages 33,000,000 32,500,000
Less wages for plant 500,000
To gross profit (33.33% of cost or 25% of sale) 35,000,000
Total 170,500,000 Total 170,500,000

4. Gimmick Stores Limited, which runs a boutique makes up its accounts annually on 31st
Ashadh. For the year ended 31st Ashadh 2073, the Profit and Loss Statement was summarized
as follows:
Rs. Rs.

© The Institute of Chartered Accountants of Nepal 161


CAP II Paper 1: Advanced Accounting

Sales 2,160,000
Less: Cost of Sales (Net after discount received) 938,400
Wages 480,000 1,418,400
Gross Profit 741,600
Less: Fixed Charges 541,600
Net Profit 200,000

On 1st Chaitra 2072, a fire occurred as a result of which no trading was possible till 1st Shrawan
2073. On that date, half of the shop was reopened for business; the other half reopened on 1st
Ashwin 2073. The Consequential Loss Policy covers gross profit and workroom wages, and
accountancy charges at Rs. 5,000 or 2% of the amount of the claim (before such charges)
whichever is higher. The insurer agreed that:
i) Turnover increases by 15% per annum
ii) Workroom wages increase by 10% per annum
iii) The period of three months from 1st Chaitra to 31st Jestha accounts for one half of the annual
turnover.
iv) Fixed charges and wages occur evenly only throughout the year.
v) Discount received equal 1% of turnover.
You are required to compute the Consequential loss Claim for Gimmick Stores Limited.
(CAP Dec. 2016 Q2b-10 Marks)
Answer:
1st Alternative: IF IT IS ASSUMED THAT THE GIVEN P & L STATEMENT IS
OF ASHADH END 2073
Computation of Consequential Loss Amount in NPR
(i) Ratio of Gross Profit:
Sales
21,60,000
Less: Cost of Sales 9,38,400
Add: Discount (1% of turnover) 21,600 9,60,000
Wages 4,80,000
14,40,000
Gross Profit 7,20,000

Thus ratio of Gross Profit 720,000/21,60,000 or 33.33%

(ii) Loss of Turnover Rs.


st st
(a) 1 Chaitra 2072 to 31 Ashadh 2073
Turnover for 8 months (normal) 21,60,000
Monthly turnover (normal) (21,60,000X1/8) 2,70,000
If total turnover for the year is ‘A’ then
A = 21,60,000+2,70,000+1/2xA (given by the question)
Therefore, A=annual turnover = 48,60,000
So, turnover for Chaitra to Jestha = 24,30,000 (half of annual turnover)
Total turnover (Chaitra to Ashadh)(24,30,000+2,70,000) 27,00,000

© The Institute of Chartered Accountants of Nepal 162


CAP II Paper 1: Advanced Accounting

(b) 1st Shrawan 2073 to 31st Bhadra 2073


Monthly turnover (normal) 2,70,000
Add: 15% increase 40,500
3,10,500
Turnover applicable for 2 months for half the shop
1/2X3,10,500X2 3,10,500
Total Loss of Turnover 30,10,500

(iii) Loss of Profit: Rs.


33.33% of Rs. 30,10,500 10,03,500
Discount @1% on turnover (1% of Rs. 30,10,500) 30,105
Workroom wages for Chaitra to Ashadh 1,60,000
(One third of Rs.4,80,000)
Workroom wages for Shrawan and Bhadra 44,000
½ X 2/12 X (4,80,000+10%)
12,37,605
Add: Accountancy charges @ 2% of above 24,752
Total Value of Claim to be lodged 12,62,357

Working Note
1. The given turnover is of only 8 months because after Chaitra the business is
completely closed till Ashadh 2073.
2. Half of the turnover is for the period of 9 months other than Chaitra, Baishakh and
Jestha. Hence for one month, it will be 1/18 of the total.
Note-
1. Due to lack of information, Gross profit ratio is computed from 8 months
transaction.
2. Same Gross profit ratio (33.33%) has been assumed for the computation of loss of
profit for the period 1st Chaitra 2072 to 31st Ashadh 2073 and 1st Shrawan 2073 to
31st Bhadra 2073.

2 nd Alternative: IF IT IS ASSUMED THAT THE GIVEN P & L STATEMENT IS OF


ASHADH END 2072 INSTEAD OF ASHAD END 2073
Computation of Consequential Loss Amount in NPR
(i) Ratio of Gross Profit:
Sales 21,60,000
Less: Cost of Sales 9,38,400
Add: Discount (1% of turnover) 21,600 9,60,000
Wages 4,80,000
14,40,000
Gross Profit 7,20,000

Thus ratio of Gross Profit 720,000/21,60,000 or 33.33%

© The Institute of Chartered Accountants of Nepal 163


CAP II Paper 1: Advanced Accounting

(ii) Loss of Turnover Rs. Rs.


(a) 1st Chaitra 2072 to 31st Ashadh 2073
Annual Turnover 21,60,000
Add: 15% increase 3,24,000
24,84,000
1/2 up to 31st Jestha 2073 12,42,000
1/18 for one month up to 31st Ashadh 2073 1,38,000
(Refer Working Note No.1)
Total 13,80,000
(b) 1st Shrawan 2073 to 31st Bhadra 2073
Annual turnover as above 24,84,000
Add: 15% increase 3,72,600
28,56,600
Turnover applicable for 2 months for half the shop
2/36X28,56,800 1,58,700
Total Loss of Turnover 15,38,700
(iii) Loss of Profit: Rs.
33.33% of Rs. 15,38,700 5,12,900
Discount @1% on turnover (1% of Rs.15,38,700) 15,387
Workroom wages for Chaitra to Ashadh 1,76,000
(One third of Rs.4,80,000 plus 10%)
Workroom wages for Shrawan and Bhadra 48,800
½ X 2/12 X (5,28,000+10%)
7,53,087
Add: Accountancy charges @ 2% of above 15,062
Total Value of Claim to be lodged 7,68,149

Working Note
Half of the turnover is for the period of 9 months other than Chaitra, Baishakh and Jestha.
Hence for one month, it will be 1/18 of the total.

Note-
1. Due to lack of information, Gross profit ratio is computed from 8 months transaction.
2. Same Gross profit ratio (33.33%) has been assumed for the computation of loss of profit
for the period 1st Chaitra 2072 to 31st Ashadh 2073 and 1st Shrawan 2073 to 31st Bhadra
2073.

5. On 19th Mangsir, 2073 premises of National Trading were destroyed by fire, but sufficient
records were saved, wherefrom the following particulars were ascertained:
Rs.
Stock at cost on 1.4.2072 36,750
Stock at cost on 31.03.2073 39,800

© The Institute of Chartered Accountants of Nepal 164


CAP II Paper 1: Advanced Accounting

Purchases less returns during 2072/73 1,99,000


Sales less return during 2072/73 2,43,500
Purchases less returns during 1.4.2073 to 19.8.2073 81,000
Sales less returns during 1.4.2073 to 19.8.2073 1,15,600

National Trading closes his books every 31st Ashadh. In valuing the stock for the balance sheet
as at 31st Ashadh 2073 Rs. 1,150 had been written off on certain stock which was a poor selling
line having the cost Rs. 3,450. A portion of these goods were sold in Ashoj, 2073 at a loss of
Rs. 125 on original cost of Rs. 1,725. The remainder of this stock was now estimated to be
worth the original cost. Subject to the above exceptions, gross profit has remained at uniform
rate throughout. The stock salvaged was Rs. 2,900.

Show the amount of the claim of stock destroyed by fire. Memorandum Trading Account to be
prepared for the period from 1-4-2073 to 19-8-2073 for normal and abnormal items.
(CAP Dec. 2017 Q3a-10 Marks)
Answer
National Trading
Memorandum Trading Account for the year ended on 31st Ashadh, 2073
Rs Rs Rs
To Opening Stock 36,750 By Sales A/c 2,43,500
To Purchases 1,99,000 By Closing Stock
To Gross Profit 48,700 As valued 39,800
Add: Amount written off 1,150 40,950
to restore stock to full cost
2,84,450 2,84,450

The normal rate of gross profit to sales is =48,700 x 100 =20%


2, 43,500
Memorandum Trading Account up to 19, Mangsir, 2073
Normal Abnormal Total Normal Abnormal Total
Items Items Items Items
Rs Rs Rs Rs Rs Rs
To Opening 37,500 3,450* 40,950 By Sales 1,14,000 1,600 1,15,600
Stock
To Purchases 81,000 - 81,000 By Loss - 125 125
To Gross Profit 22,800 - 22,800 By 27,300 1,725 29,025
(20% on Rs Closing
1,14,000) Stock
(bal.fig)
1,41,300 3,450 1,44,750 1,41,300 3,450 1,44,750
*at cost.
Calculation of Insurance Claim
Particulars Rs.

© The Institute of Chartered Accountants of Nepal 165


CAP II Paper 1: Advanced Accounting

Value of Stock on 19th Mangsir, 2073 29,025


Less :Salvage (2,900)
Loss of Stock 26,125
Therefore insurance claim will be for Rs. 26, 125 only.

© The Institute of Chartered Accountants of Nepal 166


CAP II Paper 1: Advanced Accounting

6. From the following particulars, you are required to calculate the amount of claim for
Godawari Ltd., whose business premises was partly destroyed by fire:
Sum insured (from 31st Chaitra, 2073) Rs. 4,00,000
Period of indemnity 12 months
Date of damage 1st Baishakh, 2074
Date on which disruption of business ceased 31st Magh,
2074
The subject matter of the policy was gross profit but only net profit and insured standing
charges are included.
The books of account revealed:
v) The gross profit for the financial year 2073 was Rs. 3,60,000.
vi) The actual turnover for financial year 2073 was Rs. 12,00,000 which was also the turnover
in this case.
vii) The turnover for the period from 1 st Baishakh to 31st Magh, in the year preceding the
loss, was Rs. 10,00,000.
During dislocation of the position, it was learnt that in Fagun-Chaitra 2074, there has been
an upward trend in business (compared to the figure of the previous years) and it was stated
that had the loss not occurred, the trading results for 2074 would have been better than those
of the previous years.
The Insurance company official who was appointed to assess the loss accepted this view and
adjustments were made to the pre-damaged figures to bring them up to the estimated amounts
which would have resulted in 2074.
The pre-damaged figures together with agreed adjustments were:
Period Pre damaged Adjustment to Adjusted Standard
figure be added Turnover
Rs. Rs. Rs.
Baishakh 90,000 10,000 1,00,000
Jestha to Magh 9,10,000 50,000 9,60,000
Fagun to Chaitra 2,00,000 10,000 2,10,000
Total 12,00,000 70,000 12,70,000
Gross Profit 3,60,000 46,400 4,06,400

Rate of Gross Profit 30% (actual for 2073), 32% (adjusted for 2074). Increased cost of working
amounted to Rs. 1,80,000.
There was a clause in the policy relating to savings in insured standard charges during the
indemnity period and this amounted to Rs. 28,000.
Standing Charges not covered by insurance amounted to Rs. 20,000 p.a. The annual turnover
for Baishakh was nil and for the period Jestha to Magh 2074 Rs. 8,00,000.
Answer
Short Sales
Period Adjusted Standard Turnover Actual Turnover Shortage
Rs. Rs. Rs.
Baishakh 1,00,000 - 1,00,000

© The Institute of Chartered Accountants of Nepal 167


CAP II Paper 1: Advanced Accounting

Jestha to Magh 9,60,000 8,00,000 1,60,000


10,60,000 8,00,000 2,60,000
1) Gross profit ratio for the purpose of insurance claim on loss of profit
Gross profit - Insured Standing Charges - Uninsured standing charges = Net profit
Or
Net profit +Insured Standing Charges = Gross profit - Uninsured standing charges
= 4,06,400 – 20,000 = 3,86,400
=3,86,400/12,70,000×100%= 30.425%
2) Amount allowable in respect of additional expenses is least of the following
(i) Actual expenses 180,000
(ii) Gross profit on sales during 10 months period =8,00,000×30.425%=243,400
(iii)
Gross profit on annual adjusted Turnover
× Additional expenses
Gross profit on annual adjusted Turnover
+uninsured standard charges
3,86,400
× 1,80,000 = 1,71,142 (𝑎𝑝𝑝𝑟𝑜𝑥)
386,400 + 20000
Least i.e. 1,71,142 is admissible

3) Amount of Claim
Particulars Rs.
Gross profit on short sales = 2,60,000× 30.425% 79,105
Add: Amount allowable in respect of additional 1,71,142
expense
2,50,247
Less: Savings in Insured Standing Charges (28,000)
2,22,247
On the amount of final claim, the average clause will not apply since the amount of the
policy Rs.4,00,000 is higher than gross profit on annual adjusted turnover Rs.3,86,400.
Therefore the insurance claim will be Rs.2,22,247

7. Makalu Ltd. provides the following Trading and Profit & Loss Account for year ended 31st
December, 2010:
Memorandum Profit & Loss for the year ended 31st December, 2010

Rs Rs
To Opening Stock 50,000 By Sales 800,000
To Purchases 300,000 By Closing Stock 70,000
To Wages (Rs. 20000 for skilled labor) 160,000
To Manufacturing Expenses 120,000
To Gross Profit 240,000
Total 870,000 Total 870,000
To Office Administrative Expenses 60,000 By Gross Profit 240,000

© The Institute of Chartered Accountants of Nepal 168


CAP II Paper 1: Advanced Accounting

To Advertising 20,000
TO Selling Expenses ( Fixed) 40,000
To Commission on Sales 48,000
To Carriage Outward 16,000
To Net profit 56,000
Total 240,000 Total 240,000
The company had taken out policies both against Loss of Stock and against Loss of profit, the
amounts being Rs. 80,000 and Rs. 172,000. A fire occurred on 1st May, 2011 and as a result of
which sales were seriously affected for a period of 4 months. You are given the following
further information:
a) Purchases, wages and other Manufacturing Expenses for the first 4 months of 2010 were
Rs. 100,000, Rs. 50,000 and Rs. 36,000 respectively.
b) Sales for the same period were Rs. 240,000.
c) Other sales figures were as follows:
From 1st January, 2010 to 31st April, 2010 – Rs 300,000
From 1st May, 2010 to 31st August, 2010 – Rs. 360,000
From 1st May, 2011 to 31st August, 2011 – Rs. 60,000
d) Due to rise in wages, gross profit during 2011 was expected to decline by 2% on Sale.
e) Additional expenses incurred during the period after fire amounted to Rs. 140,000. The
amount of the policy included Rs. 120,000 for expenses leaving Rs. 20,000 uncovered.
Ascertain the claim for stock and for Loss of Profit. (Inter Dec. 2011 Q2-15 Marks)

Answer
A. Claim for Loss of Stock :
Memorandum Trading Account for the period 1st January to 1st May, 2011
Particulars Rs. Particulars Rs.
To Opening Stock 70000 By Sales 240000
To Purchases 100000 By Closing Stock ( Balancing 83200
figure)
To Wages 50000
To Manufacturing Expenses 36000
To Gross Profit ( 240000 X 67200
28% )
323200 323200

Claim for loss of Stock will be limited to Rs. 80000 which is the amount of Insurance Policy.

Working Notes :
1. Rate of Gross Profit in 2010
Gross Profit
Rate of Gross Profit = --------------- X 100
Sales

© The Institute of Chartered Accountants of Nepal 169


CAP II Paper 1: Advanced Accounting

24000
G. P. Rate = ------------ X 100 = 30%
80000
In 2010 Gross Profit had declined By 2% as a result of rise in wages, hence the rate of Gross
Profit for Loss of Stock is taken at 28%.

B. Claim for Loss Of Profit


Particular Rs
Sales from 1st May , 2010 to 31st August , 2010 360000
Less : 20% Decline observed in 2011 over – 2010 ( Jan – April Rs.240000 72000
instead of Rs. 300000)
Adjusted Standard Turnover 288000
Less : Sales from 1st May , 2011 to 31st August , 2011 60000
Short – Sales 228000
Loss Of Gross Profit on Short Sale = 20% on 228000 (WN 1) 45600
Amount Admissible for Additional Expenses (WN 3) 12000
Claim before application of Average Clause 57600

1. Computation of claim after application of Average clause –


Amount of policy 172000
Gross Profit on Adjusted Annual Turnover (WN 2 ) 128000
Here, the Average Clause will not apply since the amount of the policy Rs. 172000 is
higher than Gross Profit on Annual Turnover Rs. 128000.

Working Notes :
1 Gross Profit Ratio
Particular Rs
(a) Net Profit + Insured Standing Charges = 56000+120000 176000
(b) Turnover for the previous year 800000
(c) Gross Profit Ratio (a) ÷ (b) 22%
(d) Expected decrease due to increase in wages 2%
(e) Gross Profit Ratio © - (d) 20%

2 Computation of Gross Profit on Annual turnover – 12 months to 1st May, 2011


Particular Rs
Sales for Jan- Dec; 2010 800000
Less : From 1-12010 to 30.04.2010 300000
500000
Less : 20% Downward trend 100000
400000
Add: From 01.01.2011 To 30.04.2011 240000
Annual Turnover Adjusted for the 12 months ended 01.05.2010 640000

© The Institute of Chartered Accountants of Nepal 170


CAP II Paper 1: Advanced Accounting

Gross Profit on Annual Turnover @ 20% 128000

3 Amount allowable in respect of Additional Expenses (Least of the following)-


A. Actual Expenses = 140000
B. The amount obtained by applying the formula –
Gross Profit on Adj. Annual
Turnover
Additional Expenses Allowable = Additional Expenses X -------------------------------------
(Gross Profit as above +
Uninsured Standing
Charges)

128000
Additional Expenses Allowable = ------------------ = 121081
148000
C. Gross Profit on Increased Sale as a result of Increased Cost of Working = 20% X
60000= Rs. 12000 Least i.e. Rs. 12000 is admissible.

8. Given the following details, compute the consequential loss claim.


(Inter Jun. 2010 Q2- 15 Marks)
• Accounting year ends on 31 December with turnover and net profit of Rs. 300,000 and Rs
st

18,000 respectively.
• Fire occurs on 1st January and abnormality continues up to 30th May.
• Sum insured is Rs. 55,000 with an indemnity period of 6 months and subject to 'average
clause'
• Insured and uninsured standing charges figure Rs. 36,000 and Rs. 6,000 respectively.
• Standard turnover during 1st January to 30th May in the year preceding the fire is Rs. 112,500
and the actual turnover during the interruption period is Rs. 33,750.
• Increased cost of working is Rs. 6,000 with a saving of insured standing charges Rs. 600.
• Avoidance of reduction in turnover through increased working cost of Rs. 15,000.
• 'Special circumstances clause' allows for increase of turnover (standard and annual) and
G.P. rate, by 10% and 2% respectively.

Answers:
Rs
Standard turnover during 1st January to 30th May Last Year 112,500
Add: 10% Upward trend 11,250
Expected standard turnover during interruption period 123,750
Less: Actual sales during interruption period 33,750

Short sales or (Reduced turnover) 90,000

Insured G.P. rate

© The Institute of Chartered Accountants of Nepal 171


CAP II Paper 1: Advanced Accounting

Last accounting year's net profit + Insured standing charges


= x 100
Last accounting year's turnover

18000 + 36000
= x 100
300000
= 18%

Adjusted insured G.P, rate = 18% + 2% = 20%


Profit Lost = Short sales x adjusted G.P. rate
= 90000 x 20%
= 18000
Increased working cost (W.C.) of Rs 6,000 needs apportionment (owing to the uninsured
standing charges) by applying any of below mentioned formula:

Last financial year's net profit + Insured standing charges


a) Increased W. C [
]
Last financial year's net profit + Insurable standing charges

G.P on Annual Turnover


b) Increased Cost of Working = G.P on Annual Turnover +Uninsured Standing charges.

By Applying formula mentioned in (a) above. Appropriate cost of working is

18,000 + 36,000
Rs. 6,000 x = Rs.5,400
18,000 + 42,000

The above sum of Rs.5,400 is, however, subject to the ceiling of the amount derived from the
application of adjusted G.P. rate on the avoidance of reduction in sales, viz., 20% of Rs. 15,000.
or Rs. 3,000. Having regard to Rs. 600 saving in the insured standing charges, the assessed loss
is : Rs

Profit Lost 18,000


Add : Permissible increased working cost 3000
Less :- Saving in insured standing charges 600 2,400

Assessed loss or gross claim 20,400

Since the policy is subject to ' average clause', it is necessary to determine whether the policy
is underinsured by applying the formula:

© The Institute of Chartered Accountants of Nepal 172


CAP II Paper 1: Advanced Accounting

Adjusted G.P. Rate x Adjusted annual turnover

Though the problem is silent as to the annual turnover (Preceding the date of fire) yet it is equal
to the last accounting year's turnover of Rs. 300000 as the date of fire is January 1. Hence 12
month period preceding the date of fire coincides with the accounting year.

Applying 10% upward trend, the adjusted annual sales will be Rs. 330,000.

Accordingly, 20% of Rs. 330000 = Rs.66,000 (insurable policy amount)

The Policy should have been for Rs. 66,000 as against the actual insured sum of Rs.55,000.
Hence, the policy is underinsured and owing to the partial loss, the average clause is operative,
The final net claim is
55,000
Rs.20,400 x = Rs. 17,000
66,000
This final figure can alternatively be derived by the sum of: Rs.

Policy amount 90,000 x 55,000


i) Short sales x =
15,000
Adjusted annual sales 330,000

Insured Policy Amount 2,400 x 55,000


ii) Net increased working cost x =
2,000
Insurable Policy Amount 66,000
Final net claim
17,000

9. On 20th October, 2011 the godown and business premises of Aman Ltd. were affected by fire.
From the salvaged accounting records, the following information is available:
Rs.
st
Stock of goods @ 10% lower than cost as on 31 March,
216,000
2011
Purchases less return (01.04.2011 to 20.10.2011) 280,000
Sales less return (01.04.2011 to 20.10.2011) 620,000

Additional information:
i) Sales up to 20th October, 2011 include Rs. 80,000 for which goods had not been dispatched.
ii) Purchase up to 20th October, 2011 did not include Rs. 40,000 for which purchase invoice
had not been received from suppliers, though goods have been received in godown.
iii) Past records show the gross profit rate of 25%.

© The Institute of Chartered Accountants of Nepal 173


CAP II Paper 1: Advanced Accounting

iv) The value of goods salvaged from fire is Rs. 31,000.


v) Aman Ltd. has insured their stock for Rs. 100,000.
Compute the amount of claim to be lodged to the insurance company. 5 (Inter
Jun. 2012 5B

Answer
Memorandum Trading Account
(01.04.2011 to 20.10.2011)
Particulars Rs. Particulars Rs.
By Sales Rs. (620,000 –
To Opening stock 240,000 540,000
80,000)
To Purchases Rs. (280,000 +
320,000 By Closing stock – Bal. fig. 155,000
40,000)
To Gross profit – 25% of sales 135,000
695,000 695,000
Stock destroyed by fire:
Rs.
Stock on the date of fire 155,000
Less: Stock salvaged 31,000
Stock destroyed by fire 124,000

Loss of Stock
Insurance Claim =  Amount of Policy
Value of Stock on the date of fire
Rs. 124,000
=  Rs. 100,000 = Rs. 80,000
Rs. 155,000
Working note:
Stock on 1st April, 2011 was valued at 10% lower than cost. Hence, original cost of the
𝑅𝑠.216,000
stock as on 1st April, 2011 would be: = × 100 = 𝑅𝑠. 240,000
90
10. As a bad luck in the very beginning of new year, a fire occurred on 1 st Baishakh, 2066 in the
premises of Kanchenjunga Ltd. and business was partially affected up to 30th Bhadra, 2066.
The Company was insured under a loss of profits policy for Rs. 150,000 with a six months
period of indemnity. Following further information are available:
Rs.
st th
Actual turnover from 1 Baishakh, 2066 to 30 Bhadra 2066 100,000
st th
Turnover from 1 Baishakh, 2065 to 30 Bhadra, 2065 200,000
st st
Turnover from 1 Baishakh, to 31 Chaitra, 2065 450,000
Net profit for last financial year 86,000
Insured standing charges for last financial year 43,000
Turnover for the last financial year 430,000

It has been agreed that an adjustment of 20% be made in respect of the upward trend in
turnover.

© The Institute of Chartered Accountants of Nepal 174


CAP II Paper 1: Advanced Accounting

Required:
Computation of the amount of claim under the loss of profit policy.
(CAP Jun. 2010 2a- 10 Marks)
Answer:
a) Computation of the amount of claim for the loss of profit
Reduction in turnover Rs.
st th
Turnover from 1 Baishakh, 2065 to 30 Bhadra, 2065 200,000
Add: 20% expected increase 40,000
240,000
th
Less: Actual Turnover from 2066 to 30 Bhadra, 2066 100,000
Short Sales (Reduction in turnover) 140,000
Gross profit on reduction in turnover @ 30% on Rs.140000 42,000
(see working note 1 for gross profit)
Application of Average Clause:
Amount of Policy x Reduction in GP
GP on Annual Turnover
=(150000/162000) x 42000 38,888.88

Amount of claim under the policy = Rs. 38,888.88

Working notes
(i) Rate of Gross profit for last financial Year:
Gross profit: Rs.
Net Profit 86,000
Add: Insured Standing Charge 43,000
129,000
Turnover for the last financial year 430,000
Rate of Gross Profit = (129,000 / 430,000 ) x 100%

(ii) Annual Turnover and Gross Profit there on:


Turnover from 1st Baishakh 2065 to 31st Chaitra 2065 450,000
Add: 20% expected increase 90,000
540,000
Gross profit on Rs. 5,40,000 @ 30% 162,000

11. The premises of ABC Limited were partially destroyed by fire on 1st March, 2011 and as a
result, the business was practically disorganized up to 31st August, 2011. The company is
insured under a loss of profits policy for Rs. 1,65,000 having an indemnity period of 6 months.
From the following information, prepare a claim under the policy.
Rs.
Actual turnover during the period of dislocation (1-3-2011 to 31-8-2011) 80,000
Turnover for the corresponding period (dislocation) in the 12 months
immediately before the fire (1-3-2010 to 31-8-2010) 2,40,000

© The Institute of Chartered Accountants of Nepal 175


CAP II Paper 1: Advanced Accounting

Turnover for the 12 months immediately preceding the fire (1-3-2010 to


28-2-2011) 6,00,000
Net profit for the last financial year 90,000
Insured standing charges for the last financial year 60,000
Uninsured standing charges 5,000
Turnover for the last financial year 5,00,000

Due to substantial increase in trade, before and up to the time of the fire, it was agreed that an
adjustment of 10% should be made in respect of the upward trend in turnover. The company
incurred additional expenses amounting to Rs. 9,300 immediately after the fire and but for this
expenditure, the turnover during the period of dislocation would have been only Rs. 55,000.
There was also a saving during the indemnity period of Rs. 2,700 in insured standing charges
as a result of the fire.
(CAP Jun. 2012 Q2b – 10 Marks)
Answer
Computation of insurance claim for Loss of Profit:
Rs.
1. Rate of Gross Profit:
Net profit for the last financial year 90,000
Add: Insured standing charges 60,000
150,000
Turnover for the last financial year 500,000
Rate of gross profit [Rs. 150,000 / Rs. 500,000 × 100] = 30%
2. Short sales:
Standard turnover 240,000
Add: 10% increasing trend 24,000
264,000
Less: Actual turnover for the period of dislocation 80,000
184,000
3. Annual adjusted turnover:
Annual turnover [1.3.2010 to 28.2.2011] 600,000
Add: 10% increasing trend [See Note] 60,000
660,000
4. Additional expenses:
(a) Actual expenses 9,300
(b) Gross profit on sales generated by additional expenses [WN 1] 7,500
(c) [30% of Rs. 660,000 / (30% of Rs. 660,000 + Rs. 5,000)] × Rs. 9,300 9,071
Allowable additional expenses – Least of the above three figures 7,500
5. Loss of profit:
Loss of profit on short sales [Rs 184,000 × 30%] 55,200
Add: Allowable expenses 7,500
62,700
Less: Savings in insured standing charges 2,700

© The Institute of Chartered Accountants of Nepal 176


CAP II Paper 1: Advanced Accounting

Loss of profit 60,000

6. Claim to be lodged on application of average clause:


165,000
= 𝑅𝑠. [198,000 × 60,000]
= Rs 50,000

Note:
It is assumed that trend adjustment is required on total amount of annual turnover. However,
part of the annual turnover represents trend adjusted figure. Alternatively, the students may
ignore trend and take simply annual turnover. The claim would be Rs. 55,000 which is more
than the claim computed in (6) above. So the Insurance Company would insist on trend adjusted
on annual turnover.
Working Note:
1. Gross profit on sales generated by additional expenses:
Rs.
Actual turnover during the period of dislocation 80,000
Expected turnover if no additional expenses were incurred 55,000
Sales attributable to additional expenses 25,000
Gross profit @ 30% 7,500

© The Institute of Chartered Accountants of Nepal 177


CAP II Paper 1: Advanced Accounting

12. A fire broke out in the godown of a business house on Shrawan 08, 2068. Goods costing Rs.
203,000 in a small sub-godown remain unaffected by fire. The goods retrieved in a damaged
condition from the main godown were valued at Rs. 197,000. The following particulars were
available from the books of account:

Stock on the last balance sheet date at 30.03.2068 was Rs. 1,572,000
Purchases for the period from Shrawan 01 to Shrawan 07 were Rs. 3,710,000
Sales during the same period amounted to Rs. 5,260,000
The average gross profit margin was 30% on sales.

The business house has a fire insurance policy for Rs. 1,000,000 in respect of its entire stock.
Assist the accountant of the business house in computing the amount of claim of loss by fire.
(CAP Jun. 2013 Q4a– 7 Marks)
Answer:
Calculation of Amount of Claim Amount Amount
Value of Stock on 8th Shrawan 16,00,000
(Refer W.N.)
Less:
Value of Stock remaining unaffected by fire. 2,03,000
Agreed value of damaged goods 1,97,000 4,00,000
Loss of Stock 12,00,000

Applying Average Clause:


Amount of Claim=Amount of Policy x Loss of Stock
Stock on the date of Fire
=Rs 10,00,000x12,00,000
Rs16,00,000
=Rs 750,000

Working Note:
Memorandum Trading Account for the Period from 1St Shrawan 2068 to 8Th Shrawan2068
Particulars Amount Particulars Amount
To Opening Stock 15,72,000 By Sales 52,60,000
To Purchases 37,10,000 By Closing Stock (Bal. Fig) 16,00,000

To Gross Profit (30% of Sales) 15,78,000

68,60,000 68,60,000

13. A fire occurred in the premises of M/s Careless Co. on 30th Mangsir 2070. From the following
particulars, relating to the period 1st Shrawan 2070 to 30th Mangsir 2070. You are required to
ascertain the amount of claim to be filed with the insurance company for the loss of stock.
Rs.

© The Institute of Chartered Accountants of Nepal 178


CAP II Paper 1: Advanced Accounting

i) Stock as per Balance Sheet as at 31st Ashadh 2070 99,000


ii) Purchases (including purchase of a machinery
Costing Rs. 30,000) 170,000
iii) Wages (including wages for the installation
of Machinery Rs. 3,000) 50,000
iv) Sales (including goods sold on approval basis amounting
to Rs. 49,500. No confirmation had been received in
respect of two-thirds of such Goods sold
on approval basis. 275,000
v) Sales value of goods drawn by proprietor 15,000
vi) Cost of goods sent to consignee on 15 Mangsir 2070 lying unsold
With them 16,500
vii) Sales value of goods distributed as free samples 1,500
The average rate of gross profit had been 20% in the past. This selling price had been
increased by 20% with effect from 1st Shrawan 2070.
For valuing the stocks for the Balance Sheet as at 31st Ashadh 2070, Rs. 1,000 had been
written off in respect of a slow moving item, the cost of which was Rs. 5,000. A portion of
those goods were sold at a loss of Rs. 500 on the original cost of Rs. 2,500. The remainder
of the stock was now estimated to be worth the original cost.
Subject to the above exceptions, the gross profit had remained at a uniform rate throughout.
The value of goods salvaged was estimated at Rs. 25,000. The enterprise had taken an
insurance policy for Rs. 60,000 which was subject to the average clause.
(CAP Jun. 2014 Q2b- 10 Marks)
Answer:

Memorandum Trading Account for the period from 1.4.2070 to 30.8.2070


Particulars Normal Abnormal Particulars Normal Abnormal
Rs. Rs. Rs. Rs.
Opening Stock
(W.N.1) 95000 5000 Sales (W.N. 2&1) 240000 2000
Stock lying with
customers on
Purchases 170000 approval (W.N.3) 22000
Less Machinery Goods sent to
purchase 30000 140000 consignee 16500
Goods drawn by
Wages 50000 proprietor 10000
Less: wages for instl. Free samples (W.N.
of machinery 3000 47000 5) 1000
Gross Profit (33.33%
of sales)
(W.N. 4) 80000 Loss (W.N.) 500

© The Institute of Chartered Accountants of Nepal 179


CAP II Paper 1: Advanced Accounting

Stock on the date of


_________ _________ fire 72500 2500
362000 5000 362000 5000

Working Notes
(1) Abnormal items: Rs.
Original cost of slow moving items 5000
cost of slow moving item sold 2500
Balance of slow moving items in stock 2500
Original cost of slow moving items sold 2500
Loss incurred on such sale 500
Sale proceeds of slow moving items 2000

(2) Sales as given 275000


Less: goods sold on approval basis 49500
225500
Add: sales confirmed with respect
to goods sold on approval basis
'/3 x 49,500 16500
Total sales for the period 242000
Less: sale of abnormal items 2000
Normal sales 240000

(3) Goods sold on approval basis 49500


Less: sales confirmed 16500
Goods with customers on approval basis
-sales not being confirmed 33000
Less: profit element 1/3 of Rs. 33,000 11000
Goods on approval at cost 22000

(4) Previous year G.P. margin on sales = 20%


Cost 100-20 = 80
Selling price in the current year 100+20 120
G.P Margin in current year 40/120 x 100 33.33%

(5) Sales value of free samples distributed 1500


Less: Profit on such sale value @ 33.33% 500
Cost of sample distribution 1000

Statement of claim Rs.


Stock on the date of fire
Normal items 72500
Slow moving items 2500

© The Institute of Chartered Accountants of Nepal 180


CAP II Paper 1: Advanced Accounting

75000
Less Salvage 25000
50000
Claim (applying average clause
Value of Policy/Stock on the fire date x loss of stock
= 60000/75000 x 50000 =Rs. 40000

14. A Ltd. prepares accounts on 30th September each year, but on 31st December 2013 fire
destroyed the greater part of its stock .Following information was collected from books of
account:

Stock as 1.10 .2013 Rs. 29,700,000


Purchase from 1.10.2013 to 31.12.2013 Rs. 75,000,000
Wages from 1.10.2013 to 31.12.2013 Rs. 33,000,000
Sales from 1.10.2013 to 31.12.2013 Rs. 140,000,000

The rate of gross profit is 33.33% on cost .Stock to the value of Rs. 3,000,000 was salvaged
.Insurance policy was for Rs. 25,000,000.
Additional Information:
i) Stock in the beginning was calculated at 10% less than cost.
ii) A plant was installed by firm’s own workers. They were paid Rs. 500,000 which was
included in the wages.
iii) Purchase includes the purchase of plant of Rs. 5,000,000.
You are required to calculate the claim for the loss of stock.
(CAP Jun. 2015 Q3c - 8 Marks)
Answer
Computation of claim for loss of stock
Stock on the date of fire (31.12.2013) Rs. 30,500,000
Less salvage stock Rs. 3,000,000
Loss of Stock Rs. 27,500,000
Amount of claim = insured value/cost of stock on date of fire *loss of stock
= 25,000,000/30,500,000*27,500,000
= 22,541,000
Working Note
Memorandum Trading Account for period from 1.10.2013 to 31.12.2013
Particulars Rs. Particulars Rs.
To opening 33,000,000 By sales 140,000,000
stock(29700000*100/90)
To Purchase 75,000,000 By Closing 30,500,000
Less cost of plant 5,000,000 70,000,000 Stock(balancing figure)
To wages 33,000,000 32,500,000
Less wages for plant 500,000

© The Institute of Chartered Accountants of Nepal 181


CAP II Paper 1: Advanced Accounting

To gross profit (33.33% of cost or 35,000,000


25% of sale)
Total 170,500,000 Total 170,500,000

15. D Ltd., which operates a wholesale business, had a fire on premises on September 17, 2014,
which destroyed most of the premises, although stock to the value of Rs. 3,960 was salvaged.
The company has an insurance policy (with suitable average clauses) covering Stocks for Rs.
600,000, Building for Rs. 800,000, and Loss of Profits including standing charges for Rs.
250,000 with a six months period of indemnity.
The company’s last Profit & Loss Account, for the year ended 16th July, 2014, showed the
following position:

DR CR
Particulars Amount (Rs.) Particulars Amount (Rs.)
To Opening Stock 412,500 By Sales 2,000,000
To Purchases 1,812,500 By Stock 525,000
To Insured Standing Charges 167,500 By Interest 5,000
To Other Expenses 80,000
To Net Profit for the Year 57,500
2,530,000 2,530,000

The company’s records show that the sales for period up to September 17, 2014 had been the
same as for the corresponding months in the previous year at Rs. 100,000, payments made to
trade creditors in October-November were Rs 106,680 and at the end of that month the
balances owing to trade creditors had increased by Rs. 3,320. The company’s business was
disrupted until the mid of December 2015, during which period turnover fell by Rs. 180,000
compared with the same period in the previous year. It was agreed that three quarters of the
value of the building had been lost and that at the time of the fire it had been worth Rs.
1,000,000.
Ascertain the amount of various claims to be lodged with the insurers. (CAP Jun. 2016
Q3a)
Answer

Loss of Stock
Trading Account
(for the period ended September 17, 2014)
Dr Cr
Particulars Amount Particulars Amount
To Opening Stock 525,000 By Sales 100,000
To Purchases 110,000 By Closing Stock
To Gross Profit @ 15% Sales Rs.
100,000 15,000 (Balancing Fig.) 550,000

© The Institute of Chartered Accountants of Nepal 182


CAP II Paper 1: Advanced Accounting

650,000 650,000

Estimated Stock at the time of fire Rs.550,000


Less: Salvaged Stock Rs. 3,960
Loss of Stock: Rs. 546,040

Working Notes:
1. Gross Profit shown by Last Year’s Accounts is Rs 300,000 against Rs. 2,000,000 Sales
i.e. 15% of Sales. In absence, of any information; the same GP % is assumed for
current year also.
2. Purchases = Payment to Creditors + Increase in Creditors Balance = 106,680 + 3,320 =
110,000.

Building
Loss: Three quarters of Rs. 1,000,000 : Rs. 750,000
The building was under-insured, as the insurance cover was for Rs. 800,000 only; Hence,
Average Clause is applicable. Insured Covered = 800,000 / 1,000,000 X 100% = 4/5 0or
80%.
Therefore, the amount to be claimed= 4/5 of 750,000 =Rs. 600,000

Loss of Profit:
Indemnity Period: September 17 to December 15 i.e. 3 Months
Profit Ratio for the year ending July 16, 2014
Net Profit (Less Interest) + Insured Standing Charges * 100
Sales

= (57,500-5,000) + 167,500 * 100


2000000

= 11%.
As Short Sales = Rs. 180,000,
Claim for Loss of Profit = 11% of 180,000 = Rs. 19,800.

16. A fire engulfed in the premises of a business of M/s K on the morning of 1st July 2016. The
building, equipment and stock were destroyed and the salvage recorded the following:
Building Rs. 4,000, Equipment Rs. 2,500 and Stock Rs. 20,000. The following other
information was obtained from the records saved for the period from 1st January 2016 to 30th
June 2016.

Particulars Amount (Rs.)


Sales 11,10,000
Purchases 9,37,500
Cartage Inward 17,500

© The Institute of Chartered Accountants of Nepal 183


CAP II Paper 1: Advanced Accounting

Wages 7,500
Stock in hand 31st December 2015 1,50,000
Building (Value on 31st December 2015) 3,75,000
Equipment (Value on 31st December 2015) 75,000
Depreciation provision till 31st December 2015 on:
Building 1,25,000
Equipment 22,500

No depreciation has been provided since 31st December 2015. The latest rate of depreciation is
5% p.a on building and 15% p.a on equipment by straight line method. Normally business
makes a profit of 25% on sales. You are required to prepare the statement of claim for
submission to the insurance company.
(CAP Jun. 2017 Q3b-5 Marks)
Answer

Items Cost Rs. Depreciation Rs. Salvage Claim Rs.


Rs.
Stock (w.n 2) 2,80,000 20,000 2,60,000
Building 3,75,000 1,25,000 + 9,375 4,000 2,36,625
Equipment 75,000 22,500 + 5,625 2,500 44,375
Claim Amount 5,41,000

Working Note:
1. Memorandum Trading Account for the period from 1.1.2016 to 30.6.2016
Particulars Rs. Particulars Rs.
To Opening Stock 1.1.2016 1,50,000 By Sales 11,10,000
To Purchases 9,37,500 By Closing Stock 2,80,000
To Cartage Inward 17,500 ( Balancing figure)
To Wages 7,500
To Gross Profit (25% of 11,10,000) 2,77,500
13,90,000 13,90,000

2. Stock Destroyed Account


Particulars Rs. Particulars Rs.
To Trading Account 2,80,000 By Stock Salvage Account 20,000
By Balance c/d (For Claim) 2,60,000
280,000 280,000

17. A firm has the practice of valuing its stock at cost less 10%. A fire occurred in the godown of
the firm on Baishakh 2067 (financial year 2066/67) destroying all stocks except for Rs.62,000.
The stocks were insured. Following information is available:
Rs.

© The Institute of Chartered Accountants of Nepal 184


CAP II Paper 1: Advanced Accounting

Opening Stock of F.Y.2065/66 360,000


Purchases during F.Y.2065/66 1,450,000
Sales during F.Y.2065/66 2,000,000
Closing Stock of F.Y.2065/66 225,000
Purchases during F.Y.2066/67 till the date of fire 1,460,000
Sales during F.Y.2066/67 till the date of fire 1,890,000
In Shrawan 2066, the prices were raised by 5%.

Required:
Calculate the value of stock loss. (Inter Dec. 2010 Q2a-10 Marks)

Answer
Finding the rate of Gross Profit in F.Y.2065/66
Rs.
Sales 2,000,000
Less: Cost of Goods sold
Opening Stock (360,000*10/9) 400,000
Purchases 1,450,000
Closing Stock (225,000*10/9) (250,000) 1,600,000
Gross Profit 400,000
Rate of GP on sales = (400,000/2,000,000)*100= 20%

Note: Since the stock was valued at cost less 10%, the value of given opening and closing stock
are only 90% of actual. Therefore, they are increased by 1/9th to bring them to actual cost.
(i) Finding the rate of Gross Profit in F.Y.2066/67
Rs.
Sale (per Rs.100 of 2065/66) 105 (after price rise by 5%)
Cost of goods sold 80
Gross Profit 25
Rate of GP on Sales= (25/105)*100 = 23.81%

(ii) Finding the value of closing stock of 2066/67 till the date of fire and value of loss
Rs.
Sales 1,890,000
Less:
Opening Stock (250,000)
Purchases (1,460,000)
Gross Profit (@23.81% on Sales) (450,000)
Closing Stock 270,000
Less: value of stock left out (62,000)
Amount of claim to be lodged 208,000

18. Janta Trading Ltd. gives the following Trading, Profit and Loss Account for the year ended 31st
December 2003.

© The Institute of Chartered Accountants of Nepal 185


CAP II Paper 1: Advanced Accounting

Trading, Profit and Loss Account for the year ended 31st December 2003

Rs Rs
To opening stock 50,000 By Sales 800,000
To Purchase 300,000 By Closing Stock 70,000
To wages (Rs.20,000 for skilled labor) 160,000
To Manufacturing Expenses 120,000
To Gross Profit 240,000
870,000 870,000
To Office Administrative Expenses 60,000 By Gross Profit 240,000
To Advertising 20,000
To Selling Expenses (Fixed) 40,000
To Commission on sales 48,000
To Carriage outward 16,000
To Net Profit 56,000
240,000 240,000
The company had taken out policies both against loss of stock and against loss of profit for
Rs.80,000 and Rs.172, 000 respectively. A fire occurred on 1st may 2004 and as a result of
which sales were seriously affected for a period of a 4 months. The following information is
available:

Purchase wages and other manufacturing expenses for the first four months of 2004 were
Rs.100, 000, Rs.50, 000 and Rs.36, 000 respectively.
Sales for the same period were Rs.240, 000
Other sales figures were as follows:
Rs.
st th
From 1 January 2003 to 30 April 2003 300,000
st st
From 1 May 2003 to 31 August 2003 360,000
st st
From 1 May 2004 to 31 August 2004 60,000
Due to rise in wages, gross profit during 2004 was expected to decline by 2% on sales.

Additional expenses incurred during the period after fire, amounted to Rs. 140,000. the amount
of the policy included Rs.120, 000 for expenses leaving Rs.20, 000 uncovered. Ascertain the
claim for loss of stock and loss of profit. (All working should form part of your answer)
(Inter June 2005, Q 4)
Answer:
Claim for Loss of Stock

Memorandum Trading Account


For the period 1st January to 1st May 2004
Rs Rs
To Opening Stock 70,000 By Sales 240,000

© The Institute of Chartered Accountants of Nepal 186


CAP II Paper 1: Advanced Accounting

To Purchases 100,000 By Closing Stock 83,200


(Balance figure)
To wages 50,000

To Manufacturing Expense 36,000


To Gross Profit (2.8% on sales) 67,200
323,200 323,200
Claim for loss of stock will be limited to Rs.80, 000 which is the amount of insurance policy.
Workings:
Rate of Gross Profit in 2003
=Gross Profit x 100
Sales
=240,000 x 100
800,000
= 30%
As the gross profit had declined by 2% as a result of use in wages, hence the rate of Gross
Profit for loss of stock is taken at 28%.
Claim for Loss of Stock
Short Sales
Sales from 1st May 2003 to 31st August 2003 360,000
Less: 20% decline observed in 2004 over 2003 (Jan-April 240,000 instead of Rs. 72,000
300,000)
288,000
st st
Less: Sales from 1 May 2004 to 31 August 2004 60,000
Short Sales 228,000

Gross Profit Ratio


Gross Profit Ratio = Net Profit+ Insured Standing Charges (2003) x 100
Sales(2003)
= 56000 + 120,000 x 100 = 22%
800,000
Less: expected decline 2%
20%
Loss of Gross Profit
20% on short sales (228,000)

Annual Turnover (12 months to 1st May 2004)


Rs.
Sales for Jan – Dec 2003 800,000
Less: from 1-1-2003 to 30-4-2003 300,000
500,000
Less: 20% downward trend 100,000

© The Institute of Chartered Accountants of Nepal 187


CAP II Paper 1: Advanced Accounting

400,000
Add: from 1-1-2004 to 30-4-2004 240,000
640,000
Gross Profit @ 20% 128,000

Amount allowable in respect of additional expenses: least of the following:-


i. Actual Expenses Rs. 140,000
ii. Gross Profit on sales during the identity fund (60000 x 20%) Rs.12, 000
iii. Gross Profit on annual (adjusted ) turnover x Additional Expenses
Gross Profit as above + uninsured charges.

= 128,000 x 140,000 = 121,081


148,000

Least Rs. 12,000 is admissible


Remark: On the amount of final claim, the average claim will not apply since the amount of
the policy (Rs.172, 000) is higher than Gross Profit (Rs.128, 000).

19. A fire occurred on 01-02-2007 in the premises of Z Ltd. and business was partially disorganized
up to 30-06-2007.

From the books of accounts, the following information was extracted:

Rs.
i. Actual turnover from 01-02-2007 to 30-06-2007 150,000
ii. Turnover from 01-02-2006 to 30-06-2006 420,000
iii. Turnover from 01-02-2006 to 31-01-2007 900,000
iv. Net profit for last financial year 140,000
v. Insured standing charges for the last financial year 112,000
vi. Total standing charges for the year 128,000
vii. Turnover for the last financial year 840,000

The company incurred additional expenses amounting to Rs. 13,400 which reduced the loss in
turnover. There was also a saving during the indemnity period of Rs. 4,900 in the insured
standing charges as a result of the fire.

The company holds a “Loss of Profit” policy for Rs. 248,400 having an indemnity of 6 months.
There has been a considerable increase in trade since the date of the last annual accounts and
it had been agreed that an adjustment of 15% be made in respect of the upward trend in
turnover.

Compute the claim under the policy. (Inter June 2007, Q 4,)

Answer:

© The Institute of Chartered Accountants of Nepal 188


CAP II Paper 1: Advanced Accounting

i. Calculation of short sales: Rs.


Turnover from 01-02-2006 to 30-06-2006 4,20,000
Add: 15% Upward trend in turnover 63,000
Expected Turnover 4,83,000
Less: Actual turnover from 01-02-2007 to 30-06-2007 1,50,000
Short Sales 3,33,000
ii. Rate of gross profit :
𝑁𝑒𝑡 𝑝𝑟𝑜𝑓𝑖𝑡 𝑓𝑜𝑟 𝑙𝑎𝑠𝑡 𝑓𝑖𝑛𝑎𝑛𝑐𝑖𝑎𝑙 𝑦𝑒𝑎𝑟 + 𝐼𝑛𝑠𝑢𝑟𝑒𝑑 𝑠 𝑡𝑎𝑛 𝑑 𝑖𝑛𝑔 𝑐ℎ 𝑎𝑟𝑔 𝑒 𝑠
× 100
𝑇𝑢𝑟𝑛𝑜𝑣𝑒𝑟 𝑓𝑜𝑟 𝑡ℎ𝑒 𝑙𝑎𝑠𝑡 𝑓𝑖𝑛𝑎𝑛𝑐𝑖𝑎𝑙 𝑦𝑒𝑎𝑟

𝑅𝑠.140,000+𝑅𝑠.112,000
= × 100 = 30%
𝑅𝑠.840,000

iii. Amount of gross claim: Rs.


Gross profit on short sales (30% of Rs. 3,33,000) 99,900
Add: Admissible additional expenses 12,744
1,12,644
Less: Saving in insured standing charges 4,900
1,07,744

iv. Application of Average Clause: Rs.


Annual turnover i.e. turnover from 01-02-2006 to 31-01-2007 9,00,000
Add: 1,35,000
Adjusted annual turnover 10,35,000
Gross profit on adjusted annual turnover 3,10,500
Loss of profit policy value 2,48,400

Since, the policy value is less than gross profit on adjusted annual turnover, the average clause
is applicable.
𝑅𝑠.2,48,400
Hence, the amount of net claims = 𝑅𝑠.3,10,500 × 𝑅𝑠. 1,07,744 = 𝑅𝑠. 86,195

Working Notes:
1. Admissible additional expenses: The additional expenses admissible is limited to the lower
of the following:-
a. Gross profit earned on sales during disturbed period arising from additional
expenses = 30% of Rs. 1,50,000* = 45,000
𝐺𝑟𝑜𝑠𝑠  𝑃𝑟 𝑜𝑓𝑖𝑡 𝑜𝑛 𝐴𝑛𝑛𝑢𝑎𝑙 𝑇𝑢𝑟𝑛𝑜𝑣𝑒𝑟
b. Additional expenses × 𝐺𝑟𝑜𝑠𝑠  𝑃𝑟 𝑜𝑓𝑖𝑡 𝑜𝑛 𝐴𝑛𝑛𝑢𝑎𝑙 𝑇𝑢𝑟𝑛𝑜𝑣𝑒𝑟+𝑈𝑛𝑖𝑛𝑠𝑢𝑟𝑒𝑑 𝑆 𝑡𝑎𝑛 𝑑𝑖𝑛𝑔 𝑐ℎ 𝑎𝑟𝑔 𝑒𝑠
𝑅𝑠.3,10,500
= 𝑅𝑠. 13,400 × 𝑅𝑠.3,10,500+𝑅𝑠.16,000
= Rs. 12,744
* In the absence of information it is assumed that the entire sales during disturbed
period were generated due to additional expenses.
2. In the absence of information regarding the period of last accounting year, the upward
trend has been adjusted on the entire annual turnover.

© The Institute of Chartered Accountants of Nepal 189


CAP II Paper 1: Advanced Accounting

20. PQR Ltd. give the following Trading and Profit and Loss Account for the year ended 32 Ashadh
2063.
TRADING AND PROFIT AND LOSS ACCOUNT
for the year ended 32 Ashadh 2063
Rs. Rs.
To Opening Stock 50,000 By Sales 800,000
To Purchases 300,000 By Closing Stock 70,000
To Wages (NRs. 20,000 for skilled
labor) 160,000
To Manufacturing Expenses 120,000
To Gross Profit 240,000
870,000 870,000

To Office Administrative Expenses


60,000 By Gross Profit 240,000
To Advertising 20,000
To selling expenses (Fixed) 40,000
To commission on sale 48,000
To carriage outwards 16,000
To net profit 56,000
240,000 240,000

The company had taken over policies both against loss of stock and against loss of profit, the
amounts being Rs. 80,000 and Rs. 172,000. Fire occurred on 1st Mangsir 2063 and as a result
of which sales were seriously affected for the period of four months. You are given following
further information:

1) Purchases, wages and other manufacturing expenses for the first four months of fiscal year
2062/63 were Rs. 100,000, Rs. 50,000 and Rs. 36,000 respectively.
2) Sales for the same period were Rs. 240,000.
3) Other sales figures were as follows:
From 1st Shrawan 2062 to Kartik end 2062 Rs. 300,000
st
From 1 Mangsir 2062 to Fagun end 2062 Rs. 360,000
st
From 1 Mangsir 2063 to Fagun end 2063 Rs. 60,000

4) Due to rise in wages, net profit during fiscal year 2063/64 was expected to decline by 2
percent on sales.

Additional expenses incurred during the period after fire amounted to Rs. 140,000. The
amount of the policy included Rs. 120,000 for expenses leaving Rs. 20,000 uncovered.
Ascertain the claim for stock and for loss of profit. (Inter Dec. 2007, Q 3-15 Marks)

© The Institute of Chartered Accountants of Nepal 190


CAP II Paper 1: Advanced Accounting

Answer:
Calculation of Amount of claim for Loss of Stock
Gross profit ratio in fiscal year Gross Profit for 2062/063
X 100
2062/063 Sales for 2062/063
240,000
= X 100
800,000
= 30%

In fiscal year 2063/64, gross profit and net profit is expected to decline by 2% because of rise
in wages, hence the rate of gross profit will be 28% (30% - 2%)

MEMORANDUM TRADING ACCOUNT


for the period 1st Shrawan 2063 to Kartik end 2063

NRs. NRs.
To Opening Stock on 2063/04/01 By Sales 240,000
70,000
To Purchases 100,000 By Closing Stock 83,200
(balancing figure)
To Wages 50,000
To Manufacturing Expenses 36,000
To Gross Profit (28% of Sales) 67,200
323,200 323,200

Value of stock destroyed by fire is NRs. 83,200 but claim for loss of stock will be limited to
NRs. 80,000 which is the amount of insurance policy.

Calculation of Loss of Profit


(i) Short Sales
Sales from 1st Mangsir 2062 to Fagun end 2062 360,000
Less: 20% decline observed in 2063/63 over 2062/63
(2063/04/01 to Kartik end 2063 NRs. 240,000 instead of NRs. 300,000
for the same period of previous year) 72,000
Turnover expected during dislocation period 288,000
st
Less: Actual Sales from 1 Mangsir 2063 to Fagun end 2063 60,000
Short Sales 228,000
(ii) Gross Profit Ratio
Net Profit (2062/63)+Insured Standing
= Charges
Sales (2062/63)
= NRs. 56,000 + NRs. 120,000 = 22%

© The Institute of Chartered Accountants of Nepal 191


CAP II Paper 1: Advanced Accounting

NRS. 800,000
Less: Expected decrease due = 2%
to increase in wages
Gross Profit Ratio 20%
(iii) Loss of Gross Profit on Short Sales
20% of NRs. 228,000 Short Sales NRS. 45,600
(iv) Calculation of Preceding 12 months’ Adjusted Sales
NRs.
Sales for the fiscal year (2062/63) 800,000
Less: Sales from 1st Shrawan 2062 to Kartik end 2062 300,000
500,000
Less: 20% downward trend 100,000
4,00,000
Add. sales for 1st Shrawan 2063 to Kartik end 2063 2,40,000
Adjusted sales 6,40,000
Gross profit on annual adjusted turnover @ 20% 128,000

(v) Calculation of Amount Allowable for Additional Expenses Least of the following:

NRs.
(a) Actual Expenses 140,000
(b) Gross Profit @ 20% on actual sales of NRs. 60,000 during the indemnity
period 12,000
(c) Additional Gross Profit on adjusted sales
Expenses X Gross Profit + Uninsured expenses as above
128,000
= 140,000 X
128,000 + 20,000
128,000 =121,081
= 140,000 X
148,000
Lowest of above three figures is NRs.12,000, so claim allowable for additional expenses is for
NRs. 12,000

(vi) Calculation of Amount of Claim for Loss of Profit


NRs.
Gross Profit on Short Sales as calculated in (iii) 45,600
Add: Additional Expenses allowable 12,000
Amount of Claim 57,600

21. A fire occurred in the premises of Nepal Trading on 31st January 2003 and destroyed most of
the building. The salvage value of stock was Rs. 6,000.

The insurance policy of Nepal Trading covers the following:


Rs.

© The Institute of Chartered Accountants of Nepal 192


CAP II Paper 1: Advanced Accounting

Stock 3,00,000
Building 6,00,000
Loss of Profit (including standing charges) 1,87,500
Period of indemnity 6 months

The summarized profit and loss account for the year ended 31st December 2002 is as follows:
Rs.

Turnover 15,00,000
Closing Stock 3,93,750
18,93,750
Opening Stock 3,09,375
Purchases 13,59,375
Standing Charges (Insured) 1,25,625
Variable Expenses 60,000 18,54,375
NET PROFIT 39,375

The transactions for the month of January 2003 were:


Rs.
Turnover 75,000
Payment to creditors 80,000

Trade creditors
Balance on 1.1.2003 1,13,000
Balance on 31.1.2003 1,15,500

The business was disorganized until 30.5.2003. During the period of disruption, the reduction
in turnover amounted to Rs. 1,35,000 as compared to corresponding period of previous years.

Prepare the claim to be lodged for loss of stock and loss of profit to the insurance company.
(Inter Dec. 2003 Q 3-16 Marks)
Answer:
Computation of claim for loss of stock

Trading Account for the year ended 31st December 2002


(To determine the rate of Gross Profit)
Rs. Rs.

To Opening Stock 3,09,375 By Sales 15,00,000


To Purchases 13,59,375 By Closing Stock 3,93,750
To Gross Profit 2,25,000 ________
18,93,750 18,93,750

© The Institute of Chartered Accountants of Nepal 193


CAP II Paper 1: Advanced Accounting

225000
Rate of Gross Profit = 1300000  100 = 15%

Memorandum Trading Account


For the month of January 2003
Rs. Rs.

To Opening Stock 3,93,750 By Sales 75,000


To Purchases 82,500 By Closing Stock 4,12,500
To Gross Profit (15%) 11,250 (Balancing figure) ________
4,87,500 4,87,500

Closing Stock (on 31.1.2003) 4,12,500


Less: Stock Salvaged 6,000
Value of stock destroyed 4,06,500

Insurance Policy Amount


Claim for stock = Value of stock destroyed  Value of Stock on date fine
300000
= 4,06,500  412500 Say in 2,95,636

Computation of claim for loss of profit:


(Net Profit + Insured Standing Charges)
Gross Profit Ratio = 100  Sales
(39375 + 125625)
= 100  = 11%
1500000
Rs.

Short Sales (given) 1,35,000


Gross Profit on short sales @ 11% 14,850

Claim for consequential loss of profit in Rs. 14,850 as there is no need to apply average claim.

© The Institute of Chartered Accountants of Nepal 194


CAP II Paper 1: Advanced Accounting

22. A fire destroyed the premises of a business man on 1st July 2000. From the following
information, prepare a statement of the claim to be submitted to the Insurance company:
Rs.

Opening Stock 1.1.99 (including obsolete item of Rs. 6,000) 72,000


Purchases for 1999 3,54,000
Carriage Inwards 6,000
Sales for 1999 (including sale of entire obsolete item of 1.1.99
for Rs. 5,000 and sale of scrap Rs. 7,000) 5,12,000
Closing stock 31.12.99 88,000
Purchases 1.1.2000 to 30.6.2000 2,10,000
Carriage Inwards 4,000
Sales 1.1.2000 to 30.6.2000 (including sale of scrap Rs. 3,000) 3,03,000

Opening and Closing Stock of 1999, except the obsolete item have been overvalued by 10%.
The Salvage was Rs. 9,000. The policy amount was Rs. 50,000 subject to average clause.
(Inter Jun. 2001 Q4a 10 Marks)
Answer:
Memoranda Trading Account 1999
Dr. Rs. Cr. Rs.
To Opening Stock 72,000 By Sales 5,12,000
Less: Obsolete item 6,000 Less: Obsolete item 5,000
66,000 5,07,000
Less: Over valuation Less: Sale of scrap 7,000
1/11 of 66,000 6,000 5,00,000
60,000 By Closing Stock
To Purchases 3,54,000 (88,000  20/22) 80,000
To Carriage Inwards 6,000
To Gross Profit (32%) 1,60,000
5,80,000 5,80,000

Pro-Forma Trading Account 1/1/2000 to 30/6/2000

Dr. Rs. Cr. Rs.


To Opening Stock 80,000 By Sales 3,03,000
To Purchases 2,10,000 Less: Sale of scrap 3,000
To Carriage Inwards 4,000 3,00,000
To Gross Profit 32% 96,000 By Closing Stock 90,000
3,90,000 3,90,000
Loss on Obsolete item = Cost 6,000 minus Sale value 5,000 = Rs.1,000
Calculation of Claim

Rs.

© The Institute of Chartered Accountants of Nepal 195


CAP II Paper 1: Advanced Accounting

Stock on date of fire 90,000


Less: Salvage 9,000
Actual Loss 81,000

Policy Amount (50000)  Actual Loss (81000)


Claim = Stock on date of fire (90000)
= Rs. 45,000

23. A fire occurred in the premises of Nepal Supply on the evening of 25th march,2004 when a
large part of the stock was destroyed. Salvage was Rs.15,000. Nepal supply gives you the
following information for the period January 1, 2004 to March 25, 2004:
i. Purchase – Rs.85,000
ii. Sales – Rs.90,000
iii. Goods costing Rs.5, 000 were kept by Nepal supply for Personal use.
iv. Cost Price of Stock on January 1,2004 was Rs.40,000

Over the Past few years, Nepal supply has been selling goods at a consistent gross Profit Margin
of 33-1/3 %. The insurance policy was for Rs.50, 000 with an average clause.

Nepal Supply asks you to prepare a statement of claim to be made on the Insurance
Company.
(Inter Dec. 2004 Q4)

Answer

Memorandum Trading Account for the period from 01.01.2004 to 25.03.2004

Dr Rs Cr Rs
To Opening Stock 40,000 By Sales 90,000
To Purchases 85,000 By stock in date of fire 60,000
Less: Drawing 5,000 80,000 (Balancing figure)
To Gross Profit 30,000
(33.33% of Rs.90, 000)

1, 50,000 1, 50,000

Statement of Claim
Rs.
Stock on the Date of fire 60,000
Less: Salvage Stock 15,000
Loss of Stock 45,000
Amount of Claim by applying average clause:

© The Institute of Chartered Accountants of Nepal 196


CAP II Paper 1: Advanced Accounting

Loss of Stock x Amount of Policy = 45,000 x 50,000


Value of Stock on the date of fire 60,000

24. A fire occurred on 15th December, 2008 in the premises of D Co. Ltd. From the following
figures, calculate the amount of claim to be lodged with the insurance company for the loss of
stock.
NRs.
st
Stock at cost as on 1 April, 2007 200,000
Stock at cost as on 1st April, 2008 300,000
Purchases for the year ended 31st March, 2008 400,000
st th
Purchases from 1 April, 2008 to 15 December, 2008 880,000
Sales for the year ended 31stMarch, 2008 600,000
st th
Sales form 1 April, 2008 to 15 December, 2008 1,050,000
During the accounting year 2008-09, cost of purchase rose by 10% above the
previous year’s levels while selling prices went up by 5%.

The value of stock salvaged was NRs. 20,000.


(Inter Jun. 2009 Q4- 10 Marks)
Answer D Co. Ltd.
Memorandum Trading Account
For the year ending 31-3-2008
Debit Credit
Particulars NRs. Particulars NRs.
To Opening Stock 200,000 By Sales 600,000
By Stock as on 31st March,
To Purchases 400,000 2008 300,000
To Gross Profit (Bal. Fig) 300,000
Total 900,000 Total 900,000

Rate of Gross Profit =300,000/600,000*100=50%

D Co. Ltd.
Memorandum Trading Account
For the period form 1st April 2008 to 15-Dec, 2008
Debit
Credit
At last At last
Actual year's rate Actual year's rate
NRs. NRs. NRs. NRs.
To Op. Stock 300,000 300,000 By sales 1,050,000 1,000,000
1st Apr. 2008 By Stock on15 Dec.,
To Purchases 880,000 800,000 2008
To Gross Profit (At last year's rates - 600,000
(at last year's rate 500,000 Balancing figure)
= (100000*50%) (Actual
(Actual, balancing 530,000
figure) NRs.600,000*110/100) 660,000
1,710,000 1,600,000 1,710,000 1,600,000

© The Institute of Chartered Accountants of Nepal 197


CAP II Paper 1: Advanced Accounting

NRs.
Value of closing stock on 31st December,2008 at current rates 660,000
Less: Value of goods salvaged 20,000

Amount of the claim to be lodged 640,000

© The Institute of Chartered Accountants of Nepal 198


CAP II Paper 1: Advanced Accounting

CHAPTER 3: COMPANY ACCOUNTING

3.1 Accounting for Shares & Debentures

A. Theoretical Questions

1. Sweat share
(CAP Jun. 2015 Q 6a- 2.5 Marks; Inter. Jun. 2001 Q 6-4 Marks; CAP Dec. 2014 Q6a-2.5
Marks)

Answer
Sweat shares means the equity shares issued by company to its employees or directors at a
discount or for a consideration other than cash for providing technical know-how or making
available rights in the nature of property rights or value additions.
Such issue must be authorized by a special resolution passed by the company in a general
meeting. The resolution must specify the category of employees etc., to whom shares are to be
issued, number of shares, price, etc.

2. Firm underwriting
(CAP Jun. 2015 Q6b- 2.5 Marks; CAP Dec. 2010 6e; CAP Dec. 2014 6b-2.5 Marks; Inter
Dec. 2006 Q 5a-5 Marks)

Answer
‘Firm underwriting’ signifies a definite commitment to take up a specified number of shares
irrespective of the number of shares subscribed by the public. In such a case, unless it has been
otherwise agreed, the underwriter’s liability is determined without taking into account the
number of shares taken up ‘firm’ by him, i.e. the underwriter is obliged to take up:
1. the number of shares he has applied for ‘firm’; and
2. the number of shares he is obliged to take up on the basis of the underwriting
agreement.

3. Underwriting (Inter Dec. 2008 Q6d-4 Marks)

Answer
Underwriting involves issue of shares or debentures by entering into a contract with a person
known as underwriter. Underwriting is basically done to cover the risk of non-subscription of
share or debenture. In case of non-subscription or short-subscription, the underwriter will be
liable to subscribe the non-subscribed or short-subscribed portion. However, the liability of
under-writers will be limited to the amount undertaken by him.
The function of underwriter has great economic significance. It provides an assurance to the
company that it would be able to raise the stipulated amount of capital by issue of shares or
debentures and on the basis of such an assurance; it can proceed to draw up its investment
programme.

© The Institute of Chartered Accountants of Nepal 199


CAP II Paper 1: Advanced Accounting

4. Sinking Fund (Inter Dec. 2009 Q6d-5 Marks; CAP Dec. 2011 6d)
The most common method of supplementing capital available to a company is to issue
debentures, which are usually redeemable. Redeemable debentures may be redeemed after a
fixed number of years or any time after a certain number of years has elapsed since their issue,
on giving a specified notice or by annual drawing.
Usually, according to the conditions of the issue, the company is required to create a fund by
appropriating annually a certain percentage of, or a fixed amount out of, its profit. The fund so
created is normally known as Sinking Fund or Debenture Redemption Reserve Fund. The
company invests the amount of the fund either in the purchase of securities which are readily
saleable or takes a policy that shall mature at the time the debentures will fall due for payment.
Such an arrangement would ensure that the company will have sufficient liquid funds for the
redemption of debentures at the time they shall fall due for payment. In the case of debentures
that are redeemable at premium, the appropriation to the Sinking Fund should be sufficient to
pay both the amount of debenture and the premium on redemption. On redemption of
debentures, out of the balance lying in the Sinking Fund an amount equal to the debentures so
redeemed is transferred to General Reserve.

5. Forfeiture of Shares
(Inter Jun. 2008 Q6 iii - 4 Marks; Inter Jun. 2005 Q 6c- 4 Marks)
Answer
If the shareholder fail to pay call amount and interest thereon within the time prescribed by the
company, company may decide to cancel such non-paid shares. The cancellation of such non-
paid shares is called forfeiture of shares. The articles of a company usually authorized the
directors to forfeit shares of a member on account of non-payment of a call or interest thereon
after serving him a prior notice as prescribed by the articles. Directors also have right to cancel
such a forfeiture before the forfeited shares are re-allotted. Subsequent to forfeiture, a
company, subject to any contrary provision in the articles, may re-issue the shares at any price
whatever so long as the amount receivable on re-issue together with that received from the
original allottee is equal to or is in excess of the called up value of the shares.

6. Zero- Coupon Bonds (Inter Jun. 2008 Q6 iv- 4 Marks)

Answer
A bond that pay no interest while the investor holds it. It is called Zero-Coupon Bond sold
originally at a substantial discount from its eventual maturity value, paying the investor its full
face value when it comes due, with the difference between what he paid initially and what he
finally collected representing the interest he would have received over the years it was held.

7. Differentiate between dividend yield and dividend cover.


(Inter Jun. 2012 Q5c- 5 Marks)

Answer

© The Institute of Chartered Accountants of Nepal 200


CAP II Paper 1: Advanced Accounting

Dividend Yield and Dividend Cover both are shareholder ratio or investor ratio. They indicate
how well a company is performing in relation to market price, dividend and number of shares.

Dividend Yield measures the real rate of return by comparing the dividend paid to the market
price of a share. An investor can find out the rate of his earnings on investment by calculating
this ratio. A higher dividend yield is preferred. It is calculated by:
Dividend Yield = Gross Dividend per Share ÷ Market Price per Share × 100%

Dividend Cover compares the amount of profit earned by an entity per ordinary share with the
amount of dividend paid. Thus, it shows the proportion of profits that could have been
distributed. It also reflects the dividend payment capacity of a company. It differs from EPS
only in having a different denominator. A high dividend cover is generally preferred. It can be
calculated as:

Dividend Cover (times) = Net Profit after Tax and Preference Dividend ÷ Net Dividend
on ordinary shares

8. Share and Stock. (CAP Jun. 2011 Q6a)

Answer
In today’s financial markets, the distinction between stocks and shares has been somewhat
blurred. Generally, these words are used interchangeably to refer to the pieces of paper that
denote ownership in a particular company, called stock certificates. However, the difference
between the two words comes from the context in which they are used. For example, “stock”
is a general term used to describe the ownership certificates of any company, in general, and
“shares” refers” to a the ownership certificates of a particular company. So, if investors say
they own stocks, they are generally referring to their overall ownership in one or more
companies. Technically, if someone says that own shares- the question then becomes – shares
in what company?

Bottom line, stocks and shares are the same thing. The minor distinction between stocks
and shares is usually overlooked, and it has more to do with syntax than financial or
legal accuracy.

9. Illustrate the various ways in which a company can alter its share capital.
(Inter Jun. 2002 Q 2b- 8 Marks)
Answer
A company can, if 201authorized by its articles carryout alteration of capital by altering the
provisions of its memorandum, in the following ways:
i. increase the share capital by making fresh issue of capital,
ii. consolidate its existing shares of smaller amounts into shares of larger amounts,
iii. sub-divide its share capital of larger amounts into shares of smaller amounts,
iv. decrease its unissued capital, without resulting in the reduction of the paid-up capital,
and

© The Institute of Chartered Accountants of Nepal 201


CAP II Paper 1: Advanced Accounting

v. convert its fully paid shares into stock reconvert its stock into fully paid shares.

Additionally, a company can reduce its share capital according to provisions contained in the
Companies Act, 2063. The main provision in this context are:

i. reduction of capital is possible by passing a special resolution,


ii. such reduction of paid-up capital can be carried out only after the scheme is confirmed
by the court.
iii. reduction of capital can take any of the following three forms:
- reducing or extinguishing the uncalled liability of members, or
- writing off or cancelling paid-up capital which is lost or not represented by available
assets, or
- paying off paid-up capital which is in excess of the needs of the company.

B. Practical Questions

Note: Wherever issue costs have been incurred, the amount has been netted off against equity
as per the provisions of NAS 32 rather than charging off as expenses or carrying over as an
asset.

1. The Financial Position of Zee Limited as on 31st Ashadh 2071 is given below:
Equity & Liabilities Amount Assets Amount
Issued and paid up capital Freehold Property 200,000
20,000 equity shares of Rs.10 each 200,000 Stock in trade 120,000
Reserves 180,000 Sundry debtors 100,000
% Debentures 120,000 Cash and bank balances 180,000
Sundry Creditors 100,000
600,000 600,000
It was resolved at the annual general meeting:
i) to pay dividend of 10%
ii) To issue one bonus share for every 4 shares held.
iii) To give the existing shareholders the option to buy one of Rs.10 share @ Rs.14 for every
four shares held prior to bonus distribution.
iv) To repay the debentures at the premium of 4%.
v) All the shareholders took up the option in (3) above.
You are required to pass necessary journal entries to give effect to the above transactions.
(CAP Dec. 2015 4B)
Answer

Journal Entries

© The Institute of Chartered Accountants of Nepal 202


CAP II Paper 1: Advanced Accounting

Reserves A/c Dr. 20,000


To Dividend Payable A/c 20,000
(Transfer of 10% dividend on Rs.2,00,000 equity shares to
Dividend payable account.)

Dividend Payable A/c Dr. 20,000


To Bank A/c
(Dividend Paid to the shareholders) 20,000

Reserves A/c Dr. 50,000


To Bonus to equity shareholders A/c 50,000
(Bonus payable to equity shareholders.)

Bonus to Equity shareholders A/c Dr. 50,000


To Equity Share Capital A/c 50,000
(Allotment of 5000 equity shares of Rs.10 each as bonus shares to
the equity shareholders.)

Bank A/c Dr. 70,000


To Equity Share Capital A/c 50,000
To Shares Premium A/c 20,000
(Allotment of 5,000 equity shares of Rs.10 each at Rs.4 per share
premium on right basis.)

12% Debentures A/c Dr. 1,20,000


Premium on Redemption of Debentures (loss) A/c Dr. 4,800
To Debenture holders A/C 1,24,800
(Amount payable on redemption of debentures along with the
premium)
Debenture holders A/c Dr. 1,24,800
To Bank A/c 1,24,800
(Being amount due to the debenture holders paid off)

2. Mahakali Bank Ltd. issues shares of Rs. 10 each at a premium of 10% payable as follows:
On application Rs. 2
On allotment Rs. 3 (including premium)
On first call Rs. 2
On final call Rs. 4

Mahesh who was holding 50 shares did not pay his allotment and first call and his shares were
forfeited. Suresh, who was holding 30 shares, did not pay first call and his shares also were
forfeited. Journalize transaction relating to forfeiture of shares.
(CAP Dec. 2017 Q3b-5 Marks)

© The Institute of Chartered Accountants of Nepal 203


CAP II Paper 1: Advanced Accounting

Answer
Share capital A/C –Dr 50*6 300
Securities Premium A/c –Dr (50*1) 50
To forfeited shares A/C (50*2) 100
To share allotment A/C (50*3) 150
To Share first call A/C( 50*2) 100
( Being forfeiture of Mahesh Shares)
Share Capital A/C 30*6 180
To forfeited shares A/C (30*4) 120
To share first call A/C(30*2) 60
(Being forfeiture of Suresh Shares)
Note: in case of Mahesh securities premium account has been cancelled by debiting it because
the premium has not been paid but in case of Suresh it has not been done because the premium
has been paid, premium once received cannot be cancelled.

3. Neo Trade Ltd. is running a chain of departmental store. It was incorporated in Nepal by
registration in the Office of Company Registrar with authorized share capital of Rs.5 million
and paid up capital of Rs.2 million of Rs.100 per share. The company paid the remuneration
of its promoters for their service by issuing 1,000 shares. After successful running for 3 years,
it decided to go public by offering 10,000 of its equity share at Rs.142 each fully payable on
application. The entire issue was underwritten and managed by ABC Capital Ltd. for a
commission of 3% of the issue price payable in cash. The issue was fully subscribed by public
and it got its shares listed in Stock Exchange. Currently, its shares are trading at Rs.204 each.
Further, in the course of expansion of its business, it acquired a departmental store in
Bhairahawa taking over its following assets for a total price of Rs.2.05 million payable in the
form of shares in Neo Trade Ltd. at Rs.205 each:

Furniture and fixtures Rs.450,000; Store Inventories Rs.1,250,000; and Accounts Receivable
Rs.255,000.
Neo Trade Ltd.’s retained earnings is Rs.2.31 million.
Required:
i) Journalize each of the above transaction.
ii) Draw the extracts of the Financial Position showing balance in Share Capital and Reserve
and Surplus.
(Inter Dec 10 Q 2b)
Answer
Journal Entries
Date Particulars Dr. (Rs.) Cr. (Rs.)
1 Bank A/c ….. Dr. 2,000,0000
To Share Capital A/c 2,000,000
(Being amount received from promoters
for 20,000 shares @ Rs.100 each at the
time of incorporation)

© The Institute of Chartered Accountants of Nepal 204


CAP II Paper 1: Advanced Accounting

2 Director’s Remuneration A/c* …. Dr. 100,000


To Share Capital A/c 100,000
(Being 1,000 shares @ Rs.100 each issued
to promoters as remuneration for their
service)
3 Bank A/c …. Dr. 1,420,000
To Share application A/c 1,420,000
(Being application money received for
10,000 shares @ Rs.142 each)
4 Share application and allotment A/c..Dr. 1,420,000
To Share Capital A/c 1,000,000
To Share Premium A/c 420,000
(Being shares issued and application
money transferred to share capital and
share premium accounts)
5 Equity (Share capital/prem) ** A/c … Dr. 42,600
To Bank A/c 42,600
(Being underwriting and issue
management cost paid)
6 Furniture & Fixture A/c …..Dr. 450,000
Inventories A/c…………….Dr. 1,250,000
Accounts Receivable A/c….Dr. 255,000
Goodwill A/c………………Dr. 95,000
To Vendor A/c 2,050,000
(Being assets taken over from vendor)
7 Vendor A/c ….Dr. 2,050,000
To Share Capital A/c 1,000,000
To Share Premium A/c 1,050,000
(Being purchase consideration discharged
by issuing 10,000 shares of Rs.100 each
with Rs.105 share premium)

* Amount should be charged as an expense as there are no vesting conditions. As the fair value
of services rendered is not stated, the par value of shares have been considered as the fair value
of services rendered by the directors. (NFRS 2 Share Based Payments)

** The underwriting fee cannot be accounted for as expense under the provisions of NAS 32,
“Financial Instruments: Presentation” and needs to be deducted from equity.

2) Financial Position Extracts of Neo Trade Ltd.


Rs.
Equity
(A) Share Capital

© The Institute of Chartered Accountants of Nepal 205


CAP II Paper 1: Advanced Accounting

Authorized: 50,000 shares of Rs.100 each 5,000,000


Issued and paid up:
41,000 shares of Rs.100 each fully paid up 4,100,000
(B) Reserve and Surplus
Retained Earnings (2,310,000 – 100,000 + 42,600) 2,252,600
Share Premium 1,470,000
Total 3,722,600
(C) Total (A+B) 7,822,600
(D) Less: Issue Costs 42,600
(E) Net (C – D) 7,780,000

4. National Bank Limited issued 060,000 Equity Shares of Rs. 10 each at a premium of Rs.2.50
Per share payable on application. The amount payable on allotment was fixed at Rs.4 per share
and an equivalent sum of due on a call to be made.
Total applications received were for 110,000 shares and after consulting the stock Exchange,
the following scheme of allotment was decided upon –
Category A B C Total
Grouping of Shares 1 – 100 101 – 500 Above 500
No. of applications received 1,200 175 5 1,380
No. of shares applied for 70,000 35,000 5,000 110,000
No. of shares allotted 42,000 14,000 4,000 60,000

It was decided that the excess amount received on applications would be utilized in payment of
allotment money and surplus, if any, would be refunded to the applicants.

Hanuman, who was one of the applicants belonging to category A and had applied for 100
shares, defaulted in payment of allotment money. Angadh, who belonged to category C, and
who had been allotted 800 shares failed to pay the call money. Their shares were forfeited after
the respective calls were made and reissued as fully paid up for Rs.8 and Rs. 6 per share
respectively.

Show the necessary Journal Entries in the book of the Company to record the transactions and
prepare the relate accounts. Workings should form part of your answer.
(Inter Dec 10 Q4-20 Marks)
Answer
C Cu
Application Appl
C No. No. Exce All al mul
money due icati Exce Adju Calls
at Of Of ss ot Exc ls Call Call ativ
on ss stme amo
eg Shar Shar towa mo ess in amo s in e
mon mon nt of unt
or es es Pre rd ney rece ar unt Arr call
Shar ey ey Exce recei
ie Appl Allot miu Allot rece ived re due ears s in
e recei due ss ved
s ied ted m ment ived ar Arr
ved
s ears

© The Institute of Chartered Accountants of Nepal 206


CAP II Paper 1: Advanced Accounting

6
A 100 60 120 150 450 180 240 180 - - 240 - 240 300
0
6990 4194 8388 1048 3145 1258 1677 1258 419 1677 1677
A - - - -
0 0 0 50 50 20 60 20 40 60 60
3500 1400 2800 3500 1575 9450 5600 5600 385 5600
B - - 5600 - -
0 0 0 0 00 0 0 0 00 0
230 320 320
C 1000 800 1600 2000 4500 900 3200 900 - - 3200 -
0 0 0
1800 1280 920 1280 1280
C 4000 3200 6400 8000 3600 3600 - - - -
0 0 0 0 0
1100 6000 1200 1500 4950 2250 2400 1865 534 385 6 2400 2365 344 350
00 0 00 00 00 00 00 00 40 00 0 00 60 0 0

Particulars Dr. Cr.


1.Bank A/c Dr 495,000
To Share Application A/c 495,000

2. Share Application A/c Dr. 495,000


To Share Capital A/c 120,000
To Share Premium A/c 150,000
To Share Allotment A/c 186,500
To Bank A/c 38,500

3.Share Allotment A/c Dr. 240,000


To Share Capital A/c 240,000

4. Bank A/c Dr. 53,440


Calls in Arrear A/c Dr. 60
To Share Allotment A/c 53,500

5.Share 1st & final Call A/c Dr. 240,000


To Share capital A/c 240,000

6. Bank A/c Dr. 236,560


Calls in Arrear A/c Dr. 3,440
To Share 1st & final call A/c 240,000

7. Share Capital A/c Dr. 8,600


To calls in Arrear A/c 3,500
To Share forfeiture A/c 5,100

8. Bank A/c Dr. 5,280

© The Institute of Chartered Accountants of Nepal 207


CAP II Paper 1: Advanced Accounting

Discount on issue of shares/Share premium A/c Dr. 3,320


To Share Capital A/c 8,600

9. Share forfeiture A/c Dr. 5,100


To Hanuman 300
To Angadh 4,800

10. Hanuman A/c Dr. 300


Angadh A/c Dr. 4,800
To Bank 5,100

Share Application A/c Bank A/c


To Share To Share By Share
Capital 120,000 By Bank 495,000 Application 495,000 Application 38,500
To Share To Share By Balance
Premium 150,000 Allotment 53,440 c/d 751,780
To Share
Allotment 186,500 To Share Call 236,560
To Share
To Bank 38,500 Capital 5,280
495,000 495,000 790,280 790,280

Share Allotment Share 1st & final call A/c


To Share By Share To Share
Capital 240,000 Application 186,500 Capital 240,000 By Bank 236,560
By Calls in
By Bank 53,440 Arrear 3,440
By Calls in
Arrear 60
240,000 240,000 240,000 240,000

Calls in Arrears A/c Share Capital A/c


To By
Share Share To Calls in By Share
allotment 60 Capital 3,500 Arrears 3,500 Application 120,000
To Share To share By Share
Call 3,440 Forfeiture 5,100 Allotment 240,000
To Balance By Share
3,500 3,500 b/d 600,000 Call 240,000
By Bank 5,280

© The Institute of Chartered Accountants of Nepal 208


CAP II Paper 1: Advanced Accounting

By
Discount 3,320
608,600 608,600

Share Forfeiture Share Premium


To By Share
Hanuman 300 By Share Capital 5,100 To Balance b/d 150,000 Application 150,000
To
Angadh 4,800
5,100 5,100 150,000 150,000
Discount on issue of shares
By
To Share Balance
Capital 3,320 c/d 3,320

3,320 3,320

5. Elite Limited recently made a public issue in respect of which the following information is
available:
d) No. of partly convertible debentures issued 200,000; face value and issue price NRs.100
per debenture.
e) Convertible portion per debenture 60%, date of conversion on expiry of 6 months from
the date of closing of issue.
f) Date of closure of subscription lists 1.5.2007, date of allotment 1.6.2007, rate of interest
on debenture 15% payable from the date of allotment, value of equity share for the
purpose of conversion NRs. 60 (Face Value NRs. 10).
g) Underwriting Commission 2%.
h) No. of debentures applied for 150,000.
i) Interest payable on debentures half-yearly on 30th September and 31st March.
Write relevant journal entries for all transactions arising out of the above during the year
ended 31st March, 2008 (including cash and bank entries).
(CAP Jun. 2009 Q3 15 Marks)

Answer
Journal Entries
In the books of Elite Ltd
NRs. NRs.
Date Particulars Dr. Cr.

Journal No. 1
1.5.2007 Bank A/c Dr. 15,000,000

© The Institute of Chartered Accountants of Nepal 209


CAP II Paper 1: Advanced Accounting

To Debenture Application A/c 15,000,000


(Application money received on 150,000 debentures @ NRs. 100
each)

Journal No. 2
1.6.2007 Debenture Application A/c Dr. 15,000,000
Underwriters A/c Dr. 5,000,000
To 15% Debentures A/c 20,000,000
(Allotment of 150,000 debentures to applicants and 50,000
debentures to underwriters)

Journal No. 3
1.6.2007 Underwriting Commission Dr. 400,000
To Underwriters A/c 400,000
(Commission payable to underwriters @ 2% on NRs. 20,000,000)

Journal No. 4
1.6.2007 Bank A/c Dr. 4,600,000
To Underwriters A/c 4,600,000
(Amount received from underwriters in settlement of account)

Journal No. 5
30.9.2007 Debenture Interest A/c Dr. 1,000,000
To Bank A/c 1,000,000
(Interest paid on debentures for 4 months @ 15% on NRs.
20,000,000)

Journal No. 6
30.10.2007 15% Debentures A/c Dr. 12,000,000
To Equity Share Capital A/c 2,000,000
To Securities Premium A/c 10,000,0000
(Conversion of 60% of debentures into shares of NRs. 60 each with
a face value of NRs. 10)

Journal No. 7
31.3.2008 Debenture Interest A/c Dr. 750,000
To Bank A/c 750,000
(Interest paid on debentures for the half year)

Working Note:
Calculation of Debenture Interest for the half year ended 31st March, 2008
On NRs. 8,000,000 for 6 months @ 15% = NRs. 600,000
On NRs. 12,000,000 for 1 months @ 15% = NRs. 150,000
NRs. 750,000

© The Institute of Chartered Accountants of Nepal 210


CAP II Paper 1: Advanced Accounting

6. Himalaya Power Ltd. issued 05,000,000 Equity shares of Rs.10 each. The whole issue was
underwritten by A, B and C as below
A 1,500,000 shares
B 2,500,000 shares
C 1,000,000 shares
Applications were received for 4,850,000 shares of which the marked applications were as
follows:
A 1,200,000 shares
B 2,500,000 shares
C 850,000 shares
Calculate the number of shares to be taken up the underwriters.
(Inter Jun 10 Q 5c-4 Marks)
Answer:
A B C
Gross Liability (3:5:2) 1,500,000 2,500,000 1,000,000
Less: Marked applications 1,200,000 2,500,000 _850,000
300,000 Nil 150,000
Less: Unmarked applications * in 3:5:2 ratio 90,000 150,000 60,000
210,000 (150,000) 90,000
Less: Surplus of B allocated to A & C in 3:2 ratio 90,000 (150,000) 60,000
Number of shares to be taken up by the underwriters 120,000 Nil 30,000
Note
* 4,850,000 share - (1,200,000+ 2,500,000 + 850,000) = 300,000 Share

7. On 1st December, 2010 Smart Choice Limited purchases for immediate cancellation 2,000 12%
Debentures (face value Rs. 100) of its own. Due dates for interest payment is 31st March and
30th September.
Pass the necessary journal entries relating to the cancellation if:
i) Debentures are purchased at Rs. 92 ex – interest.
ii) Debentures are purchased at Rs. 92 cum – interest.
(Inter Jun 11 Q3b-10 Marks)
Answer
Journal Entries in the books of Smart Choice Ltd.
Particulars Dr Rs. Cr Rs.
(i) When Debentures are purchased ex – interest:
12% Debenture Account 2,00,000
Interest on Debenture Account 4,000
To Bank Account 1,88,000
To Gain on Cancellation of Debenture 16,000
(Being cancellation of 2,000 12% Debentures purchased at Rs.
92 ex interest and interest paid for two month)
(ii) When Debentures are purchased cum – interest:

© The Institute of Chartered Accountants of Nepal 211


CAP II Paper 1: Advanced Accounting

12% Debenture Account 2,00,000


Interest on Debenture Account 4,000
To Bank Account 1,84,000
To Gain on Cancellation of Debenture Account 20,000
(Being cancellation of 2,000 12% Debentures purchased at Rs.
92 cum – interest)

8. On 31st Ashadh, 2065, A Ltd.’s Financial Position showed 10,000, 12 % debentures of Rs. 100
each outstanding. Interest on debenture is payable on 30th Poush and 31st Ashadh every year.
On 1st Mangsir, 2065, the company purchased 500 of its own debentures as investment at Rs.
97 ex- interest.
Required:
Journal entries supposing:
i) The company cancels all its own debentures on 1st Ashadh, 2066.
ii) The company resells all its own debentures at Rs. 105 cum- interest on 1st Ashadh, 2066.
(CAP Jun 10 Q2b-10 Marks)
Answer
Cr. Dr.
2065 12% Debenture a/c 50,000
Mangsir 1 Debentures Interest Account 2,000
To Bank 50,500
To Gain on acquisition 1,500
(Purchase of 500 of own Debentures of Rs. 100
each at Rs. 97 ex- interest. Amount of interest paid
being Rs. 12/100 * 4/12 * 50 =Rs. 2,000)
Poush 30 Debenture Interest Account 57,000
2065 To bank 57,000
(Payment of half yearly interest on 9500
debentures)
Ashadh Debenture Interest A/C 57,000
31, 2066 To bank A/c 57,000
(Payment of half yearly interest on 9500
debenture)
2nd option
2066 Bank 52,500
Ashadh To 12% debenture a/c 50,000
1st To debenture holder’s a/c 2,500
(Resale of 500 own debenture at Rs. 105 cum-
interest)

9. Arial Constructions Ltd issues 80,000 equity shares of Rs. 10 each at a premium of 25%, Rs.
4 per share being payable along with application and the balance including premium being
payable on allotment. Applications total 91,000 shares out of which applications for 11,000

© The Institute of Chartered Accountants of Nepal 212


CAP II Paper 1: Advanced Accounting

shares are rejected while all other application are fully accepted. Allotment money on 200
shares is not received. After due notices have been served, the Board of directors decided to
forfeit the shares. Later all these shares are issued as fully paid up @ Rs. 9 per share.
Pass necessary journal entries for the above transaction crediting Share Premium Account only
when the amount of the premium has been received.
(CAP Jun. 2014 Q3b- 7 Marks)
Answer
Journal Entries

Bank A/c Dr. 3,64,000


To Equity share application A/c 3,64,000
(Application money received for 91,000 equity shares @Rs. 4 per
share)

Equity Share Application A/c Dr. 3,20,000


Equity Share Allotment A/c Dr. 4,80,000
To Equity Share Capital A/c 8,00,000
(Credit to equity share application a/c for application money
@Rs.4 per share and that part of allotment money which is
payable towards the capital i.e. @Rs.6 per share on allotment of
80,000 equity shares of Rs.10 each at a premium of 25%.)

Equity share application A/c Dr. 44,000


To Bank A/c 44,000
(Return of application money to application for 11,000 shares
because of rejection of their application.)

Bank A/c Dr. 6,78,300


To Equity Share allotment A/c 4,78,800
To Share premium A/c 1,99,500
(Receipt of allotment money on 79,800 shares @Rs.8.50 per
share, Rs. 6 per share towards Share capital and Rs.2.5 as share
premium.)

Equity Share Capital A/c 2,000


To Equity Share Allotment A/c 1,200
To Forfeited Shares A/c 800
(Forfeiture of 200 equity shares on which application money @
Rs.4 per share has been received for non-payment of allotment
money.)
Bank A/c Dr.
Forfeited Shares A/c Dr. 1,800
To Equity Share Capital A/c 700

© The Institute of Chartered Accountants of Nepal 213


CAP II Paper 1: Advanced Accounting

To Share Premium A/c 2,000


(Being reissue of 200 equity shares as fully paid up @Rs.9 per 500
share)

Forfeited Shares A/c Dr. 100


To Gain on forfeiture A/c 100
(Gain Recognized.)

10. From the following information, calculate the value of an equity share:
i) The subscribed share capital of a company consists of 10 lakhs 13% preference shares of
Rs. 10 each and Rs. 20 lakhs equity shares of Rs. 10 each. All the shares are fully paid up.
ii) The average annual profits of the company after providing depreciation but before taxation
are Rs. 18,000,000. It is considered necessary to transfer Rs. 3,450,000 to general
reserve before declaring any dividend. Rate of taxation is 30%.
iii) The normal return expected by investors on equity shares from the type of business carried
on by the company is 20%.
iv) Ignore corporate dividend Tax.
(CAP Jun. 2015 Q4c- 5 Marks; CAP Dec. 2014 4c-5 Marks)
Answer:
Rs.
Average Annual profits before 18,000,000
Less: Income Tax @ 30% 5,400,000
12,600,000
Less: Transfer to general reserve 3,450,000
Amount available for dividend 9,150,000
Less: Preference dividend @13% on Rs. 10,000,000 1,300,000
Amount available for equity dividend 7,850,000

Rate of Dividend = 7,850,000/20,000,000 x100 = 39.25%


Normal Rate of Dividend = 20%
Value of an equity share = 39.25 / 20 x 10 = 19.63

11. Butwal Hydro Power Company Ltd. came up with public issue of 3000,000 equity shares of
Rs. 10 each at Rs. 15 per share. A, B and C took underwriting of the issue in 3: 2: 1 ratio.
Applications were received for 2,700,000 shares. The marked applications were received as
under:

A 800,000 shares B 700,000 shares C 600,000 shares


Commission payable to underwriters is at 5% on the face value of shares.
i) Compute the liability of each underwriter as regards the number of shares to be taken up.
ii) Pass journal entries in the books of Butwal Hydro Power Company Ltd. to record the
transactions relating to underwriters.
(CAP Jun. 2016 4b- 5 Marks)

© The Institute of Chartered Accountants of Nepal 214


CAP II Paper 1: Advanced Accounting

Computation of liability of underwriters in respect of shares


(In shares)
Particulars A B C
Gross liability in agreed ratio of 3: 2: 1 15,00,000 10,00,000 5,00,000
Less: Unmarked applications (Subscribed (3,00,000) (2,00,000) (1,00,000)
shares- marked shares) in 3: 2: 1
Marked shares as per agreed ratio 12,00,000 8,00,000 4,00,000
Less: Marked applications actually received (8,00,000) (7,00,000) (6,00,000)
Shortfall / surplus in marked shares 4,00,000 1,00,000 (2,00,000)
Surplus of C distributed to A & B in 3:2 ratio (1,20,000) (80,000) 2,00,000
Net liability for underwriting shares 2,80,000 20,000 Nil

Journal Entries in the books of Butwal Hydro Power Company Ltd.


Particulars Rs Rs
A's Account Dr 42,00,000
B's Account Dr 3,00,000
To Share Capital Account 30,00,000
To Securities Premium Account 15,00,000
(Being the shares to be taken up by the underwriters)
Equity (Share Capital/Premium) Dr. 15,00,000
To A's Account 7,50,000
To B's Account 5,00,000
To C's Account 2,50,000
(Being the underwriting commission due to the
underwriters)
Bank Account Dr. 34,50,000
To A's Account 34,50,000
(Being the amount received from underwriter A for the
shares taken up by him after adjustment of his
commission)
B's Account Dr. 2,00,000
To Bank Account 2,00,000
(Being the amount paid to underwriter B after
adjustment of the shares taken by him against
underwriting commission due to him)
C's Account Dr. 2,50,000
To Bank Account 2,50,000
(Being the underwriting commission paid to C)

Note: C had sold in excess of the underwriting obligation and hence he will not be required to
purchase any shares but will get commission for underwriting.

© The Institute of Chartered Accountants of Nepal 215


CAP II Paper 1: Advanced Accounting

12. A company issues 1000, 14% debentures of Rs. 1,000 each at a premium of 20%. 60 % of
the issue was underwritten by Mr. Akash Chettri & Co. for a commission @ 1.50% of the
issue price of debentures underwritten. Applications were received for 800 debentures
which were accepted and payments were received in full. Give the journal entries.
(CAP Dec. 2012 4a-5 Marks)

Answer
The Journal Entries are as follows.
S.N. Particulars Debit Credit
1 Bank Account 960,000
To 14% Debentures 960,000
Allotment of 800 debentures @Rs 1200 as per Directors Resolution of ……………
2 Akash Chettri & Co 144,000
To 14% Debentures 144,000
Allotment of 120 debentures to the Underwriters in pursuance of their agreement
dated………..
3 14% Debenture A/c Dr. 10,800
To Akash Chettri & Co 10,800
Commission @1.5% on 600 Debentures at the issue price of Rs 1200
4 Bank 133,200
To Akash Chettri & Co 133,200
Amount Received form the underwriter from settlement

Note: Since Akash Chettri & Co underwrite 60% of the issue, the company must itself be
treated as an underwriter of the remaining 40% . In the absence of any information, the
applications for 800 debentures must be deemed to have marked 60% in favor of
underwriters i.e. 480 debentures. The underwriters are, therefore, liable to take up 120
debentures (600-480).

13. A company issued 1,50,000 shares of Rs. 10 each at a premium of Rs. 10. The entire issue was
underwritten as follows:
X-90,000 shares (firm underwriting 12,000 shares)
Y- 37,500 shares (firm underwriting 4,500 shares)
Z- 22,500 shares (firm underwriting 15,000 shares)
Total subscriptions received by the company (excluding firm underwriting and marked
applications) were 22,500 shares.
The marked applications (excluding firm underwriting) were as follows:
X-15,000 shares; Y- 30,000 shares; Z- 7,500 shares

Commission payable to underwriters is at 5% of the issue price. The underwriting contract


provides that credit for unmarked applications be given to the underwriters in proportion to the
shares underwritten and benefit of firm underwriting is to be given to individual underwriters.

© The Institute of Chartered Accountants of Nepal 216


CAP II Paper 1: Advanced Accounting

Required:
i) Determine the liability of each underwriter (number of shares)
ii) Compute the amounts payable or due from underwrites; and
iii) Pass Journal Entries in the books of the company relating to underwriting.
(CAP Dec. 2018 Q2b-10 Marks)
Answer
(i) Computation of total liability of underwriters in shares
(in shares)
X Y Z Total
Gross liability 90,000 37,500 22,500 1,50,000
Less: Marked applications
(excluding firm underwriting ) (15,000) (30,000) (7,500) (52,500)
75,000 7,500 15,000 97,5000
Less: Unmarked applications in the
ratio of gross liabilities of 12:5:3
(excluding firm underwriting )
Less: Firm underwriting (13,500) (5,625) (3,375) (22,500)
61,500 1,875 11,625 75,000
Less: Surplus of Y and Z adjusted (12,000) (4,500) (15,000) (31,500)
in X's balance (2,625+3,375) 49,500 (2,625) (3,375) 43,500
Net liability
Add: Firm underwriting (6,000) 2,625 3,375 00
Total liability 43,500 - - 43,500
12,000 4,500 15,000 31,500
55,500 4,500 15,000 75,000

(ii) Calculation of amount payable to or due from underwriters


X Y Z Total
Total Liability in shares 55,500 4,500 15,000 75,000

Amount receivable @ Rs 20
11,10,000 90,000 3,00,000 15,00,000
from Underwriter (in Rs)
Less: Underwriting
(90,000) (37,500) (22,500) (1,50,000)
Commission Payable @ 5% of
Rs 20 (in Rs)
Net amount receivable (in Rs) 10,20,000 52,500 2,77,500 13,50,000

(iii) Journal Entries in the books of the company (relating to underwriting)


Rs Rs

© The Institute of Chartered Accountants of Nepal 217


CAP II Paper 1: Advanced Accounting

1 X Dr. 11,10,000
Y Dr. 90,000
Z Dr. 3,00,000
To Share capital A/c 7,50,000
To Securities Premium A/c 7,50,000
(Being allotment of shares to underwriters)

2 Equity (Share Capital/Premium) A/c * Dr.


To X 1,50,000
To Y 90,000
To Z 37,500
(Being amount of underwriting commission 22,500
payable)
3 Bank A/c Dr.
To X
To Y 13,50,000
To Z 10,20,000
(Being net amount received by underwriting 52,500
for shares allotted less underwriting 2,77,500
commission)

* The underwriting fee cannot be accounted for as expense under the provisions of NAS 32,
Financial Instruments: Presentation and needs to be deducted from equity.

14. Give Journal entries in respect of the following:


i) 10% Preference Capital of Rs. 6,00,000 was redeemed along with the dividend due for one
year by conversion into equity shares of Rs. 10 each issued at 20% premium.
ii) Preference Capital of Rs. 8,00,000 was redeemed at 15% premium by conversion into
equity shares of Rs. 10 each issued at 8% discount.
iii) Debentures of Rs. 6,00,000 were redeemed at 5% discount by conversion into equity shares
of Rs. 100 each issued at a premium of Rs. 14 per share.
iv) Debentures of Rs. 10,00,000 were redeemed at 12½% premium by conversion into equity
shares issued at 25% premium.
(Inter Jun. 2001 Q 2a -8 Marks)
Answer
Journal Entries
Dr. Cr.
Preference Share Capital A/c Dr. 6,00,000
Preference Share Dividend A/c Dr. 60,000
To Equity Share Capital 5,50,000
To Securities Premium 1,10,000
(Being conversion of preference shares into 55,000 Equity shares each at Rs. 2
premium per share.)

© The Institute of Chartered Accountants of Nepal 218


CAP II Paper 1: Advanced Accounting

ii)Preference Share Capital 8,00,000


Reserves (Loss on settlement of liability) 1,20,000
Share Discount A/c 80,000
To Equity Share Capital 10,00,000
(Conversion of Preference Capital 1,00,000 Equity Shares of Rs. 10 each at 0.80 per
share discount.)

iii) Debentures A/c 6,00,000


To Equity Share Capital A/c 5,00,000
To Securities Premium A/c 70,000
To Reserves (Gain on settlement of 30,000
liability) A/c
(Conversion of debentures into equity at Rs. 14 per share)
iv) Debentures A/c 10,00,000
Reserves (Loss on settlement of liability) 1,25,000
To Equity Share Capital 9,00,000
To Securities Premium 2,25,000
(Being conversion of debentures into 90,000 Equity Shares of Rs. 10 each at Rs. 2.50
per share.)

15. i) A holds 1,000 shares of Rs. 10 each on which he has not paid allotment money of Rs. 3 per
share.
ii) B holds 2,000 shares of Rs. 10 each on which he has not paid first call of Rs. 3 per share.
iii) C holds 3,000 shares of Rs. 10 each on which he has not paid final call of Rs. 2 per share.
The directors forfeited the above shares and later on reissued them to D as fully paid at Rs. 8
per share.
Give the necessary Journal Entries in respect of the above.
(Inter Jun. 2001 Q 2b-8 Marks)

Journal Entries
Dr. (Rs.) Cr. (Rs.)
i) Share Capital A/c 60,000
To Allotment Arrears 3,000
To First Call A/c 9,000
To Final Call A/c 12,000
To Forfeited Shares 36,000
(Forfeiture of shares for nonpayment of dues as per Board Resolution dated........)

ii) Bank A/c 48,000


Forfeited Shares A/c 12,000
To Share Capital 60,000
(Issues of 6,000 / Shares at Rs. 8, per share and discount on reissue adjusted.)

© The Institute of Chartered Accountants of Nepal 219


CAP II Paper 1: Advanced Accounting

iii) Forfeited Shares A/c 24,000


To Capital Reserve 24,000
(Balance on former account transferred to the latter)

16. The share capital of Share Limited consisted of 50,000 equity shares of Rs. 10 each, fully paid.
The company has an accumulated reserve fund of Rs. 500,000, out of profits. It issued further
10,000 equity shares during 2001 at a premium of Rs. 30 per share and the entire amount had
been realized.

During 2001, an independent valuation of its assets increased as thus:

Land and building by Rs. 6,00,000


Plant and machinery by 3,00,000
Stores and spares by 2,50,000

The valuation also reduced the amounts in respect of the following:

Goodwill by Rs. 1,50,000


Patents by 1,00,000

At the end of 2001, it was decided to redeem 3,000 debentures of Rs. 100 each at 5 per cent
premium, to adopt the new valuation amounts, and to allot two equity shares of Rs. 10 each as
fully paid bonus shares for every equity share held.

It was also decided that the balance of reserve fund, after carrying out all the above
arrangements, was to be capitalized. Pass journal entries in the books of the Shree Limited.
(Inter June 2002, Q 2a-14 Marks)
Answer

Shree Limited
Journal Entries

1. Bank Dr 4,00,000
To Equity share Capital 1,00,000
To Securities Premium 3,00,000
(Issued 10,000 equity share at a premium of Rs. 30 per share)

2. Land & Building Dr 6,00,000


Plant & M/C Dr 3,00,000
To Revaluation Reserve 9,00,000
(Upward valuation of Non-current Assets credited to Revaluation Reserve. Stores & spares are
assumed to be inventory and therefore are not revalued)

© The Institute of Chartered Accountants of Nepal 220


CAP II Paper 1: Advanced Accounting

3. Impairment Loss Dr 2,50,000


To Goodwill 1,50,000
To Accumulated Impairment Loss 1,00,000
(Impairment on goodwill charged to goodwill and on patents credited to accumulated
impairment loss)

4. Debentures Dr 3,00,000
Loss on Settlement of Financial Liability Dr. 15,000
To Debenture holders 3,15,000
(3000 Debentures redeemable at 5% premium)

5. Debenture holders Dr 3,15,000


To Bank 3,15,000
(Debenture holders paid off)

6. Securities premium Dr 3,00,000


Accumulated Res. Fund Dr 5,00,000
Retained Earnings Dr. 4,15,000
To Bonus to Equity Shareholders 12,00,000
(Bonus shares payable to Equity shareholders)

7. Bonus to Equity shareholders Dr 12,00,000


To Equity share capital 12,00,000
(120,000 bonus shares distributed to 60,000 equity shares.

Note: In the absence of information it is assumed that there is sufficient balance in the
reserves/retained earnings for the purpose of issuance of bonus shares..

17. A holds 200 shares of Rs. 10 each on which he has paid Rs. 2 as application money. B holds
400 shares of Rs. 10 each on which he has paid Rs. 2 per shares as application money and Rs.
3 per shares as allotment money. C holds 300 shares of Rs. 10 each and has paid Rs. 2 on
application, Rs. 3 on allotment and Rs. 3 on the first call. They all fail to pay their arrears on
the second and final call of Rs. 2 per share and the directors, therefore, forfeited their shares.

The shares are re-issued subsequently for Rs. 12 per share, fully paid-up. Journalize the
transactions relating to the forfeiture and re-issue in the books of the issuing company.
(Inter Jun. 2004 Q 5a -8 Marks)
Answer

Date Particulars Dr. Rs. Cr. Rs.


Share Capital A/c (900  Rs. 10) 9,000
To Share Allotment A/c 600

© The Institute of Chartered Accountants of Nepal 221


CAP II Paper 1: Advanced Accounting

To Share first call A/c 1,800


To Share final call A/c 1,800
To Forfeited shares A/c 4,800
(Forfeiture of 900 shares of Rs. 10 each
for non-payment of allotment, first &
final money as per board resolution No.
…. Dated……)

Bank A/c (900  Rs. 12) 10,800


To Share Capital A/c 9,000
To Share Premium A/c 1,800
(Re-issue of 900 shares of Rs. 10/- each
@ Rs. 12 as per board resolution No….
Dated….)

Forfeited Shares A/c 4,800


To Reserve A/c 4,800
(Profit on re-issue transferred to
reserve/equity)

Workings:

Shareholders Money Received on Money Received on


Application Allotment 1st Final Allotment 1st Final
Call Call Call Call
A 200 – – – 200 200 200
B 400 400 – – – 400 400
C 300 300 300 – – – 300
Total 900 700 300 – 200 600 900
Money Rs. 2 Rs. 3 Rs. Rs. 2 Rs. 3 Rs. 3 Rs. 2
Receivables 3
Rs. 1,800 Rs. 2,100 Rs. Rs. 600 Rs. Rs.
900 1,800 1,800

18. On 1st January, 2005 Sumitra Limited issued 25,000, 12 % Debentures of Rs 100 each at par.
According to the terms of the issue, interest was payable half yearly on 30 June and 31
December and the company reserved the right to buy any number of debentures in the open
market to be held as investment or to be cancelled at any time.
During the accounting year ended 31 December 2005, the company purchased 200 of its
debentures at Rs. 102 cum-interest on 30 April and 800 of its debentures at Rs. 99 ex-interest
on 30 November.

© The Institute of Chartered Accountants of Nepal 222


CAP II Paper 1: Advanced Accounting

On 31 March 2006, the company sold one-half of the debentures purchased on 30 November
2005 at Rs 104 cum-interest. On 31 August 2006, the company purchased 1,500 debentures at
Rs. 97.50 ex-interest and on 30 September 2006, it cancelled all the debentures purchased in
2005 and still lying with it.

Prepare 12 % Debentures account, Own Debentures account, Debenture Interest account and
Interest on own debentures account. Ignore creation of debenture redemption reserve.
(Inter Jun. 2008 Q5-15 Marks)

Answer:
Note: Own debentures need to be cancelled upon purchase as per the requirements of
NFRS.

Books Sumitra Limited


Own Debenture Account
Amount Amount
Date Particular (Rs.) Date Particular (Rs.)
2005 2005
30-Apr To Bank A/C 19,600 30-Apr By Debentures A/c 20,000
30-Apr To Gain on debentures 400
30-Nov To Bank A/C 79,200 30-Nov By Debentures A/c 80,000
30-Nov To Gain on debentures 800
31-Aug To Bank A/C 146,250 31-Aug By Debentures A/c 150,000
31-Aug To Gain on debentures 3,750

Debenture Interest A/c


Amount Amount
Date Particular (Rs.) Date Particular (Rs.)
2005
30-Apr To Bank A/C 800
To Debenture holder’s
30-June A/c 148,800
To Debenture holder’s
30-Nov A/c 4,000
To Debenture holder’s
31-Dec A/c 144,000
2006
To Debenture holder’s
30-June A/c 145,200
To Debenture holder’s
31-Aug A/c 3,000

© The Institute of Chartered Accountants of Nepal 223


CAP II Paper 1: Advanced Accounting

To Debenture holder’s
31-Dec A/c 137,400

Debentures A/c
Amount Amount
Date Particular (Rs.) Date Particular (Rs.)
2005 2005
30-Apr 20,000 1-Jan By Bank 2,500,000
By Own Deb. A/c
30-Nov 80,000
31-Dec By Bal c/d 2,400,000
2006 2006
31-Aug By Own Deb. A/c 150,000 1-Jan By Bal b/d 2,400,000
31-Dec By bal c/d 2,290,000 31-Mar By Bank 40,000

Trans.
Date Particulars Units Price Interest Principal FV Gain/Loss
1-Jan-05 Issued 25000 100 - 2500000 2500000 0
30-Apr-05 Purchased -200 102 800 19600 20000 400
30-Jun-05 Interest 24800 100 148,800 0 0 0
30-Nov-05 Purchased -800 99 4,000 79200 80000 800
31-Dec-05 Interest 24000 100 144,000 0 0 0
31-Mar-06 Re-issued * 400 104 1,200 40400 40000 400
(Less payable accounted for on re-
issue (Rs. 1,200) which is not
30-Jun-06 Interest 24400 100 145,200
interest as it pertains to period held
by the company)
31-Aug-06 Purchased -1500 97.5 3,000 146250 150000 3750
31-Dec-06 Interest 22900 100 137,400 0 0 0

* Note
Bank A/C Dr. 41,600
To Debentures A/c 4,0000
To Debenture holder’s A/c 1,200
To Gain on re-issue of debentures 400
(Debenture should have been measured at amortized cost and no gain/loss should have been
recognized. However, calculation cannot be made due to lack of information about maturity)

Question No. 2 'a'


M/s Biswas Ltd. with an authorized capital of NRs.30,000,000 offered to public 400,000
ordinary shares of NRs.50 each at a premium of NRs.5 each. The payment was to be made as;
NRs.15 on application,
NRs.25 on allotment (including premium) and
NRs.15 on first and final call.

© The Institute of Chartered Accountants of Nepal 224


CAP II Paper 1: Advanced Accounting

Application totaled. 800,000 shares; shares were allotted on a pro-rata basis. Amit who had
applied for 800 shares and to whom 400 shares had been allotted failed to pay the balance of
allotment money due from him. His shares were forfeited and then reissued to Tanka as NRs.40
(including premium of NRs.5) per share as fully paid up. Rojina, another shareholder, failed to
pay the call money on 100 shares held by her. Her shares were also forfeited. Later, these shares
were reissued as fully paid to Suchitra for NRs.60 per share.
Expenses of the issue of shares came to NRs.12,000.
Pass necessary journal entries in the books of M/s Biswas Ltd.
(Inter Jun. 2009 Q2a- 11 Marks)
Answer
M/s Biswas Ltd.
Journal Entries
S.N. Particulars Debit Credit
1. Bank Account Dr. 12,000,000
To Share Application A/c 12,000,000
(Being Share application money received for
800,000 share at NRs. 15 per Share))

2. Share Application A/c Dr. 12,000,000


To Share Capital A/c 6,000,000
To Share Allotment A/c 6,000,000
(Being Share Application money transferred to
share capital account & excess money received
transferred to share allotment account)

3. Share Allotment A/c Dr. 10,000,000


To Share Capital A/c 8,000,000
To Share Premium A/c 2,000,000
(Being allotment of 400,000 number of shares
made at NRs. 25 per Share)

4. Bank Account Dr. 3,996,000


Calls in Arrears A/c Dr. 4,000
To Share Allotment A/c 4,000,000
(Being Share Allotment money received except
one shareholder holding 400 shares who failed to
pay the allotment money)

5. Share First & Final Call A/c Dr. 6,000,000


To Share Capital A/c 6,000,000
(Being first and final call made on share issued)

6. Bank Account Dr. 5,992,500


Calls in Arrears A/c Dr. 7,500
To Share First & Final Call A/c 6,000,000
(Being share first & final call money received
except 400 shares held by Mr. Amit and 100
shares held by Rajina who failed to pay call
money)

© The Institute of Chartered Accountants of Nepal 225


CAP II Paper 1: Advanced Accounting

7. Share Capital A/c Dr. 25,000


Share Premium A/c Dr. 2,000
To Share Forfeiture A/c 15,500
To Calls in Arrears A/c 11,500
(Being 400 shares held by Amit who failed to
pay the allotment and first and final call and 100
Shares held by Rojina who failed to pay the call
money forfeited)

8. Bank Account Dr. 16,000


Share Forfeiture A/c Dr. 6,000
To Share Capital A/c 20,000
To Share Premium A/c 2,000
(Being Shares forfeited from Amit issued at
NRs. 40 including premium of NRs.5 to Tanka)

9. Bank Account Dr. 6,000


To Share Capital A/c 5,000
To Share Premium A/c 1,000
(Being Shares forfeited from Rojina reissued at
NRs. 60 to Suchitra)

10. Share Forfeiture A/c Dr. 9,500


To Capital Reserve A/c 9,500
(Being credit balanced on share forfeited account
transferred to Capital Reserve A/c)

11. Equity (share Capital/Reserves) A/c Dr. 12,000


To Bank Account 12,000
(Being NRs.12,000 paid for share issue
accounted under Deferred Revenue Expenditure
account)

Working Note;
a. Calculation of Calls in Arrears of Amit NRs.
Amount paid at the time of application = 800×15 = 12,000
Amount utilized in application = 400×15 = 6,000
Excess amount = 12,000 – 6,000 = 6,000
Calls in arrear at the time of Allotment = 400×25 – 6,000 = 4,000
Calls in arrear at the time of First & Final Call = 400×15 = 6,000

b. Calculation of Calls in arrears of Rojina NRs.


Calls in arrear at the time of First & Final Call = 100×15 = 1,500

© The Institute of Chartered Accountants of Nepal 226


CAP II Paper 1: Advanced Accounting

3.2 Preparation & Presentation of Financial Statements

A. Theoretical Questions

1. What are the differences between Cash Flow Statement and Fund Flow Statement?
(CAP Jun. 2009 Q5b- 5 Marks)
Answer
The differences between cash flow statement and fund flow statement are as per below:
i) Cash flow statement deals with the change in cash position between two points of time.
Fund flow statement deals with the changes in working capital position.
ii) Cash flow statement contains opening as well as closing balances of cash and cash
equivalents. The fund flow statement does not contain any such opening and closing
balance.
iii) Cash flow statement records only inflow and outflow of cash. Fund flow statement
records sources and application of funds.
iv) Fund flow statement can be prepared from the cash flow statement under indirect
method. However, a cash flow statement cannot be prepared from fund flow statement.
v) A statement of changes in working capital is usually prepared along with fund flow
statement. No such statement is prepared along with the cash flow statement.

B. Practical Questions

1. ABC Fertilizers presents the following Financial Position as on 31-03-2069 and 32-03-
2068. You are required to prepare cash flow statement: (CAP Dec. 2012 Q 2a-10 Marks)

(Rs in
'000)
31-03-2069 32-03-2068
Equity Share Capital 8,500 7,000
General Reserve 3,800 4,000
Profit & Loss - 250
Share Premium Account 1,500 750
Share Holders Fund 13,800 12,000
Secured Loans 4,800 5,000
Unsecured Loans 5,350 4,000
Loan Funds 10,150 9,000
Total Sources 23,950 21,000

PPE
Gross Block 22,400 21,000
Less: Accumulated Depreciation 3,450 3,200
Net Block 18,950 17,800
Capital Work In Progress 1,860 -

© The Institute of Chartered Accountants of Nepal 227


CAP II Paper 1: Advanced Accounting

Investments 1,650 2,320


Current Asset, Loans & Advances
Inventories 2,510 2,600
Debtors 1,090 1,200
Cash & Bank Balance 120 280
Loans 1,700 200
Advance Tax - 500
(A) 5,420 4,780
Less Creditors 1,050 1,200
Outstanding Expense 30 -
Tax Provision - 500
Proposed Dividend 3,400 2,800
(B) 4,480 4,500
Net Current Asset 940 280
Miscellaneous Expenditure 550 600
Total Applications 23,950 21,000

Other Information:
i) PPE costing Rs. 400,000 (accumulated depreciation Rs. 300,000) were sold for Rs.
150,000.
ii) Actual tax liability for fiscal year 2067-068 was Rs. 500,000.
iii) Loan represents loan given to other companies.
iv) Interest on loan funds for fiscal year 2068-069 was at Rs. 1,421,000 and interest &
dividend income were Rs. 402,000.
v) Investments costing Rs. 2,000,000 were sold for 2,500,000.

Answer:
CASH FLOW STATEMENT
(Rs in ‘000)
Cash Flow from Operation Activities Rs.
Change in General Reserve -200
Change in Profit & Loss -250
Proposed Dividend 3400
Profit Before Tax 2950
Add: Depreciation 550
Add: Miscellaneous Exp 50
Less: Profit on Sale of PPE -50
Less: Profit on Sale of Investments -500 50
Fund Flow from Operations 3000
Add: Interest Paid 1421
Less: Interest & Dividend Received -402
Add/(Less): Changes in Working Capital

© The Institute of Chartered Accountants of Nepal 228


CAP II Paper 1: Advanced Accounting

Inventories 90
Debtors 110
Creditors -150
Outstanding Expense 30 80
Cash Flow from Operation Activities (Before Tax) 4099
Less: Advance Tax for 2068-2069 0
Cash Flow from Operation Activities (After Tax) A 4099
Cash Flow from Financing Activities
Issue of Shares 2250
Repayment of Secured Loans -200
Raising of Unsecured Loans 1350
Interest Payment -1421
Dividend Payment -2800
(B) -821
Cash Flow from Investment Activities
Purchase of PPE -1800
Sale of PPE 150
Capital WIP -1860
PPE(NET) -3510
Purchase of Investments -1330
Sale of Investments 2500
Investments (Net) 1170
Loans -1500
Interest & Dividend Income 402
C -3438
Increase/ Decrease in Cash & Bank Balance (A+B+C) -160
Add: Opening Cash & Bank Balance 280
Closing Cash & Bank Balance 120

2. Financial Position of Raj Motors is given below:

Liabilities Amount (in 000 Rs.) Assets Amount (in 000 Rs.)
31.03.2069 31.03.2070 31.03.2069 31.03.2070
Share Capital 500 500 Land & Building 300 300
9% Debentures 200 160 Machinery 164 180
Sundry Creditors 230 216 Accumulated Dep. (80) (88)
Profit & Loss 40 54 Stock in Trade 200 228
Contingency Reserve 140 110 Sundry Debtors 170 162
Outstanding Expenses 30 48 Cash & Bank 120 110
Balances
Current Investment 262 190
Prepaid Expenses 4 6

© The Institute of Chartered Accountants of Nepal 229


CAP II Paper 1: Advanced Accounting

1,140 1088 1,140 1088

The following information is furnished:

i) An Old Machinery, which has original cost of Rs. 30,000, was sold for Rs. 10,000. The
accumulated depreciation in respect of the said machinery amounts to Rs. 16,000.
ii) One New Machinery was acquired for Rs. 46,000.
iii) 9 % Debentures was redeemed at a discount of 4% of their face value (at the beginning of
the year).
iv) Dividend at 12% was declared and paid in cash.
v) Income Tax-Liability of Rs. 30,000 paid was debited to contingency reserve.

You are required to prepare Cash Flow Statement on Indirect Method as per the NAS.
(CAP Dec. 2013 Q2a-10 Marks)

Answer
A. Cash Flow from Operating Activities Rs Rs
Net Profit Before Tax (Rs 54,000 - Rs 40,000 + Rs 60,000 74,000
dividend paid)
Add: Adjustment for Depreciation (W.N.1) 24,000
Interest on debentures (9% X Rs 1,60,000) 14,400
Loss on Sale Machinery (W.N.2) 4,000
1,16,400
Less: Profit on redemption of Debentures -1,600
1,14,800
Less: Income Tax Paid -30,000
84,800
Add: Increase in Outstanding Expenses 18,000
Decrease in Sundry Debtors 8,000
Decrease in Current Investment 72,000 98,000
1,82,800
Less: Decrease in Sundry Creditors 14,000
Increase in Stock In Trade 28,000
Increase in Prepaid Expenses 2,000 -44,000
Net Cash from Operating Activities 1,38,800
B. Cash Flow from Investing Activities
Sale of old Machinery 10,000
Purchase of Machinery -46,000
Net Cash used in Investing Activities -36,000
C. Cash Flow from Financing Activities
Redemption of Debentures(Rs. 40000-Rs. 1,600) -38,400
Payment of Dividend -60,000

© The Institute of Chartered Accountants of Nepal 230


CAP II Paper 1: Advanced Accounting

Payment of Interest on Debentures -14,400


Net Cash from Financing Activities -1,12,800
Net Decrease in Cash & Cash Equivalent during the year -10,000
Cash & Cash Equivalent at the beginning of the year 1,20,000
Cash & Cash Equivalent at the end of the year 1,10,000
Accumulated Depreciation
Particulars Amount Particulars Amount
To Machinery A/c 16000 By Balance b/d 80000
To Balance c/d 88000 By Depreciation 24000
104000 104000

Machinery A/c
Particulars Amount Particulars Amount
To balance B/d 1,64,000 By Accumulated Depreciation 16,000
To Bank 46,000 By Bank 10,000
By Loss on Sale 4,000
By Balance c/d 1,80,000
2,10,000 2,10,000

It is assumed that current investments cannot be liquidated within short duration of 3 months;
therefore it has not been considered as part of cash & cash Equivalents

3. The summarized Financial Position of Jackson Ltd. as on 31st December 2014 and 2015 are as
follows: (CAP Dec. 2016 Q2a-10 Marks)
Amounts in Rs.
Liabilities 2014 2015 Assets 2014 2015
Share Capital 100,000 100,000 Building 46,800 45,000
General Reserve 38,400 42,000 Plant & Machinery 38,280 42,030
Creditors 9750 6380 Goodwill 13,000 13,000
Tax Provision 19,000 21,000 Investment 10,000 11,250
Stock 30,000 28,000
Debtors 22,070 22,300
Less: Prov. for doubtful debt (1,000) (1,200)
Cash 8,000 9,000
Total 167,150 169,380 Total 167,150 169,380

After taking the following information into account, prepare a cash flow statement for the year
ended on 31st December 2015.
i) Profit for year 2014/2015 was Rs. 8,600. Against this depreciation of Rs. 3,050 had been
charged and increase in provision for doubtful debt was Rs.200/-.
ii) Income Tax Rs. 18,000 was paid during the year charged against the provision and in
addition Rs. 20,000 was charged against profit and carried to the provision.
iii) An interim dividend of Rs. 5,000 was paid in January 2015.

© The Institute of Chartered Accountants of Nepal 231


CAP II Paper 1: Advanced Accounting

iv) Additional Plant was purchased in September 2014 for Rs. 5,000
v) Investments (cost Rs. 5,000) were sold for Rs. 4,800 in 2015 and on 1st March 2015 another
investment was made for Rs. 6,250.
Answer:
a) Cash Flow Statement for the year ended on 31st December 2015.

Particular Rs. Rs.


1. Cash Flow from Operating Activities
Net profit before tax 28,800
Adjustments:
Depreciation 3,050
Increase in provision for doubtful debt 200
Decrease in stock 2,000
Decrease in creditor (3,370)
Increase in Debtor (230)
Income Tax Paid (18,000)
Net Cash from Operating Activities 12,450
2. Cash Flow from Investing Activities
Investment Purchased (6,250)
Sale of Investment 4,800
Plant Purchased (5,000)
Net Cash Flow from Investing Activities (6,450)
3. Cash Flow from Financing Activities
Payment of Interim Dividend (5,000)
Net increase in cash equivalent 1,000
Add: Opening Cash Balance 8,000
Closing Cash Balance 9,000

Adjusted Profit/Loss Memorandum


Particulars Rs. Particulars Rs.
To Provision for Tax 20,000 By Profit 28,800
To General Reserve 3,600
To Loss on Sale of Investment 200
To Interim Dividend 5,000

28,800 28,800
Provision for Tax Account
Particulars Rs. Particulars Rs.
To Bank (Tax Paid) 18,000 By Balance b/d 19,000
To Balance c/d 21,000 By Tax Expense (Provision) 20,000

39,000 39,000

© The Institute of Chartered Accountants of Nepal 232


CAP II Paper 1: Advanced Accounting

4. Mahabir Digital Library Society showed the following position on Ashadh 31, 2073.
Balance sheet as on Ashadh 31, 2073
Liabilities Amount(Rs.) Assets Amount(Rs.)
Capital fund 793,000 Electrical fittings 150,000
Expense payable 7,000 Furniture 50,000
Digital Book license 400,000
Investment in securities 150,000
Cash at bank 25,000
Cash in hand 25,000
800,000 800,000

Receipts and Payment Accounts for the year ended Ashadh 31, 2073 is given below:
Receipts Amount(Rs.) Payments Amount(Rs.)
To, Balance b/d By, Electrical charges 7,200
Cash at bank 25,000 By, Postage & stationary 5,000
Cash in hand 25,000 By, Telephone charges 5,000
By, Digital book License 60,000
To, Entrance fee 30,000 By, O/s expense paid 7,000
To, Membership subscription 200,000 By, Rent 88,000
To, Sale proceeds of old By, Investment in securities
1,500 40,000
paper (Magh 01, 2072)
To, Hire of lecture hall 20,000 By, Salaries 66,000
To, Interest on securities 8,000 By, Balance c/d
Cash at bank 20,000
Cash in hand 11,300
309,500 309,500

You are required to prepare Income and Expenditure Account for the year ended Ashadh
31, 2073 and Balance sheet as on Ashadh 31, 2073 after making following adjustments:
i) Membership subscription included Rs. 10,000 received in advance.
ii) Provide for Outstanding rent Rs. 4,000 and Salaries Rs. 3,000
iii) Digital book license to be amortized at 10% including additions, electrical fitting and
furniture are to be depreciated at 10%.
iv) 75% of entrance fees to be capitalized.
v) Interest on securities is to be calculated @ 5% p.a. including for purchase made during the
year.
(CAP Dec. 2016 Q3a-10 Marks)
Answer:

IF IT IS ASSUMED THAT THE GIVEN BALACE SHEET IS OF ASHADH END 2072


INSTEAD OF ASHAD END 2073

Mahabir Digital Library Society

© The Institute of Chartered Accountants of Nepal 233


CAP II Paper 1: Advanced Accounting

Income and expenditure account for the year ended Ashadh 31, 2073
Dr. Cr.
Expenditure Amount Incomes Amount
To, Electric charges 7,200 By, Entrance fees 7,500
To, Postage & stationary 5,000 25% of 30000
To, Telephone charges 5,000
By Membership
To, Rent 88,000 200,000
subscription
Less: Received in
Add: Outstanding 4,000 92,000 (10,000) 190,000
Advance
To, Salaries 66,000
By, Sale proceeds of
Add: Outstanding 3,000 69,000 1,500
paper
By, Hire of lecture hall 20,000
To, Amortization & By, Interest on securities
8,000
Depreciation (WN-1) (WN-2)
Electrical fittings 15,000 Add: Receivable 500 8,500
Furniture 5,000
By, Deficit -excess of
Books 46,000 66,000 16,700
expenditure over Income
244,200 244,200

Mahabir Digital Library Society


Balance sheet as on Ashadh 31, 2073
Liabilities Amount Assets Amount

Capital fund 793,000 Electrical fittings 150,000


Add: Entrance fees
22,500 Less: Depreciation (15,000) 135,000
capitalization
Less: Excess of expenses
(16,700) 798,800 Furniture 50,000
over income
Less: Depreciation (5,000) 45,000
Outstanding Expense Digital Books 460,000
Rent 4,000 Less: Amortization (46,000) 414,000
Salaries 3,000 7,000
Investment in
190,000
Securities
Membership subscription in advance 10,000 Accrued interest 500 190,500

Cash at bank 20,000


- Cash in hand 11,300
815,800 815,800
Working Notes

© The Institute of Chartered Accountants of Nepal 234


CAP II Paper 1: Advanced Accounting

1 Depreciation & Amortization 2 Interest on securities


Electrical fittings @10% 15,000 Interest @5% p.a. on 150000 full year 7,500
Furniture @10% 5,000 Interest @5% p.a. on 40000 half year 1,000
Digital Books @10% 46,000 Total 8,500
Total 66,000 Less: Received (8,000)
Receivable 500

Alternate -2

IF IT IS ASSUMED THAT THE GIVEN BALACE SHEET IS OF ASHAD END 2073

Income and Expenditure Account for the year ended 31st Ashadh 2073
Expenditure Rs. Income Rs.
To Electrical Charges 7,200 BY Entrance Fees (25% of 7,500
30,000)
To Postage and Telegram 5,000 By Membership Subscription 190,000
(200,000 -10,000)
To Telephone Charges 5,000 By Sale proceeds of old paper 1,500
To Amortization of Digital 44,444 By Hire of Lecture Hall 20,000
License Book (400,000 * 10/90)
To Rent (88,000 + 4,000) 92,000 By Interest on Securities (110,000 6,500
*5% + 40,000 *5% *1/2)
To Salaries (66,000 + 3,000) 69,000
To Depreciation on Electrical 16,667
fittings (150,000 *10/90)
To Depreciation on Furniture 5,556 By Deficit 19,367
(50,000 * 10/90)
294,867 294,867

Balance Sheet as on 31.3.2073


Liabilities Rs. Assets Rs.
Capital Fund 793,000 Electrical Fittings 150,000
Expenses payable 7,000 Furniture 50,000
Digital Book License 400,000
Investment in Securitas 150,000
Cash 25,000
Bank 25,000
800,000 800,000

© The Institute of Chartered Accountants of Nepal 235


CAP II Paper 1: Advanced Accounting

5. Mr. Vijay runs a wholesale business where all purchases and sales are made on credit. He
furnishes the following closing balances:
31-12-2005 31-12-2006
Rs. Rs.
Sundry Debtors 70,000 92,000
Bills Receivable 15,000 6,000
Bills Payable 12,000 14,000
Sundry Creditors 40,000 56,000
Stock 1,10,000 1,90,000
Bank 90,000 87,000
Cash 5,200 5,300

Summary of the Cash Transactions during the year 2006:


i) Deposited to bank after payment of shop expenses @ Rs. 600 p.m.; wages @ Rs. 9,200
p.m. and personal expenses @ Rs. 1,400 p.m. Rs. 7,62,750
ii) Withdrawals Rs. 1,21,000
iii) Cash Payments to suppliers Rs. 77,200 for suppliers and Rs. 25,000 for Furniture.
iv) Cheques collected from customers but dishonored Rs. 5,700
v) Bills accepted by the customers Rs. 40,000
vi) Bills endorsed Rs. 10,000
vii) Bills discounted Rs. 20,000 and discount Rs. 750
viii) Bills Matured and duly collected Rs 16,000
ix) Bills Accepted Rs. 24,000
x) Paid to suppliers by cheque Rs. 3,20,000
xi) Received Rs. 20,000 on maturity of one Life Insurance Policy of Mr. Vijay by Cheque.
xii) Rent received Rs. 14,000 by cheque.
xiii) A Building was purchased on 30-11-2006 for opening a branch for Rs. 350,000 and some
expenses were incurred the details of which are not maintained.
xiv) Electricity and telephone bills paid by cash Rs. 18,700; due Rs. 2,200

Other Transactions:
(a) Claim against the firm for damage Rs.1,55,000 is under legal dispute. Legal expenses Rs.
17,000. The firm anticipates defeat in the suit.
(b) Goods returned to suppliers Rs. 4,200
(c) Goods returned by customers Rs. 1,200
(d) Discount offered by suppliers Rs. 2,700
(e) Discount offered to customers Rs. 2,400
(f) The business is carried on at the premises owned by Mr. Vijay. 50 % of the ground floor
space is used for the business and rest 50 % space is let out for an annual rent of Rs. 20,000.

Prepare the Statement of Profit or Loss of Mr. Vijay for the year ended 31-12-2006 and the
Financial Position as on that date.
(Inter Jun. 2008 Q1- 20 Marks)
Answer:

© The Institute of Chartered Accountants of Nepal 236


CAP II Paper 1: Advanced Accounting

Books of Mr. Vijay


Statement of Profit or Loss
For the year ended 31-12-2066
Dr. Cr.
Rs. Rs. Rs. Rs.
Opening Stock 110,000 Sales 959,750
Purchases 454,100 Less: Return 1,200 958,550
Less: Return 4,200 449,900 Closing Stock 190,000
Gross Profit 588,650
1,148,550 1,148,550
Wages 110,400 Gross Profit 588,650
Electricity &
Telephone Charges 20,900 Discount 2,700
Legal Expenses 17,000
Discount 3,150
Shop Expenses 7,200
Provision for claims
for damages 155,000
Shop Rent (Notional) 20,000
Net Profit 257,700
Total Rs. 591,350 Total Rs. 591,350

Financial Position
As on 31-12-2006

Amount Amount
Liabilities Rs Assets Rs.
Capital 238,200 Building 372,000
Add Fresh Capital Furniture 25,000
- Life Policy 20,000 Stock 190,000
- Rent 14,000 Sundry Debtors 92,000
- Notional Rent 20,000 Bills Receivable 6,000
Net Profit 257,700 Cash At Bank 87,000
549,900 Cash in Hand 5,300
Less: Drawing 16,800
533,100
Sundry Creditors 56,000
Bills Payables 14,000
Outstanding Expenses
- Legal Expenses 17,000
- Electricity & Telephone
expenses 2,200

© The Institute of Chartered Accountants of Nepal 237


CAP II Paper 1: Advanced Accounting

Provision for claims for


damages 155,000
Total Rs. 777,300 Total Rs. 777,300

Working Notes:
Statement of Affairs as on 31-12-2005
Amount
Liabilities Amount Rs. Assets Rs.
Sundry Creditors 40,000 Stock 110,000
Bills Payable 12,000 Debtors 70,000
Capital (Bal. fig) 238,200 Bills Receivable 15,000
Cash At Bank 90,000
Cash in Hand 5,200
290,200 290,200

Sundry Debtors A/c


Dr. Cr.
Rs. Rs.
To Balance B/d. 70,000 By Bills Receivable 40,000
To Bills Receivable - B/R
Dishonored 3,000 By Bank 5,700
To Bank - Cheque4 Dishonored 5,700 By Cash 897,150
To Sales (Credit Sales) (Bal. Fig) 959,750 By Return inwards 1,200
By Discount 2,400
By Balance c/d 92,000
1,038,450 1,038,450

Sundry Creditors A/c


Dr. Cr.
Rs. Rs.
To Bank 320,000 By Balance B/d 40,000
To Cash 77,200 By Credit Purchase (Bal Fig) 454,100
To Bills Payable 24,000
To Bills Receivable 10,000
To Return Outward 4,200
To Discount Received 2,700
To Balance C/d 56,000
494,100 494,100

Bills Receivable A/c


Dr. Cr.
Rs. Rs.

© The Institute of Chartered Accountants of Nepal 238


CAP II Paper 1: Advanced Accounting

By Sundry Creditors - Bills


To Balance B/d 15,000 Received 10,000
To Sundry Debtors-Bills accepted 40,000 By Discount on Bills 750
By Bank 19,250
By Bank (Bills Collected) 16,000
By sundry Debtors - Bills
Dishonored (Bal. Fig) 3,000
By Balance C/d 6,000
55,000 55,000

Bills Payable A/c


Dr. Cr.
Rs. Rs.
To Bank (Bal. Fig) 22,000 By Balance B/d 12,000
By Sundry Creditors- Bills
To Balance C/d 14,000 Accepted 24,000
36,000 36,000

Cash Book
Dr. Cr.
Bank
Cash Rs. Bank Rs. Cash Rs. Rs.
To Balance B/d 5,200 90,000 By Bank 762,750
To Sundry Debtors 897,150 By Cash 121,00
(Bal.Fig)
To Cash 762,750 By Shop Expenses 7,200

To Bank 121,000 By Wages 110,400


To Sundry Debtors 5,700 By Drawing 16,800
To Bills Receivable 19,250 By Bills Payable 22,000
To Bills Receivable 16,000 By Sundry Creditors 77,200 320,000
To Capital (Life Insurance) 20,000 By Furniture 25,000

To Capital ( Rent) 14,000 By Sundry Debtors 5,700


By Electricity & 18,700
Telephone
Charges
By Building (Bal. 372,000
Fig)
By Balance C/d 5,300 87,000
1,023,350 927,700 1,023,350 927,700

6. The following list of account balances relates to Virat as at 31st Ashadh, 2067:

© The Institute of Chartered Accountants of Nepal 239


CAP II Paper 1: Advanced Accounting

Rs'000 Rs'000
Sales revenue 473,300
Purchases 310,500
Operating expenses 18,400
Loan stock interest 5,000
Dividends paid 15,500
Leasehold building at cost (note (ii)) 200,000
Plant and equipment at cost (note (ii)) 124,800
Deferred development expenditure (note (iii)) 75,000
Joint venture (note (iv)) 62,000
Depreciation at 1st Shrawan, 2066 – leasehold 56,000
– plant and equipment 48,800
– development expenditure 15,000
Trade receivables 49,200
Inventory – 1st Shrawan, 2066 27,500
Bank 12,100
Trade payables 82,200
Ordinary shares of Rs 100 each 100,000
10% Convertible Loan stock – issued 2065 100,000
Deferred tax at 1st Shrawan, 2066 (note (v)) 11,400
Profit and loss reserve at 1st Shrawan, 2066 13,300
9,00,000 9,00,000
The following notes are relevant:
(i) The cost of the inventory at 31st Ashadh, 2067 was Rs.37.7 million (excluding joint
venture inventory – see note (iv)).
(ii) Non-current assets:
On 1st Shrawan, 2066, Virat’s leasehold building was revalued at Rs.270 million by an
independent surveyor. The lease was for a 25-year period when Virat acquired it. The
directors wish to incorporate the revalued amount in Virat’s financial statements. The
revaluation reserve will be deemed to be realized in line with the remaining life of the
lease.
Plant is depreciated at 20% per annum on the reducing balance basis.
All depreciation is charged to cost of sales.
(iii) The deferred development expenditure relates to a new product. The project was
successfully completed on 1st Shrawan, 2065 and sales of the new product commenced
on that date. The development costs are being depreciated on a straight-line basis over
the expected product life of five years. Early in the current year, a review of the sales
figures for the new product showed that they were disappointing. In view of this, Virat
has estimated that the present value of the expected net future cash flows from sales of
the new product is Rs.30 million; however, Virat has been approached by a rival
company with an offer of Rs.40 million for the rights to the product. At this stage, Virat
intends to continue to market and sell the product.
(iv) On 1st Shrawan, 2066, Virat entered into a joint venture with two other companies. Each
venturer contributes its own assets and pays its own expenses. The agreement stipulates

© The Institute of Chartered Accountants of Nepal 240


CAP II Paper 1: Advanced Accounting

that the joint venture will be terminated on 31st Ashadh, 2070. Virat is entitled to 30%
of the joint venture’s total revenues. The joint venture is not a separate entity. Details
of Virat’s joint venture transactions are:
Rs'000
Plant and equipment at cost 70,000
Share of joint venture sales revenues (30% of total sales revenues) (18,000)
Related cost of sales excluding depreciation 8,000
st
Inventory 31 Ashadh, 2067 2,500
st
Accounts receivable 31 Ashadh, 2067 3,500
st
Accounts payable 31 Ashadh, 2067 (4,000)
Net balance included in the above list of balances 62,000

Plant should be depreciated on a straight-line basis. It is not expected to have any


residual value at the end of the joint venture.
(v) The directors have estimated the required provision for income tax for the year to 31st
Ashadh, 2067 is Rs.15 million. The deferred tax provision at 31st Ashadh, 2067 is to be
adjusted to reflect the tax base of the company’s assets being Rs.70 million less than
their carrying values. Rs.28·8 million of this Rs. 70 million is attributable to the
revaluation of the leasehold. Virat’s rate of income tax is 25%.
(vi) The directors have proposed a final ordinary dividend of Rs.24 per share. Virat
discloses proposed dividends as part of shareholders’ funds.
Required:
Prepare for Virat, in accordance with International Accounting Standards as far as the
information permits:
a) the Statement of Profit or Loss;
b) the Statement of Changes in Equity for the year to 31st Ashadh, 2067; and
c) a Financial Position as at 31st Ashadh, 2067.
Notes to the financial statements are not required. (CAP Jun. 2010 Q1- 20 Marks)

Answer:

a) Virat – Statement of Profit or Loss & Other Comprehensive Income- for the year to 31st
Ashadh, 2067

Rs '000
Sales revenue (473,300 + 18,000 (w (i)) 491,300
Cost of sales (w (ii)) (366,000)
Gross profit
125,300
Operating expenses
(18,400
)
Impairment of non-current asset (60,000 – 40,000) (w (iv))

© The Institute of Chartered Accountants of Nepal 241


CAP II Paper 1: Advanced Accounting

(20,000
)
Profit on the ordinary activities before interest
86,900
Loan interest (5,000 + 5,000) (10,000)
Profit before tax
76,900
Taxation (w (iii)) (13,900)
Profit for the period
63,00
0

Other Comprehensive Income


Revaluation of PPE
126,0
00
Deferred Tax on Revaluation
(7,200
)
Total Comprehensive Income
181,8
00

b) Virat – Statement of Changes in Equity – for the Year to 31st Ashadh, 2067

Ordinary Revaluation Accumulated Total


Shares reserve profits
Rs '000 Rs '000 Rs '000 Rs '000

Balance at 1st Shrawan, 2066 100,000 nil 13,300 113,300


Surplus on revaluation of leasehold (w (iv)) - 126,000 - 126,000
Deferred tax (w (iii)) - (7,200) - (7,200)
Transfer to realized profits re leasehold
(126,000 – 7,200 deferred tax)/18 years) - (6,600) 6,600 -
Net profit for the period (63,000) - - 39,000 39,000
Dividends paid - - (15,500) (15,500)

Balance at 31st Ashadh, 2067 100,000 112,200 67,400 279,600

c) Virat – Financial Position as at 31st Ashadh 2067

© The Institute of Chartered Accountants of Nepal 242


CAP II Paper 1: Advanced Accounting

Non-current assets Rs '000 Rs '000


Development expenditure (w (iv)) 30,000
Property, plant and equipment (w (iv)) 368,300
_______
398,300
Current Assets
Inventory (37,700 + 2,500) 40,200
Trade receivables (49,200 + 3,500) 52,700
Bank 12,100
105,000
Total assets
503,300
_______

Equity and liabilities


Ordinary shares Rs. 100 each 100,000
Reserves:
Accumulated profits (see (b) above) 67,400
Revaluation reserve (see (b) above) 112,200 179,600
______ _______
279,600
Non-current liabilities
10% Convertible loan stock 100,000
Deferred tax (70,000 × 25%) 17,500 117,500
______

Current liabilities (w (v)) 106,200


Total equity and liabilities
503,30
0

Working Notes

(i) Since Virat pays for its own assets and provides its own liabilities, the arrangement has
been classified as Joint Operation as per the provisions of NFRS 11, Joint Arrangements.
Share of Virat in the assets, liabilities and revenue & expenses have therefor been
proportionately consolidated in the respective statement of financial position & profit or
loss. Had the arrangement been classified as Joint Venture, Virat would have to opt for
equity method of accounting as per the provisions of NAS 28.
Proposed dividends are not accounted for in the financial statements. Disclosure could be
made in the financial statements if required however the question specifically states that
no notes are required

(ii) Cost of sales: Rs.’000

© The Institute of Chartered Accountants of Nepal 243


CAP II Paper 1: Advanced Accounting

Opening inventory 27,500


Purchases 310,500
Joint venture cost of sales 8,000
Depreciation (w (iv)) – leasehold 15,000
– Plant (15,200 + 17,500) 32,700
– Development expenditure 10,000
Closing inventory (37,700)
_______
366,000
_______

(iii) Taxation Rs. ‘000


Current tax provision for year 15,000
Deferred tax (see below) (1,100)
_______
13,900
_______
The difference between the tax base of the assets and their carrying value of Rs.70
million would require a deferred tax of Rs.17·5 million (at 25%). Of this Rs.7·2 million
(Rs.28·8 million × 25%) belonging to revaluation gain is taken to the revaluation
reserve leaving a balance of Rs.10·3 million. The opening provision was Rs.11·4
million, thus Rs.1·1 million (Rs.11·4 million – Rs.10·3 million) is released to the
income statement.

(iv) Non-current assets/depreciation/revaluation:

Development expenditure:
The capitalized development expenditure has suffered impairment due to falling sales
of the related product. Therefore, it should be written down to its recoverable amount,
which is the higher of its value in use (Rs.30 million) or its realizable value (Rs.40
million). Thus it should be written down to Rs.40 million.

Impairment loss on development expenditure Rs.'000


Carrying amount (Rs.70 – 15) million 60,000
Less: Higher of realizable value and value in use (i.e. realizable value) 40,000
20,000

Depn. on development expenditure

Amortization has been based on the revised carrying value of Rs.40 million and its
remaining life of 4 years.
Depn. on development expenditure = Rs 40/4 = Rs 10 million

© The Institute of Chartered Accountants of Nepal 244


CAP II Paper 1: Advanced Accounting

a) Leasehold Building Rs. '000


Cost 200,000
Depreciation at 1 Shrawan 2066 (of 7yrs@8milion each year) (56,000)
Net book value prior to revaluation 144,000
(difference on revalued amount and net book value)
Valuation 1 Shrawan 2066 270,000
Revaluation surplus 126,000

Depreciation year to 31 Ashadh 2067 (270,000/18 (i.e. 25 – 7) years) (15,000)

Net book value at 31 Ashadh 2067=Rs. (270-15) million=


255,000

_______
b) Plant: Rs. '000
Cost – non joint venture 124,800
– joint venture 70,000
_______
194,800
Depreciation for year
– non joint venture (124,800 – 48,800) × 20% 15,200
– joint venture (70,000/4 years) 17,500
_______
32,700
Net book value (194,800 – (48,800 + 32,700)) 113,300
_______

Net book value of property, plant and equipment (b+c)


(255,000 + 113,300) 368,300
_______

(v) Current liabilities Rs. '000


Per trial balance 82,200
Joint venture 4,000
Accrued loan interest ((10% × 100,000) – 5,000) 5,000
Taxation 15,000
_______
106,200

7. From the following details relating to the accounts of XYZ Ltd. Prepare Cash flow statement
for the year ended 16.07.2011.

© The Institute of Chartered Accountants of Nepal 245


CAP II Paper 1: Advanced Accounting

16.07.2011 16.07.2010
Liabilities Rs. Rs.

Share Capital 5,00,000 4,00,000


Reserve 2,00,000 1,80,000
Profit & Loss 1,50,000 1,30,000
Debentures 2,00,000 2,50,000
Provision for Income Tax 1,40,000 1,50,000
Sundry Creditors 1,70,000 1,90,000
Proposed Dividend 1,40,000 1,30,000
15,00,000 14,30,000
Assets
Goodwill 1,90,000 2,00,000
Plant & Machinery 5,29,250 3,98,000
Debenture Discount 15,000 18,000
Prepaid Expenses 15,750 14,000
Investments 1,60,000 2,00,000
Sundry Debtors 2,90,000 3,40,000
Stock 1,80,000 1,50,000
Cash and Bank 1,20,000 1,10,000
15,00,000 14,30,000

a) 15% Depreciation has been charged in the accounts on Plant & Machinery.
b) Old machine costing Rs. 1,00,000 (WDV Rs. 40,000) have been sold for Rs. 70,000.
c) A machine costing Rs. 20,000 (WDV Rs. 6,000) has been disregarded.
d) Rs. 20,000 profit has been earned by sale of investments.
e) Debentures have been redeemed at 5% premium.
f) Rs. 45,000 income tax has been paid and adjusted against Provision for Income Tax
Account.
(CAP Jun. 2012 Q1 – 20 Marks)
Answer
Cash flow statement of XYZ Ltd. for the year ended as on 16.07.2011
Particulars Rs. Rs.
Cash Flows from Operating Activities:
Increase in reserves 20,000
Increase in P/L 20,000
Add: Goodwill written off 10,000
Plant & machinery disregarded 6,000
Depreciation on plant & machinery 93,397
Discount on debentures written off 3,000
Premium on debentures redeemed 2,500
Provision for income tax 35,000
Proposed dividend 140,000

© The Institute of Chartered Accountants of Nepal 246


CAP II Paper 1: Advanced Accounting

Decrease in sundry debtors 50,000 339,897


379,897
Less: Profit on sale of plant & machinery 30,000
Profit on sale of investments 20,000
Increase in prepaid expenses 1,750
Increase in stock 30,000
Decrease in sundry creditors 20,000
Income tax paid 45,000 (146,750)
Net Cash from Operating Activities 233,147
Cash Flows from Investing Activities:
Sale of plant & machinery 70,000
Sale of investment 60,000
Purchase of plant & machinery (270,647)
Net cash used in Investing Activities (140,647)
Cash Flows from Financing Activities:
Issue of share capital 100,000
Redemption of debentures (52,500)
Dividend paid (130,000)
Net cash used in Financing Activities (82,500)
Net increase in Cash & Cash Equivalents 10,000
Cash & Cash Equivalents at beginning 110,000
Cash & Cash Equivalents at end 120,000

Working Notes:
1.
Plant & Machinery Account
Particulars Rs. Particulars Rs.
To Balance b/d 398,000 By Bank 70,000
To Profit on sale 30,000 By Assets w/o 6,000
By Accumulated
To Bank 270,647 93,397
Depreciation
By Balance c/d 529,250
698,647 698,647

2.
Provision for Tax Account
Particulars Rs. Particulars Rs.
To Bank 45,000 By Balance b/d 150,000
To Balance c/d 140,000 By Current Tax expense 35,000
185,000 185,000

3.
Investment Account

© The Institute of Chartered Accountants of Nepal 247


CAP II Paper 1: Advanced Accounting

Particulars Rs. Particulars Rs.


To Balance b/d 200,000 By Bank 60,000
To Profit on sale 20,000 By Balance c/d 160,000
220,000 220,000

4. Addition to plant & machinery during the year and depreciation:


Particulars Rs. Rs.
Closing balance (WDV) of total plant & machinery 529,250
Less: Closing balance (WDV) of opening plant & machinery:
Opening WDV of plant & machinery 398,000
Less: WDV of machinery sold 40,000
Less: WDV of machinery disregarded 6,000
Balance of opening machinery (WDV) 352,000
Less: Depreciation @ 15% thereof 52,800 299,200
Closing balance (WDV) of machinery purchased during the
230,050
year
Original cost of machinery purchased during the year (Rs. 230,050 / 0.85) 270,647
Depreciation @ 15% thereof 40,597
Total depreciation for the year [Rs. 52,800 + Rs. 40,597] 93,397

8. The books of Mr. Z showed the following information:

1.1.2011 31.12.2011
Rs. Rs.
Bank balance - 50,000
Debtors - 87,500
Creditors - 46,000
Stock 50,000 62,500
Non-current assets 7,500 9,000

The following are the details of the bank transactions:


Rs.
Receipt from customers 3,40,000
Payments to creditors 2,80,000
Capital brought in 5,000
Sale of Non-current assets 1,750
Expenses paid 49,250
Drawings 25,000
Purchase of Non-current assets 5,000

Other information:
(i) Cost of goods sold Rs. 2,60,000
(ii) Gross profit 25% on cost of goods sold

© The Institute of Chartered Accountants of Nepal 248


CAP II Paper 1: Advanced Accounting

(iii)Book value of assets sold Rs. 2,500


Prepare Statement of Profit or Loss account for the year ended 31.12.2011 and Financial
Position as at that date. (CAP Jun. 2012 3a (ii) – 8 Marks)

Answer
In the books of Mr. Z
Statement of Profit or Loss
For the year ended 31st December, 2011
Particulars Rs. Particulars Rs.
Opening stock 50,000 By Sales 325,000
Purchases 272,500 By Closing stock 62,500
Gross profit 65,000
387,500
Expenses 49,250
Loss on sale of asset 750
Depreciation 1,000
Net profit 14,000
65,000

Financial Position
As on 31st December, 2011
Equity & Liabilities Rs. Assets Rs.
Capital: Non-current Asset (Less: Acc 9,000
Opening balance 169,000 dep 1,000)
Add: Net profit 14,000 Stock 62,500
Add: Additional capital 5,000 Debtors 87,500
188,000 Bank balances 50,000
Less: Drawings 25,000 163,000
Creditors 46,000
209,000 209,000

Working Notes:
1. Opening Balance Sheet:
Financial Position as on 1st January, 2011
Liabilities Rs. Assets Rs.
Capital – Bal. figure 169,000 Non-current Asset 7,500
Creditors 53,500 Debtors 102,500
Stock 50,000
Bank balance 62,500
222,500 222,500

2. Opening bank balances:


Bank Account

© The Institute of Chartered Accountants of Nepal 249


CAP II Paper 1: Advanced Accounting

Particulars Rs. Particulars Rs.


To Balance b/d – Bal. figure 62,500 By Creditors 280,000
To Debtors 340,000 By Expenses 49,250
To Capital bought in 5,000 By Drawings 25,000
To Non-Current assets - Sales 1,750 By Non-Current assets 5,000
By Balance c/d 50,000
409,250 409,250

3. Opening debtors:
Debtors account
Particulars Rs. Particulars Rs.
To Balance b/d – Bal. figure 102,500 By Bank 340,000
To Sales (WN 8) 325,000 By Balance c/d 87,500
427,500 427,500

4. Opening creditors:
Creditors account
Particulars Rs. Particulars Rs.
To Bank 280,000 By Balance b/d – Bal. figure 53,500
To Balance c/d 46,000 By Purchases (WN 7) 272,500
326,000 326,000

5. Depreciation:
Non-Current asset account
Particulars Rs. Particulars Rs.
To Balance b/d 7,500 By Bank (Sale) 1,750
To Bank 5,000 By Loss on sale 750
By Acc. Depreciation – Bal.
1,000
figure
By Balance c/d 9,000
12,500 12,500

6. Gross Profit = Rs. 260,000 × 25% = Rs. 65,000.


7. Cost of goods sold = Opening stock + Purchases – Closing stock
Rs. 260,000 = Rs. 50,000 + Purchases - Rs. 62,500
Purchases = Rs. 272,500.
8. Sales = Cost of goods sold + gross profit
= Rs. 260,000 + Rs. 65,000
= Rs. 325,000.
Note:
The question was printed as if there were no opening bank balance, opening debtors and opening
creditors. If students are aware of this anomaly in the question, then answer to this question
would be as follows:

© The Institute of Chartered Accountants of Nepal 250


CAP II Paper 1: Advanced Accounting

In the books of Mr. Z


Trading and Profit and Loss Account
For the year ended 31st December, 2011
Particulars Rs. Particulars Rs.
Opening stock 50,000 Sales 325,000
Purchases 272,500 Closing stock 62,500
Gross profit c/d 65,000
Expenses 49,250
Loss on sale of asset 750
Depreciation 1,000
Net profit c/d 14,000
65,000

Balance Sheet
As on 31st December, 2011
Liabilities Rs. Assets Rs.
Non-current Asset (Less:
Capital: 9,000
Acc dep 1,000)
Opening balance 57,500 Stock 62,500
Add: Net profit 14,000 Debtors 87,500
Add: Additional capital 5,000 Bank balances 50,000
56,500 Advance to suppliers 53,500
Less: Drawings 25,000 51,500
Advance from customers 102,500
Loan 62,500
Creditors 46,000
262,500 262,500

Working Notes:
1. Opening balance sheet:
Financial Position as on 1st January, 2011
Liabilities Rs. Assets Rs.
Capital – Bal. figure 57,500 Non-current assets 7,500
Stock 50,000
57,500 57,500

2. Bank balances / Loan:


Bank Account
Particulars Rs. Particulars Rs.
To Debtors 340,000 By Creditors 280,000
To Capital bought in 5,000 By Expenses 49,250
To Non-current assets 1,750 By Drawings 25,000
To Loan – Bal. figure 62,500 By Non-current assets 5,000

© The Institute of Chartered Accountants of Nepal 251


CAP II Paper 1: Advanced Accounting

By Balance c/d 50,000


409,250 409,250

3. Advances from customers:


Debtors account
Particulars Rs. Particulars Rs.
To Sales (WN 8) 325,000 By Bank 340,000
To Advances – Bal. figure 102,500 By Balance c/d 87,500
427,500 427,500

4. Advances to suppliers:
Creditors account
Particulars Rs. Particulars Rs.
To Bank 280,000 By Purchases (WN 7) 272,500
To Balance c/d 46,000 By Advances – Bal. figure 53,500
326,000 326,000

5. Depreciation:
Fixed Assets account
Particulars Rs. Particulars Rs.
To Balance b/d 7,500 By Bank – Sale 1,750
To Bank 5,000 By P/Loss A/c - loss on sale 750
By Acc. Depreciation – Bal.
1,000
figure
By Balance c/d 9,000
12,500 12,500

6. Gross Profit = [Rs. 260,000 × 25% = Rs. 65,000.


7. Cost of goods sold = Opening stock + Purchases – Closing stock
Rs. 260,000 = Rs. 50,000 + Purchases - Rs. 62,500
Purchases = Rs. 272,500.
8. Sales = Cost of goods sold + gross profit
= Rs. 260,000 + Rs. 65,000
= Rs. 325,000.

9. The following are the ledger balances of Sri Babubhaj for the year ended 31st March 2015.
(Rs.)
Capital Account 2,00,000
Drawing Account 15,250
Sundry Creditors 56,562
Loans taken on Mortgage 20,000
Sundry Debtors 17,800
Goodwill 10,000

© The Institute of Chartered Accountants of Nepal 252


CAP II Paper 1: Advanced Accounting

Interest on Mortgage Loan 400


Cash in Hand 364
Bad Debts Provision (Cr.) 450
Plant & Machinery 22,500
Sales less Returns 1,50,210
Cash at Bank 6,756
Electric Power 3,250
Discount Paid 1,216
Dividends Received 3,250
Bills Payables 11,575
st
Investment reserve as at 1 April, 2014(Cr.) 1,250
Purchase less Returns 1,65,265
Salaries 15,200
Rent and Rates 3,756
Electricity Charges 2,120
Insurances 1,000
Advertisement 17,256
st
Stock on 1 April, 2014
Raw Materials 51,265
Finished Goods 12,625
Factory Land & Buildings 21,200
Bad Debts 450
Carriage Inwards 5,210
Wages 18,560
st
Investments at Market Price as on 1 April, 2014 36,150
Bills Receivables 16,263
Outstanding Expenses 2,100
General Expenses 4,065
Prepaid Insurances 200
Discount 2715
a) Factory buildings were constructed on land purchased for Rs. 10,000 during the year but
this was wrongly posted to purchase account.
b) Provide depreciation at 10% on plant and 5% on buildings.
c) Interest has to be provided on mortgage loan at 6% per annum keeping in view the fact that
a sum of Rs. 5,000 was repaid on 30th June, 2014.
d) The market value of the investments as at 31st march, 2015 was Rs. 37,000. The investment
reserve account represents the difference between the cost and market value.
e) Electricity charges were paid in advance to the extent of Rs. 155.
f) Some investments were sold during the year for Rs. 1,260 realizing a profit of Rs. 125 when
compared to their market value on 1st April, 2014. The sale proceeds were, however,
credited to the sales account.
g) Provision for bad debts should be maintained at 10%.

© The Institute of Chartered Accountants of Nepal 253


CAP II Paper 1: Advanced Accounting

h) Out of the bills receivables, one bill for Rs. 1,200 matured for the payment in the last week
of March, 2015. However, the bankers informed that they could not collect the said bill on
31st March, 2015. The information was not recorded in the books.
i) The closing stock was as follows:
Raw materials Rs. 85,263
Finished goods Rs. 64,987

You are required to prepare the Statement of Profit or Loss and the Financial Position.
(CAP Jun. 2017 Q1-20 Marks)
Answer

SHRI BABUBHAJ
Statement of Profit or Loss
For the Year Ending 31st March, 2015
Rs.
Sales 1,48,950
Cost of Sales
Opening Stock of FG 12,625
Cost of Production
Opening Stock 51,265
Purchase less return 1,55,265
2,06,530
Less; Closing Stock 85,263 1,21,267
Wages 18,560
Carriage Inwards 5,210
Electric Power 3,250
Depreciation on
Plant & Machinery 2,250
Factory Building 1,060 3,310
Less: Closing Stock of FG (64,987)
Gross Profit c/d 49,715
2,13,937
Profit on Sale of Investment 125
Discount Received 2,715
Dividend Received 3,250

Salaries 15,200
General Expenses 4,065
Rent and Rates 3,756
Electricity 1,965
Advertising 17,256
Insurances 1,000
Interest on Mortgage Loan 1,275
Provision for bad Debts 1900

© The Institute of Chartered Accountants of Nepal 254


CAP II Paper 1: Advanced Accounting

Add: Bad Debts 450


2,350
Less: Existing Provision 450 1,900
Discount Allowed 1,216
Net Profit 8,172
55,805

Financial Position of Shri Babubhaj as on 31st March, 2015

Capital & Liabilities Assets

Rs. Rs. Rs. Rs.

Capital 2,00,000 Non-Current Assets


Add: Profit 8,172 Goodwill
10,000
2,08,172 Factory Land &
Less: Drawing 15,250 1,92,922 Building 21,200
Addition during
The year 10000
Investment Reserve 3,235 31,200
Loan on Mortgage 20,000 Less: Depreciation1,060
30,140
Add: Interest accrued 875 20,875
Sundry Creditors 56,562 Plant & Machinery 22,500
Bills Payables 11,575 Less: Depreciation 2,250
20,250
Expenses Outstanding 2,100 Investment at Market Value
37,000
Suspense Account 9 Current Assets
Stock in Trade(assumed at cost)
Raw Materials 85,263
Finished Goods 64,987 150,250
Sundry Debtors 19,000
Less: Provision for
Bad debts 1,900 17,100
Bills Receivables 15,063
Cash In Hand 364
Cash at Bank 6,756
Prepaid Expenses 355
2,87,278 2,87,278

Notes:

© The Institute of Chartered Accountants of Nepal 255


CAP II Paper 1: Advanced Accounting

(i) Rs. 10,000 has been deducted from purchases and added to Land & Buildings.
(ii) Rs. 1200 has been deducted from Bills Receivables and added to the sundry
debtors.
(iii) Rs. 1260 has been deducted from Sales, being sale proceeds of investments the
cost Rs. 1135 has been from investments.
(iv) The cost of investments is Rs. 35,015 i.e. Rs. 36,150 less Rs. 1135. Since these
have to be shown at Rs. 37,000, Rs. 1985(i.e. Rs. 37000-Rs. 35015) has been
added to investment Reserve which now becomes Rs. 3235. Strictly, speaking the
cost, of investments sold being less than the market value, the profit should be
more than Rs. 125.
However, the cost of each investment would be different hence this point has been
ignored.

(v) Interest on Mortgage Loan:


Interest on Rs. 25,000 @ 6% for 3 months 375
Interest on Rs. 20,000@6% for 9 months 900
1275
Less: Paid 400
Outstanding 875

10. Mr. Bhandari commenced business as a cloth merchant on 1st Shrawan, 2068 with a capital Rs.
20,000. On the same day, he purchased furniture and fittings for Rs. 10,000 in cash.
Other information:
Particulars Amount (Rs.)
Sales (inclusive of cash sales Rs. 47,000) 1,29,000
Purchases( inclusive of cash purchases Rs. 14,000) 1,01,000
Mr. Bhandari’s drawings (50% cloth) 13,200
Salaries paid to staffs 12,300
Bad debts written off 500
Business expenses paid 11,700

Mr. Bhandari took cloth worth Rs. 700 from the shop for private use and paid Rs.500 to his
son, but he omitted to record these transactions in books. On 31st Ashadh, 2069 his sundry
debtors were Rs. 5, 200 and sundry creditors were Rs. 13,600. Stock in hand on 31st Ashadh,
2069 was Rs. 18,550.

From the above transactions obtained from his books kept on single entry, you are asked to
prepare financial statements for the year ended 31st Ashadh, 2069.
(CAP Jun. 2017 Q3a-10 Marks; Inter Jun. 2009 Q5b- 10Marks)
Answer

Statement of Profit or Loss


For the year ended 31.03.2069

© The Institute of Chartered Accountants of Nepal 256


CAP II Paper 1: Advanced Accounting

Dr Cr
Particulars NRs Particulars NRs.
Purchase 101,000 Sales 1,29,000
Less: Drawings (7,300) 93,700 Closing Stock 18,550
(6,600+700)
Gross Profit 53,850
Total 147,550
Salaries to staff 12,300
Bad Debts written off 500
Business expenses 11,700
Net Profit 29,350
Total 53,850 53,850

Statement of Financial Position


As on 31.03.2069
Capital 20,000 Furniture and Fittings 10,000
Add: Net Profit 29,350 Sundry Debtors 5,200
Less: Drawings (14,400) 34,950 Cash 14,800
(W.N-3)
Sundry Creditors- 13,600 Closing Stock 18,550
48,550 48,550

Working Notes
Sundry Debtors Account
Dr Cr
Particulars Amount Particulars Amount
To Credit Sales (129,000- 82,000 By Cash 76,300
47,000)
By Bad Debts 500
By Balance c/d 5,200
82,000 82,000

Sundry Creditors Account


Dr Cr
Particulars Amount Particulars Amount
To Cash 73,400 By Credit Purchase 87,000
(101,000-14,000)
To Balance c/d 13,600
87,000 87,000

Cash Account
Dr Cr

© The Institute of Chartered Accountants of Nepal 257


CAP II Paper 1: Advanced Accounting

Particulars Amount Particulars Amount


To Capital 20,000 By Furniture & Fittings 10,000
To Cash Sales 47,000 By Purchases 14,000
To Sundry Debtors (W.N-1) 76,300 By Sundry Creditors(W.N-2) 73,400
By Salaries to staff 12,300
By Drawings (6,600+500) 7,100
By Business Expenses 11,700
By Balance c/d 14,800
1,43,300 143,300

11. The following is the Financial Position of a concern on 31st Ashadh, 2073:

Liabilities Rs. Assets Rs.


Capital 10,00,000 Fixed Assets 4,00,000
Creditors (Trade) 1,40,000 Stock 3,00,000
Profit & Loss 60,000 Debtors 1,50,000
Cash & Bank 3,50,000
12,00,000 12,00,000

The management estimates the purchases and sales for the year ended 31st Ashadh, 2074 as
under:

Particulars Up to 32.2.2074 (Rs.) Ashadh 2074 (Rs.)


Purchases 14,10,000 1,10,000
Sales 19,20,000 2,00,000

It was decided to invest Rs. 1,00,000 in purchases of Non-current assets, which are depreciated
@ 10% on cost.

The time lag for payment to Trade Creditors for purchase and receipt from Sales is one month.
The business earns a gross profit of 30% on turnover. The entire sales and purchases are taken
on credit basis. The expenses against gross profit amount to 10% of the turnover. The amount
of depreciation is not included in these expenses.

Draft a Financial Position as of 31st Ashadh, 2074 assuming that creditors are all Trade
Creditors for purchases and debtors for sales and there is no other item of current assets and
liabilities apart from stock and cash and bank balances. (CAP Jun. 2018 Q2a-10 Marks)

Answer:
Projected Financial Position of
As on 31st Ashadh, 2074

© The Institute of Chartered Accountants of Nepal 258


CAP II Paper 1: Advanced Accounting

Liabilities Rs Assets Rs
Capital 10,00,000 Non-current assets 4,00,000
Profit & Loss Account Additions 1,00,000
as on
1st Shrawan, 60,000 5,00,000
2073
Add: Profit for 3,74,000 4,34,000 Less: Depreciation (50,000) 4,50,000
the year
Creditors 1,10,000 Stock in trade 3,36,000
(Trade)
Sundry Debtors 2,00,000
Cash & Bank Balances 5,58,000
15,44,000 15,44,000
Working Notes:
Projected Statement of Profit or Loss
For the year ended 31st Ashadh, 2074
Particulars Rs Particulars Rs
Opening Stock 3,00,000 Sales 21,20,000
Purchases 15,20,000 Closing Stock 3,36,000
Gross Profit c\d (30% on sales) 6,36,000 (balancing figure)
24,56,000 24,56,000
Sundry Expenses (10% on sales) 2,12,000
Depreciation 50,000
Net Profit 3,74,000
6,36,000 6,36,000

Cash and Bank Account


1 Shrawan, 2073 to 31st Ashadh, 2074
st

Rs. Rs
.
To Balance b/d 3,50,000 By Sundry Creditors 15,50,000
To Sundry Debtors 20,70,000 (Rs1,40,000+Rs 14,10,000)
(Rs 1,50,000 + Rs By Expenses 2,12,000
19,20,000)
By Non-current assets 1,00,000
By Balance c/d 5,58,000
24,20,000 24,20,000

12. From the following Financial Position of Mr. Ram, prepare a Cash Flow Statement as per NAS
3 for the year ended 31.03.2074:

Financial Position of Mr. Ram

© The Institute of Chartered Accountants of Nepal 259


CAP II Paper 1: Advanced Accounting

As on 1.4.2073 As on 31.03.2074
Liabilities:
Ram’s Capital Account 5,00,000 6,12,000
Sundry creditors 1,60,000 1,76,000
Mrs. Ram’s loan 1,00,000 -
Long term loan from bank 1,60,000 2,00,000
9,20,000 9,88,000

Assets:
Land 3,00,000 4,40,000
Plant & Machinery 3,20,000 2,20,000
Stock 1,40,000 1,00,000
Debtors 1,20,000 2,00,000
Cash 40,000 28,000
9,20,000 9,88,000

Additional information:
A machine costing Rs. 40,000 (accumulated depreciation there on Rs. 12,000) was sold for Rs.
20,000. The provision for depreciation on 1.4.2073 was Rs. 1,00,000 and on 31.03.2074 was
Rs. 1,60,000. The net profit for the year ended on 31.03.2074 was Rs. 1,80,000.
(CAP Jun. 2018 Q2b-10 Marks)
Answer
Cash Flow Statement of Mr. Ram for the year ended 31.03.2074

Rs.
(i)Cash flow from operation activities
Net Profit (given) 1,80,000
Adjustment for Depreciation on Plant & Machinery 72,000
(W.N.2) 8,000 80,000
Loss on sale of Machinery (W.N.1) 2,60,000
Operating profit before working capital changes 40,000
Decrease in Stock (80,000)
Increase in Debtors 16,000 ( 24,000)
Increase in Creditors 2,36,000
Net cash from operating activities
(ii) Cash flow from investing activities: 20,000
Sale of Machinery (1,40,000)
Purchase of Land(4,40,000-3,00,000) (1,20,000)
Net cash used in investing activities
(iii) Cash flow from financing activities: (1,00,000)
Payment of Mrs. Ram’s Loan (68,000)
Drawings(W.N. 3) 40,000
Loan from bank (1,28,000)

© The Institute of Chartered Accountants of Nepal 260


CAP II Paper 1: Advanced Accounting

Net cash used in financing activities (12,000)


Net decrease in cash 40,000
Cash balance as on 1.4.2073 28,000
Cash balance as on 31.03.2074

Working Notes:
Plant & Machinery A/c
Rs. Rs.
To Balance b/d 4,20,000 By Bank-sales 20,000
( 3,20,000+1,00,000) By Acc. Dep. A/c 12,000
By Loss on sale(40,000-
20,000-12,000) 8,000
By balance c/d
(2,20,000+1,60,000) 3,80,000
4,20,000 4,20,000

Accumulated Depreciation on Plant & Machinery A/c


Rs. Rs.
To Plant & Machinery A/c 12,000 By Balance b/d 1,00,000
To Balance c/d 1,60,000 By Depreciation 72,000
1,72,000 A/c(Bal.fig) 1,72,000

Mr. Ram’s Drawing


Rs.
Opening Capital 5,00,000
Add: Net profit 1,80,000
6,80,000
Less Closing Capital (6,12,000)
Drawings 68,000

13. Following is the cash flow abstract of Alpha Ltd. for the year ended 31st March, 2008:
Inflows Rs. Outflows Rs.
Opening balance: Payment to creditors 90,000
Cash 10,000 Salaries and wages 25,000
Bank 70,000 Payment of overheads 15,000
Share capital 500,000 Non-current assets acquired 400,000
Collection from Debtors 350,000 Debentures redeemed 50,000
Sale of fixed assets 70,000 Bank loan repaid 250,000
Taxation 55,000
Dividends 100,000

© The Institute of Chartered Accountants of Nepal 261


CAP II Paper 1: Advanced Accounting

Closing balance:
Cash 5,000
Bank 10,000
1,000,000 1,000,000
Required:
Prepare Cash Flow Statement for the year ended 31st March, 2008 in accordance with Nepal
Accounting Standard. (CAP Dec. 2010 Q4c -5 Marks)

Answer
Cash Flow Statement
for the year ended 31.3.2008
Rs. Rs.
Cash flow from operating activities
Cash received from customers 3,50,000
Cash paid to suppliers (90,000)
Cash paid to employees (salaries and wages) (25,000)
Other cash payments (overheads) (15,000)
Cash generated from operations 2,20,000
Income tax paid (55,000)
Net cash from operating activities 1,65,000
Cash flow from investing activities
Payment for purchase of Non-current assets (4,00,000)
Proceeds from sale of fixed assets 70,000
Net cash used in investment activities (3,30,000)
Cash flow from financing activities
Proceeds from issue of share capital 5,00,000
Bank loan repaid (2,50,000)
Debentures redeemed (50,000)
Dividends paid (1,00,000)
Net cash from financing activities 1,00,000
Net decrease in cash and cash equivalents (65,000)
Cash and cash equivalents at the beginning of the year 80,000
Cash and cash equivalents at the end of the year 15,000

14. Following Financial Position pertains to Rising Nepal Ltd. as on 31st Ashadh 2065:
Rising Nepal Ltd.
Financial Position as on 31st Ashadh 2065

Liabilities Current Year(Rs)


Share Capital 13,00,000
Reserve and Surplus 4,50,000
Total 17,50,000

© The Institute of Chartered Accountants of Nepal 262


CAP II Paper 1: Advanced Accounting

Assets Current Year(Rs)


Non-current assets-Gross 11,00,000
Less: Accumulated Depreciation 2,50,000
Net Block 8,50,000
Current Assets
Cash in Hand 40,000
Cash at Bank 3,25,000
Sundry Debtors 6,60,000
Advance to Supplier 2,50,000
Stock in Hand 50,000
Sub-total 13,25,000
Less: Current Liabilities
Sundry Creditors 3,00,000
Advance from Customers 1,25,000
Sub-total 4,25,000
Net Current Assets 9,00,000
Total 17,50,000

Following matters were not considered while preparing above Balance Sheet:
Advance to Supplier includes Raw material advance amounting to Rs. 1,75,000 given to a
Limited company in India. The company was liquidated on December 2007 (Poush
2064).Liquidator made public notice for any claims against the company during the course
of liquidation. Rising Nepal Limited was aware of the process only on Shrawan 2065. The
court declared the company bankrupt on Shrawan. There was no possibility of recovering
the advance in any circumstances. Similarly, Cheques issued by customers amounting to
Rs. 60,000 was received by the company on 2nd Shrawan 2065. Further, there was a fire in
one of its plants on first week of Bhadra causing significant damage to the plant and assets
thereon. The book value of the plant along with all the Non-current assets was Rs. 3,50,000.
However, no estimation of loss is finalized yet.

The Financial Statements of the company were approved on 15th Bhadra 2065.

Prepare the Financial Position after considering the provisions of applicable Nepal
Accounting Standard. (CAP Dec. 2009 5b -10 Marks)

Answer

Rising Nepal Ltd


Financial Position as on Ashadh end 2065

Liabilities Current Year(NRs)


Share Capital 1,300,000
Reserve and Surplus 275,000

© The Institute of Chartered Accountants of Nepal 263


CAP II Paper 1: Advanced Accounting

Total 1,575,000
Assets Current Year(NRs)
Non-current assets-Gross 1,100,000
Less: Accumulated Depreciation 250,000
Net Block 850,000
Current Assets
Cash in Hand 40,000
Cash at Bank 325,000
Sundry Debtors 660,000
Advance to Supplier 75,000
Stock in Hand 50,000
Sub-total 1,150,000
Less: Current Liabilities
Sundry Creditors 300,000
Advance from Customers 125,000
Sub-total 425,000
Net Current Assets 725,000
Total 1,575,000

Disclosure of Plant and Machinery destroyed by fire

A material damage was made by fire to plant and machinery with book value of Rs. 3,50,000.
The process of estimation of the loss is being carried out for the same. The amount of loss and
the recovery of the same by way of insurance claim can be ascertained only upon completion
of the process of assessment of the damage thereon.

Note: Neither the adjustment nor the disclosure is required in case of cheques received in the
ordinary course of the business.

15. The following information pertains to the business of Mr. Mohan for the year ended on March
31, 2003. You have been requested to prepare for him a Statement of Profit or Loss for the
ended on March 31, 2003 and a Financial Position as on that date.

a. Assets and Liabilities as on March 31, 2002:

Rs. Rs.
Bills Receivable 50,000 Building 1,40,000
Debtors Creditors 72,000
Stock-in trade 1,50,000 Bills Payable 32,000
Machinery 94,000

b. Summarized cash transactions for the year ended March 31, 2003:

© The Institute of Chartered Accountants of Nepal 264


CAP II Paper 1: Advanced Accounting

Rs. Rs.
To Cash balance as on 1-4- 1,400 By Bank overdraft as on 1-4- 10,000
2002 2002
To Receipts from Debtors 5,80,000 By Salaries 24,000
To Bills Receivable 2,00,000 By Bills payable 2,86,000
By Wages 31,600
By Payments to Creditors 2,94,000
By Office Expenses 16,000
By Drawings 90,000
By Balance as on 31-3-2003 29,800
In Hand 4,800
At Bank 25,000
Total 7,81,400 Total 7,81,400

c. Other Information:

Rs. Rs.
Sales 8,14,000 Bills Receivable received during the 2,18,000
year
Discount to customers 4,000 Bills payable issued during the year 3,00,000
Purchases 6,00,000 Stock-in-Trade as on 31-3-2003 1,06,000
Discount allowed by 2,000 Sundry Debtors as on 31-3-2003 90,000
suppliers

Provide depreciation on Plant and Machinery at 15% and on Land and Building at 5%.
(Inter June 2003, Q No. 1-16-10 Marks)

Answer:
Statement of Profit or Loss
(For the year ended March 31, 2003)

Rs. Rs.
Opening Stock 1,50,000 Sales 8,14,000
Purchase 6,00 ,000 Closing Stock 1,06,000
Wages 31,600 Discount Received 2,000
Gross Profit 1,38,400
Salaries 24,000
Office Expenses 16,000
discount allowed 4000
Depreciation:
Machinery 14,100 21,100
Building 7,000
Net Profit 75,300

© The Institute of Chartered Accountants of Nepal 265


CAP II Paper 1: Advanced Accounting

Balance Sheet as at March 31, 2003

Liabilities Rs. Rs. Assets Rs. Rs.


Capital (5) 3,99,400 Machinery 94,000
Less: Drawing 90,000 Less: Depreciation 14,100 79,900
3,10,000
Add: Profit 75,300 3,84,700 Building 1,40,000
Less: Depreciation 7,000 1,33,000
Creditors (2) 76,000
Bills payable 46,000 Debtors 90,000
(4)
Bills Receivable 68,000
(3)
Stock-in-Trade 1,06,000
Cash:
On Hand 4,800
At Bank 25,000 29,800
5,06,700 5,06,700

Working Notes:
1. Debtors A/c
Rs. Rs
To Balance b/d 78,000 By Cash 5,80,000
(Balancing Figure)
To Sales 8,14,000 By Discount 4,000
By Bills Receivable 2,18,000
By Balance c/d 90,000
8,92,000 8,92,000

2. Creditors A/c
Rs. Rs
To Cash 2,94,000 By Balance b/d 72,000
To Discount 2,000 By Purchases 6,00,000
To Bills Payable 3,00,000
To Balance c/d 76,000
6,72,000 6,72,000

3. Bills Receivable A/c


Rs. Rs
To Balance b/d 50,000 By Cash 2,00,000
To Sundry Debtors 2,18,000 By Balance c/d 68,000

© The Institute of Chartered Accountants of Nepal 266


CAP II Paper 1: Advanced Accounting

2,68,000 2,68,000

4. Bills Payable A/c


Rs. Rs
To Cash 2,86,000 By Balance b/d 32,000
To Balance c/d 46,000 By Sundry Creditors 3,00,000
3,32,000 3,32,000

5. Statement of Affairs as on 31 March, 2002 A/c


Rs. Rs
Creditors 72,000 Bills Receivable 50,000
Bills Payable 32,000 Debtors (1) 78,000
Bank Overdraft 10,000 Stocks-in-Trade 1,50,000
Capital (Balancing Figure) 3,99,400 Machinery 94,000
Building 1,40,000
Cash in hand 1,400
5,13,400 5,13,400

16. Nepal Travel Agency sells tickets for Inland Transport Ltd., Sita Airlines and Indian Railways.
The rate of commission due to the agency on account of sales of tickets are 10%, 7.5% and 5%
respectively on the sale price of tickets. The firm closes its books on 31st December every year.
The balances as on 31st December 2003 were as follows:

Particulars Dr. (Rs.) Cr. (Rs.)


Capital 50,000
Deposits from customers of Inland Transport Ltd. 10,000
Deposits from General Public 10,000
Interest due for half year on above 500
Auditors 1,500
Advertising 1,000
Rate and Taxes 500
Fixtures and Fittings 20,000
Motor Car 18,000
Debtors for Rail Tickets 5,000
Debtors for Air Ticket 2,000
Rent paid in advance 1,250
Bank Balance 27,250
73,500 73,500

Other available particulars are:

a. From the Bank Statement, returned cheques and the paying in slips for the year
ended 31st December, 2004.

© The Institute of Chartered Accountants of Nepal 267


CAP II Paper 1: Advanced Accounting

Rs. Rs.
Bank Cheques and 9,23,000 Electricity 5,000
Cash
Payment for Tickets: Rates and Taxes 3,000
Inland Transport Ltd. 6,20,000 Interest paid to public on
their deposits 1,000
Sita Airlines 1,69,000 Amount paid to Auditors 1,500
Indian Railways 84,000 Advertising 6,250
Rent Paid for 4 quarters 5,000 Bank Balance as on
31.12.2004 55,500

b. Weekly expenditure defrayed from cash receipts before banking staff wages Rs.
1,100 Petty Expenses Rs. 50. In addition to the above, the owner has drawn Rs. 650
per month to meet personal expenses and spent Rs. 350 per month for maintenance
of a car in the interest of the agency, out of the cash receipts before banking.

c. Liabilities of the firm as on 31st December, 2004 include:

Auditors' Fees 1,500 Inland Transport Ltd. 5,500


Advertising Charges 1,250 Sita Airlines 16,000
Rates and Taxes 1,000 Indian Railways 11,000

d. Customers deposits on 31st December, 2004 were for Inland Transport Ltd. Rs.
8,000.
e. Debtors for air and rail tickets on 31st December 2004 were Rs. 2,500 and 800
respectively.
f. Depreciation on car and fixtures is allowed at the rate of 20% and 10% of the last
year's balance respectively.
g. Owner agrees to treat the cash differences if any, as his drawings.

Required: Draw a Statement of Profit or Loss showing commission earned for each class of
tickets sold for the year ending 31st December 2004 and a Financial Position as at that date.
(June 2004, Q No. 3-16 Marks)

Answer:

Nepal Travel Agency


Statement of Profit or Loss
For the year ended 31st December, 2004
Dr Cr
Particulars Rs. Particulars Rs.

© The Institute of Chartered Accountants of Nepal 268


CAP II Paper 1: Advanced Accounting

Wages 57,200 Commission for sale of


tickets (See Note iii)
Electricity 5,000 Inland Transport Ltd. 69,500
Rent 5,000 Sita Airlines 15,000
Rates and Taxes Indian Railways 5,000
(Rs. 6,250 + Rs. 1,250 - Rs. 6,500 Net Loss 2,600
1,000)
Depreciation on:
Fixtures & Fittings 2,000
Motor Car 3,600 5,600
Auditors Fees 1,500
Interest 1,000
Petty Expenses 2,600
Car Maintenance 4,200
92,100 92,100

Financial Position as on 31st December, 2004

Equity & Liabilities Rs. Assets Rs.


Capital 50,000 Fixtures & Fitting 20,000
Less: Net Loss 2,600 Less: Depreciation 2,000
Less: Drawing 9,700 18,000
Deposits 10,000 Motor Car 18,000
Add: Interest due 500 10,500 Less: Depreciation 3,600
Customer's Deposit for Ticket 8,000 14,400
O/S
Outstanding Advertising 1,250 Bank Balance 55,500
charges
Outstanding Rates & Taxes 1,000 Sundry Debtors
Due to Inland Transport Ltd. 5,500 (Rs. 800 + Rs. 2500) 3,300
Due to Sita Airlines 16,000 Rent Prepaid 1,250
Due to Indian Railway 11,000
92,450 92,450

Working Notes:

i. Cash Book (Cash and Bank)

Particulars Rs. Particulars Rs.


To Balance C/d 27,250 By Payment made for
tickets
To Collection from sale of Inland Transport Ltd. 6,20,000
ticket

© The Institute of Chartered Accountants of Nepal 269


CAP II Paper 1: Advanced Accounting

(See Note ii) Sita Airlines 1,09,000


Inland Transport Ltd. 6,93,500 Indian Railways 84,000
Sita Airlines 1,99,500 By Rent 5,000
Indian Railways 1,04,200 By Electricity 5,000
By Rates & Taxes 3,000
By Interest 1,000
By Auditors 1,500
By Advertising 6,250
By Wager (Rs. 1,100  52) 57,200
By Car Maintenance (Rs. 4,200
35012)
By Petty Expenses (Rs. 52 2,600
 50)
By Drawings:
Monthly Drawings
(Rs. 650  12) 7,800
Cash Difference 9,700
1,900
By Balance C/d 55,500
10,23,95 10,23,950
0

ii. Statement showing the calculation of cash received against sale of tickets.

Particulars Inland Sita Airlines Indian Railways


Transport Ltd. Rs. Rs.
Rs.
A. Opening Balance – 2,000 5,000
B. Total Sales 6,95,000 2,00,000 1,00,000
6,95,000 2,02,000 1,05,000
C. Less: Closing Balance – 2,500 800
D. Less: Adjustment for
deposits 2,000 – –
E. Cash Received
(A+B - C - D) 6,93,000 1,99,500 1,04,200

iii. Statement showing the calculation of commission on sale of tickets.

Particulars Inland Transport Sita Airlines Indian


Ltd. Rs. Railways
Rs. Rs.
A. Payment made 6,20,000 1,69,000 8,4,000

© The Institute of Chartered Accountants of Nepal 270


CAP II Paper 1: Advanced Accounting

B. Amount still
due on 5,500 16,000 11,000
31.12.2004
C. Purchase after
commission (A 6,25,500 1,85,000 95,00
+ B)
D. Commission 69,500 15,000 5,000
 10  7.5   5
625000  90 185000  92.5 95000  95
     
E. Gross Sale (C + 6,95,000 2,00,000 1,00,000
D)

iv. It is assumed that rent prepaid in 2003 has not been adjusted in 2004.

17. Mr. Praveen is a dealer in fertilizers. He purchases some chemicals and mixes them to
manufacture the fertilizer. He has prepared the following statements. He requires you to
examine and prepare a Statement of Profit or Loss Account and Financial Position, after taking
into account the additional information furnished.
Statement of Profit or Loss
For the year ended 16-07-2006
Particulars Rs. Particulars Rs.
Stock as on 16-07-06 8,00,000 Sales 5,00,000
Purchases 8,00,000 Creditors 10,00,000
Expenses 7,00,000 Stock as on 16-07-2005 2,00,000
Other Income 1,00,000
Net Loss 5,00,000
23,00,000 23,00,000

Financial Position
As on 16-07-2006
Liabilities Rs. Assets Rs.
Capital as on 16.07.05 3,00,000 Debtors 2,50,000
Add: Loss 5,00,000 8,00,000 Cash 20,000
Loan taken on 16.7.05 @ 12% p.a. Bank 80,000
1,00,000
Surplus (Diff.) 4,50,000 Closing Stock 10,00,000

13,50,000 13,50,000
On enquiry, the following additional information was obtained:
1. He is entitled to a rebate of 10% on the listed price of chemicals. The purchase
during the year ended 16th July 2006 of the said chemicals at the listed price
amounted to Rs. 4,00,000.
2. The details of the expenses are as under.

© The Institute of Chartered Accountants of Nepal 271


CAP II Paper 1: Advanced Accounting

Rs
Mixing Wages 2,00,000
Administration and Selling Expenses 2,00,000
Mixing Equipments purchased 1,00,000
Construction of Factory Sheds 1,00,000
Advance for Materials 1,00,000
7,00,000
th
3. A certain raw material was received on 18 July 2006, the invoice for the material
amounting to Rs. 1,00,000 was included in the purchase.
4.A customer who has purchased goods for Rs. 1,00,000 has not taken delivery. The
value of the stock was inadvertently included in the closing stock as on 16th July
2006.
5. He had entered into a Joint Venture with Mr. Shyam Lal. The Sales made on account
of joint venture was included in the sales for Rs. 2,00,000. He had paid Rs. 1,00,000
for the purchase of raw materials. It had been included in the expenses. His share
of profit in the venture was determined at Rs. 50,000. The amount paid for settling
the account was wrongly debited to the Debtors Account.
6. Sum received for cash sales for Rs. 5,00,000 was included in creditors.
7. An invoice for Rs. 50,000 for the goods received on 10th July 2006, and included in
the stock was received only on 17th July 2006.
8. Other outstanding as on 16th July 2006 were:
(a) For the creation of factory shed Rs.50,000
(b) Other Expenses Rs.25,000
9. Provide depreciation @ 5% on the factory sheds and @ 10% on the equipment on
the closing balance.
10.The Bank Balance of Rs. 80,000 included the encashment of his personal deposit
of Rs. 50,000 with the Bank. He had forgotten to credit Account with this amount.
(Inter Dec. 2006 Q 3a-15 Marks)
Answer:

Mr. Praveen
Statement of Profit or Loss
for the year ended on 16-07-2006
Dr Cr
Particulars Rs. Particulars Rs.
Opening Stock 2,00,000 Sales 8,00,000
Purchases 8,10,000 Closing Stock
Mixing Wages 2,00,000 In hand
7,00,000 8,00,000
In transit
Administrative & Selling 2,25,000 1,00,000
Expenses 1,00,000
Depreciation: 50,000

© The Institute of Chartered Accountants of Nepal 272


CAP II Paper 1: Advanced Accounting

Equipment 10,000 Other Income


Factory Shed 7,500 Income from Joint Venture
Interest on Loan 12,000
Net Profit
2,85,500

5,40,000 5,40,000

Mr. Praveen
Financial Position as on Ashadh 32, 2063

Liabilities Rs. Assets Rs.


Capital : Non-current assets:
Opening Balance 3,00,000 Factory Shed 1,50,000
Add: Capital 50,000 Less: Depreciation 7,500 1,42,500
introduced 2,85,000 6,35,000 Mixing Equipment 1,00,000
Add: Profit for the 1,00,000 Less: Depreciation 10,000 90,000
year 12,000 Current Assets:
Loan Stock in trade 8,00,000
Interest Payable 5,50,000 Sundry Debtors 2,00,000
Creditors: 25,000 Rebate Receivable 40,000
-Trade 50,000 Cash in hand 20,000
-Expenses Cash at Bank 80,000
-Capital Expenses
Total 13,72,500 Total 13,72,500

Working Notes

Rs.
(i) Sales as shown 5,00,000
Add: Credit to Sundry Creditors 5,00,000
10,00,000
Less: Joint Venture Sales 2,00,000
8,00,000
(ii) Stock as shown on 16.07.06 8,00,000
Add: Goods purchased but in transit 1,00,000
Less: Goods sold but not delivered 1,00,000
8,00,000

© The Institute of Chartered Accountants of Nepal 273


CAP II Paper 1: Advanced Accounting

(iii) Profit on Joint Venture:


JV Sales 2,00,000
Cost of Sales 1,00,000
Profit 1,00,000
Share of Praveen 50,000
(iv) Purchases: As given 8,00,000
Add: Outstanding 50,000
8,50,000
Less: Rebate Receivable 40,000
8,10,000

18. XYZ Limited has given the following information for the preparation of cash flow statement
for the year ended 32nd Ashadh, 2075.

Particulars Rs. in 000’


Net profit after tax 50,000
Dividend (Including Dividend Tax) 17,070
Provision for Income Tax 10,000
Income Tax paid during the year 8,496
Loss on sale of assets (Net) 80
Book value of the assets sold 370
Depreciation charged to Profit & Loss 40,000
Amortization of government grant 12
Profit on sale of investment 200
Carrying amount of the investment sold 55,530
Interest received on investments 5,012
Interest Expenses of the year 20,000
Interest paid during the year 21,040
Increase in working capital (excluding cash & bank balance) 112,150
Purchase of fixed assets 106,300
Proceeds from calls in arrear 4
Receipt of grant for capital projects 24
Proceeds from long-term borrowing 51,960
Proceeds from short-term borrowing 41,150
Opening Cash & Bank balance 10,006
Closing Cash & Bank Balance 13,976
nd
You are required to prepare the Cash Flow Statement for the year ended 32 Ashadh, 2075.
(CAP Jun. 2019 Q3a – 10 Marks)
Answer:
a)
XYZ Limited
Cash Flow Statement
For the year ended 32nd Ashadh, 2075
Particulars Rs. in 000’ Rs. In 000’
Cash Flow From Operating Activities

© The Institute of Chartered Accountants of Nepal 274


CAP II Paper 1: Advanced Accounting

Net Profit before Tax (50,000+10,000) 60,000


Adjustments for:
Depreciation 40,000
Loss on sale of assets (net) 80
Amortization of government grant (12)
Profit on sale of Investments (200)
Interest income on Investments (5,012)
Interest Expenses 20,000
Cash Flow from Operating Activities before changes
in Working Capital 1,14,856
Less: Increase in working capital (excluding cash &
bank balance) (1,12,150)
Add: Proceeds from short-term borrowing 41,150
Cash Flow from Operating Activities before income
tax paid 43,856
Less: Income Tax paid (8,496)
Cash Flow from Operating Activities 35,360

Cash Flow from Investing Activities


Sale of assets (370-80) 290
Sale of investments (55,530+200) 55,730
Interest Income on Investments 5,012
Purchase of fixed assets (1,06,300)
Cash Flow from Investing Activities (45,268)

Cash Flow from Financing Activities


Proceeds from calls in arrears 4
Receipts of grant for capital projects 24
Proceeds from long-term borrowing 51,960
Interest paid (21,040)
Dividend (including dividend tax paid) (17,070)
Cash Flow from Financing Activities 13378
Net increase in Cash & Cash Equivalent 3,970
Cash & Cash Equivalent at the beginning of the period 10,006
Cash & Cash Equivalent at the end of the period 13,976

19. The Receipts and Payments account of Dadhikot Club prepared on 32nd Ashadh, 2075 is as
follows:
Receipts and Payments Account

Dr. Cr.
Receipts Amount (Rs.) Payments Amount (Rs.)
To Balance b/d 450 By Expenses (including
To Annual income payment for sports
4,590
from subscription material Rs. 2,700) 6,300
Add: Outstanding of By Loss on sale of
last year received this 180 furniture (cost price
Rs. 450)

© The Institute of Chartered Accountants of Nepal 275


CAP II Paper 1: Advanced Accounting

year
4,770 By Balance c/d 180
Less: Prepaid of last (90) 90,450
year 4,680
To Other fees 1,800
To Donation for
building 90,000
96,930
96,930
Additional information:
The club had balances as on 1/4/2074:
Furniture Rs. 1,800; investment at 5% Rs. 27,000; Sports material Rs. 6,660;
Balance as on 32.3.2075;
Subscription receivable Rs. 270; Subscription received in advance Rs. 90;
Stock of sports material Rs. 1,800
Do you agree with above receipts and payments account? If not, prepare correct receipts and
payments account and income and expenditure account for the year ended 32nd Ashadh, 2075
and balance sheet as on that date. (CAP Jun. 2019 Q4a – 10 Marks)

Answer:
Corrected Receipts and Payments Account of Dadhikot Club for the year
ended 32nd Ashadh, 2075

Receipts Amount Payments Amount


Rs. Rs. Rs.
To Balance b/d 450 By Expenses (Rs. 3,600
To Subscription Annual 4,590 6,300-Rs 2,700)
income By Sports material 2,700
Less: Receivable as on (270) By Balance c/d(cash 90,720
31.3.2075 in hand and at bank
Add: Advance received 90
for the year 2075-2076
Add: Receivable as on 180
31.3.2074
Less: Advance received a (90) 4,500
on 31.3.2074
To Other fees 1,800
To Donation for building 90,000
To Sale of furniture 270
97,020 97,020

Income and Expenditure Account of Dadhikot Club for the year


ended 32nd Ashadh, 2075

Expenditure Amount Income Amount

© The Institute of Chartered Accountants of Nepal 276


CAP II Paper 1: Advanced Accounting

To Sundry expenses 3,600 By Subscription 4,590


6,660 1,800
To Sports material Balance By Other fees
as on 1.4.2074 2,700 1,350
Add: Purchases (1,800) 7,560 By Interest on investment
Less: Balance as on (5%on Rs 27,000)
180
32.3.2075 By Deficit : Excess of 3,600
To Loss on sale of furniture --------- expenditure over income
11,340 11,340

Balance sheet of Dadhikot Club as on 32nd Ashadh, 2075


Liabilities Amount Assets Amount
(Rs) (Rs)
Capital fund 36,000 Furniture 1,800
Less: Excess of (3,600) 32,400 Less: Sold (450) 1,350
expenditure 5% Investment 27,000
over income Interest accrued on 1,350
investment
Building fund 90,000 Sports material 1,800
Subscription Subscription receivable 270
received in advance 90 Cash in hand and at 90,720
1,22,490 bank 1,22,490
Balance Sheet of Dadhikot Club as on 1st Shrawan , 2074
Liabilities Amount Assets Amount
Rs Rs
Subscription received in advance 90 Furniture 1,800
Capital Fund(balancing figure) 36,000 Investment 27,000
Sports material 6,660
Subscription receivable 180
Cash in hand and at 450
36,090 bank 36,090

© The Institute of Chartered Accountants of Nepal 277


CAP II Paper 1: Advanced Accounting

3.3 Winding up of Companies

C. Theoretical Questions

1) Winding up of a company by court’s order (Inter Jun. 2011 Q6 d – 5 Marks)

Answer
It is also known as compulsory winding up of a company. According to the Section 139(4) (f)
of the Company Act, 2063, the Court can issue an order to dissolve the company if its
shareholders file a suit to the Court claiming the company is acting against their interest.

2) Briefly explain about the preferential creditors and their order of payment in the case of a
company under liquidation.
(CAP Jun. 2010 4c – 5 Marks; Inter Jun. 2008 Q2b- 5 Marks)
Answer
Preferential creditors in case of liquidation of a company are those creditors who get priority
in payment over others though they are unsecured creditors by virtue of provisions of
insolvency (Damasahi) Act 2063.

According to insolvency Act, expenses relating to liquidation, wages and remuneration payable
and gratuity, provident fund, leave encashment gets priority over other creditors. The amount
available with liquidator from the disposal of assets of the company needs to pay in the
following priority:
a) Payment of liquidation expenses and liquidator’s remuneration.
b) Payment of wages and remuneration to workers and employees outstanding at the time of
start of liquidation process or at the time of order of restructure of the company, however,
directors engaged in the capacity of employee are not entitled to get payment under this
priority.
c) Payment of accumulated sick leave, home leave, gratuity and provident fund to workers
and employee outstanding at the time order of liquidation or at the time of restructure of
the company.
d) Payment to other creditors whose claims are accepted by liquidator.

Furthermore, The Insurance Act, 2049 and Bank and financial institution Act, 2063 have
separate priority order in respect of payment in case of liquidation of company engaged in the
business of Insurance and Banking respectively and provision of Insolvency Act regarding
priority in payment would not apply to above mentioned companies.

© The Institute of Chartered Accountants of Nepal 278


CAP II Paper 1: Advanced Accounting

D. Practical Questions

1. M. Ltd. resolved on 32nd Ashadh, 2067 that the company be wound up voluntarily. The
following was the trial balance extracted from its books as on that date:

Dr. Cr.
Rs. Rs.
Equity shares of Rs. 10 2,00,000
9% Preference shares of Rs. 10 each 1,00,000
Plant (Less depreciation w/off Rs. 85,000) 2,15,000
Stock in trade 2,50,000
Sundry Debtors 55,000
Sundry Creditors 75,000
Bank Balance 74,000
Preliminary Expenses 6,000
Accumulated Loss (Balance as on 1-4-2066) 30,000
Trading loss for the year 2066/67 24,000
Preference dividend for the year 2066/67 6,000
Outstanding expenses (Including mortgage interest) 25,000
4% Mortgage loan 2,00,000
Total 6,30,000 6,30,000

On 1st Shrawan, 2067 the liquidator sold to N. Ltd. Plant for Rs. 2,05,000 and stock in trade
for Rs. 2,00,000. The sale was completed in Shrawan, 2067 and the consideration satisfied as
to Rs. 2,62,200 in cash and as to the balance in 6% Debentures of the purchasing company
issued to the liquidator at a premium of 2%.

The remaining steps in the liquidation were as follows:


(a) The liquidator realized Rs. 52,000 out of the book debts and the cost of collection amounted
to Rs. 2,000.
(b) The loan mortgage was discharged on 31st Shrawan, 2067 along with interest from 31st
Magh 2066. Creditors were discharged subject to discount of 2% and outstanding expenses
excluding mortgage interest were settled for Rs. 2,000;
(c) On 30th Poush 2067 six month’s interest on debentures was received from N. Ltd.
(d) Liquidation expenses amounting to Rs. 3,000 and liquidator’s remuneration of 3% on
disbursements to members were paid on 30th Poush, 2067 when:
▪ The preference shareholders were paid out in cash; and
▪ The debentures of N. Ltd. and the balance of cash were distributed ratably among the
equity shareholders.
Prepare the Liquidator’s Statement of Account showing the distribution.
(CAP Dec. 2011, Q3a ii-8 Marks)

© The Institute of Chartered Accountants of Nepal 279


CAP II Paper 1: Advanced Accounting

Answer
M. Ltd. (in liquidation)
Liquidator’s Statement of Account from 1st Shrawan, 2067 to 30th Poush,
2067
Particulars Rs. Rs. Particulars Rs. Rs.
Balance at Bank 74,000 Liquidators Remuneration 7,302
Realization from: Liquidation Expenses 3,000
Sundry Debtors 52,000 Mortgage loan with Interest 204,000
Less: Cost of Collection 2,000 50,000 Creditors including Outstanding Exp. 75,500
N Ltd.: 6% Pref. Shareholders @ Rs. 10 each 100,000
6% Debenture @ Rs. 102 142,800 Equity Shareholders:
Cash 262,200 405,000 6% Debentures 142,800
Interest on Debentures – 6 months 4,200 Cash – Approximately
03 paisa per share 598 143,398
533,200 533,200

Working note:
(i) Calculation of Liquidators Remuneration:
Particulars Rs. Rs.
Total realization including debentures in N Ltd. 533,200
Distribution to outsiders:
Liquidation expenses 3,000
Mortgage loan with interest 204,000
Creditors including outstanding expenses 75,500 282,500
Amount available for liquidators remuneration & members settlement 250,700
Liquidators remuneration [Rs. 250,700 × 3 / 103] 7,302
(ii) It is assumed that loan is secured by a floating charge.
(iii) Calculation of debentures to be issued: value of Assets sold 205,000
Add: value of stock sold 200,000
Total 405,000
Less: cash received 262,200
6% debenture @ Rs. 102 = 142,800
(Assumption: face value of debenture to be issued at Rs. 100 each)

2. The Ultra Optimist Ltd. went into liquidation. Its assets realized Rs. 350,000 excluding
amounts realized by sale of securities held by the secured creditors. The following was the
position:
Particulars Rs.
Share Capital: 10,000 shares of Rs. 10 each......................... 100,000
Secured Creditors (Securities realized Rs. 40,000)................ 35,000
Preferential Creditors........................................................... 6,000
Unsecured Creditors.............................................................. 140,000

© The Institute of Chartered Accountants of Nepal 280


CAP II Paper 1: Advanced Accounting

Debentures having a floating charge on the assets of the


250,000
company
Liquidation Expenses............................................................. 5,000
Liquidator's Remuneration.................................................. 7,500
Prepare the Liquidator's Final Statement of Account
(Inter Jun. 2011 5b – 10 Marks)

Answer
The Ultra Optimist Limited
Liquidator's Final Statement of Account
Particulars Rs. Particulars Rs.
Assets Realized: Liquidator's Remuneration 7,500
Surplus from Securities 5,000 Liquidation Expenses 5,000
Other Assets 350,000 Debentures with floating charge 250,000
Preferential Creditors 6,000
Unsecured Creditors – 61.79% 86,500
355,000 355,000
Note:
Percentage of Unsecured Creditors discharged:
Balance available after satisfying Preferential Creditors............................ Rs. 86,500
Total Unsecured Creditors as per books..................................................... Rs. 140,000
Percentage covered by surplus balance (Rs. 86,500/Rs. 140,000 × 100)... 61.79%

3. Company A went into liquidation on 31st March, 2009 when the following Balance Sheet was
prepared:
Capital & Liabilities Rs. Assets Rs.
Paid up Capital: Goodwill 50,000
19,500 Shares of Rs.10 each 1,95,000 Leasehold Property 48,000
Sundry Creditors: Plant & Machinery 65,500
Preferential 24,200 Stock 56,800
Partly Secured 55,310 Sundry Debtors 64820
Unsecured 99,790 1,79,300 Cash 2,500
Bank O/D - Unsecured 12,000 Accumulated Loss 98,680
3,86,300 3,86,300

The liquidator realized the assets as follows:


Rs.
Leasehold property which was used in the first instance to pay the partly
secured creditors pro rata 35,000
Plant and Machinery 51,000
Stock 39,000
Sundry Debtors 58,500
Cash 2,500

© The Institute of Chartered Accountants of Nepal 281


CAP II Paper 1: Advanced Accounting

The expenses of liquidation amounted to Rs. 1,000 and the liquidator's remuneration was
agreed at 2.5% on the amount realized, including cash, and 2% on the amount paid to the
unsecured creditors.
From the information provided, you are required to prepare the Liquidator's Final Accounts
showing the distribution. (Inter Dec. 2009 Q5b-10 Marks)

Answer
Liquidator's Final Statement of Account
Particulars Amount Particulars Amount
Rs. Rs.
Assets Realized: Liquidator's Remuneration:
Cash 2,500 2.5% on Rs. 1,86,000 4,650
Stock 39,000 2% on 24,200 484
Sundry Debtors 58,500 2% on Rs.1,18,300 2,366 7,500
Plant and 51,000 Liquidation Expenses 1,000
Machinery
Leasehold Property 35,000 Partly Secured Creditors 35,000
Preferential Creditors 24,200
Unsecured Creditors 1,18,300
1,86,000 1,86,000

Working Notes:
(i) Goodwill valued at Rs. 50,000 in books is valueless.
(ii) Calculation of total amount due to the unsecured creditors:
Rs.
Unsatisfied balance of Partly Secured Creditors 20,310
Unsecured Creditors 99,790
Bank Overdraft 12,000
1,32,100
(iii) It is assumed that the securities in the hands of partly secured creditors are realized
by the liquidator on behalf of them and liquidator's remuneration is calculated
accordingly.
4. The following information is given to you:
Financial Position of Yan Ltd. as on 31st March, 2012
Liabilities Amount Assets Amount
Rs. Rs.
Share Capital: Land & Building 100,000
2,000 14% preference shares of Plant & Machinery 250,000
Rs. 100 each, fully paid up 200,000 Patents 40,000
1,000 Equity shares of Rs. 100 Stock at Cost 55,000
each, Rs. 75 paid up 75,000 Sundry Debtors 110,000
Cash at Bank 75,500

© The Institute of Chartered Accountants of Nepal 282


CAP II Paper 1: Advanced Accounting

3,000 Equity shares of Rs. 100 180,000 Accumulated Loss 83,500


each, Rs. 60 paid up
14% Debentures having floating 100,000
charge on all assets 14,000
Interest Outstanding 145,000
Creditors
Total 714,000 Total 714,000
The company went into liquidation on the above date.
Preference dividends were in arrear for two years. The arrears are payable automatically on
liquidation. Creditors include a loan of Rs. 50,000 on the mortgage of Land & Building. The
assets were realized as follows:
Rs.
Land & Building 120,000
Plant & Machinery 200,000
Patents 30,000
Stock 60,000
Sundry Debtors 80,000

The expense of liquidation amounted to Rs. 10,900. The liquidator is entitled to a commission
of 3% on all assets realized except cash and a commission of 2% on the amounts distributed
among unsecured creditors. Preferential creditors amount to Rs. 15,000. Assume the payment
was made on September 30, 2012. Prepare the Liquidator's Statement of Accounts.
(CAP Jun. 2013 Q2a– 10 Marks)
Answer
Liquidator's Statement of Account as at 30th September 2012
Receipts Estimated Value Payments Amount
Value Realized Paid
Assets realized: Liquidator's remuneration:
Cash 75,500 75,500 3% on NPR 490,000 = 14,700
Debtors 110,000 80,000 2% on NPR 95,000= 1,900 16,600
Stock 55,000 60,000 Expenses of liquidation 10,900
Plant & Machinery 250,000 200,000 Debentures having a floating charge
Patents 40,000 30,000 100,000
Land & Building: Interest o/s 14,000
Surplus from securities : Interest up to 30th
(100000-50,000) = 50,000 September 2012 (6month) 7,000 121,000
Creditors:
(120000-50000)= 70,000 Preferential 15,000
Other 80,000 95,000
Preferential shareholders 200,000
Arrears of preference dividend 56,000
Equity shareholders:
NPR 15 on 1,000 shares 15,000
NPR 0.25 on 4,000 shares 1,000 16,000
515,500 515,500

© The Institute of Chartered Accountants of Nepal 283


CAP II Paper 1: Advanced Accounting

5. The available cash with the liquidator of a Company after realizing all Assets and paying all
Liabilities is Rs. 3,30,000.
The issued Capital of the company is made up of:
30,000 10% Preference Shares of Rs. 10 each fully paid.
30,000 Equity Shares of Rs. 10 each fully paid.
30,000 Equity Shares Rs. 10 each Rs. 8 paid up per share.
30,000 Equity Shares of Rs. 5 each Rs. 3 paid up per share.
Preference dividend has been paid to date.
Prepare Liquidators' Statement of Account assuming that the required calls were made and the
amounts due were received in full.
(Inter Jun. 2001 Q 5a-8Marks)

Answer:
(i) Total Amount to be paid
Preference capital 3,00,000
Equity capital 3,00,000
Equity capital (30,000  8) 240,000
Equity capital (30,000  3) 90,000
9,30,000
Less: Available amount 3,30,000
Total deficiency 6,00,000

(ii) Loss to Equity Capital


60,00,000  3,00,000 + 3,00,000 + 1,50,000 80%

(iii) Loss per Equity Share


Equity shares of Rs. 10 each loss Rs. 8 per share Rs. 8
Equity shares of Rs. 5 each will suffer 80% of Rs. 5 Rs. 4

(iv) Amount to be called


a. First type shareholder has paid full Rs. 10, they suffer a loss of Rs. 8 per share
therefore refund Rs. 2 per share
b. Second type of shareholder has paid Rs. 8, per share and they also lose Rs. 8
per share they will not pay any amount nor will they be refunded any amount.
c. Third type of shareholder has paid Rs. 3 per share. They have suffered a loss
of Rs. 4 per share. Therefore they will be called upon to pay Rs. 1 per share.

Liquidators Statement of Account


Dr. Receipts Rs. Cr. Payments Rs.
To Balance b/d 3,30,000 By Preference capital 3,00,000
To Equity Capital (30,000  1) 30,000 By Equity Capital (30,000  2) 60,000
3,60,000 3,60,000

© The Institute of Chartered Accountants of Nepal 284


CAP II Paper 1: Advanced Accounting

6. After continuous struggle of years together, Kathmandu Metal Craft Ltd. Finally went into
liquidation. The following was the position:

Realization from asset Rs. 3,50,000


(excluding sale of securities held by secured creditors)
Share Capital: 1,000 shares of Rs. 100 each Rs. 1,00,000
Secured Creditors (Securities realized Rs. 40,000) Rs. 35,000
Preferential Creditors Rs. 6,000
Unsecured Creditors Rs. 1,40,000
Debentures having a floating charge on the asset of the company Rs. 2,50,000
Liquidation Expenses Rs. 5,000
Liquidator's Remuneration Rs. 7,500
prepare the Liquidator's final statement of account.
(Inter Jun. 2003 Q 5 -8 Marks)
Answer
Kathmandu Metal Craft Ltd.
Liquidator's Final Statement of Account
Rs. Rs.
To Assets Realized 3,50,000 By Liquidator's Remuneration 7,500
To Surplus from secured 5,000 By Liquidation Expenses 5,000
creditors By Debentures having a floating charge 2,50,000
By Preferential Creditors 6,000
By Unsecured creditors
(@ Rs. 86,500/1,40,000)
(Balancing figure) 86,500
3,55,000 3,55,000

Note: It is clear that the shareholders will not get anything as the amount is not sufficient even
to make full payment to unsecured creditors.

7. Reliable Ltd. went into liquidation. It has a total fund of Rs. 61,000 available for its
shareholders after discharging all its liabilities including preference shareholders. The details
regarding share capital are as follows:
Class A - 2000 Equity shares of Rs. 100 each (fully paid)
Class B - 1600 Equity shares of Rs. 80 each (Rs. 70 paid up)
Class C - 1400 Equity shares of Rs. 50 each (Rs. 40 paid up)
Show how much amount is receivable/payable to each class of shareholders.
(Inter Jun. 2006, Q 6a -5 Marks)

Answer:
Statement showing the nominal value and paid up value of shares.

© The Institute of Chartered Accountants of Nepal 285


CAP II Paper 1: Advanced Accounting

Paid up Total Total paid


FV per
Class Number value per nominal up value
share Rs.
share Rs. value Rs. Rs.
A 2000 100 100 2,00,000 2,00,000
B 1600 80 70 1,28,000 1,12,000
C 1400 50 40 70,000 56,000
3,98,000 3,98,000

Rs.
Amount to be returned to shareholders 3,68,000
Less: total amount available for them 61,000
Total Deficiency 3,07,000
% of deficiency on nominal value of shares
𝑇𝑜𝑡𝑎𝑙 Amount of Deficiency
= × 100
Nominal Value of Shares
3,07,000
= × 100
3,98,000
= 77.1357%
Statement showing the amount returnable to each class of equity shareholders.

Particulars Class A Class B Class C


No. of shares 2,000 1,600 1,400
Face value per share Rs 100 80 50
Paid up amount per share. Rs 100 70 40
Less: Deficiency per share
(77.1357% of Face Value) 77.1357 61.7086 38.5679
Amount to be returned to shareholders
per share
(Paid up value – Deficiency) 22.8643 8.2914 1.4321
Total amount to be refunded to
shareholders (No. of shares × per share 45,728.60 13,266.24 20,004.94
amount to be returned to shareholders)
Say Rs 45,729.00 13,266.00 20,005.00

8. The following is the Financial Position of X Ltd. as on 31st December 1999.

Liabilities Amount Rs. Assets Amount Rs.


Authorized and Subscribed Land & Building 190,000
Capital:
Plant & Machinery 120,000
20,000 6% non-cumulative
preference shares of Rs. 10 each 200,000 Patents
10,000

© The Institute of Chartered Accountants of Nepal 286


CAP II Paper 1: Advanced Accounting

10,000 equity shares of Rs. 10 Stock


each, Rs. 9 paid 90,000 45,000
Sundry Debtors
10,000 equity shares of Rs. 10 90,000
each, Rs. 5 paid 50,000 Cash at Bank
30,000
6% Mortgage Debentures Investment
(Holding a floating charge on all 40,000
the assets of the Company) 100,000 Accumulated Loss
70,500
Interest outstanding on
debenture 6,000

Loan secured by hypothecation


of stock 40,000

Trade Creditors 80,000

Creditors for Salaries and


Wages
15,000
Liability for workmen's
compensation 2,000

Owing to Govt. for Telephone


and Purchases 2,500

Owing to Govt. for Taxes


10,000
Total 595,500 Total 595,500

The company went into voluntary liquidation on 1st January 2000 and the liquidator was
appointed with a remuneration of 2% of assets realized with the exception of cash and 2% of
the amount distributed among unsecured creditors other than preferential creditors. The
dividend on preference shares was not paid for the year 1998 and 1999. Stock realized Rs.
30,000 and the other assets excluding cash realized Rs. 400,000. All assets were realized and
payment made on June 30, 2000. Prepare the liquidator's final statement of accounts, assuming
the expenses of liquidation were Rs. 4,450.
(Inter Dec. 2007 Q 5-10 Marks)
Answer:
X Ltd (in liquidation)
Liquidators Final Statement of Account
as on June 30 2000

© The Institute of Chartered Accountants of Nepal 287


CAP II Paper 1: Advanced Accounting

Receipts Amount Payments Amount


Rs. Rs.
Balance at Bank 30,000 Liquidators Remunerations 11,050
Liquidation Expenses 4,450
Realization from Assets 4,30,000
Debenture Holders:
Call on 10,000 Equity 6% Mortgage
Shareholders of Rs. 10 each, Debentures1,00,000
Rs. 5 paid up Interest Outstanding 6,000 1,09,000
(Rs. 2.70 per shares) 27,000 Interest for six month 3,000

Preferential Creditors:
Creditors for Salaries
and Wages 15,000
Liability for Workmen 27,000
Compensation 2,000
Owing to Govt. for Taxes
10,000

Unsecured Creditors:
Loan secured by 1,22,500
hypothecation of Stock 40,000
Trade Creditors 80,000 2,00,000
Owing to Govt. for
Telephone & Purchases 2,500

Preference Shareholders 13,000


10,000 Equity Shareholder of
Rs. 10 each, Rs. 9 paid up
(Rs. 1.30 per Share)
Total 4,87,000 Total 4,87,000

Working Note:

Calculation of Total Asset Realization

Realization from assets except stock & Cash 400,000


Stock Realized 30,000
Total Assets Realization except Cash 430,000

Calculation of Amount to be called on from equity shareholders

Cash at Bank 30,000

© The Institute of Chartered Accountants of Nepal 288


CAP II Paper 1: Advanced Accounting

Realization from Assets 430,000


Total amount available 460,000

Liquidators Remuneration 8,600


(2% of total assets realized except cash)
Balance 4,51,400

Liquidation Expenses 4,450


Balance 4,46,950

Debenture Holders:
6% Mortgage Debentures 100,000
Interest Outstanding 6000
Balance 3,40,950

Preferential Creditors:
Creditors for Salaries and Wages 15,000
Creditors for Salaries and Wages 2,000
Owing to Govt. for Taxes 10,000
Balance 3,13,950

Unsecured Creditors:
Loan secured by hypothecation of Stock 40,000
Trade Creditors 80,000
Owing to Govt. for Telephone & Purchases 2500
Balance 191,450

Liquidators Remuneration 2450


(2% of Amount distributed to Unsecured Creditors
i.e. 2% of Rs. 122,500)
Balance 1,89,000

Debenture Interest from 1.1.2000 to 30.6.2000 3000


Balance 186,000

If we collect Rs. 4 each on 5 paid 40,000


up Equity Share, the collection will be

Amount available before payment to 186,000


Preference Shareholders

Total amount available will be 2,26,000

© The Institute of Chartered Accountants of Nepal 289


CAP II Paper 1: Advanced Accounting

Amount to be paid to Preference Shareholders 200,000

Balance Available to Equity Shareholders 26,000

Amount refundable to each equity shareholders Rs. 26000/20000


=Rs. 1.30

Hence Amount to be collected from Rs. 5 paid


up Equity Shareholders Rs. 4 – 1.30 = 2.70

Amount to be refunded to each Rs. 9 paid up


Equity Shareholders = Rs. 1.30

© The Institute of Chartered Accountants of Nepal 290


CAP II Paper 1: Advanced Accounting

3.4.1 Accounting for Acquisition, Absorptions, Amalgamation and Merger

A. Practical Questions

1.
Financial Position of Himal Ltd.
Rs. Rs.
Share Capital 20,00,000 Noncurrent Assets 15,00,000
General Reserve 15,00,000 Investment 2,50,000
Current Liabilities 15,00,000 Current Assets 32,50,000
50,00,000 50,00,000

Financial Position of Hill Ltd.


Rs. Rs.
Share Capital 10,00,000 Noncurrent Assets 3,00,000
General Reserve 5,00,000 Goodwill 1,00,000
Current Liabilities 2,00,000 Current Assets 14,00,000
Proposed dividend 1,00,000
18,00,000 18,00,000
Himal Ltd. absorbed Hill Ltd. on following terms & conditions:
a) Hill Ltd declares a dividend of 10% before absorption for the payment of which it
is to retain sufficient amount of cash.
b) The net worth of Hill Ltd. is valued at Rs. 14,50,000.
c) The purchase consideration is satisfied by the allotment of fully paid shares of Rs.
100 each in Himal Ltd.
Following additional information are also to be taken in to consideration:
• Himal Ltd. holds 2,500 shares of Hill Ltd. at a cost of Rs 2,00,000.
• The stock of Hill Ltd. includes items valued at Rs. 50,000 from Himal Ltd. (cost Rs.
37,500)
• The creditors of Hill Ltd include Rs. 15,000 due to Himal Ltd.
Show ledger account in the books of Hill Ltd. to give effect to the above and Financial
Position of Himal Ltd. after completion of absorption.
(CAP Dec. 2009 Q1 -20 Marks)
Answer
Books of Hill Ltd.
Realization Account
Particulars Amount Rs. Particulars Amount Rs.
To Good will 1,00,000 By Current Liabilities 2,00,000
To Noncurrent Assets 3,00,000 By Himal Ltd. 10,87,500
To Current Assets 13,00,000 By Share Capital 2,50,000
17,00,000 By Equity Shareholders 1,62,500
a/c ( loss) 17,00,000

© The Institute of Chartered Accountants of Nepal 291


CAP II Paper 1: Advanced Accounting

Equity Shareholders (outside) Account


Particulars Amount Rs. Particulars Amount Rs.
To Realization 2,50,000 By Share Capital 10,00,000
To Realization Loss 1,62,500 By General reserve 5,00,000
To Shares in Himal Ltd 10,87,500
15,00,000 15,00,000

Statement of Financial Position Of Himal Ltd.


( After completion of absorption)

Capital & Liabilities Amount Assets Amount


Rs. Rs.
Share Capital 30,87,500 Noncurrent Assets 18,00,000
30875 shares @ Rs. 100 Himal Ltd. 15,00,000
fully paid Hill Ltd. 3,00,000

Reserve & Surplus 16,25,000 Investment 50,000


General reserve 15,25,000
Gain on Business Current Assets 45,47,500
combination 1,00,000
16,85,000
Current Liabilities 63,97,500 63,97,500

Workings:
Adjusted Financial Position of Hill Ltd.
Capital & Liabilities Amount Rs Assets Amount Rs.
Share Capital 10,00,000 Fixed assets 3,00,000
General Reserve 5,00,000 Goodwill 1,00,000
Current Liabilities 2,00,000 Current assets 13,00,000*
17,00,000 17,00,000

Hill Ltd retains Rs. 1,00,000 in cash for dividend ( 10%) ( 14,00,000-1,00,000)

1. Total Purchase consideration ( based on net worth of Hill Ltd.) is Rs. 14,50,000.
2. Himal Ltd. holds 2,500 shares in Hill Ltd. The percentage of holding is 25%
3. The net purchase consideration to pay Rs. 14,50,000 * ¾ = 10,87,500
4. Calculation Of Current assets
Current assets Of Himal Ltd. 32,50,000
Add Dividend 25,000
32,75,000
Less intercompany amount 15,000
32,60,000
Current Assets Of Hill Ltd. 13,00,000

© The Institute of Chartered Accountants of Nepal 292


CAP II Paper 1: Advanced Accounting

Less unrealized profit 12,500


( Rs. 50,000-37,500)
12,87,500
Total Current Assets 45,47,500

5. Calculation of Current Liabilities


Himal Ltd. 15,00,000
Hill Ltd. ( 2,00,000-15,000) 1,85,000
16,85,000

6. Calculation of Goodwill
Net Assets Acquired (A) = 300,000 + 1,287,500 -200,000 = 1,387,500
Purchase Consideration Paid (B)=1,087,500+ 200,000 = 1,287,500
Since the purchase consideration is less than the value of net assets acquired, the acquirer
needs to access the genuineness of the fair value of assets & liabilities acquired. If the
purchase is a genuine bargain purchase, the excess of fair value over purchase consideration
paid shall be recognized as bargain.

2. White Ltd. agreed to acquire the business of Green Ltd. as on 32nd Ashadh, 2068. The
summarized Financial Position of Green Ltd. at that date was as follows:

Statement of Financial Position as on 32nd Ashadh, 2068


Liabilities Rs. Assets Rs.
Share Capital in fully paid equity
shares of Rs. 10 each 6,00,000 Goodwill 1,00,000
General Reserve 1,70,000 Land and Buildings 2,30,000
Reserves (Profit) 1,10,000 Plant and Machinery 4,10,000
12% Debentures 1,00,000 Stock in Trade 1,68,000
Creditors 20,000 Debtors 36,000
Cash at Bank 56,000
10,00,000 10,00,000

The consideration payable by White Ltd. was agreed as follows:


i) A cash payment equivalent to Rs. 2.50 for every Rs. 10 share in Green Ltd.
ii) The issue of 90,000 shares of Rs.10 each fully paid up in White Ltd. having an agreed
value of Rs. 15 per share.
White Ltd. also agreed to discharge the 12% Debentures of Green Ltd. at a premium of 20%
by allotment of its 14% Debentures at 96 percent. When computing the agreed consideration,
the directors of White Ltd. valued the following assets at values noted against them:
Rs.
Land and Buildings 7,50,000
Plant and Machinery 4,50,000

© The Institute of Chartered Accountants of Nepal 293


CAP II Paper 1: Advanced Accounting

Stock in Trade 1,42,000


Debtors Subject to an allowance of 5% to cover doubtful debts.

The cost of liquidation of Green Ltd. came to Rs. 5,000 which was borne by White Ltd.
Give ledger accounts to close the books of the Green Ltd.
(CAP Dec. 2011, Q2b-10 Marks)
Answer
In the books of Green Ltd.
Realization Account
Particulars Rs. Particulars Rs.
To Goodwill 100,000 By 12% Debenture 100,000
To Land & Buildings 230,000 By Sundry Creditors 20,000
To Plant & Machinery 410,000 By White Ltd. – Consideration 1,500,000
To Stock in Trade 168,000
To Sundry Debtors 36,000
To Bank 56,000
To Shareholders A/c – Profit 620,000
1,620,000 1,620,000
White Ltd. Account
Particulars Rs. Particulars Rs.
To Realization Account 1,500,000 By Bank 150,000
By Shares in White Ltd. 1,350,000
1,500,000 1,500,000
Equity Shareholders Account
Particulars Rs. Particulars Rs.
To Bank 150,000 By Equity Share Capital 600,000
To Shares in White Ltd. 1,350,000 By General Reserve 170,000
By Reserves (Profit) 110,000
By Realization Account – Profit 620,000
1,500,000 1,500,000

Cash Book (Bank Column)


Particulars Rs. Particulars Rs.
To Balance b/d 56,000 By Realization Account 56,000
To White Ltd. 150,000 By Equity Shareholders Account 150,000
206,000 206,000
Working Note:
Calculation of purchase consideration:
Rs.
Cash [Rs. 2.5 × 60,000] 150,000
Shares [Rs. 15 × 90,000] 1,350,000
Total 1,500,000

© The Institute of Chartered Accountants of Nepal 294


CAP II Paper 1: Advanced Accounting

3. X Ltd. and Y Ltd. amalgamate to form a new company XY Ltd. The Financial Position of
these two companies on the date of amalgamation was as under:
X Ltd. Y Ltd. X Ltd. Y Ltd.
Equity and Liabilities Assets
(Rs.) (Rs.) (Rs.) (Rs.)
Share Capital Goodwill 80,000
Equity Share of Rs. 100
each 800,000 300,000 Land and Building 450,000 300,000
7% Preference Share of
Rs. 100 each 400,000 300,000 Plant & Machinery 620,000 500,000
5 % Debenture 200,000 - Furniture and Fittings 60,000 20,000
General Reserve - 100,000 Sundry Debtors 275,000 175,000
Retained Earnings 431,375 97,175 Stores and Stock 225,000 140,000
Sundry Creditors 100,000 210,000 Cash at Bank 120,000 55,000
Secured Loan - 200,000 Cash in Hand 41,375 17,175
Preliminary Expense 60,000
Total 1,931,375 1,207,175 Total 1,931,375 1,207,175
The terms of amalgamation are as under:
a) The assumption of liabilities of both companies.
Issue of 5 preference share of Rs. 20 each in XY Ltd. @ Rs. 18 paid up at premium of Rs.
4 per share for each preference share held. Issue of 6 Equity shares of Rs. 20 each in XY
Ltd. @ Rs. 18 paid up at a premium of Rs. 4 per share for each equity share held in both
companies .In addition, necessary cash should be paid to equity shareholders of both
companies as is required to adjust the rights of shareholder of both the Companies in
accordance with intrinsic value of the share of both the companies.
Issue of such amount of fully paid 6 % debenture in XY Ltd. as is sufficient to discharge
the 5 % debenture in X Ltd. at a discount of 5% after takeover.
b) The assets and liabilities are to be taken at book values stock and debtors for which provision
at 2 % and 2.5% respectively to be made.
The XY Ltd. to issue 15,000 new equity shares of Rs. 20 each Rs. 18 paid up at a premium
of Rs. 4 per share so as to have sufficient working capital.

Required:
Prepare Ledger accounts in the books of X Ltd. and Y Ltd. to close their books.
(CAP Dec. 2014 Q1-20 Marks; CAP Jun. 2015 Q1- 20 Marks)
Answer
Books of X Ltd.
Realization Account Dr Cr.
Particulars Rs. Particulars Rs.
To Goodwill 80,000 By 5% debenture 200,000
To land and Building 450,000 By Sundry Creditors 100,000
By XY Ltd (purchase
To Plant and machinery 620,000 Consideration) 1,560,000

© The Institute of Chartered Accountants of Nepal 295


CAP II Paper 1: Advanced Accounting

To Furniture and Fitting 60,000 By Equity Shareholders A/c( Loss) 51,375


To sundry debtors 275,000
To stores and stock 225,000
To cash at bank 120,000
To Cash in Hand 41,375
To preference shareholder
(excess payment) 40,000
Total 1,911,375 Total 1,911,375

Equity Shareholder account Dr Cr


Particulars Rs. Particulars Rs.
To preliminary Expenses 60,000 By share Capital 800,000
To Realization Account 51,375 By Retained Earnings A/c 431,375
To Equity Share in XY Ltd 1,056,000
To Cash 64,000
1,231,375 1,231,375

XY Ltd Dr. Cr.


Particulars Rs. Particulars Rs.
To Realization A/C 1,560,000 By Equity Shares in Xy Ltd
For Equity 10,56,000
For Preference 440000 1,496,000
By cash 64,000
1,560,000 1,560,000

Books of Y Ltd
Particulars Rs. Particulars Rs.
To land and Building 300,000 By Sundry Creditors 210,000
To Plant and machinery 500,000 By Secured Loan 200,000
To Furniture and Fitting 20,000 By XY Ltd ( purchase
790,000
To sundry debtors 175,000 Consideration)
To stores and stock 140,000 By Equity shareholder A/c
37,175
To cash at bank 55,000 (loss)
To Cash in Hand 17,175
To preference shareholder 30,000
1,237,175 1,237,175

XY Ltd Account
Particulars Rs. Particulars Rs.
To Realization A/C 790,000 By Equity Shares in XY Ltd
For Equity 396000
For Preference 330000 726,000

© The Institute of Chartered Accountants of Nepal 296


CAP II Paper 1: Advanced Accounting

By cash 64,000
790,000 790,000

Equity Shareholder Account


Particulars Rs. Particulars Rs.
To Equity Share in XY Ltd 396,000 By Share Capital 300,000
To Realization 37,175 By Retained Earnings A/C 97,175
To cash 64,000 By Reserve 100,000
497,175 497,175

Working note 1
Purchase consideration X Ltd (Rs.) Y Ltd (Rs.)
Payable to preference shareholder
Preference share @ 22 per share 440,000 330,000
Equity Shares at Rs 22 per share 1,056,000 396,000
Cash ( WN 2 ) 64,000 64,000
1,560,000 790,000
Working Note 2
Value of Net Assets X Ltd Y Ltd
Goodwill 80,000
land and Building 450,000 300,000
Plant and machinery 620,000 500,000
Furniture and Fittings 60,000 20,000
Debtor Less 2.5% 268,125 170,625
Stock less 2 % 220,500 137,200
Cash at Bank 120,000 55,000
Cash In Hand 41,375 17,175
1,860,000 1,200,000
Less
Debenture 200,000 -
Creditors 100,000 210,000
Secured Loan - 200,000
1,560,000 790,000
Payable in Shares 1,496,000 726,000
Payable in cash 64,000 64,000

4. Following are the summarized Financial Position of Company Rishi Ltd. and Muni Ltd.,
as at Ashadh 31, 2073.
Liabilities Rishi Muni Assets Rishi Muni
Ltd.(Rs.) Ltd.(Rs.) Ltd.(Rs.) Ltd.(Rs.)
Share Capital Goodwill 20,000 -

© The Institute of Chartered Accountants of Nepal 297


CAP II Paper 1: Advanced Accounting

Equity Shares of NRs. Other Non-current


2,000,000 1,500,000 2,400,000 1,150,000
100 Each Assets
10% Preference Shares
700,000 400,000 Trade Receivables 625,000 615,000
of NRs 100 Each
General Reserve 240,000 170,000 Inventory 412,000 680,000

Retained Earnings - 15,000 Cash at Bank 38,000 155,000


Own Debenture
12% Debenture of NRs.
600,000 200,000 (Nominal Value of 192,000 -
100 Each
NRs. 200,000/-)
Discount on Issue of
Trade Payables 560,000 315,000 2,000
Debentures
Retained Earnings 411,000 -
Total 4,100,000 2,600,000 Total 4,100,000 2,600,000

On Shrawan 01, 2073, Rishi Ltd. adopted the following scheme of reconstruction:
a) Each equity share shall be sub-divided into 10 equity share of Rs. 10 each fully paid up.
50% of the equity share capital would be surrendered to the company.
b) Preference dividend are in arrear for 3 years. Preference shareholders agreed to waive
80% of the dividend claim and accept payment for the balance.
c) Own debenture of Rs. 80,000/- (Nominal Value) were sold at NRs. 98 cum interest and
remaining own debentures were cancelled.
d) Debentures holders of Rs. 300,000/- agreed to accept one machinery of book value of
Rs. 320,000/- in full settlement.
e) Trade payables, Trade receivables and Inventory were valued at Rs. 500000/- Rs.
600000/- and Rs. 400000/- respectively.
f) The company paid Rs. 20,000/- as penalty to avoid capital commitments of Rs. 400,000/-

On Shrawan 02, 2073, a scheme of absorption was adopted. Rishi Ltd. would take over Muni
Ltd. The purchase consideration was fixed as below:
a) Equity shareholders of Muni Ltd. will be given 50 equity shares of Rs. 10 each fully paid
up, in exchange for every 5 shared held in Muni Ltd.
b) Issue of preference shares of NRs. 10 each in the ratio of 4 preference shares of Rishi Ltd.
for every 5 preference shares held in Muni Ltd.
c) Issue of 12% debentures of Rs. 100 each of Rishi Ltd. for every 12% debentures in Muni
Ltd.
Pass necessary Journal Entries in the books of Rishi Ltd., and draw a resultant Balance Sheet
as at Shrawan 02, 2073. (Make suitable assumptions required, if any.)
(CAP Dec. 2016 Q1-20 Marks; Inter Jun. 2006 Q3 (Similar) -15 Marks)
Answer:
Journal Entries in the Books of Rishi Ltd.
Debit Credit
S.N. Particulars
(Amount NRs.) (Amount NRs.)

© The Institute of Chartered Accountants of Nepal 298


CAP II Paper 1: Advanced Accounting

On Shrawan 01,2073
Equity share capital A/c (Rs.100) 2,000,000
Dr.
1 To, Equity share capital A/c (Rs.10)
2,000,000
(Being sub-division of one share of NRs. 100 each
into 10 shares of NRs. 10 each.)
Equity share capital A/c (Rs.10) Dr. 1,000,000
2 To, Capital reduction A/c
1,000,000
(Being reduction of capital made by 50%)
Reserves A/c Dr. 42,000
To, Bank A/c
(Being payment in cash of 20% of arrears of 3
3
years preference dividend. Since Pref shares are 42,000
financial liability, effect of their adjustment is not
given on capital reduction)
12% Debentures A/C Dr. 200,000
To Own Debentures A/C 192,000
To Reserves A/C 8,000
4
(Own debentures should have been cancelled at
the time of purchase itself as the financial liability
is extinguished. Error corrected.)
Bank A/c Dr. (80000*98%) 78,400
Reserves A/c Dr. (Interest) 1,600
To 12% Debentures A/c 80,000
(Being debentures re-issued. The amount of
5
debentures should have been remeasured at
Amortized cost and no finance cost would be
recognized principally. This could not be done due
to lack of information)
12% Debenture A/c Dr. 300,000
Impairment Loss (Reserves) Dr. 20,000
To Machinery A/c
6
(As the debenture-holders took the machinery for 320,000
NRs. 300,000, it is an indication that the
machinery was impaired)
Trade Payables A/c Dr. 60,000
To, Reserves A/c
7 (Being trade payable revalued. Since these are 60,000
financial liability, effect of their adjustment is not
given on capital reduction)

© The Institute of Chartered Accountants of Nepal 299


CAP II Paper 1: Advanced Accounting

Capital reduction A/c Dr. 1,000,000


To, Trade Receivables A/c 25,000
To, Inventory A/c 12,000
To, Goodwill A/c 20,000
To, Discount on debentures A/c 2,000
To, Profit & Loss 411,000
8
To, Bank A/c 20,000
To, Reserve A/c 510,000
(Being asset revalued, losses written off and
penalty paid off through capital reduction account
and the balance of capital reduction transferred to
reserve)
Business Purchase A/c Dr. (WN-1) 15,32,000
To, Liquidators of Muni Ltd. A/c
9
(Being Purchase consideration payable to Muni 15,32,000
Ltd.)
Non-current Asset A/c Dr. 1,150,000
Inventory A/c Dr. 680,000
Trade receivables A/c Dr. 615,000
Cash at Bank A/c Dr. 155,000
To, Trade payables A/c 315,000
10
To, 12% Debentures A/c of Muni Ltd. 200,000
To, Gain on Bargain Purchase (BF) 553,000
To, Business Purchase A/c 15,32,000
(Being takeover of all assets & liabilities of Muni
Ltd. by Rishi Ltd.)
Liquidators of Muni Ltd. A/c Dr. 15,32,000
To, Equity share capital A/c 1,500,000
11
To, 10 % preference share capital A/c 32,000
(Being purchase consideration paid)
12% Debenture of Muni Ltd. A/c Dr.
To, 12% Debenture A/c 200,000
12
(Being Rishi Ltd. Issued their 12% Debenture 200,000
against 12% debenture of Muni Ltd.)

Equity & Liabilities Amount Assets Amount


Share Capital
Equity Shares of NRs. 100 Each 2,500,000 Non-current Assets 3,230,000
10% Preference Shares of NRs 100 Each 732,000 Trade Receivables 1,280,000
General Reserve 240,000 Inventory 1,015,000
Other Reserves (Incl Cap Reduction) 1,067,400 Cash at Bank 209,400

© The Institute of Chartered Accountants of Nepal 300


CAP II Paper 1: Advanced Accounting

12% Debenture of NRs. 100 Each 380,000


Trade Payables 815,000
Total 5,734,400 Total 5,734,400

Working Notes (WN)


Financial Position of Rishi Ltd. (prior to absorption of Muni)

Equity & Liabilities Amount Assets Amount


Share Capital Non-current Assets 2,080,000
Equity Shares of NRs. 100 Each 1,000,000 Trade Receivables 600,000
10% Preference Shares of NRs 100 Each 700,000 Inventory 400,000
General Reserve 240,000 Cash at Bank 54,400
Other Reserves (Incl Cap Reduction) 514400
12% Debenture of NRs. 100 Each 180,000
Trade Payables 500,000
Total 3,134,400 Total 3,134,400

1 Purchase consideration computation


Equity share capital [(15000*50/5)*NRs. 10] 1,500,000
10% Preference share capital [(4000*4/5)*NRs. 10] 32,000
Total purchase consideration 15,32,000

5. The following particulars are available in respect of the business of Lucky Ltd.:
i) Profits earned for the years:
2012/2013 Rs. 500,000 2013/2014 Rs. 600,000 2014/2015 Rs. 550,000
ii) Normal rate of return = 10%
iii) Capital Employed = Rs. 3,000,000
iv) Present value of an annuity of one rupee for 5 years at 10% discount rate = Rs. 3.80
v) The profits included non- recurring profits on an average basis Rs. 30,000 a year.
You are required to calculate the value of goodwill of the company:
(a) As per five years purchase of super profits:
(b) As per capitalization of super profits method: and
(c) As per annuity method.
(CAP Dec. 2016 Q5c-5 Marks)
Answer
Average profits for the last three years
Rs. 550,000
(Rs. 500,000+ Rs. 600,000+ Rs. 550,000)/3
Less: Non –recurring profits Rs. 30,000
Recurring Profits Rs. 520,000
Less: Normal Profits = 10% of Rs. 3,000,000 Rs. 300,000

© The Institute of Chartered Accountants of Nepal 301


CAP II Paper 1: Advanced Accounting

Super Profit Rs. 220,000


(i) Goodwill at five year's purchase of super profits = Rs. 220,000 X 5= 1,100,000.
(ii) Goodwill as per capitalization of super profits @ 10%= (Rs. 220,000X100)/10= Rs.
2,200,000.
(iii) Goodwill as per annuity method = Rs. 220,000 X Rs. 3.80 = Rs. 836,000.

6. Batliboi & Co. Ltd. carried on manufacturing business. Its products were sold to wholesalers
and the company had its own retail shop. Adhikary & Co. (P) Ltd. carried on similar
manufacturing business, but all goods produced were sold through the company's own retail
shops. The summarized Financial Position of the two companies as at 31st March, 2014 were
as follows:
Equity & Batliboi Adhikary Batliboi Adhikary
Assets
Liabilities & Co & Co. & Co & Co.
Share Capital Non-Current Assets:
Authorized equity Freehold Properties at
shares of Rs. 10 4,000,000 600,000 cost 1,000,000 250,000
Issued & fully paid Plant & Machinery at
up 2,500,000 600,000 cost less depreciation 1,300,000 100,000
Retained Earnings 340,000 90,000 Current Assets:
Creditors 420,000 70,000 Stock 480,000 120,000
Debtors 230,000 80,000
Bank 250,000 210,000
Total 3,260,000 760,000 3,260,000 760,000

The Original cost of Plant and Machinery was:


Batliboi & Co. Ltd. Rs. 26,00,000
Adhikary & Co. (P) Ltd. Rs. 2,00,000
The following arrangements were made and carried out on April 1, 2014.
a) Batliboi & Co. Ltd. purchased from the shareholders of Adhikary & Co. (P) Ltd. all the
issued shares @ Rs. 14 per share.
b) The shareholders of Adhikary & Co. (P) Ltd. took over one of the freehold properties of
Adhikary & Co. (P) Ltd. for Rs. 60,000, at the book value of the same. It was agreed that
the amount should be set off against the amount due to them under (a) above and the
balance due to them to be satisfied by the issue of an appropriate number of equity shares
in Batliboi & Co. Ltd. at Rs. 19.50 per share.
The necessary transfer in regard to the setting off the price of the property taken over by the
shareholders against the amount due to them from Batliboi & Co. Ltd. were made in the books
of the two companies.
c) All manufacturing was to be carried on by Batliboi and Co. Ltd. and all retail business is
to be carried on by Adhikary & Co. (P) Ltd. in this connection.

© The Institute of Chartered Accountants of Nepal 302


CAP II Paper 1: Advanced Accounting

i) Batliboi & Co. Ltd. purchased the whole of Adhikary & Co. (P) Ltd.'s plant and
machinery for Rs. 1,50,000 and certain of their free-hold property (cost Rs. 1,00,000)
at Rs. 1,20,000.
ii) Adhikary & Co. (P) Ltd. purchased Batliboi & Co. Ltd.’s freehold retail shop
Buildings (Cost to Batliboi & Co. Ltd., Rs. 75,000) at Rs. 60,000 and took over the
retail stock at Rs. 80,000 at the book value.
d) Batliboi & Co. Ltd. drew a cheque in favor of Adhikary & Co. (P) Ltd. for the net amount
due, taking into account all the matters mentioned above.
e) Immediately after the transfer of shares in (a) above. Adhikary & Co. (P) Ltd. declared and
paid dividend of Rs. 60,000 (ignore income-tax).
You are required to prepare the Balance Sheets of Batliboi & Co. Ltd. and Adhikary & Co. (P)
Ltd. immediately after the completion of the above transaction.
(CAP Dec. 2017 Q1-20 Marks)
Answer
Financial position of Batliboi & Co. as on 1st April, 2014

Liabilities Rs. Assets Rs.


Share Capital Non-current Assets
Authorized Freehold properties
4, 00,000 shares of Rs. 10 as per last SFP
Each 40, 00,000 10, 00,000
Issued, subscribed & paid-up Addition during the
2, 90,000 shares of Rs. 10 each Year 1, 20,000
Fully paid of which 40,000 1120000
Shares were issued pursuant Less: sold during the 75,000
To contract without pay- Year 1045000
ment being received in cash 29, 00,000 Plant and Machinery:
As per 26, 00,000
previous SFP
Reserved & Surplus Addition during 1, 50,000
The year
Share premium 3, 80,000 27, 50,000
Retained Earnings 3, 25,000 Less: Depreciation 13, 00,000
Current Liabilities & 14, 50,000
Provisions: Investments
Sundry Creditors 4, 20,000 Shares in subsidiary 7, 80,000
Company
Current, Assets, Loans & Advances
Stock in trade 4, 00,000
Sundry debtors 2, 30,000
Cash at Bank 1, 20,000
40, 25,000 40, 25,000
Working Notes:
(1) Value of shares in Adhikary & Co. (P) Ltd. Rs.

© The Institute of Chartered Accountants of Nepal 303


CAP II Paper 1: Advanced Accounting

60,000 Shares @ Rs. 14 per share 8, 40,000


Less: Value of freehold property 60,000
Net amount due 7, 80,000

No of shares issued 07, 80,000/ Rs. 19.5= 40,000


Amount credited to Share Capital 4, 00,000
Amount credited to Share premium 3, 80,000
7, 80,000
(2) Rs. 15,000 loss on the sale of Building to Adhikary & Co. (P) Ltd. has been
debited to the profit & Loss Amount.
(3) Shares in subsidiary has been accounted for using the cost method as provided
for in NAS 27 & therefore, dividends are recognized in the income for the
period rather than crediting to investment cost. Alternatively, the equity method
of accounting as per NAS 28 maybe followed in which case the amount of
investment and retained earnings for the period will differ.
(4) Amount payable Adhikary & Co. (P) Ltd. Rs.
Value of assets purchased from Adhikary & Co. (P) Ltd. 2, 70,000
Less: Value of assets sold 1, 40,000
1, 30,000
Value of freehold property taken over by shareholders
Of Adhikary & Co. (P) Ltd. 60,000
1, 90,000
(5) Cash & Bank Balance:
As given 2, 50,000
Add: Dividend received 60,000
3, 10,000
Less: paid to Adhikary & Co. Ltd. including 1, 90,000
1, 20,000

Statement of Financial Position of Adhikary & Co. (P) Ltd.


as on 1st April, 2014

Liabilities Rs. Assets Rs.


Share capital Fixed Assets
Authorized Freehold properties:
60,000 shares of Rs. 10 each 6, 00,000 as per last Balance
Issued, subscribed & paid up Sheet 2, 50,000
60,000 shares of Rs. 10 Addition during the year 60,000
Each full paid 6, 00,000 3, 10,000
Reserved and Surplus Less: Sold during the year 1, 60,000
1,50,000
Plant & Machinery
Retained Earnings 1, 00,000 Cost 2, 00,000
Current Liabilities & Provision Less: Sold (Cost) 2, 00,000 nil

© The Institute of Chartered Accountants of Nepal 304


CAP II Paper 1: Advanced Accounting

Sundry Creditors 70,000 Current Assets, Loan & Advances


Stock in trade 2,00,000
Sundry debtors 80,000
Cash at Bank 3,40,000
7, 70,000 7,70,000
Working Notes: Rs.
1) Profit & Loss Account (given) 90,000
Add: Profit on sale of machinery and freehold property 70,000
1, 60,000
Less: Dividend paid 60,000
1, 00,000

2) Freehold properties have been reduced by Rs. 1, 00,000 transferred to Batilboi & Co.
& Rs. 60,000 taken over by the shareholders of Adhikari & Co. (P) Ltd.
3) Cash at Bank: Balance as given Rs.
Add: Received from Batliboi & Co. 2, 10,000
1, 90,000
4, 00,000
Less: Dividend Paid 60,000
3, 40,000

© The Institute of Chartered Accountants of Nepal 305


CAP II Paper 1: Advanced Accounting

7. Ram, the owner of Ram Ltd. and Sita, the owner of Sita Ltd., got married. So, they agreed to
amalgamate their business. The scheme envisaged a share capital, equal to the combined
capital of Ram Ltd. & Sita Ltd. for the purpose of acquiring the assets, liabilities and
undertakings of the two companies in exchange for share in Ram & Sita Ltd.
The Summarized Financial Position of Ram Ltd. & Sita Ltd. as on 32nd Ashadh, 2075 (the date
of amalgamation) are given below:
Summarized Financial Position as on 32nd Ashadh, 2075
Equity & Liabilities Ram Ltd. Sita Ltd. Assets Ram Ltd. Sita Ltd.
(Rs.) (Rs.) (Rs.) (Rs.)
Shareholders Fund: Non-current Assets:
a. Share Capital 6,00,000 8,40,000 excluding goodwill 7,20,000 10,80,000
b. Reserves 10,20,000 6,00,000
Current Assets:
Inventories 3,60,000 6,60,000
Current Liabilities: Trade Receivables 4,80,000 7,80,000
Bank Overdraft - 5,40,000 Cash at Bank 3,00,000 -
Trade Payables 2,40,000 5,40,000
18,60,000 25,20,000 18,60,000 25,20,000

The consideration was to be based on the net assets of the companies as shown in the SFP
above, but subject to an additional payment to Ram Ltd. for its goodwill to be calculated as its
weighted average net profits for the three years ended 32nd Ashadh, 2075.
The profit had been
2072/73 Rs. 3,00,000; 2073/74 Rs. 5,25,000; 2074/75 Rs. 6,30,000.

The shares of Ram & Sita Ltd. were to be issued to Ram Ltd. & Sita Ltd. at a premium and in
proportion to the agreed net assets value of these companies.
In order to raise working capital, Ram & Sita Ltd. proceeded to issue 72,000 shares of Rs. 10
each at the same rate of premium as issued for discharging the purchase consideration to Ram
Ltd. & Sita Ltd.
You are required to prepare:
a) Calculate the number of shares issued to Ram Ltd. & Sita Ltd.
b) Prepare required journal entries in the books of Ram & Sita Ltd.
c) Prepare the SFP of Ram & Sita Ltd. after recording necessary journal entries.
(CAP Dec. 2018 Q1-20 Marks)
Answer
a) Calculation of number of shares issued to Ram Ltd. & Sita Ltd.
Amount of share capital as per SFP Rs.
Ram Ltd. 6,00,000
Sita Ltd. 8,40,000
14,40,000
Share of Ram Ltd. = 14,40,000*[21,60,000/(21,60,000+14,40,000) ]
= Rs. 8,64,000 or 86,400 Shares

© The Institute of Chartered Accountants of Nepal 306


CAP II Paper 1: Advanced Accounting

Securities Premium= 21,60,000 - 8,64,000 = Rs. 12,96,000


Premium per share = 12,96,000/86,400 = Rs. 15 per share

Share of Sita Ltd. = 14,40,000*[14,40,000/(21,60,000+14,40,000) ]


= Rs. 5,76,000 or 57,600 Shares
Securities Premium= 14,40,000-5,76,000= Rs. 8,64,000
Premium per share = 8,64,000/57600= Rs. 15 per share

b) Journal Entries in the books of Ram & Sita Ltd.


i) Business Purchase Account Dr. 36,00,000
To, Liquidator of Ram Ltd. 21,60,000
To, Liquidator of Sita Ltd. 14,40,000
(Being the amount of purchase consideration payable to liquidator of Ram
Ltd. & Sita Ltd. for assets taken over)

ii) Goodwill Account Dr. 5,40,000


Non-current Assets Account Dr. 7,20,000
Inventory Account Dr. 3,60,000
Trade Receivables Account Dr. 4,80,000
Cash at Bank Account Dr. 3,00,000
To, Trade Payables Account 2,40,000
To, Business Purchase Account 21,60,000
(Being assets & liabilities of Ram Ltd. taken over)

iii) Non-current Assets Account Dr. 10,80,000


Inventory Account Dr. 6,60,000
Trade Receivables Account Dr. 7,80,000
To, Trade Payables Account 5,40,000
To, Bank Overdraft Account 5,40,000
To, Business Purchase Account 14,40,000
(Being assets & liabilities of Sita Ltd. taken over)

iv) Liquidator of Ram Ltd. Account Dr. 21,60,000


To, Equity Share Capital Account 8,64,000
To, Securities Premium Account 12,96,000
(Being the allotment of shares as per agreement for discharge of purchase
consideration)

v) Liquidator of Sita Ltd. Account Dr. 14,40,000


To, Equity Share Capital Account 5,76,000
To, Securities Premium Account 8,64,000
(Being the allotment of shares as per agreement for discharge of purchase
consideration)

© The Institute of Chartered Accountants of Nepal 307


CAP II Paper 1: Advanced Accounting

vi) Bank Account Dr. 18,00,000


To, Equity Share Capital Account 7,20,000
To, Securities Premium Account 10,80,000
(Being Equity Share Capital raised for working capital )

c)
Statement of Financial Position
Ram & Sita Ltd.
as on 32-03-2075
Equity & Liabilities Amount Assets Amount (Rs.)
(Rs.)
Shareholders Fund Non-current Assets
a. Share Capital 21,60,000 Other Non-current 18,00,000
b. Securities Premium 32,40,000 Assets
Goodwill 5,40,000
Current Liabilities Current Assets
Bank Overdraft 5,40,000 Inventories 10,20,000
Trade Payables 7,80,000 Trade Receivables 12,60,000
Cash at Bank 21,00,000
67,20,000 67,20,000

Working Notes:
1. Calculation of Goodwill of Ram Ltd.
Year Profit Weight Weighted Amount
2072/73 3,00,000 1 3,00,000
2073/74 5,25,000 2 10,50,000
2074/75 6,30,000 3 18,90,000
Total 6 32,40,000
Weighted Average Amount = 32,40,000/6 = 5,40,000
Goodwill = Rs. 5,40,000

2. Calculation of Net Assets

Ram Ltd. Sita Ltd.


Assets
Goodwill 5,40,000 -
Non-current Assets 7,20,000 10,80,000
Inventory 3,60,000 6,60,000
Trade Receivables 4,80,000 7,80,000
Cash at Bank 3,00,000 -
Less: Liabilities
Bank Overdraft - 5,40,000
Trade Payables 2,40,000 5,40,000

© The Institute of Chartered Accountants of Nepal 308


CAP II Paper 1: Advanced Accounting

Net Assets/ Purchase Consideration 21,60,000 14,40,000

8. The Balance Sheets of X Co. Ltd. and Y Co. Ltd. as on 31st Chaitra, 2074 are as follows:
X Co. Ltd.
Liabilities Rs. Assets Rs.
Share Capital: Fixed Assets:
Authorized Capital: Goodwill 80,000
10,000 shares of Other 8,00,000 8,80,000
Rs. 100 each 10,00,000 Current Assets,
Loans and Advances 9,00,000
Issued Capital:
10,000 shares of Rs.100 each
fully paid 10,00,000
Reserves and Surplus:
Capital Reserve 2,00,000
General Reserve 70,000 2,70,000
Unsecured Loans 2,00,000
Current Liabilities and
Provisions :
Sundry Creditors 3,10,000
17,80,000 17,80,000

Y Co. Ltd.

Liabilities Rs. Assets Rs.


Share Capital: Fixed Assets: 16,00,000
Authorized Capital: Current Assets, Loans
2,00,000 share of Rs.10 and Advances:
each 20,00,000 Bank 2,00,000
Other 6,60,000 8,60,000
Issued Capital:
80,000 shares of Rs.10
each fully paid 8,00,000

Reserves and Surplus: 8,00,000


General Reserve 5,00,000
Secured Loans
Current Liabilities and
Provisions:
Sundry Creditors 3,60,000
24,60,000 24,60,000

It was proposed that X Co. Ltd. should be taken over by Y Co. Ltd. The following arrangement

© The Institute of Chartered Accountants of Nepal 309


CAP II Paper 1: Advanced Accounting

was accepted by both the companies:


a. Goodwill of X Co. Ltd. is considered valueless.
b. Arrears of depreciation in X Co. Ltd. amounted to Rs. 40,000.
c. The holder of every 2 shares in X Co. Ltd. was to receive:
(i) as fully paid at per 10 shares in Y Co. Ltd. and
(ii) so much cash as in necessary to adjust the rights of shareholders of both the companies
in accordance with the intrinsic value of the shares as per their SFP subject to necessary
adjustments with regards to goodwill and depreciation.
You are required to:
(a) Determine the composition of purchase consideration; and
(b) Show the SFP after absorption.
(CAP Dec 18 Q3a-10 Marks)
Answer
a)
(a)Computation and Composition of Purchase Consideration
(i) Valuation of shares of X Co. Ltd. Rs.
Share Capital 10,0,000
Capital Reserve 2,00,000
General Reserve 70,000
12,70,000

Less: Goodwill, being valueless 80,000


Arrear of Depreciation 40,000 1,20,000
Value of Net Assets 11,50,000
No. of Shares 10,000

Intrinsic value per share Rs.115

(ii) Valuation of Shares of Y Co. Ltd.


Share Capital 8,00,000
General Reserve 8,00,000
16,00,000
No. of Shares 80,000
Value per share Rs. 20

On the basis of intrinsic values, every holder of two shares in X Co. Ltd. will receive
10 shares in Y Co. Ltd. plus cash for the balance. The intrinsic value of the two
shares in X Co. Ltd. is Rs. 230 and that of 10 shares in Y Co. Ltd. is Rs.200 .
Therefore, for each lot of two shares in X Co. Ltd. , a shareholder will receive Rs.30
in cash (Rs.230-200).

Y Co . Ltd. will therefore satisfy the purchase considerations as follows:


50,000 Shares of Rs.10 each issued at Rs. 20 each 10,00,000

© The Institute of Chartered Accountants of Nepal 310


CAP II Paper 1: Advanced Accounting

Cash 1,50,000
11,50,000

(b) Y Co. Ltd.


STATEMENT OF FINANCIAL POSITION
as on 31st Chaitra, 2074
Liabilities Rs. Liabilities Rs.
Share Capital: Non-current Assets
Authorized 16,00,000
2,00,000 shares of Addition on 23,60,000
Rs.10 each 20,00,000 acquisition 7,60,000
-
Issued and Investment 15,60,000
Subscribed Current Assets, Loans
1,30,000 Shares of &Advances 50,000
Rs.10 each fully paid Cash at Bank
(Issued for
consideration other
than cash:50,000
Shares of Rs.10 each 13,00,000
fully paid )
Reserve and Surplus 5,00,000
: 8,00,000
Share Premium 5,00,000
General Reserve 2,00,000
Secured Loans
Unsecured Loans
Current Liabilities & 6,70,000
Provisions : 39,70,000 39,70,000
Sundry Creditors

9. The following Financial Position of Best Ltd. and Better Ltd. are given as on 31st March 2010:
(Rs. In Lakhs)
Best Better Best Better
Capital & Liabilities Assets
Ltd. Ltd. Ltd. Ltd.
Shares of Rs. 100 each fully paid 20 10 Property 25 15
Reserve and Surplus 10 8 Investments 5 -
Other Liabilities 20 2 Current Assets 20 5
50 20 50 20
The following further information is given:
a) Investment of Best Ltd. includes Rs. 3 lakhs representing shares in Better Ltd. having a face
value of Rs. 2 lakhs.

© The Institute of Chartered Accountants of Nepal 311


CAP II Paper 1: Advanced Accounting

b) Better Ltd. issued Bonus Shares on 1st April, 2010 in the ratio of one shares for every two
shares held, out of Reserve and Surplus.
c) It was agreed that Best Ltd. will take over the business of Better Ltd., on the basis of the
latter's Financial Position, the consideration taking the form of allotment of shares in Best
Ltd.
d) The value of shares in Best Ltd. was considered to be Rs. 150 and the shares in Better Ltd.
were valued at Rs. 100 after the issue of bonus shares. The allotment of shares is to be made
on the basis of these values.
e) Liabilities of Better Ltd. included Rs. 1 lakh due to Best Ltd. for purchases from it, on which
Best Ltd. made a profit of 25% on the cost. The goods of Rs. 50,000 out of the said purchases
remained in stock on the date of the above Balance Sheet.
Make the closing ledger in the books of Better Ltd. and the opening journal entries in the
books of Best Ltd., and prepare the Balance Sheet as at 1st April, 2010 after the takeover.
(Inter Jun. 2010 Q3- 15 Marks)

Answer
Ledger Accounts in the Books of Better Ltd.
Property Account
Particulars Rs. Particulars Rs.
To Balance b/d 1,500,000 By Realization A/c - Transfer 1,500,000
1,500,000 1,500,000
Current Assets Account
Particulars Rs. Particulars Rs.
To Balance b/d 500,000 By Realization A/c - Transfer 500,000
500,000 500,000
Liabilities Account
Particulars Rs. Particulars Rs.
To Realization A/c – Transfer 200,000 By Balance b/d 200,000
200,000 200,000
Realization Account
Particulars Rs. Particulars Rs.
To Property A/c 1,500,000 By Liabilities A/c 200,000
To Current Assets A/c 500,000 By Best Ltd. – Consideration 1,500,000
By Shareholders A/c – Loss 300,000
2,000,000 2,000,000
Reserve and Surplus Account
Particulars Rs. Particulars Rs.
To Share Capital A/c – Bonus 500,000 By Balance b/d 800,000
To Sundry Shareholders A/c 300,000
800,000 800,000
Share Capital Account
Particulars Rs. Particulars Rs.
To Sundry Shareholders A/c 1,500,000 By Balance b/d 1,000,000

© The Institute of Chartered Accountants of Nepal 312


CAP II Paper 1: Advanced Accounting

By Reserve & Surplus A/c 500,000


1,500,000 1,500,000
Best Limited Account
Particulars Rs. Particulars Rs.
To Realization A/c – P.C. 1,500,000 By Sundry Shareholders – 1/5th 300,000
By Shares in Best Ltd. 1,200,000
1,500,000 1,500,000
Shares in Best Limited
Particulars Rs. Particulars Rs.
To Best Limited 1,200,000 By Sundry Shareholders A/c 1,200,000
1,200,000 1,200,000
Sundry Shareholders Account
Particulars Rs. Particulars Rs.
To Realization A/c – Loss 300,000 By Share Capital A/c 1,500,000
To Best Ltd. 300,000 By Reserve & Surplus A/c 300,000
To Shares in Best Ltd. 1,200,000
1,800,000 1,800,000
Working Note:
Calculation of Purchase Consideration
Rs.
Issued Capital of Better Ltd. (after bonus issue) at Rs. 100 each 1,500,000
Less: Shares held by Best Ltd. 300,000
Shares held by outsiders, valued at Rs. 100 per Share 1,200,000
Purchase consideration has been discharged by Best Ltd. by way of issue of shares for Rs.
800,000 at a premium of Rs. 400,000. This gives the value of Rs. 150 per share.
Journal Entries in the Books of Best Ltd.
Particulars Dr. (Rs.) Cr. (Rs.)
Property A/c 1,500,000
Current Assets A/c (Net of unrealized gain) 490,000
To Liabilities A/c 200,000
To Liquidator of Better Ltd. 1,200,000
To Shares in Better Ltd. 300,000
To Reserve & Surplus (Bargain purchase) A/c 290,000
(Being Assets and Liabilities of Better Ltd. taken and
cancellation of investments, held in Better Ltd. Assumed that
there was not Goodwill on initial investment date due to lack of
information. )
Liquidator of Better Ltd. 1,200,000
To Share Capital A/c 800,000
To Share Premium A/c 400,000
(Being discharge of purchase consideration by the issue of equity
shares of Rs. 8,00,000 at a premium of Rs. 50 per share)
Sundry Creditors A/c 100,000

© The Institute of Chartered Accountants of Nepal 313


CAP II Paper 1: Advanced Accounting

To Sundry Debtors A/c 100,000


(Being amount due from Better Ltd., and included in Creditors
taken over, cancelled against Sundry Debtors)

Best Limited
Statement of Financial Position
as at 1st April 2010 (After absorption)
Equity and Liabilities Rs. Assets Rs.
Share Capital: Property Plant and Equip 4,000,000
28,000 Shares of Rs. 100 each 2,800,000 Investments 200,000
Share Premium 400,000 Current Assets 2,390,000
Reserves & Surplus 1,290,000
Current Liabilities 2,100,000
6,590,000 6,590,000

10. The following are the Financial Positions of Big Ltd. and Small Ltd. as on 31st March, 2012.
(Rs. in Crore)
Big Ltd. Small Ltd.
Equity share capital – Rs. 10 each 50 40
10% Preference share capital – Rs. 100 each - 60
Reserves & surplus 200 150
Loans – secured 100 100
Total funds 350 350

Applied for:
Non-current assets less depreciation 150 150
Net current assets 200 200
Total assets 350 350

The present worth of Non-current assets of Big Ltd. is Rs. 200 crores and that of Small Ltd. is
Rs. 429 crores. Goodwill of Big Ltd. is Rs. 40 crores and of Small Ltd. is Rs. 75 crores.
Small Ltd. absorbs Big Ltd. by issuing equity shares at par in such a way that intrinsic net
worth is maintained.
Goodwill account is not to appear in the books. Non-current assets are to appear at old figures.
a) Show the balance sheet of Small Ltd. after absorption.
b) Prepare a statement showing intrinsic value of shares to prove that it is maintained as
before.

(Inter Jun. 2012 Q3-15 Marks)


Answer Small Ltd.
Balance Sheet as on 1st April, 2011
Rs. in crore

© The Institute of Chartered Accountants of Nepal 314


CAP II Paper 1: Advanced Accounting

Equity Share Capital – Rs. 10 each (Rs. 40 Crores + Rs. 25 Crores) 65


10% Preference Share Capital – Rs. 100 each 60
Reserves & Surplus (Rs. 150 Crores + Rs. 225 Crores) 375
Loans – Secured (Rs. 100 Crores + Rs. 100 Crores) 200
Total Funds 700
Applied for:
Fixed Assets less Depreciation (Rs. 150 Crores + Rs. 150 Crores) 300
Net Current Assets (Rs. 200 Crores + Rs. 200 Crores) 400
Total Assets 700
Working notes:
(i) Calculation of intrinsic value of shares of both companies:
(Rs. in crore)
Big Ltd. Small Ltd.
Equity Share Capital 50 40
Reserve & Surplus 200 150
Value of Goodwill – as agreed 40 75
Increase in value of Fixed Assets 50 279
340 544
Number of Equity Shares 5 crores 4 crores
Intrinsic Value per Share Rs. 68 Rs.136
Ratio of intrinsic value of shares in the two companies = 68 : 136 or 1 : 2
Since the shares are to be issued at par, the number of equity shares of Rs.10 each to be
issued by Small Ltd. to holders of 5 crore equity shares of Big Ltd. to maintain the intrinsic
net worth = 5 Crores / 2 = 2.5 Crores.
Therefore, purchase consideration is Rs. 25 Crores (Rs. 10 × 2.5 Crores).
(ii) Calculation of Capital Reserve / Goodwill on Absorption:
Rs. in Crores
Net Assets taken over Rs.(150 + 200 – 100) 250
Purchase Consideration 25
Capital Reserve 225
(b) Statement showing intrinsic value of shares:
Rs. in Crores
Equity Share Capital 65
Reserve & Surplus 375
Unrecorded value of Goodwill Rs.(40 + 75) Crores 115
Unrecorded increase in value of Fixed Assets Rs. (50 + 279) Crores 329
884
Number of Equity Shares 6.5 Crores
Intrinsic Value per Share Rs. 136

11. Sudeep and Mayur had been carrying business independently. They agree to amalgamate and
form a new company Sumeru Ltd. with an authorized share capital of Rs. 2,00,000 divided in
to 40,000 equity shares of Rs. 5 each..

© The Institute of Chartered Accountants of Nepal 315


CAP II Paper 1: Advanced Accounting

On 31st Ashadh 2066, the respective Financial Position of Sudeep and Mayur were as follows:
Description Sudeep Rs Mayur Rs.
Non-current Assets 3,17,500 1,82,500
Current assets 1,63,500 83,875
4,81,000 2,66,375
Less: Current Liabilities 2,98,500 90,125
Representing Capital 1,82,500 1,76,250

Additional information:
a) Revalued figures of Non-current assets were: Sudeep Rs 3,55,000 and Mayur Rs. 1,95,000
and revalued figure of current Assets were, Sudeep Rs. 1,49,750 and Mayur Rs. 78,875.
b) The debtors of Sudeep includes Rs. 21,675 to be received from Mayur.
c) The purchase consideration is satisfied by issue of the following shares and debentures:
30,000 equity shares of Sumeru Ltd to Sudeep and Mayur in the proportion of the
profitability of their respective business based on the average net profit during the last three
years which were as follows:
Sudeep Rs Mayur Rs
2064 Profit 2,24,788 1,36,950
2065 Loss / Profit ( 1,250) 1,71,050
2066 Profit 1,88,962 1,79,500
d) 15% Debenture in Sumeru Ltd. at par to provide an income equivalent to 8 % return on
capital employed in their respective business as on 31st Ashadh 2066 after revaluation of
assets.
You are required to:
i) Compute the amount of Debentures and shares to be issued to Sudeep and Mayur
ii) A Statement of Financial Position of Sumeru Ltd. showing the position immediately after
amalgamation.
(CAP Jun. 2011 Q1-20 Marks)
Answer
Sudeep Rs Mayur Rs
(1) Average Profit:
Rs (2,24,788-1,250+1,88,962)/3 1,37,500
Rs ( 1,36,950+1,71,050+1,79,500)/3 1,62,500
(2) Equity Shares issued:
(a) Ratio of distribution 1375 1625
(b) No. of shares 13,750 16,250
(c) Amount:
13,750@ Rs. 5 68,750
16,250 @ Rs. 5 81,250

(3) Capital employed (after revaluation of assets)


Non-current assets 3,55,000 1,95,000
Current Assets 1,49,750 78,875

© The Institute of Chartered Accountants of Nepal 316


CAP II Paper 1: Advanced Accounting

5,04,750 2,73,875
Less: Current Liabilities 2,98,500 90,125
2,06,250 1,83,750

(4)Debenture Issued
8% return on capital employed 16,500 14,700
15% Debenture to be issued to provide equivalent income
Sudeep Rs 16,500*100/15 1,10,000
Mayur Rs 14,700*100/15 98,000

Working No.1 Calculation of Purchase Consideration:


Description Sudeep Rs Mayor Rs Total Rs.

Equity shares issued 68,750 81,250 1,50,000


15% Debenture issued 1,10,000 98,000 2,08,000
1,78,750 1,79,250 3,58,000

WN 2: Calculation of Goodwill

(a) Net Assets taken over:


Fixed Assets 3,55,000 1,95,000 5,50,000
Current assets 1,28,075 78,875* 2,06,950
4,83,075 2,73,875 7,56,950
Less: Current Liabilities 2,98,500 68,450** 3,66,950
184,575 2,05,425 3,90,000

(b) Purchase Consideration 1,78,750 1,79,250 3,58,000

(c ) Goodwill (a-b) (5,825) (26,175) (32,000)


The acquirer should assess whether this transaction is a bargain purchase and if yes, the amount
of gain should be recognized in profit/loss for the period.
* Rs 1,49,750- Rs 21,675 ** Rs 90,125- Rs 21,675

Statement of Financial Position of


Sumeru Ltd as on 31st Ashadh 2066

Rs.
Non-current Assets 5,50,000
Current assets, Loans & Advances Rs. 2,06,950
Less; Current Liabilities Rs 3,66,950 ( 1,60,000)
3,90,000

Represented by:
Share Capital** 1,50,000

© The Institute of Chartered Accountants of Nepal 317


CAP II Paper 1: Advanced Accounting

Reserves 32,000
15% Debenture 2,08,000 3,90,000

**
Share Capital:
Authorized share capital 40,000 shares of Rs 5 2,00,000
Issued and subscribed capital:
30,000 equity shares of Rs 5 each 1,50,000
( all the above shares have been allotted as fully
Paid up without payments being received in cash.)

12. On 31stAshadh 2072 Varun Limited with an issued and subscribed capital of Rs.700,000
divided into 70,000 fully paid equity shares of Rs.10 each had net tangible assets of Rs.950,000
and Goodwill amounting to Rs.50,000. On this date Paul Limited took over Varun Limited’s
business for Rs.1,300,000 payable as to Rs.100,000 cash and Rs.1,200,000 in 100,000 equity
shares of Rs.10 each valued at Rs.12 per share. However, it was decided that the shares so
allotted should be recorded by both the companies in the books at par value only.

On the date of purchase of business, Varun Limited’s debtors included Rs.20,000 due from
Paul Limited for goods sold to it at the profit of 25% on cost. Paul Limited had sold only 1/4th
of these goods by the date of absorption, the remaining goods lying unsold in stock.

You are required to pass necessary journal entries in the books of both the companies.
(CAP Jun. 2016 Q2b- 10 Marks)
Answer

Working Note:
Summarized Balance Sheet of Varun Limited as on 31stAshad 2072:

Share capital Goodwill 50,000


Issued and Subscribed Net Sundry Tangible Assets 9,50,000
70,000 equity shares of Rs.10
Each fully paid up 7,00,000
Reserves 3,00,000

The consideration on the basis of par value of shares would be:


Rs.
Cash 1,00,000
1,00,000 Equity shares of Rs.10 each 10,00,000
11,00,000

In the books of Varun Limited


Journal Entries

© The Institute of Chartered Accountants of Nepal 318


CAP II Paper 1: Advanced Accounting

Date Particulars Dr Cr
Amount Amount
2072 Realization A/c Dr. 10,00,000
31/3 To Sundry Net Tangible Assets 9,50,000
To Goodwill 50,000
(Being transfer of goodwill and other sundry
tangible assets to realization account.)
31/3 Paul Limited A/c Dr. 11,00,000
To Realization A/c 11,00,000
(Being amount calculated on the basis of par
value of shares due to Paul Limited for
business sold to it.)
31/3 Bank. A/c Dr. 1,00,000
Equity Shares in Paul Limited Dr. 10,00,000
To Paul Limited A/c 11,00,000
(Being receipt of cash and equity shares in
Paul Limited at par from Paul Limited in
discharge of the purchase consideration for
the business sold to it.)
31/3 Equity Share Capital A/c Dr. 7,00,000
Reserves A/c Dr. 3,00,000
Realization A/c Dr. 1,00,000
To Equity Shareholders A/c 11,00,000
(Transfer of equity share capital account,
Reserves and profit on realization to the
Equity Shareholders A/c.)
31/3 Equity Shareholders A/c Dr. 11,00,000
To Bank A/c 1,00,000
To Equity Shares in Paul Ltd. A/c 10,00,000
(Being shares allotted)

In the books of Paul Limited


Journal Entries

Date Particulars Dr Cr
Amount Amount
2072 Business Purchase A/c Dr. 11,00,000
31/3 To Liquidator of Varun Limited A/c 11,00,000
(Being amount calculated on the basis of par
value of shares payable to liquidator of Varun
Limited for the business purchased.)
31/3 Goodwill A/c Dr. 1,50,000
Net Sundry Assets A/c Dr. 9,50,000

© The Institute of Chartered Accountants of Nepal 319


CAP II Paper 1: Advanced Accounting

To Realization A/c 11,00,000


(Incorporation of the net sundry tangible
assets taken over and the amount paid for
goodwill..)
31/3 Liquidator of Varun Limited A/c Dr. 11,00,000
To Bank A/c 1,00,000
To Equity Share Capital Account 10,00,000
(Payment of Cash and allotment of 1,00,000
equity shares of Rs.10 each (recorded at par)
to the liquidators of Varun Limited in
discharge of the consideration.)
31/3 Sundry Creditors Dr. 20,000
To Sundry Debtors A/c 20,000
(Elimination of mutual owing.)
31/3 Goodwill A/c Dr. 3,000
To Stock A/c 3,000
(Being elimination of unrealized profit in
respect of 3/4th of goods purchased earlier
from Varun Limited and remaining unsold on
the date of absorption.)

13. The summarized Financial Position of Krishna Ltd. As on 31st Ashadh, 2073 was as follows:
Liabilities Amount Assets Amount
(Rs.) (Rs.)
Equity Shares of Rs. 10 fully 30,00,000 Goodwill 5,00,000
Export
paid Profit Reserves 8,50,000 Tangible Assets 30,00,000
General Reserves 50,000 Stock 10,40,000
Retained Earnings 5,50,000 Debtors 1,80,000
9% Debentures 5,00,000 Cash & Bank 2,80,000
Trade Creditors 1,00,000 Preliminary Expenses 50,000
50,50,000 50,50,000
st
Radha Ltd. agreed to absorb the business of Krishna Ltd. with effect from 1 Shrawan, 2073.
iii) The purchase consideration settled by Radha Ltd. as agreed:
- 4,50,000 equity shares of Rs. 10 each issued by Radha Ltd. by valuing its share @
Rs. 15 per share.
- Cash payment equivalent to Rs. 2.50 for every share in Krishna Ltd.
iv) The issue of such an amount of fully paid 8% debentures in Radha Ltd. at 96% as is sufficient
to discharge 9% debentures in Krishna Ltd. at a premium of 20%.
v) Radha Ltd. will take over the tangible assets at 100% more than the book value, stock at Rs.
7,10,000 and debtors at their face value subject to a provision of 5% for doubtful debts.
vi) The actual cost of liquidation of Krishna Ltd. was Rs. 75,000. Liquidation cost of Krishna Ltd.
is to be reimbursed by Radha Ltd. to the extent of Rs. 50,000.
vii) Statutory reserves are to be maintained for 1 more year.
You are required to:

© The Institute of Chartered Accountants of Nepal 320


CAP II Paper 1: Advanced Accounting

(i) Close the books of Krishna Ltd. by preparing realization account, Radha Ltd. Account,
shareholders account and debenture account.
(ii) Pass journal entries in the books of Radha Ltd. regarding acquisition of business
(CAP Jun. 2017 Q2a-10 Marks)
Answer
(i) Purchase consideration computation Rs.
Cash payment for(3,00,000xRs.2.5) 7,50,000
Equity Shares(4,50,000xRs.15) 67,50,000
75,00,000
In the books of Krishna Ltd.
Realization Account
Rs. Rs.
To Goodwill 5,00,000 By 9% Debentures 5,00,000
To Tangible Assets 30,00,000 By Creditors 1,00,000
To Stock 10,40,000 By By Radha Ltd. 75,00,000
To Debtors 1,80,000 (Purchase
To Cash & Bank A/c 2,55,000 consideration)
(2,80,000-25,000)
To Cash & Bank A/c 25,000
(Realization expenses)
To Profit on realization
Transfer to shareholders 31,00,000
81,00,000 81,00,000
Equity Shareholders A/c
Rs. Rs.
To Preliminary expenses 50,000 By Equity Share Capital 30,00,000
To Equity Shares in Radha Ltd. 67,50,000 By Export Profit 8,50,000
To Cash & Bank A/c ReservesReserves
7,50,000 By General 50,000
By Retained Earnings 5,50,000
By Realization A/c 31,00,000
75,50,000 75,50,000
9%DebenturesAccount
Rs. Rs.
To Realization A/c 5,00,000 B Balance b/d 5,00,000
Radha
y Ltd.
Rs. Rs.
To Realization A/c 75,00,000 By Share Capital 67,50,000
By Bank A/c 7,50,000
75,00,000 75,00,000

Journal Entries in the books of Radha Ltd.


Rs. Rs.

© The Institute of Chartered Accountants of Nepal 321


CAP II Paper 1: Advanced Accounting

1 Business Purchase A/c Dr. 75,00,000


To Liquidator of Krishna Ltd 75,00,000
(Being business of Krishna Ltd. taken over)
2 Tangible Assets Dr. 60,00,000
Stock Dr. 7,10,000
Debtors Dr. 1,71,000
Cash & Bank A/c Dr. 2,55,000
Goodwill A/c (Bal.fig.) Dr. 11,19,000
To Liability for 9% Debentures 6,25,000
To Creditors 1,00,000
To Business Purchase account (Being assets 75,00,000
and liabilities taken over)

3 Amalgamation Adjustment A/c Dr. 8,50,000


To Export Profit Reserves 8,50,000
(Being statutory Reserves taken over)
4 Goodwill Dr. 50,000
To Bank A/c 50,000
(Liquidation expenses reimbursed))
5 Liquidator of Krishna Ltd. Dr. 75,00,000
To Equity Share Capital 45,00,000
To Securities Premium 22,50,000
To Bank A/c 7,50,000
(Being purchase consideration discharged)

6 Liability for 9% Debentures ( 5,00,000x 120/100) Dr. 6,25,000


To 8% Debentures (6,00,000x100/96) 6,25,000
(Being liability of debenture holders’ discharged.
Debentures should be recognized at amortized cost
as per the provisions of NFRs. However, full amount
has been recognized as liability due to lack of
information)

14. The financial position of two companies Big Ltd. and Small Ltd. as on 31.3.2001 was as under:
Big Ltd. (Rs.) Small Ltd. (Rs.)

Assets:
Goodwill 40,000 25,000
Building 2,00,000 80,000
Machinery 6,00,000 1,30,000
Investments 70,000 45,000
Stock 2,90,000 1,60,000
Debtors 3,00,000 1,80,000

© The Institute of Chartered Accountants of Nepal 322


CAP II Paper 1: Advanced Accounting

Cash & Bank 30,000 20,000


Preliminary Expenses 20,000 10,000
15,50,000 6,50,000

Liabilities:
Share Capital:
Equity Shares of Rs. 10 each 10,00,000 3,00,000
7% Preference shares of Rs. 100 each 2,00,000 –
8% Preference shares of Rs. 100 each – 1,50,000
General Reserve 1,00,000 45,000
Retirement Gratuity Fund 70,000 45,000
Workmen's Compensation Fund – 20,000
Sundry Creditors 1,80,000 90,000
15,50,000 6,50,000
Big Ltd. absorbs Small Ltd. on the following terms and conditions:
i. 8% Preference shareholders are to be paid at 10% premium by issue of 7%
preference shares of Big Ltd.
ii. Goodwill of Small Ltd. is valued at Rs. 60,000, Buildings are valued at Rs. 1,35,000
and Machinery at Rs. 1,45,000. Investments are worth Rs. 46,500.
iii. Stock and book debts of Small Ltd. are valued at 10% below their book values.
iv. Equity shareholders of Small Ltd. will be issued 05 shares of Big Ltd. at 10%
premium for every 4 shares held.
v. Rs. 20,000 liquidation expenses will be borne by Big Ltd.
Close the books of Small Ltd. and show the acquisition entries in the books of Big Ltd.
Also draft the Financial Position after absorption.
(Inter Dec 2001, Q 1-20 Marks)

Answer:
In the books of Small Ltd.
Realization Account
Rs. Rs.

To Sundry Assets 640000 By Gratuity Fund 45000


(650000-10000) By Sundry Creditors 90,000
To Preference Shareholders 15000 By Big Ltd. 597500
(Premium of redemption (Purchase Consideration)
To Bank- (Liquidation Exp.) 20000
To Equity Shareholders 57500
732500 732500

Shareholders Account
Equity Preference Equity Preferenc
(Rs.) (Rs.) (Rs.) e (Rs.)

© The Institute of Chartered Accountants of Nepal 323


CAP II Paper 1: Advanced Accounting

To Prelim Expenses 10,000 By Share capital 3,00,000 1,50,000


To 7% Pref. Shares of 1,65,000 By Realization A/c 15,000
Big Ltd.
To Equity shares of 4,12,500 (Premium on
Big Ltd. redemption)
By Workmen
compensation Fund 20,000
By General Reserve 45,000
By Realization A/c 57,500
(Profit on
realization)
4,22,500 1,65,000 4,22,500 1,65,000

Big Limited Account


Rs. Rs.

To Realization Account 5,97,500 By Bank 20,000


By 7% Preference shares 1,65,000
By Equity Shares 4,12,500
5,97,500 5,97,500

In the books of Big Ltd.

Journal Entries Rs. Rs.

Goodwill A/c Dr. 80,000


Building A/c Dr. 1,35,000
Machinery A/c Dr. 1,45,000
Investment A/c Dr. 46,500
Stock A/c Dr. 1,44,000
Debtors A/c Dr. 1,62,000
Bank A/c Dr. 20,000
To Gratuity Fund 45,000
To Sundry Creditors 90,000
To Liquidators of Small Ltd. 5,97,500
7,50,500 7,50,500
(Being assets and liabilities taken over as per agreed valuation.)
Liquidators of Small Ltd. Dr. 5,97,500
To Bank 20,000
To 7% Preference shares Capital 1,65,000
To Equity Shares Capital 3,75,000
To Share Premium 37,500

© The Institute of Chartered Accountants of Nepal 324


CAP II Paper 1: Advanced Accounting

5,97,500 5,97,500
(Being purchase consideration satisfied as above.)
General Reserve Dr. 20,000
To Preliminary Expenses 45,000
(Being Preliminary expenses written off)

Statement of Financial Position


as at 31st March, 2001

Sources Rs.

Share Capital:
3,650 7% Preference shares of Rs. 10 each 3,65,000
1,37,500 Equity shares of Rs. 10 each fully paid up 13,75,000
(1650 Preference shares and 37,500 equity shares were issued without
payment received in cash)
Reserve & Surplus:
Share Premium 37,500
General Reserve 80,000
18,57,500

Application Rs Rs.
Fixed Assets:
Goodwill 1,20,000
Building 3,35,000
Machinery 7,45,000 12,00,000
Investments 1,16,500
Current Assets
Stock 4,34,000
Debtors 4,62,000
Cash & Bank 30,000
9,26,000
Less Current Liabilities
Gratuity Fund 1,15,000
Sundry Creditors 2,70,000 3,85,000
Net Current Assets 5,41,000
18,57,500

Working Notes:
Rs.
Purchase Consideration
Building 135,000
Machinery 145,000

© The Institute of Chartered Accountants of Nepal 325


CAP II Paper 1: Advanced Accounting

Investments 46,500
Stock 144,000
Debtors 162,000
Cash & Bank 20,000
712,500
Less: Liabilities:
Gratuity Fund 45,000
Sundry Creditors 90,000 135,000
Net Assets Acquired 517,500
To be satisfied as under:
7% Redeemable Preference shares 165,000
Equity shares issued at 10% premium 30000 x 5/4 = 37500 412,000
shares of Rs. 10 each at a premium of 10%
577,500
Liquidation Expenses 20,000
Total Purchase Consideration 597,500
Goodwill 80,000

15. Enterprise Ltd. has 2 division A and B. Division A has been making constant profits while
division B has been in variably suffering losses. On 31st March 2003, the division's Financial
Position was:
(Rupees in crores)
A B Total
Non-Current Assets (Cost) 250 500 750
Depreciation 225 400 625
25 100 125
Current Assets 200 500 700
Less: Current Liabilities 25 400 425
Rs. 175 100 275
Rs. 200 200 400
Financed by:
Loan funds – 300 300
Capital: Equity Rs. 10 each 25 – 25
Surplus 175 -100 75
Rs. 200 200 400
Division B along with its assets and liabilities was sold for Rs. 25 crores to Turnaround Ltd. a
new company, who allotted 1 crores equity shares of Rs. 10 each at a premium of Rs. 15 per
share to the members of Enterprise Ltd. in full settlement of the consideration, in proportion
to their shareholding in the company.

Assuming that there are no other transactions, you are asked to:
i. Pass Journal entries in the books of Enterprise Ltd.
ii. Prepare the Statement of Financial Position of Enterprise Ltd. after the entries in (i)

© The Institute of Chartered Accountants of Nepal 326


CAP II Paper 1: Advanced Accounting

iii. Prepare the Statement of Financial Position of Turnaround Ltd.


(Inter Jun 04, Q 4a-8 Marks)

Answer:
Journal of Enterprise Ltd.
(Rs. in crores)

Particulars L.F. Dr. (Rs.) Cr. (Rs.)

Realization A/c 1,000


To Non-current assets 500
To Current Assets 500
(Being the transfer of assets to Realization A/c)
Accumulated Depreciation 400
Current Liabilities. 400
Loan Funds 300
To Realization A/c 1,100
(Being the transfer of depreciation and liabilities)
Turnaround Ltd. 25
To Realization A/c 25
(Being purchase consideration due)
Realization A/c 125
To Gain on sale of Discontinued 125
Operation (Through P/L)
(Being the transfer of profit on sale of division B)
Investment in shares of T Ltd. A/c 25
To Turnaround Ltd. 25
(Being the allotment of Rs. 1 crore equity shares of Rs. 10 each at a premium of
Rs. 15 per share to the members of Enterprise Ltd. in full settlement of the
consideration)

SFP of Enterprise Ltd.


(after Sale of Division B)

[Rs. in crore]
Equity & Liabilities Rs. Assets Rs.
Equity share capital 25 Non-Current Assets 250
Surplus (125+75) 200 Less: Depreciation 225 25
Current Liabilities 25 Current Assets 200
Investment 25
250 250
Extract of Notes to the financial statements:

© The Institute of Chartered Accountants of Nepal 327


CAP II Paper 1: Advanced Accounting

The profit or loss for the period includes gain on sale of discontinued division (Division B)
against which Turnaround Company issued 1 crore units of shares @ Rs 25 each..

SFP of Turnaround Ltd.


(Rs. in crores)

Liabilities Rs. Assets Rs.


1 crore Equity Shares of 10 Goodwill 125
Rs. 10 each
Securities Premium 15 Non-Current Assets 100
Current Liabilities 400
Loan Funds 300 Current Assets 500
725 725

16. During July 2006, Mera Peak Ltd agreed to acquire the business of Sharma Ji as on June 30,
2006 and take over all his assets (other than any balance at bank and cash) and to discharge his
creditors. The purchase price is to be the book value of assets taken over less liability subject
to:
a) An addition of Rs. 10,000 for goodwill
b) An increase of 20% in the book value of the fixture and fittings, and
c) A deduction of 5% from total of debtors to allow for possible bad debts.
The purchase price is to be satisfied by the issue to Sharma Ji fully paid share of Rs. 10 each
in Mera Peak Ltd, at their market value of Rs. 12.50 per share and any fraction amount in
cash. Having being instructed to determine the purchase price you ascertain the followings:

i. No Statement of Financial Position had been prepared as on June 30, 2006, the last one
being that on March 31, 2006 which showed the following position:
Rs. Rs.
Capital 27,850 Furniture & Fittings 16,000
Creditors 6,200 Stock in Trade 14,700
Debtors 2,700
Balance at Banks 600
Cash in hand 50
34,050 34,050

ii. Sharma Ji kept no proper set of books, but there is available a file of unpaid suppliers’
invoices and a notebook of debtors for credit sales.
iii. Stock had not been taken on June 30, 2006, but it was agreed that it should be taken to be
such an amount as would produce a gross profit percentage on sale of 33.33% for the
three months ended June 30, 2006.
iv. An analysis of the bank account for the three months ended June 30, 2006 shows
Rs. Rs.

© The Institute of Chartered Accountants of Nepal 328


CAP II Paper 1: Advanced Accounting

Paid into bank cheques Cheques drawn suppliers 17,050


from credit customers 18,400
Cash deposited 6,570 Expenses (including rent) 1,540
Drawings 700
New fittings 600
24,970 19,890

v. Payments out of takings during the three months were: cash purchase Rs. 950, expenses
Rs. 650, and drawings Rs. 100.
vi. After retaining a float of Rs. 50, Sharma Ji paid into bank at the end of each day the
balance of his cash takings.
vii. On June 30, 2006 creditors amounted to Rs. 6,400 and debtors to Rs. 3,600.
viii. It was agreed that depreciation and taxation should be ignored.

You are required:


a) To prepare Sharma Ji’s Profit & Loss statement for the three months ended June 30,
2006 and his Financial Position on that date.
b) To compute the purchase price payable and show how it is to be satisfied.
(Inter June 2007, Q 1-20 Marks)

Answer:
Columnar Cash Book
(For 3 months Ending June 30, 2006)

Dr. Cr.
Cash Bank Cash Bank
(Rs.) (Rs.) (Rs.) (Rs.)
To Balance b/d 50 600 By Cash purchases 950
To Cheques from credit 18,400 By Sundry expenses 650 1,540
customers
To Cash (lodgments) 6,570 By Drawings 100 700
To Takings (balancing 8,270 By Cheques to sundry 17,050
figure) creditors
By New fittings 600
By Bank (lodgments) 6,570
By Balance c/d 50 5,680
8,320 25,570 8,320 25,570
To Balance b/d 50 5,680

Sharma Ji Profit & Loss Statement & Financial Position

Profit & Loss Statement Of Sharma Ji


For the month ending June 30, 2006

© The Institute of Chartered Accountants of Nepal 329


CAP II Paper 1: Advanced Accounting

Rs. Rs. Rs. Rs.


To Opening Stock 14,700 By Sales:
Payments Received 18,400
Add: Debtors as on
30.6.1990 3,600
22,000
Less: Outstanding
Debtors as on 2,700
312.3.1990
Credit Sales 19,300
Add: Cash Sales 8,270 27,570
To Purchases 17,050 18,200 By Closing stock 14,520
(Balancing Figure)
Add: Creditors as on 6,400
30.6.1990
23,450
Less: Creditors as on 6,200
30.3.1990
Credit purchases 17,250
Add: Cash purchases 950
To Gross Profit c/d
(33 1/3% on Rs. 27,570) 9,190
42,090 42,090
To Expenses (inclusive rent) 2,190 By Gross Profit b/d 9,190
To Net profit 7,000
9,190 9,190

Financial Position Statement of Sharma Ji


As on June 30, 2006
Liabilities Rs. Rs. Assets Rs.
Sundry creditors 6,400 Fixture and Fittings 16,600
Capital: Balance on April 27,850 Stock 14,520
1990
Add: Net profit for 3 months 7,000
34,850
Less: Drawings 800 34,050
Sundry debtors 3,600
Balance at Bank 5,680
Cash in hand 50
Total 40,450 Total 40,450

b) Computation of purchase price to be paid to Sharma Ji

© The Institute of Chartered Accountants of Nepal 330


CAP II Paper 1: Advanced Accounting

Rs. Rs.
Goodwill 10,000
Fixtures & Fittings 16,600
Add 20% 3,320 19,920
Debtors 3,600
Less 5% 180 3,420
Stock 14,520
47,860
Less: Liabilities 6,400
Purchase price 41,460
To be satisfied as under
By the issue of 3,316 shares of Rs. 10
each at Rs. 12.50 41,450
In Cash (remaining amount) 10
Total 41,460

Working Note:
In order to find out the takings on account of cash sales, Cash Book must be prepared

17. The business condition of Madan Ltd. and Mohan Ltd. were in high competition and they were
selling their product in the market with very low profit margin and spending a big chunk of
money in promotional expenditure. They came across with the problem of human resource,
trying to pull other company's marketing personnel with high remuneration but unable to
increase sales as well as profit margin due to which both of them had incurred losses in the
recent years. Their respective financial position as on 31st July, 1995 were as follows:

Madan Ltd.
Liabilities Amount Rs. Assets Amount Rs.
Issued Capital: Plant 40,000
10,000 shares of Rs. 10 Furnitures & Fittings 4,600
each fully paid 1,00,000 Patents 2,500
Creditors 18,560 Stock 42,460
Bank Overdraft 6,050 Debtors 15,630
Export Profit A/C 10,000 Accumulated Reserves 29,420
Total 1,34,610 Total 1,34,610

Mohan Ltd.
Liabilities Amount Rs. Assets Amount Rs.
Issued Capital: Plant 21,000
12000 shares of Rs. 5 Furniture & Fittings 3,280
each fully paid 60,000 Patents 8,000
Creditors 8,310 Goodwill 10,000
Accumulated Reserves 640 Stock 16,990

© The Institute of Chartered Accountants of Nepal 331


CAP II Paper 1: Advanced Accounting

Statutory Reserve 10,000 Debtors 9,550


Cash 10,130
Total 78,950 Total 78,950
In order to eliminate competition and provide for more economical working as well as to
make it possible to introduce fresh capital, the following arrangements were made and
carried into effect:
b) Both companies were to be wound up, a new company Saddam Ltd. being formed to
take over both businesses.
c) Saddam Ltd. took over the floating assets of both companies as book values except cash
of Mohan Ltd. and following non-current assets were taken at:

Assets Madan Ltd. (Rs.) Mohan Ltd. (Rs.)


Goodwill 1000 1000
Patents 500 2000
Plant 27,000 11,000
Furniture & Fitting 3000 2300
d) The consideration for the assets of Madan Ltd. was satisfied by the issue of 1,200 9%
preference shares of Rs. 10 each and Rs. 64,490 in Rs. 10 equity shares of Saddam Ltd.
fully paid and the balance in cash and for the assets of Mohan Ltd. Rs. 34,300 in Rs. 10
equity shares of Saddam Ltd. and the balance in cash.
e) The liquidator of Madan Ltd. transferred the preference shares to a loan creditors for
Rs. 12,000 in satisfaction of his claim and the equity shares were distributed pro rata
among the shareholders of each of the original companies, the cash being just sufficient
to satisfy the creditors of each company and the expenses of liquidation which amount
to Rs. 500 and Rs. 300 respectively for Madan Ltd. and Mohan Ltd.

f) In order to provide the necessary cash, Saddam Ltd. issued 100 7.5% debentures of Rs.
100 each at a discount of 5% and 1,800 9% preference shares of Rs. 10 each at par,
which were fully paid.

You are required (i) to prepare the Financial Position of Saddam Ltd. showing the position after
the completion of purchase, (ii) to prepare statement showing cash paid and received by
Saddam Ltd and the liquidator of Madan Ltd. and (iii) to prepare the shareholders account in
the books of Madan Ltd.
(Inter Dec. 2007 Q1-20 Marks)

Answer:

Saddam Ltd.
Statement of Financial Position
as on 31st July 1995
Liabilities Amount Rs. Assets Amount Rs.

© The Institute of Chartered Accountants of Nepal 332


CAP II Paper 1: Advanced Accounting

Equity Share Capital of Goodwill 2,000


Rs. 10 each fully paid up 98,790 Patents 2,500
Plant 38,000
9% Preference Share Furnitures & Fittings 5,300
Capital of Rs. 10 each 30,000 Stock 59,450
fully paid up Debtors 25,180
Cash 5,860
7.5% Debentures of Rs. Discount on Issue of 500
100 each 10,000 Debentures
Total 1,38,790 Total 1,38,790

Cash Account
In the books of Saddam Ltd
Dr Cr
Particulars Amount Rs. Particulars Amount
Rs.
To 9% Preference Share By Liquidator of Madan Ltd. 13,100
Capital A/C 18,000
By Liquidator of Mohan Ltd. 8,540
To 7.5% Debentures A/C 9,500
By Balance c/d 5,860

Total 27,500 Total 27,500

Cash Account
In the books of Madan Ltd
Dr Cr
Particulars Amount Rs. Particulars Amount
Rs.
To Saddam's A/C 13,100 By Realization A/C 500
(Liquidation Exp.)

By Bank Overdraft A/c 6,050

By Creditors A/c 6,550


Total 13,100 Total 13,100

Shareholders Account
In the books of Madan Ltd
Dr Cr

© The Institute of Chartered Accountants of Nepal 333


CAP II Paper 1: Advanced Accounting

Particulars Amount Rs. Particulars Amount


Rs.
To Accumulated Reserves 29,420 By Equity Share Capital A/C 1,00,000
To Realization A/c 16,090
(Realization Loss) By Export Profit A/C 10,000
To Equity Share of 64,490
Saddam Ltd.
Total 1,10,000 Total 1,10,000

Working Notes:

(i) Calculation of Purchase Consideration


Madan Ltd. Mohan Ltd.
(Rs.) (Rs.)
Goodwill 1000 1000
Patents 500 2000
Plant 27,000 11,000
Furniture & Fitting 3000 2300
Stock 42,460 16,990
Debtors 15,630 9,550
Total Purchase Consideration 89,590 42,840

(ii) Settlement of Purchase Consideration


Madan Ltd. Mohan Ltd.
(Rs.) (Rs.)
9% Preference Shares of Saddam Ltd 12,0000
(1,200 shares of Rs. 10 each)
Rs. 10 Equity Shares of Saddam Ltd 64,490 34,300
Cash 13,100 8,540
Total Purchase Consideration 89,590 42,840

Realization Account
In the books of Madan Ltd
Dr Cr
Particulars Amount Rs. Particulars Amount Rs.
To Plant 40,000 By Saddam Ltd.’s A/c 89,590
(Purchase consideration)
To Furniture & Fittings 4,600
By Creditor's A/C 10
To Patents 2,500 (Short amount of Pmt.)

To Stock 42,460 By Shareholder's A/C 16,090


(Realization Loss)

© The Institute of Chartered Accountants of Nepal 334


CAP II Paper 1: Advanced Accounting

To Debtors 15,630

To Cash – Liquidation 500


Exp.
Total 105,690 Total 1,05,690

(iii)
Creditor A/C
In the Books of Madan Ltd.
Particulars Amount Rs. Particulars Amount Rs.
To 9% preference share of By balance b/d 18,560
Saddam Ltd. 12,000
To Cash 6,550
To Realisation 10
Total 18560 Total 18,560

18. The Financial Position of B. Ltd and C Ltd. as on 31st December 2004 were as follows:

B Ltd. C Ltd. B Ltd. C Ltd.


Rs. Rs Rs. Rs
50,000 12% Preference Goodwill - 150
shares of Rs.100 each 5,000 - Land & Building 7,400 -
15,00,000 Equity Shares of Plant & Machinery 16,380 -
Rs.10 each 15,000 - Furniture 270 500
400,000 Equity Shares of Patents 600 -
Rs.10 each - 4,000 Motor Vehicles - 705
Capital Reserve 4,800 - Stock 4,050 2,600
General Reserve 3,500 1,000 Debtors 800 1,290
Accumulated Profit 600 150 Cash at Bank 100 155
Creditors 700 250
29,600 5,400 29,600 5,400

A new company, D Ltd. was formed with an authorized capital of Rs. 4 crores divided into
50,000 preference shares of Rs.100 each and 35, 00,000 equity shares of Rs.10 each. B. Ltd.
and C. Ltd. merged into D Ltd. on the following terms:-
i. D Ltd. allotted to B Ltd. 50,000, 13% fully paid preference shares and 20 lakh fully paid
equity shares to satisfy the claims of B Ltd.’s preference shareholders and equity
shareholders respectively.
ii. D Ltd. allotted to C Ltd. 4, 40,000 fully paid equity shares to be distributed among C
Ltd.’s shareholders in full satisfaction of their claims.
iii. Mr. D. was allotted 5,000 fully paid equity shares in consideration of his services for
making this merger possible. The company debited the amount to Preliminary Expenses
Account.

© The Institute of Chartered Accountants of Nepal 335


CAP II Paper 1: Advanced Accounting

iv. Expenses on the liquidation of B Ltd. and C Ltd. totaled Rs.3, 000 and were borne by D
Ltd.

D Ltd. made a public issue of 2 lakh equity shares of Rs.10 each at a premium of Rs.2 per
share. The issue was underwritten at a commission of 2½% on the issue price of the shares.
The issue was fully subscribed for by the public. D Ltd. paid Rs.85,000 in cash as its
preliminary expenses.

Show important Ledger Accounts to close the books of B Ltd. pass journal entries in the books
of D Ltd. and prepare D Ltd.’s Financial Position immediately after all the above mentioned
transactions have been recorded.
(Inter Dec. 2005 Q1-20 Marks)
Answer:
B Ltd.’s Ledger
Realization Account
Dr Cr
Rs.(‘000) Rs. (‘000)
To Land and Buildings 7,400 By creditors 700
To Plant and Machinery 16,380 By D Ltd.
To Furniture 270 - Consideration 25,000
To Patents 600 By Equity Shareholders A/c
To Stock 4,050 - Transfer to loss 3,900
To Debtors 800
To Bank 100
29,600 29,600

D Ltd.
Dr Cr
Rs.(‘000) Rs. (‘000)
To Realization Account - 25,000 By 13% Preference shares in D Ltd. 5,000
Consideration By Equity Shares in D Ltd 20,000
25,000 25,000

13% Preference Shares in D Ltd.


Dr Cr
Rs.(‘000) Rs. (‘000)
To D Ltd 5,000 By Preference Shareholders Account 5,000
5,000 5,000

Preference Shareholders Account


Dr Cr
Rs.(‘000) Rs. (‘000)

© The Institute of Chartered Accountants of Nepal 336


CAP II Paper 1: Advanced Accounting

To 13% Preference Share in 5,000 By 12% Preference Shares Capital t 5,000


D Ltd-settlement
5,000 5,000

Equity Shares in D Ltd.


Dr. Cr
Rs.(‘000) Rs. (‘000)
To D Ltd. 20,000 By Equity Shareholder Account 20,000
20,000 20,000

Equity Shareholders Account


Dr. Cr
Rs.(‘000) Rs. (‘000)
To Realization Account – loss 3,900 By Equity Share Capital
To Equity Shares in D Ltd- 20,000 Account 15,000
settlement By Capital Reserve 4,800
By General Reserve 3,500
By Profit and Loss Account 600
23,900 23,900

D Ltd.’s Journal

Rs.(‘000) Rs.(‘000)
Business Purchase Account 29,400
To Liquidator of B. Ltd 25,000
To Liquidator of C. Ltd 4,400
(Consideration payable for business of B Ltd. and C Ltd.)

Land and Buildings 7,400


Plant and Machinery 16,380
Furniture 770
Patents 600
Motor Vehicles 705
Stock 6,650
Debtors 2,090
Bank 255
To Creditors 950
To Gain on Business Purchase 4,500
To Business Purchase Account 29,400
(Incorporation of Assets, Liabilities and Reserves of B Ltd and C Ltd.;
Rs.5400 the excess of total consideration paid for the two companies
being adjusted against the balances of General Reserve and Profit and

© The Institute of Chartered Accountants of Nepal 337


CAP II Paper 1: Advanced Accounting

Loss Account and then against the balance of Capital Reserve (as
explained in Working Note)

Liquidator of B Ltd. 25,000


Liquidator of C Ltd. 4,400
To 13% Preference Share Capital Account 5,000
To Equity Share Capital Account 24,400
(Allotment of 50,000 13% Preference Share of Rs.100 each and
20,00,000 Equity Share of Rs.10 each of B Ltd and of 4,40,000 Equity
Shares of Rs.10 each to C Ltd. in discharge
Of consideration of their business)
2,400
Bank Account 2,400
To Equity Share Applications and Allotment Account
(Receipt of Application money @ Rs.12 per share in respect of 2 lakh
Equity Shares of Rs.10 each issued at premium of Rs.2 per share)
2,400
Equity Share Applications an Allotment Account 2000
To Equity Share Capital Account 400
To Share Premium Account
(Allotment of 2 lakh Equity Shares of Rs.10 each at a
Premium of Rs.2 per share)
60
Underwriting Commission Account 60
To Underwriter’s
Underwriting commission @ 2½% on the issue price of the shares
underwritten payable to underwriters.)
60
Underwriters 60
To Bank account
(Payment of underwriting commission to underwriter.)
50
Preliminary Expenses Account 50
To Equity Share Capital Account
Allotment of 5,000 fully paid equity shares of Rs.10 each to Mr. D in
consideration of his services.)
85
Preliminary Expenses A/c 85
To Bank Account
(Preliminary Expenses paid in cash)
3
Expenses on Liquidation of B ltd. and C Ltd. Account 3
To Bank Account

© The Institute of Chartered Accountants of Nepal 338


CAP II Paper 1: Advanced Accounting

(Payment of expenses incurred on Liquidation of B Ltd.


And C Ltd.)
3
Gain on Business Purchase 3
To Expenses on Liquidation of B Ltd. and C Ltd Account
(Transfer of Expenses on Liquidation of B Ltd. and C
Ltd)
60
Equity A/c 60
To Underwriting Commission Account
(Underwriting commission netted from equity)

Financial Position of Sheet of D Ltd. as at……………

Liabilities Rs Assets Rs.


(‘000) (‘000)

Share Capital :
Authorized :
50,000 Preference Shares Land and Building 7,400
of Rs.100 each 5,000 Plant & Machinery 16,380
35,00,000 Equity Shares of Rs.10 each Furniture 770
35,000 Patents 600
40,000 Motor Vehicles 705
Issued and Subscribed:
50,000 13% Preference Shares of Rs.100 Current Assets, Land and
each, fully paid. 5,000 Advances
(A) Current Assets
26,45,000 Equity Shares of 10 each fully
paid 26,450
Less: Issue Costs (60) Stocks 6,650
(of the above shares, all the preference
shares and 24,45,000 equity shares have
been allotted as fully paid up pursuant to Debtors 2,090
contractors without payments being
received in cash)

Reserves and Surplus


Securities Premium 400
Gain on Business Purchase 4497
General Reserve (Preliminary Expenses) Cash at Bank 2,507
Current Liabilities and Provisions (135) (B) Loan and Advances Nil

© The Institute of Chartered Accountants of Nepal 339


CAP II Paper 1: Advanced Accounting

Current Liabilities 950


Creditors Nil
(B) Provisions
37,102 37,102
* Preliminary expenses may be netted off against gain on bargain purchase and can be
shown as retained earnings with adequate disclosure in the financial statements.

Working Notes:

(i) Calculation of gain on business purchase


B C
Purchase Consideration 25,000 4,400
Less: Net Assets Acquired 28,900 5,000
3,900 600
4,500

(ii) Cash Book (Bank Columns Only)


Dr Cr
Rs.(‘000) Rs.(‘000)
To Business Purchase 255 By Underwriters 60
Account
By Preliminary Expenses 85
To Equity Share Applications 2,400 Account
and Allotment Account
By Expenses on Liquidation 3
of B Ltd. and C Ltd. Account

By Balance c/d 2,507


2,655 2,655
To Balance c/d 2,507

(iii)

Rs.(‘000)
Amount paid to D Ltd in the form of shares 50
Preliminary expenses paid in cash 85
Total Preliminary expenses 135

19. Arun Ltd has been in operation, for any many years. However, due to complexities in operating
business, it decided to sell the business to Barun Ltd. as on December 31, 2004. On that date
its Financial Position was as under:

© The Institute of Chartered Accountants of Nepal 340


CAP II Paper 1: Advanced Accounting

Rs Rs
200, 000 shares of Re. 1 each 200,000 Freehold property 150,000
9% Debentures 100,000 Plant & Machinery 15,000
Sundry Creditors 30,000 Stock in Trade 35,000
Reserve Fund 50,000 Sundry Debtors 40,000
Retained Earnings 20,000 Bills Receivable 20,000
Goodwill 40,000
Cash 100,000
Total 400,000 400,000

Barun Ltd. agreed to take over the assets (exclusive of cash and goodwill) at the amount stated
in the SFP, to discharge the liabilities to the creditors, and to pay Rs.100, 000 towards
Goodwill.

The purchase price was to be discharged by the allotment of 120,000 shares of Re.1 each in the
capital of the Barun Ltd. at Rs.1.25 per share, and the balance in cash. Arun Ltd went into
voluntary liquidation. The expenses of liquidation amounted to Rs. 3,000

Write up necessary Ledger Accounts to close the books of Arun Ltd. and give the initial journal
entries in the books of the Barun Ltd. Necessary workings should form part of your answer.
(Inter Jun. 2005 Q2 -16 Marks; Inter Jun. 2003 Q2 -8 Marks)

Answer:

Calculation of purchase consideration Rs


Goodwill (specially agreed value) 100,000
Freehold Property 150,000
Plant and Machinery 15,000
Stock in Trade 35,000
Sundry Debtors 40,000
Bills Receivable 20,000
360,000

Books of Arun Ltd. (Vendor)


Realization Account
To Sundry Assets (taken By Barun Co. Ltd
over by the vendor) - Purchase Consideration 360,000
Free hold property 150,000
Plant and Machinery 15,000
Stock in trade 35,000
Sundry debtors 40,000
Bills receivable 20,000
Goodwill 40.000

© The Institute of Chartered Accountants of Nepal 341


CAP II Paper 1: Advanced Accounting

To Reconstruction 60,000
360,000 360,000

Reconstruction Account
To cash : By Realization :
Liquidation expenses incurred 3,000 Gain on transfer of business 60,000
on direct discharge of creditors By Reserve Fund 50,000
etc. By Retained Earnings 20,000
To shareholders 127,000
130,000 130,000

Cash Account
To balance b/d 100,000 By 9% Debentures 100,000
To Barun Co. Ltd 210,000 By Sundry Creditors 30,000
By Reconstruction 3,000
(Liquidation expenses)
By shareholders 177,000
310,000 310,000

Barun Co. Ltd., (Vendee)


To Realization : 360,000 By shares in Barun Co. ltd. 150,000
Purchase consideration By cash 210,000

360,000 360,000

Shareholders’ Account
To shares in Barun Co. Ltd. 150,000 By share capital 200,000
To cash 177,000 By Reconstruction 127,000

327,000 327,000

Journal of Barun Co. Ltd (Vendee)


Debit Credit
Goodwill 100,000
Freehold Property 150,000
Plant and Machinery 15,000
Stock in Trade 35,000
Sundry Debtors 40,000
Bills Receivable 20,000
To Arun Co. Ltd. (Vendor) 360,000
(Towards acquisition of the assets of Arun co. Ltd)

© The Institute of Chartered Accountants of Nepal 342


CAP II Paper 1: Advanced Accounting

Arun Co. Ltd (Vendor) 360,000


To Share Capital 120,000
To Share Premium 30,000
To Cash 210,000
(Towards discharge of Purchase consideration)

20. X Ltd. takes over Y Ltd. on Shrawan 01, 2069 and discharges consideration for the
business as follows:
a. Issued 042,000 fully paid equity shares of Rs. 10 each at par to the equity shareholders
of Y Ltd.
b. Issued fully paid up 15% preference shares of Rs. 100 each to discharge the preference
shareholders (Rs. 170,000) of Y Ltd. at a premium of 10%.
c. It is agreed that the debentures of Y Ltd. (Rs. 50,000) will be converted into equal
number and amount of 13% debentures of X Ltd.
Calculate the purchase consideration of X Ltd. (CAP Dec. 2012 Q3b ii-3 Marks)

Answer
Particulars Rs.
Equity Shares(42000*10) 420,000
Preference Share Capital 170,000
Add: Premium On redemption 17,000 187,000
Purchase Consideration 607,000

Note: There is no relevance of (c)

© The Institute of Chartered Accountants of Nepal 343


CAP II Paper 1: Advanced Accounting

3.4.2 Reconstruction

A. Theoretical Questions

B. Practical Questions

1. The paid-up capital of Tele Ltd. amounted to Rs.250,000 consisting of 25,000 equity shares of
Rs.10 each. Due to losses incurred by the company continuously, the directors of the company
prepared a scheme for reconstruction which was duly approved by the court. The terms of
reconstruction were as under:
In lieu of their present holdings, the shareholders are to receive:
Fully Paid Equity Shares equal to 2/5th of their holding.
5% Preference Shares fully paid-up to the extent of 20% of the above new equity shares.
3,000 6% Second Debentures of Rs.10 each.

An issue of 2,500 5% First Debentures of Rs.10 each was made and fully subscribed in cash.

The assets were reduced as follows:


Goodwill from Rs.150,000 to Rs.75,000.
Machinery from Rs. 50,000 to Rs.37,500.
Leasehold Premises from Rs.75,000 to Rs. 62,500.
Required:
Show the journal entries to give effect to the above scheme of reconstruction.
(CAP Dec. 2010 3b (i) -7 Marks)

In the books of Tele Ltd.


Journal Entries
Rs. Rs.
Share Capital A/c (Old) Dr. 250,000
To Equity Share Capital A/c (2/5 of Rs. 2,50,000) 100,000
To 5% Pref. Shares Capital A/c (20/100xRs.1,00,000) 20,000
To 6% Second Debentures A/C 30,000
To Capital Reduction A/c 100,000
(Being conversion of 25,000 Equity Shares and balance being transferred to Capital
Reduction A/c in accordance with the Scheme of internal reconstruction as per Special
Resolution dated ----- as confirmed by the Court Order dated---)

Bank A/c Dr. 25,000


To 5% First Debenture A/c 25,000
(Being issue of Rs.25,000 5% First Debentures for cash as per scheme of internal
reconstruction)
Capital Reduction A/c Dr. 100,000

© The Institute of Chartered Accountants of Nepal 344


CAP II Paper 1: Advanced Accounting

To Goodwill A/c 75,000


To Plant & Machinery A/c 12,500
To Leasehold Premises A/c 12,500
(Sundry Assets written down as per scheme of internal reconstruction)

* The company will need to further assess impairment of assets and goodwill as continuous
losses are indication of impairment of assets under NAS 36.

2. The following is the Financial Position as at 1st January of Everest Ltd.


Liabilities Rs Assets
Subscribed Capital: Non-Current Assets 1,080,000
(Inc. Goodwill 100,000)
3000 cum, pref, Share of Rs. 100 each 300,000 Investments 20,000
7500 Equity Shares of Rs.100 each 7,550,000 Stock in Trade 200,000
Securities Premium on Preference 12,000 Trade Debtors 154,500
Share
General Reserve 80,000 Bank Balance 62,500
Trade Creditors 375,000
Total 1,517,000 Total 15,517,000
Contingent Liability: Preference Dividends In arrears Rs.66,000.

The Board of Directors of the Company decided upon the following Scheme of Reconstruction
i) Preference Shares are to be converted into 13% unsecured Debentures of Rs. 100 each in
regard to 80% of the dues (including arrears of dividends), and for the balance, Equity
Shares of Rs.50 paid up issued. The Authorized Capital of the Company permitted the
Issue of Additional Shares.
ii) Equity Shares would be reduced to share of Rs.50 each paid up
iii) All Equity Shareholders agree to pay the balance in Cash.
iv) Goodwill has lost its value and is to be written off fully. Investments are to reflect their
market value of Rs. 30,000. Obsolete items in stock of Rs. 50,000 are to be written off.
Bad Debts to the extent of 5% of the Total Debtors would be provided for. Other Non-
Current Assets to be written down By Rs. 150,000.
The Scheme was duly approved and put into effect.
The Company carried on trading for six months and after writing off Depreciation at 20% p.a.
on the revised value of Non-current Assets and made a Net profit of Rs.80,000. The half- yearly
working resulted in an increase of Sundry Debtors by Rs.60,000. Stock by Rs.80,000 and cash
by Rs. 40,000.

Show the Journal Entries necessary in the Company's books to give effect to the scheme and
draw the Statement of Financial Position as at 30th June
(Inter. Dec 10 Q5a-15 Marks)

© The Institute of Chartered Accountants of Nepal 345


CAP II Paper 1: Advanced Accounting

Answer
a) 1 Computation of number of Equity Shares to be issued afresh
Amount due to preference Shareholders = Capital + Arrears Dividend (300000 + 366,000
66000) 292,800
Amount to be settled by way of 13% Unsecured Debentures at 80% of the above 73,200
Balance Amount (i.e.20%) to be settled by issuing Equity shares 1,464
Number of Equity Shares of Rs.100 each to be issued as Rs.50 paid up = Rs.73200/ shares
Rs.50

2 Computation of Cash /Bank balance after six mounts


Receipts Rs. Payments Rs.
To balance b/d – given 62,500
To Equity Share Capital – Call money of 448,200
Rs.50 on (1464 + existing 7500) share
To Addition to Cash Balance due to six 40,000 By Balance c/d (after six 550,700
months operations – given months )
Total 550,700 Total 550,700

3 Journal Entries in the books of Everest Ltd


S.No Particulars Dr. Cr.
1 Cumulative Preference Share Capital A/c Dr. 300,000
Loss on extinguishment of financial liability A/C Dr. 66,000
To 13% Unsecured Debentures A/c 292,800
To Equity Share Capital A/c 73,200
(Being old Financial liability extinguished and new financial liability & equity
recognized – refer WN 1)
2 Equity Share Capital A/c Dr. 375,000
To Capital Reduction A/c 375,000
(Being Equity Share of Rs.100 each reduced as Rs.50 paid up-presently 7500 Equity
Shares at Rs.50 per share as per scheme.)
4 Cash /bank A/c Dr. 448,200
To Equity Share Capital A/c 448,200
(Being call money of Rs.50 each for (1464 + 7500)= 8964 Equity Shares, including
those allotted to preference Shareholders as per scheme)
5 Investments A/c 10,000
Capital Reduction (Balancing figure) 297,725
To Goodwill A/c 100,000
To Stock A/c 50,000
To Other Non-Current Assets A/c 150,000
To Debtors A/c 7,725
( Being the change in value of assets as per Scheme, now recorded)
6 Capital Reduction A/c 11,275
To General Reserve A/c 11,275

© The Institute of Chartered Accountants of Nepal 346


CAP II Paper 1: Advanced Accounting

(Being the transfer of Capital Reduction A/c balance – see WN 4 below)

4 Capital Reduction Account


Particulars Rs Particulars Rs
To Cum. pref. Shareholders – 66,000 By Equity Share capital – 375,000
Dividend Arrears Reduction
To Sundry Assets – w/off – refer J.E 5 297,725
To General Reserve – balancing 11,275
figure
Total 375,000 Total 375,000

5 Statement of Financial Position of Everest Ltd as at 30th June


Liabilities Rs. Assets Rs.
Share Capital : 8,964 Equity 896,400 Non-Current Assets: Gross * 747,000
Shares of Rs. 100 each, fully paid 830,000
up Less : Depreciation 83,000
Securities premium (given) 12,000 Investments (at Market Value) 30,000
General Reserve (80,000+11,275) 91,275 Current Assets
Profit and Loss A/c (6 months 80,000 Stock in trade (200,000 – 50,000 + 230,000
Profit ) 80,000)
13% Unsecured Debentures 292,800 Debtors (154,500 + 60,000-7,725) 206,775
Trade Creditors (balancing figure) 392,000 Cash and Bank Balances (WN 2) 550,700
Total 1,764,475 Total 1,764,475

*Revised Gross Blok of Non-current Assets= 1,080,000 – Goodwill 100,000 – Reduction


150,000 = 830,000

6 Determination of Trade Creditors figure


Trade Creditors has been determined as Balancing Figure. Alternatively, it can be computed
independently, in the following manner
Profit upto30th June – given 80,000
Add: Depreciation ( Non Cash Item ) 83,000
Cash from Operations 163,000
Less: Change in Current Assets (i.e. Increase in Assets represent cash
outflow)
Debtors (60,000)
Stock (80,000)
Cash Balance (40,000)
Net Cash outflow from operations – representing an increase in creditors 17,000
Add: Opening Balance of Creditors 375,000
Revised Balance of Creditors 392,000

© The Institute of Chartered Accountants of Nepal 347


CAP II Paper 1: Advanced Accounting

3. Grand Limited is in the hands of Receiver, Mr. L for debenture holders who hold a charge on
all the assets except uncalled capital. The following statement shows the position as regards to
the creditors as on 31st Ashadh 2072:

Liabilities Amount Assets Amount


(Rs.) (Rs.)
Share Capital Rs.360,000 in - Cash in hand of the
shares of Rs.60 each, Rs.30 receiver, Mr. L 270,000
paid up Property, machinery and
First debentures 300,000 Plant etc. cost Rs.390,000
Second debentures 600,000 estimated at 150,000
Unsecured creditors 450,000 Charged under debenture 420,000
Uncalled capital 180,000
Deficiency 750,000
1,350,000 1,350,000
Mr. A holds first debentures of Rs.300,000 and second debentures of Rs.300,000. He is also
an unsecured creditor for Rs.90,000. Mr. B holds second debenture of Rs.300,000 and is an
unsecured creditor for Rs.60,000.
The following scheme of reconstruction is proposed and approved:
a) Mr. A is to cancel Rs.210,000 of the total debt owing to him, to advance Rs.30,000 in cash
and to take first debentures (in cancellation of those already issued to him) for Rs.510,000
in settlement of all his claims.
b) Mr. B is to accept Rs.90,000 in cash in satisfaction of all claims by him.
c) Unsecured creditors (other than Mr. A and Mr. B) are to accept four shares of Rs.7.5 each
fully paid in satisfaction of 75% of every Rs.60 of their claim. The balance of 25% is to be
postponed and to be payable at the end of three years from the date of the Court’s approval
of the scheme. The nominal share capital is to be increased accordingly.
d) Uncalled capital is to be called up in full and Rs.52.50 per share cancelled, thus making
the shares of Rs.7.5 each.
Assuming that the company keeps sectional ledgers, give necessary journal entries and the
balance sheet of the company before and after the scheme has been carried into effect.
(CAP Dec 15 Q1-20 Marks)
Answer:
Before passing the necessary journal entries, it would be better to prepare the SFP since the
statement does not provide clear position. It does not include figures for share capital and, on
the assets side, it includes “Uncalled Capital” which should not be included.

Financial Position as on 31st Ashadh 2072 (before Reconstruction)


Equity & Liabilities Amount Assets Amount
Share Capital 6,000 shares of 1,80,000 Cash in hand of the Receiver, 2,70,000
Rs.60 each, Rs.30 paid up Mr. L
First debentures 3,00,000 Property, Machinery and Plant
Second Debentures 6,00,000 at cost (Refer Working Note 1) 3,90,000

© The Institute of Chartered Accountants of Nepal 348


CAP II Paper 1: Advanced Accounting

Unsecured Creditors 4,50,000 Deficiency (Acc. Losses) 8,70,000


15,30,000 15,30,000

Journal Entries
First Debentures Dr. 3,00,000
Second Debentures Dr. 3,00,000
Unsecured Creditors A/c Dr. 90,000
To Mr. A’s A/c 6,90,000
(Total amount due to Mr. A transferred to his account)
Second Debentures A/c Dr. 3,00,000
Unsecured Creditors A/c Dr. 60,000
To Mr. B’s A/c 3,60,000
(Total amount due to Mr. B transferred to his account)
Bank A/c Dr. 30,000
To Mr. A’s A/c 30,000
(The amount paid by Mr. A under the approved scheme of
reconstruction.)
Mr. A’s A/c Dr. 7,20,000
To First Debentures A/c (New) 5,10,000
To Gain on settlement of financial liability A/c 2,10,000
(The issue of First Debenture worth Rs.5,10,000 to Mr. A in full
satisfaction of his claims)
Mr. B’s A/c Dr. 3,60,000
To Bank A/c 90,000
To Gain on settlement of financial liability A/c 2,70,000
(The payment of Rs.90,000 to Mr. B in full satisfaction of his claims
in accordance with the scheme)
Unsecured Creditors A/c Dr. 2,25,000
To Share Capital A/c 1,50,000
To Gain on settlement of financial liability A/c 75,000
(The issue of Rs.7.50 fully paid shares to unsecured creditors in
accordance with the sanctioned scheme, Refer Working Note 2)
Bank A/c Dr. 1,80,000
To Share Capital A/c 1,80,000
(The balance on shares (Rs.30 per share) called up in accordance
with the scheme.)
Share Capital A/c Dr. 3,15,000
To Reconstruction A/c 3,15,000
(The cancellation of Rs.52.50 per share on 6,000 shares in
accordance with the scheme.)
Gain on Settlement of Financial Liability A/c……………….Dr. 4,55,000
Reorganization A/c Dr. 3,15,000
To Accumulated Losses A/c 8,70,000

© The Institute of Chartered Accountants of Nepal 349


CAP II Paper 1: Advanced Accounting

Statement of Financial Position as on 31st Ashadh 2072 (After Reconstruction)


Liabilities Amount Assets Amount
Share Capital 26,000 shares of 1,95,000 Fixed Assets:
Rs.7.5each fully paid up Property, Machinery and Plant 3,90,000
Secured Loans at cost
First debentures 5,10,000 Current Assets:
Current Liabilities and Provisions Cash and Bank Balance 3,90,000
Sundry Creditors 75,000
7,80,000 7,80,000
Working Note:
1. Property plant and equipment has been shown at cost rather than realizable value as no
value in use is provided for us to be able to determine impairment loss, if any.
2. Four shares of Rs.7.5 fully paid (Rs.30 in all) given to those who have claim of Rs.60 each
i.e. 50% is being paid in the form of shares and 25% is being cancelled and 25% being
postponed. Hence against the total claim of Rs.3,00,000 (other than Mr. A and Mr. B),
Rs.1,50,000 will be given in the form of shares, Rs. 75,000 will be cancelled and balance
Rs.75,000 will be postponed. No entry is required for postponement.

4. The Financial Position of Unstable Company Ltd. as on July 16, 2003 is:

Liabilities Rs. Assets Rs.

Share Capital Freehold Land & 34,000


Building
Authorized, Issued and Fully Paid: Plant 96,000
10,000, 9% Cumulative Preference Tools & Dies 27,300
Share of Rs. 10 each 1,00,000 Investments 15,000
15,000 Equity Shares of Rs. 10 each 1,50,000 Stock 42,500
10% Debenture 60,000 Debtors 53,400
Interest due 4,200 64,200 Research & Development
Bank overdraft 20,000 Expenditure 18,000
Creditors 50,000 Retained Earnings 98,000
3,84,200 3,84,200
It was decided to reconstruct the company for which necessary resolution was passed and
sanctions were obtained after completing all legal formalities.

a. Prepare journal entries to record the transactions, in the books, including cash.
b. Prepare the SFP of the company as on July 17,2003, after completion of the scheme.
i. The following assets are to be revalued as:
Rs.
Plant 59,000
Tools & Dies 15,000

© The Institute of Chartered Accountants of Nepal 350


CAP II Paper 1: Advanced Accounting

Stock 30,000
Debtors 48,700
ii.The research and development expenditure and debit balance of retained earnings are to be
written off.
iii.Piece of land recorded in the books at Rs. 6,000 is valued at Rs. 14,000 and is to be taken
over by the debenture holders in part payment of principal. The remaining freehold land and
building is to be valued at Rs. 40,000.
iv.A creditor of Rs. 18,000 has agreed to accept a second mortgage debenture of 11% per
annum (secured on plant) for Rs. 15,500 in settlement of his debt. Other creditors totaling Rs.
10,000 agree to accept a payment of Rs. 0.85 in the rupee for immediate settlement.
v. The entire investments are to be taken over by the bank at valuation of Rs. 22,000.
vi.The ascertained loss is to be met by writing down the equity shares to Rs. 1 each and
preference shares to Rs. 8 each.
vii.The equity shareholders agree to subscribe for two new ordinary shares at par for every
share held. The cash is all received.
viii.The costs of the scheme amounted to Rs. 3,500. These have been paid and to be written
off. Debenture interest has also been paid.
(Inter Dec. 2003, Q 4-16 Marks)
Answer:
Reconstruction Account Dr. 1,82,500
To Plant Account 37,000
To Tools and Dies Account 12,300
To Stocks Account 12,500
To Debtors 4,700
To Research & Dev. Expenditure Account 18,000
To Retained Earnings 98,000
(Value of various assets written down and elimination of
fictious assets as per scheme of reconstruction)

Freehold Land and Building Account Dr. 20,000


Investment Account Dr. 7,000
To Reconstruction Account 27,000
(Increment in the value of assets as per reconstruction
scheme.)

Creditors Account Dr. 28,000


To 11% Second Mortgage Debentures Account 15,500
To Bank Account 8,500
To Reconstruction Account 4,000
(The discharge of part of the creditors.)

10% Debentures Account Dr. 14,000


To Freehold Land & Building Account 14,000

© The Institute of Chartered Accountants of Nepal 351


CAP II Paper 1: Advanced Accounting

(Claims of debenture holders in part, in respect of principles


discharged by way of transfer of freehold property)

Bank Account Dr. 22,000


To Investment Account 22,000
(Investment taken over by bank)

Equity Share Capital A/c (Rs. 10) Dr. 1,50,000


Preference Share Capital A/c (Rs. 10) Dr. 1,00,000
To Equity Share Capital A/c (Rs. 1) 15,000
To Preference Share Capital A/c (Rs. 8) 80,000
To Reconstruction Account 1,55,000
(The writing down of equity and preference share)

Bank Account Dr. 30,000


To Equity Share Capital 30,000
(30,000 equity shares issued at Rs. 1 each)

Reconstruction Account Dr. 3,500


Outstanding Interest on Deb. A/c Dr. 4,200
To Bank Account 7,700
(Payment of outstanding interest and reconstruction expenses)

Working Notes:
Reconstruction Account

Rs. Rs.

To Plant 37,000 By Freehold Land & 20,000


Building
To Tools and Dies 12,300 By Investment Account 7,000
To Stocks 12,500 By Creditor 4,000
To Provision for bad debts 4,700 By Equity Share Capital 1,35,000
To Research & Development By Preference Share Capital
Expenditure 18,000 20,000
To Retained Earnings 98,000
To Bank Account 3,500 _______
1,86,000 1,86,000

Appreciation in the value of Land & Building is computed as follows:


Rs.
Increase in the value of piece of land taken over by debenture holders 8,000
Increase in the value of remaining portion 12,000
20,000

© The Institute of Chartered Accountants of Nepal 352


CAP II Paper 1: Advanced Accounting

5. Following is the Financial Position of KLM Ltd. for the year ending 31-3-2009

KLM Ltd
Balance Sheet as at 31-03-2009
Liabilities NNRs.. Assets NNRs..
Share capital Fixed assets 2,000,000
5,000 equity shares of NRs. 100 each500,000
2000 10% preference Investment 500,000
share of NRs. 100 each 200,000

Current assets 1,750,000


Secured loans
12.5% debentures (NRs. 100 each) 1,400,000 Profit & Loss Account 750,000
Long term loan 1,500,000

Current Liabilities
Director Loan 200,000
Creditors 1,000,000
Provision for taxation 200,000
Total 5,000,000 Total 5,000,000

Due to decline in sales and adverse market conditions for the past few years the
company is facing heavy losses. The management decided to restructure the company
on 01-04-2009 on the following terms and conditions:
a) Equity shares are to be reduced to equal number of equity shares of NRs. 50 each.
b) All preference capital are reduced to 12% Preference share of NRs. 75 each and the
preference shareholders agree to forego their right to dividend which are in arrears
for last three years.
c) The debenture holders agree to surrender their existing debentures and agree for
equivalent number of fresh 15% debenture of NRs. 75 each.
d) Long term loan providers agree to forego 50% of their claims and agree to take
equity shares for the balance amount.
e) Taxation liability is to be reduced to NRs. 50,000.
f) Creditors have the option to either accept 60% of their claims in cash in full
settlement or to convert their claims into equity shares of NRs.50 each. 90% of the
creditors opted for equity shares in satisfaction of their claims.
g) Fixed assets are to be written down by 25%.
h) The investments made by the company also have lost their market value. They are
to be valued at 50%.
i) Directors are forced to refund NRs. 100,000 in cash, previously taken as bonus even
when the company was at loss. Directors loan amounting to NRs. 50,000 are to be
waived and the balance is to be settled by converting into equity shares.
j) Provision for bad debts is to be maintained at NRs. 240,000
Pass journal entries, prepare reconstruction account and redraft the company's
balance sheet immediately after reconstruction.
(Inter Jun. 2009 Q1- 20Marks)

© The Institute of Chartered Accountants of Nepal 353


CAP II Paper 1: Advanced Accounting

Answer
KLM Ltd.
Journal Entries
(Recording the transactions of Reconstruction)
Particulars Debit Credit
i) Equity Share Capital (NRs. 100) A/c 500,000
To Equity Share Capital (NRs. 50) A/c 250,000
To Reconstruction A/c 250,000
(Equity Shareholders right of NRs. 100 per shares reduced to a share of NRs.50 each
vide Board Resolution dated ------, the amount of sacrifice credited to Reconstruction
Account)

Particulars Debit Credit


ii) 10% Preference share capital (NRs. 100) A/c 200,000
To 12% Pref share capital (NRs.75) A/c 150,000
To Reconstruction A/c 50,000
(10% Preference Share of NRs. 100 each converted into equals number of 12%
Preference Share of NRs. 75 each vide Board Resolution dated ------, the amount of
sacrifice credited to Reconstructive A/c)

Particulars Debit Credit


iii) 12.50% Debenture (NRs. 100) A/c 1,400,000
To 15% Debenture (NRs. 75) A/c 1,050,000
To Reconstruction A/c 350,000
(12.50% Debenture of NRs. 100 each converted into equivalent number of 15%
Debenture of NRs. 75 each vide Board Resolution dated -------, the amount of sacrifice
credited to Reconstruction A/c)

Particulars Debit Credit


iv) Long Term Loan A/c 1,500,000
To Equity Share capital (NRs. 50) 750,000
To Reconstruction A/c 750,000
(Long Term loan foregone 50% of claim credited to Reconstruction A/c and equity
share of 50 each issued to settle their claim vide Board Resolution dated ----------)

Particulars Debit Credit


v) Provision for taxation A/c 150,000
To. Reconstruction A/c 150,000
(Tax liability being settle at NNNRs..50,000 and the balance amount credited to
Reconstruction A/c as per the scheme of reconstruction dated ----)

Particulars Debit Credit


vi) Creditors A/c 1,000,000
To Equity Share Capital (NNRs..50 ) A/c 900,000
To Bank A/c 60,000
To Reconstruction A/c 40,000
(Allotment of equity shares of NRs. 50 each to 90% of creditors opting to take equity
share in satisfaction of their claim and the remaining creditors being settled by payment
of cash as per the scheme of reconstruction dated----)

© The Institute of Chartered Accountants of Nepal 354


CAP II Paper 1: Advanced Accounting

Particulars Debit Credit


vii) Reconstruction A/c 500,000
To Property Plant & Equip A/c 500,000
(Fixed assets being written down by 25% as per the scheme of reconstruction dated----
--)

Particulars Debit Credit


viii)Reconstruction A/c 250,000
To Investment A/c 250,000
(Investments being depreciated by 50% as per the scheme of reconstruction dated -----
--)

Particulars Debit Credit


ix) Directors Loan A/c 200,000
To Reconstruction A/c 50,000
To Equity share capital (NRs. 50)A/c 150,000
(Directors Loan to be settled by issue of 3,000 numbers of equity share of NRs. 50
each and the balance amount waived creditors to Reconstruction A/c as per the
scheme of reconstruction dated ------)

Particulars Debit Credit


x) Bank A/c 100,000
To Reconstruction A/c 100,000
(Bonus amounting NNRs..100,000 being refunded by directors in cash as per the
scheme of reconstruction dated ------)

Particulars Debit Credit


xi) Reconstruction A/c 240,000
To Provision for Bad Debts A/c 240,000
(Provision for bad debts being maintained at NRs. 240,000 as per the scheme of
Reconstruction dated ---)

Particulars Debit Credit


xii) Reconstruction A/c 750,000
To Profit and Loss A/c 750,000
(Debit balance in Profit and loss A/c being transferred to Reconstruction A/c as per the
scheme of reconstruction dated ------)

© The Institute of Chartered Accountants of Nepal 355


CAP II Paper 1: Advanced Accounting

KLM Ltd.
Reconstruction Account
Debit Credit
Particulars NNRs.. Particulars NNRs..
To Profit/Loss A/c 750,000 By Equity Share Cap A/c 250,000
To Property Plant & Equip A/c 500,000 By 10% Pref Share A/c 50,000
To Investment A/c 250,000 By 12.5% Deb A/c 350,000
To Provision for bad debts A/c 240,000 By Long-Term Loan A/c 750,000
By Provision for taxation A/c 150,000
By Creditors A/c 40,000
By Bank A/c 100,000
By Director's loan Ac 50,000
Total 1,740,000 Total 1,740,000

KLM Ltd.
Balance sheet (And reduced)
As on 01-04-2009
Equity & Liabilities NNRs.. Assets NNRs..
Share Capital Property Plant & Equip 1,500,000
41,000 Equity Share of NRs.50 2,050,000 Investments 250,000
each (WN- 1)
2,000 , 12% Preference Share of 150,000 Current Assets (WN 2) 1,550,000
NRs.75 each
Secured Loan
15% debentures of NRs.75 each 1,050,000

Current Liabilities
Provision for taxation 50,000
Total 3,300,000 Total 3,300,000

Working Note:
1. Computation of value of Equity Share Capital after Reconstruction NNRs..
(a) Equity –Share include to existing shareholder ( 5,000×50) 250,000
(b) Equity –Share include Long term loan holder at 50% of
their claim (1,400,000×50%) 750,000
(c) Equity Share in settlement of exiting loan 150,000
(d) Equity Share to 90% of creditor ( 9,000,000×90%) 900,000
2,050,000
2. Computation of Value of Current assets
Value of Assets through forwarded 1,750,000
Less: Provision for bad or doubtful debts (240,000)
Add: Bonus refund by Director 100,000
Less: Payment to creditors 60,000 40,000
1,550,000

6. The summarized financial position of Shyam Limited as on 32nd Ashadh, 2075 is as under:
Particulars Amount (Rs.)
Liabilities
Share Capital

© The Institute of Chartered Accountants of Nepal 356


CAP II Paper 1: Advanced Accounting

500,000 equity shares of Rs. 10 each fully paid up 5,000,000


9%, 20,000 Preference shares of Rs. 100 each fully paid up 2,000,000
Reserves & Surplus
Accumulated Loss (1,460,000)
Non-current Liabilities
10 % secured debentures 1,600,000
Current Liabilities
Interest due on debentures 160,000
Trade Payables 500,000
Loan from Directors 100,000
Bank Overdraft 100,000
Provision for Tax 100,000
Total 8,100,000
Assets
Non- Current Assets
Fixed Assets:
a. Tangible Assets
Land & Building 3,000,000
Plant & Machinery 1,250,000
Furniture & Fixtures 250,000
b. Intangible Assets
Goodwill 1,000,000
Patents 500,000
Current Assets
Trade Investments 500,000
Trade Receivables 500,000
Inventory 1,000,000
Discount on issue of Debentures 100,000
Total 8,100,000
Note: Preference dividend is in arrears for last two years.
Mr. Ram holds 60% of debentures and Mr. Shyam holds 40% of debentures. Moreover,
Rs. 100,000 and Rs. 60,000 were also payable to Mr. Ram and Mr. Shyam respectively as
trade payable.

The following scheme of reconstruction has been agreed upon and duly approved.
i) All the equity shares to be converted into fully paid equity shares of Rs. 5 each.
ii) The preference shares to be reduced to Rs. 50 each and the preference shareholders
agreed to forego their arrears of preference dividends, in consideration of which 9 %
preference shares are to be converted into 10% preference shares.
iii) Mr. Ram and Mr. Shyam agreed to cancel 50 % each of their respective total debt
including interest on debentures. Mr. Ram and Mr. Shyam also agreed to pay Rs.
100,000 and Rs. 60,000 respectively in cash and to receive new 12% debentures for
the balance amount.
iv) Persons related to trade payables, other than Mr. Ram and Mr. Shyam also agreed to
forego their 50% claims.
v) Directors also waived 60% of their loans and accepted equity shares for the balance.
vi) Capital commitments of Rs. 300,000 were cancelled on payment of Rs. 15,000 as
penalty.

© The Institute of Chartered Accountants of Nepal 357


CAP II Paper 1: Advanced Accounting

vii) Directors refunded Rs. 100,000 of the fees previously received by them.
viii) Reconstruction expenses paid Rs. 15,000.
ix) The taxation liability of the company was settled for Rs. 75,000 and was paid
immediately.
x) The assets were revalued as under:
Land & Building Rs. 3,200,000; Plant & machinery Rs. 600,000; Inventory Rs.
750,000; Trade Receivables Rs. 400,000; Furniture & Fixtures Rs. 150,000; Trade
Investments Rs. 450,000.
You are required to prepare necessary journal entries for all the above mentioned transactions
including written off of intangible assets. And also prepare Bank Account and Reconstruction
Account. (CAP Jun. 2019 Q1 20 Marks)

Answer:
Journal Entries in the books of Shyam Limited
Dr. Cr.
i. Equity Share Capital (Rs. 10 each) A/c Dr. 50,00,000
To, Equity Share Capital (Rs. 5 each) A/c 25,00,000
To, Reconstruction A/c 25,00,000
(Being conversion of 5,00,000 equity shares of Rs. 10
each fully paid into same number of fully paid equity
shares of Rs. 5 each as per the scheme of
reconstruction)
ii. 9 % Preference Share Capital (Rs.100 each) A/c Dr. 20,00,000
To, 10 % Preference Share Capital (Rs.50 each) A/c 10,00,000
To, Reconstruction A/c 10,00,000
(Being conversion of 9% preference shares of Rs. 100
each fully paid into same number of fully paid
Preference shares of Rs. 50 each and claims of
preference dividends settled as per the scheme of
reconstruction )
iii. 10% Secured Debentures A/c Dr. 9,60,000
Trade Payables A/c Dr. 1,00,000
Interest on Debentures Outstanding A/c Dr. 96,000
Bank A/c Dr. 1,00,000
To, 12% Debentures A/c 6,78,000
To, Reconstruction A/c 5,78,000
(Being Rs. 11,56,000 due to Ram, including Trade
Payables, cancelled and 12% debentures allotted for
the amount after waiving 50% as per scheme of
reconstruction )
iv. 10% Secured Debentures A/c Dr. 6,40,000
Trade Payables A/c Dr. 60,000
Interest on Debentures Outstanding A/c Dr. 64,000
Bank A/c Dr. 60,000
To, 12% Debentures A/c 4,42,000
To, Reconstruction A/c 3,82,000
(Being Rs. 7,64,000 due to Shyam, including Trade
Payables, cancelled and 12% debentures allotted for

© The Institute of Chartered Accountants of Nepal 358


CAP II Paper 1: Advanced Accounting

the amount after waiving 50% as per scheme of


reconstruction.
v. Trade Payables A/c Dr. 1,70,000
To, Reconstruction A/c 1,70,000
(Being remaining payables sacrificed 50% of their
claim)
vi. Directors Loan A/c Dr. 1,00,000
To, Equity Share Capital A/c 40,000
To, Reconstruction A/c 60,000
(Being Director's loan claim settled by issuing 12,000
equity shares of Rs. 5 each as per scheme of
reconstruction)
vii. Reconstruction A/c Dr. 15,000
To, Bank A/c 15,000
(Being payment made towards penalty of 5% for
cancellation of capital commitments of Rs. 3 lakhs)
viii. Reconstruction A/c Dr. 15,000
To, Bank A/c 15,000
(Being payment made of reconstruction expenses)
ix. Bank A/c Dr. 1,00,000
To, Reconstruction A/c 1,00,000
(Being refund of fees by directors credited to
reconstruction account)
x. Provision for Tax A/c Dr. 1,00,000
To, Bank A/c 75,000
To, Reconstruction A/c 25,000
(Being payment of tax liability in full settlement
against provision for tax)
xi. Land & Building A/c Dr. 2,00,000
To, Reconstruction A/c 2,00,000
(Being appreciation in value of land & building
recorded)
xii. Reconstruction A/c Dr. 50,15,000
To, Goodwill A/c 10,00,000
To, Patent A/c 5,00,000
To, Accumulated Loss A/c 14,60,000
To, Discount on issue of debenture A/c 1,00,000
To, Plant & Machinery A/c 6,50,000
To, Furniture & Fixture A/c 1,00,000
To, Trade Investment A/c 50,000
To, Inventory A/c 2,50,000
To, Trade Receivable A/c 1,00,000
To, Accumulated Reserve A/c (Balancing Figure) 7,75,000
(Being writing off of losses and reduction in value of
assets as per scheme of reconstruction , balance of
reconstruction account transferred to Reserve)

© The Institute of Chartered Accountants of Nepal 359


CAP II Paper 1: Advanced Accounting

Bank Account
Particulars Amount Particulars Amount
To, Reconstruction 1,00,000 By, Balance b/d 1,00,000
(Ram) 60,000 By, Reconstruction A/c- Penalty 15,000
To, Reconstruction 1,00,000 By, Reconstruction A/c- 15,000
(Shyam) Expenses 75,000
To, Reconstruction A/c By, Provision for Tax A/c 55,000
(Refund from director) By, Balance c/d
Total 2,60,000 2,60,000

Reconstruction Account
Particulars Amount Particulars Amount
To, Bank- Penalty 1,00,000 By, Equity Share Capital A/c 25,00,000
To, Bank- Expenses 30,000 By, 9%Preference Share Capital 10,00,000
To, Goodwill A/c 10,00,000 A/c 5,78,000
To, Patent A/c 5,00,000 By, Mr. Ram A/c 3,82,000
To, Profit & Loss A/c 14,60,000 By, Mr. Shyam A/c 1,70,000
To, Discount on issue of By, Trade payables A/c 60,000
debenture A/c 1,00,000 By, Director's Loan A/c 1,00,000
To, Plant & Machinery A/c 6,50,000 By, Bank A/c 25,000
To, Furniture & Fixture A/c 1,00,000 By, Provision for Tax A/c 2,00,000
To, Trade Investment A/c 50,000 By, Land & Building A/c
To, Inventory A/c 2,50,000
To, Trade Receivable A/c 1,00,000
To, Accumulated Reserve A/c 7,75,000
(Balancing Figure)

Total 50,15,000 50,15,000

© The Institute of Chartered Accountants of Nepal 360


CAP II Paper 1: Advanced Accounting

CHAPTER 4 PARTNERSHIP ACCOUNTS

A. Theoretical Questions

1. Briefly explain about Payment of Firm's Debts and Separate Debts on dissolution of a
partnership firm. (Inter Dec. 2001 Q6d -4 Marks)

Answer:
When there are joint debts due from the partnership and also any separate debts due from any
partner, firms property must be first applied in the payment of the debt of the firm and if there
is any surplus then the share of each partner in such surplus must be applied in payment of his
separate debts or paid to him, if he has no debts. So also separate property of any partner
must first be applied in payment of his separate debts, and the surplus, if any, in the payment
of the debts of the firm. Where partnership assets are insufficient to meet out the liabilities of
the partnership, the creditors of the firm can resort to the partners personal property outside
the firm provided their personal and private creditors have already been paid out of it.

2. Capitalization basis for Valuing Goodwill


(Inter Jun. 2001, Q 6a- 4 Marks)

Answer:
Under the capitalization basis the value of the business is determined by applying the normal
rate of return. If such value is higher than the capital employed in the business the difference
is the amount of goodwill.
The following steps should be adopted:
i) The normal rate of return as applicable to that type of business should be determined.
ii) Find out the average profit of the partnership firm for which goodwill is to be determined.
iii) Determine the capital employed in the partnership firm for which goodwill is to be
determined.
iv) The normal value of the business should be found out by dividing the average profit by
normal rate of return.
v) Deduct average profit employed from the normal value of the business to arrive at
goodwill.

3. Accounting of goodwill. (Inter Jun. 2004 Q6b(i) -4 Marks)

Answer:
Goodwill is the estimated value of aggregate future profit that a firm is expected to earn as
compared to other similar firms operating in the same area. The partners, who gain in terms
of profit sharing ratio, have to pay for such gain as a proportion to value of goodwill. The
partners, who lose in terms of profit sharing ratio, receive payments for the sacrifice as a
proportion to value of goodwill. It is generally argued that goodwill is accounted for in the
books of the organization that has goodwill. However, this is not allowed in accounting.

© The Institute of Chartered Accountants of Nepal 361


CAP II Paper 1: Advanced Accounting

Goodwill is accounted for only in the books of the organization that has paid a price to
purchase it. In the case of partnership firm, the value of goodwill is adjusted among the
partners whenever there is reconstitution of the firm, i.e., admission, retirement or change in
the profit sharing ratio

4. Fixed capital and fluctuating capital of partners (Inter Dec. 2006 Q5b-5 Marks)

Answer:
Partners’ capital accounts can be maintained either on ‘fixed capital system’ or on ‘fluctuating
capital system’. In case of fixed capital system, two accounts for each partner, i.e. partner’s
capital account and partner’s current account are maintained. The partner’s capital account is
credited with the original amount of capital introduced by the partners into business. It is to
be credited subsequently with extra capital introduced by the partners or debited with the
amount of capital permanently withdrawn by the partners. No other adjustments are made with
this account. The partner’s current account is maintained for making all entries relating to
interest, share of profit, drawings, etc. The balance in this account will go on fluctuating but
the balance of the capital account will remain fixed. That is why the system is termed as ‘fixed
capital system’.
In case of fluctuating capital system, only one account is maintained for each partner. This
account is termed as his ‘capital account’. All entries relating to introduction for fresh capital,
drawings, interest, profit, etc. are made in this account. The balance in this capital account,
therefore, goes on fluctuating. The system is, therefore, called as ‘fluctuating capital system

5. Briefly explain the concept of piecemeal distribution of assets in case of dissolution of a


partnership firm. (Inter Jun. 2002 Q1b- 5 Marks)

Answer:

In case of dissolution of a partnership firm, the assets are realized and the liabilities repaid in
small installments over a period of time. In such circumstances, the choice is either to distribute
whatever is collected or wait till the whole amount is collected. Usually, the first is the case. In
order to ensure that the distribution of cash among the partners is in proportion to their interest
in the partnership concern, either of the following two methods may be adopted.

i. Maximum loss method: In this method, each installment realized is considered to be the
final payment, i.e., outstanding assets and claims are considered worthless and partners
accounts are adjusted on that basis each time when a distribution is made by following
either the Garner Vs. Murray rule or the Profit-sharing ratio rule.

ii. Highest Relative Capital Method: In this method, to determine a partners share in the
available 'piecemeal' cash, partners' capitals are notionally adjusted according to their share
of profits on the basis of relatively lowest capital, called basic capital. Their actual capitals
are then compared with the notionally adjusted capitals to determine the excesses of the

© The Institute of Chartered Accountants of Nepal 362


CAP II Paper 1: Advanced Accounting

remaining partner's capitals. This elimination process is repeated until only one partner's
capital shows an excess. This ultimate excess is paid out first as it implies relatively highest
capital. Thereafter, the partner next to highest capital also participates, and so on.

6. Highest Relative Capital Method (Inter Dec. 2006 Q5d-5 Marks)

Answer:
According to this method, the partner who has the highest relative capital that is, whose capital
is greater in proportion to his profit sharing ratio, is first paid off. This method is also called
as proportionate capital method. For determining the amount by which the capital of each
partner is in excess of his relative capital, partners’ capitals are first divided by figures that are
in proportion to their profit sharing ratio; the smallest quotient will indicate the basic capital.
Having ascertained the partner who has the smallest basic capital, the amount of capital of
other partners proportionate to the profit sharing ratio of the basic capital is calculated. These
may be called as their hypothetical capitals. The amount of hypothetical capital of each partner
is then subtracted from the amount of his actual capital; the resultant figure will be the amount
of excess capital held by him. By repeating the process once or twice, as may be necessary
between the partners having excess capital, the amount by which the capital of each partner is
in excess will be ascertained. The partner with the largest excess capital will be paid off first,
followed by payment to the other or others who rank next to him until the capitals of partners
are reduced to their profit sharing ratio.

7. Super profits in partnership firms (CAP Jun. 2010 Q6a – 2.5 Marks)

Answer:
Among various basis of determining the sharing and paying for joining or sacrificing the
profits of a firm one basis is super profit method. Under this method, it is assumed that any
partner can get normal profits by joining any average firm in the market. However, focus should
be given on super profits of the firm. Such super profit implies that the profit can be earned by
a firm over and above all ordinary firms in the industry/market. This is the excess amount of
profit earned by a firm over the past years and expected to continue the same in the future.
If and only if the average profit of the firm is more than the normal profit in the market there
comes into existence the super profit or goodwill.

8. Consequences of insolvency of a partnership firms (CAP Jun. 2010 Q6e- 2.5 Marks)

Answer:
Following consequences arise when a partner is insolvent:
-The partner ceases to be a partner
-The firm is dissolved unless there is a contrary contract not to do so.
-The firm and existing partner cannot be held liable after the date of declaration of
insolvency. Similarly, the estate of insolvent partner cannot be made liable for any
subsequent act of the firm or partners thereon.

© The Institute of Chartered Accountants of Nepal 363


CAP II Paper 1: Advanced Accounting

9. Need for revaluation of partnership assets

Answer:
The partnership assets need to be revalued mainly in the following situation:
• Admission of a new partner
• Retirement or death of a partner
• Change in profit or loss sharing ratio
• Sale or dissolution of partnership

10. Concepts of capital accounting in partnership business (CAP Dec. 2010 6c-2 Marks)

Answer
There are two methods of accounting- Fixed Capital Method and Fluctuating Capital Method.
In fixed capital method, generally initial capital contributions by the partners are credited to
partners’ capital accounts and al subsequent transactions and events are dealt with through
current accounts. On the contrary, under the fluctuating capital method, no current account is
maintained. All such transactions and events are passed through capital accounts.

11. Explain the major considerations, from an accountants view point, at the time of conversion of
partnerships to a limited company.

Answer
While converting partnership firms into a company, first consideration should be given to
ascertain position of the firm up to the cutoff point. It includes ascertaining past losses by
preparing a Reconstruction Account. The amount of loss so ascertained is ultimately written
off by adjusting the same to the debit of the old partner’s capital accounts. In case creditors
agree to join, then whole or part of the accounts standing to their credit is transferred to their
capital account as agreed. If fresh capital or loan is raised, the same is to be credited to
respective parties capital or loan accounts. Other considerations may be of apportionment of
shares among partners. Partners may agree to continue, in the proposed company, in the same
profit sharing ratio as in partnership or it may be different. If net value placed on the business
is more than the capital contributed by the partners the same should be credited to respective
partner’s capital accounts in the old profit sharing ratio. Similarly, due consideration should be
given if different priority is given for repayment of capital amongst partners or new
shareholders.

B. Practical Questions

Note: Solution relating to partnership accounts have been carried forward as it were
previously solved as both NFRS and NFRS for SME are not applicable mandatorily. Also, NAS
for Micro Entities are optional and therefore not taken into account for solving the questions.

© The Institute of Chartered Accountants of Nepal 364


CAP II Paper 1: Advanced Accounting

1. Mr. A and Mr. B have been carrying on business in partnership under the name M/s X & Co.
as equal partners for the purpose of sharing profits or losses of the firm. The firm's Balance
Sheet is as shown below:

Balance Sheet of M/s X & Co. as t 31.3.2001

Liabilities Rs. Assets Rs.

Capital Accounts: Fixed Assets:


Mr. A 68,000 Goodwill 20,000
Mr. B 56,000 1,24,400 Freehold premises 41,400
Sundry Creditors 19,600 Current Assets:
Stock 28,400
Sundry Debtors 32,200
Cash on hand 22,00
1,44,000 1,44,000

It is agreed that Mr. A should retire on 1st April, 2001 and his son Mr. C should join the firm
on the same day.
Mr. C should be entitled to one-third of the firm's profit or loss in future.

For the purpose of A's retirement and Mr. C's admission, Goodwill was revalued at Rs. 44,000
and Freehold premises was revalued at Rs. 48,000. Other terms agreed were that enough money
was to be introduced to enable Mr. A to be paid out and to leave Rs. 20,000 cash for working
capital requirements. Mr. B and Mr. C should bring such sums so as to make their capital
proportionate to their future share of profit.

Mr. A agreed to make a gift by transfer from his Capital Account one-half of the amount, which
Mr. C should bring in as his capital.

Mr. B and Mr. C paid in cash the amount due from them and the amount due to Mr. A was paid
on the same day.

You are asked to draw up the Journal entries to recovered these transactions in the books of the
firm and prepare the Balance Sheet after the retirement of Mr. A.
(Inter Dec. 2001 Q3-16 Marks)
Answer
A's Capital + Increase
Rs. 68,000 + 15,300 = 83,300
B's Capital + Increase
Rs. 56,400 + 15,300 = 71,700
1,55,000

Less: Excess cash (22,000 - 20,000) 2,000 (-)

© The Institute of Chartered Accountants of Nepal 365


CAP II Paper 1: Advanced Accounting

(Working capital)
Total Capital of Mr. B and Mr. C. 1,53,000

Mr. C's Share = 1/3


Mr. B's Share = 1 - 1/3 = 2/3

Therefore, Mr. C has to provide = 1,53,000  1/3 = 51,000


Mr. B's Capital should be = 1,53,000  2/3 = 1,02,000
B's Existing capital = Rs. 71,700/-
Excess to be brought in by B = 1,02,00 - 71,700 = 30,300/-
Gift made by Mr. A to Mr. C = 1/2 of C's Capital
= 51,000  1/2 = 25,500/-
A's Capital A/c = 83,300
Less: Gifted to Mr. C = 25,500
Balance Due 57,800

Journal Entries:

1.4.2001
Goodwill A/c 24,000
Freehold premises A/c 6,600
To Mr. A's Capital A/c 15,300
To Mr. B's Capital A/c 15,300
(Being the increase in revaluation of assets transferred to Mr. A and Mr. B in equal Ratio)
25,500
Mr. A's Capital A/c
To Mr. C's Capital A/c 25,500
(Being gift of Rs. 25,500 given by Mr. A to Mr. C i.e. half of the capital Amount due from
Mr. C.)
55,800
Cash A/c
To Mr. B's Capital A/c 30,300
To Mr. C's Capital A/c 25,500
(Being Cash required paid in by Mr. B and Mr. C)
57,800
Mr. A's Capital A/c
To Cash A/c 57,800
(Being the amount due to Mr. A paid)

Balance Sheet of M/s X and Co. as at 1.4.2001

Liabilities Rs. Assets Rs.

Capital Accounts: Fixed Assets:

© The Institute of Chartered Accountants of Nepal 366


CAP II Paper 1: Advanced Accounting

Mr. B 1,02,000 Goodwill 44,000


Mr. C 51,000 Freehold premises 48,000
Current Liabilities: Current Assets:
Sundry Creditors 19,600 Stock 28,400
Sundry Debtors 32,200
Cash 20,000
1,72,600 1,72,600

2. P, Q and R are partners in a firm with Capitals of Rs. 1,70,000, Rs. 1,02,000 and Rs. 68,000
respectively and they share profits and losses in the same proportion. Their Balance Sheet on
31st March, 2000 is as under:

Balance Sheet

Liabilities Rs. Assets Rs.


Capitals 3,40,000 Goodwill 50,000
Current A/c P 45,000 Building 2,50,000
Current A/c Q 25,000 Machinery 80,000
Undistributed Profit 10,000 Stock 1,20,000
Loan 12% (1.4.99) 1,00,000 Debtors 45,000
Creditors 80,000 Bank Balance 42,000
Outstanding Expenses 7,000 R's Current A/c 20,000
6,07,000 6,07,000

A private limited company was formed to take over the partnership business from 1st
April, 2000 on the following terms and conditions:
i) All assets except goodwill and debtors are to be taken over along with creditors at
book value/agreed value.
ii) Rs. 5,000 of the debtors are irrecoverable and the remaining debtors are taken over
by P at 10% less than book value.

iii) Building is worth Rs. 4,00,000 and Machinery Rs. 1,00,000 for the purpose of
takeover.
iv) The loan was paid off by the firm along with interest due and the outstanding
expenses were fully settled for Rs. 6,000.
v) Purchase price was paid by allotting 40,000 Equity Shares of Rs. 10 each fully paid
at par and the balance was paid in cash. Shares were divided among partners in
proportion to their capitals.
Give the necessary Ledger Accounts to close the books of the partnership firm.
(Inter Jun. 2001 Q3 – 16 Marks)

Answer:
Purchase Consideration
Assets Taken over Rs.

© The Institute of Chartered Accountants of Nepal 367


CAP II Paper 1: Advanced Accounting

Building 4,00,000
Machinery 1,00,000
Stock 1,20,000
Bank Balance 42,000
6,62,000
Less: Creditors Purchase Price 80,000
5,82,000
40,000 Equity Shares of Rs. 10 each at per Cash 4,00,000
1,82,000
5,82,000
Realization Account

Dr. Rs. Cr. Rs.


To Goodwill 50,000 By Creditors 80,000
To Building 2,50,000 By Loan 1,00,000
To Material 80,000 By Outstanding Expenses 7,000
To Stock 1,20,000 By P (Debtors)
To Debtors 45,000 (45,000-5,000 = 40,000 x
To Bank 42,000 90/100) 36,000
To Bank (Loan) 1,00,000 By Pvt. Ltd. Co. 5,82,000
To Bank (Interest) 12,000
To Bank (Expenses) 6,000
To Profit P 50,000
To Profit Q 30,000
To Profit R 20,000
8,05,000 8,05,000

Bank Account
Dr. Rs. Cr. Rs.
To Pvt. Ltd. Co. 1,82,000 By Realization (Loan) 1,00,000
To R's Capital 10,000 By Realization (Interest) 12,000
By Realization (Expenses) 6,000
By P's Capital 34,000
By Q's Capital 40,000
1,92,000 1,92,000

Pvt. Ltd. Co. Account


Dr. Rs. Cr. Rs.
To Realization A/c 5,82,000 By Shares in Ltd. Co. 4,00,000
By Bank 1,82,000
5,82,000 5,82,000

© The Institute of Chartered Accountants of Nepal 368


CAP II Paper 1: Advanced Accounting

Capital Accounts
Dr. P Q R Cr. P Q R
To 36000 - - By Balance 170000 1020000 68000
Realization
To Shares in 200000 120000 80000 By 5000 3000 2000
Ltd. Co. Undistributed
profit
To Current - - 20000 By Realization 50000 30000 20000
A/c
To Bank A/c 34000 40000 - To Current A/c 45000 25000 -
To Bank A/c - - 10000
270000 160000 100000 270000 160000 100000

Shares in Ltd. Co. Account


Dr. Rs. Cr. Rs.
To Pvt. Ltd. Co. 4,00,000 By P's Capital A/c 2,00,000
By Q's Capital A/c 1,20,000
By R's Capital A/c 80,000
4,00,000 4,00,000

3. L, M and N were carrying on business in partnership sharing profits and losses equally. On
June 30, 2001, N retired from the firm and L and M agreed to share profits and losses in the
ratio of 3:2. N continued his association with the firm in the capacity of an advisor. The parties
agreed that as from July 1, 2001:

(i) N was to be credited with a retainership fee of Rs. 2,000 per month while he remained
the advisor.

(ii) The partners decided to bring into the accounts of the firm the various assets which up to
now were unrecorded. The various assets as on June 30, 2001 were: goodwill Rs.
60,000, office equipment Rs. 18,000, library books Rs. 3,000.
(iii) The values of the office equipment and library books were to be retained in the books but
goodwill was not to be recorded as a permanent asset.
(iv) N's Capital Account was to bear the whole cost of Rs. 10,000, a payment on March 30,
2001, for providing an annuity for a long-service employee who retired on that date.

The firm's profit for the year ended September 31, 2001, which is deemed to have accrued
evenly, amounted to Rs. 90,000, after deduction of the cost of the pension but before adjusting
for any of the events listed above. Other relevant figures are:
Credit balances on Capital Accounts Drawings during the year ended
October 1, 2000 (Rs.) September 30, 2001 (Rs.)
L 45,000 7,000

© The Institute of Chartered Accountants of Nepal 369


CAP II Paper 1: Advanced Accounting

M 35,000 9,000
N 55,000 6,000

All relevant entries to partners' entitlements are effected in their Capital Accounts. You are
required to write up, in columnar form, the Capital Accounts of L, M and N for the year ended
September 30, 2001 transferring the balance in N's Capital Account to N's Loan Account.
(Inter Jun. 2002, Q1a – 15 Marks)

Answer:
M/S L, M and N
Capital Accounts of Partners for the year ended September 30, 2001
L M N L M N

To Goodwill 36,000 24,000 – By Balance b/d 45,000 35,000 55,000


To Drawings 7,000 9,000 6,000 By P/L 25,000 25,000 25,000
Adjustment A/C
(Profit)
To P & L – – 10,000 By P/L 11,400 7,600 –
Adjustment A/C Adjustment A/C
(Annuity charge) (Post retirement)
[Refer W.N. - 1]
To N's Loan A/C – – 91,000 By Revaluation 7,000 7,000 7,000
A/C [Refer W.N.
2]
To Balance C/d 65,400 61,600 – By Goodwill A/C 20,000 20,000 20,000
108,400 61,600 107,000 108,400 94,600 107,000

N Loan A/C
Working Notes: 1

1. Profit and Loss Adjustment A/C

To Partners Capital A/C By Profit B/d 90,000


(Pre-retirement profit)
L 25,000 By N's Capital A/c 10,000
M 25,000 (Annuity charge Borne by
N 25,000 N)
75,000
To Partners' Capital A/C
(Post-retirement profit)
L 11,400
M 7,600
19,000
To N's Loan A/c 6,000

© The Institute of Chartered Accountants of Nepal 370


CAP II Paper 1: Advanced Accounting

(Retainership fee) ----------- -----------


100,000 100,000
====== ======
2. Revaluation A/C

To Office 18,000 By L's Capital A/c 7,000


Equipment
To Library Books 3,000 By M's Capital A/c 7,000
By N's Capital A/c 7,000
(Revaluation profit)
------------ ------------
21,000 21,000
======== ========

3. Treatment of Goodwill

a. Goodwill A/c Dr. 60,000


To L's Capital A/c 20,000
To M's Capital A/c 20,000
To N's Capital A/c 20,000
(Value of Goodwill brought into account)

b. L's Capital A/c Dr. 36,000


M's Capital A/c Dr. 24,000
To Goodwill A/c 60,000
(Value of Goodwill written off in the New Profit sharing ratio of 3:2)

4. The Balance Sheet of a partnership firm consisting of three partners - Rabin, Keshav and
Chhabi as on March 31, 2004, was as under:

Liabilities Rs. Assets Rs.


Bank overdraft 20,000 Plant and Machinery 32,000
Creditors for supplies 30,000 Buildings 50,000
Creditors for expenses 18,000 Receivables 48,000
Capital Accounts: Stock in Trade 40,000
Rabin 30,000 Cash and Bank balances 8,000
Keshav 20,000
Chhabi 20,000 70,000
Current Accounts:
Rabin 16,000
Chhabi 24,000 40,000
1,78,000 1,78,000

© The Institute of Chartered Accountants of Nepal 371


CAP II Paper 1: Advanced Accounting

The profit and losses were shared in the ratio of fixed capitals as on March 31, 2004. The
partners agreed that due to old age Rabin would retire from the firm when its goodwill will
be valued and proportionate share be credited to Rabin. It was decided also that buildings
appearing in the books at cost would be valued at their market value of Rs. 80,000 and allotted
to Rabin in satisfaction of his dues. Any excess or deficit would be settled in cash. It was also
agreed that stock in trade and receivables would be taken at 90% of the book value and an
unrecorded liability of bonus of Rs. 10,000 to staff be brought into books. Goodwill of the
firm was to be taken at Rs. 70,000.

After Rabin's retirement, the business was carried on by Keshav and Chhabi, sharing profits
and losses equally and till 30 June, 2004, the firm had made a net profit of Rs. 30,000 after
crediting each partner's capital account with Rs. 500 p.m. as salary. No drawings were made
by the partners in the quarter.

Keshav and Chhabi now find that they cannot continue the business and decided sell it to a
private limited company as and from July 1, 2004. The company is to take over the entire
business for a consideration of Rs. 90,000, which the vendors agree to take 40% in 14%
secured debentures and the balance in cash. To enable the company to pay the vendors and
also leave it with a working capital of Rs. 20,000, the company makes issue of equity shares
of Rs. 10 each at par.

Show the balance sheet of the company after the takeover of the business of Keshav and
Chhabi. (Inter Jun. 2004 Q1 -20 Marks)

Answer:

Balance Sheet of ……………… Company as on July 1, 2004

Liabilities Rs. Assets Rs.


Share Capital: Fixed Assets:
Issued, subscribed and paid up: Goodwill 16,600
7,400 Equity shares of Rs. 10 each Plant and Machinery 32,000
fully paid up 74,000 Net Current Assets:
(Of the above, 5,400 shares have Cash 20,000
been issued for consideration other Others (Net) 41,000
than cash)
Secured Loans:
14% secured Debentures 36,000
Total 1,10,00 Total 1,10,000
0

Working Notes:

© The Institute of Chartered Accountants of Nepal 372


CAP II Paper 1: Advanced Accounting

i. Goodwill Rs. 70,000: Rabin's shares Rs. 30,000 to be credited to his Capital Account by
debiting Capital Account to Keshav and Chhabi with Rs. 15,000 each.

ii. Profit and Loss Adjustment Account

Dr. Cr.
Rs. Rs.
To Sundry Debtors (Discount) 4,800 By Premises 30,000
To Stock in Trade 4,000
To Bonus due to staff 10,000
To Profit on adjustment transferred
to:
Rabin's Capital A/c 4,800
Keshav's Capital A/c 3,200
Chhabi's Capital A/c 3,200 11,200

Total 30,000 Total 30,000

iii.Rabin's Capital Account

Dr. Cr.
Rs. Rs.
To Premises 80,000 By Balance B/d 30,000
To Cash A/c 800 By Transfer from current A/c 16,000
By Keshav's Capital A/c
(for goodwill) 15,000
By Chhabi's Capital A/c
(for goodwill) 15,000
By Profit and Loss Adjustment 4,800
Total 80,800 Total 80,800

iv. Revised Balance Sheet as at 31st March, 2004 (After Rabin's retirement)

Liabilities Rs. Assets Rs.


Bank overdraft 20,000 Plant and Machinery 32,000
Creditors for goods 30,000 Sundry Debtors 43,200
Creditors for expenses 28,000 Stock in Trade 36,000
Capital Account: Cash 7,200
Keshav (Rs. 20,000-15,000+3,200) 8,200
Chhabi (Rs. 20,000+24,000+3,200-
15,000) 32,200
Total 1,18,400 Total 1,18,400

© The Institute of Chartered Accountants of Nepal 373


CAP II Paper 1: Advanced Accounting

v. Balance Sheet of Keshav & Chhabi as at 30th June, 2004

Liabilities Rs. Assets Rs.


Capital Account Plant and Machinery 32,000
Keshav: Net Current Assets:
Balance as on 1.4.04 8,200 Balance as on 1.4.04 8,400
Add: Salary 1,500 Add: Profits 30,000
Add: Share of Profits 15,000 24,700 Add: Partners Salary 3,000 41,400
Chhabi:
Balance as on 1.4.04 32,000
Add: Salary 1,500
Add: Share of profit 15,000 48,700

Total 73,400 Total 73,400

vi. Goodwill = (Purchase consideration - Net Assets take over) = (90,000 - 73,400) = Rs.
16,600

vii. Purchase consideration Rs. 90,000 : to be paid in cash 60% i.e., Rs. 54,000 and balance
to be discharged by issue of debentures Rs. 36,000

viii. Cash required = Rs. 54,000 + Working capital of Rs. 20,000 = Rs. 74,000

Therefore, shares to be issued = 7,400 shares of Rs. 10 each.

5. Ram, Shyam and Pitamber were in partnership sharing profits and loss in the ratio of ½, 1/3
and 1/6 respectively. Due to certain differences between the partner, it was decided to dissolve
the partnership firm on 31.3.2005, when the balance sheet of the firm was as follows:-

Balance Sheet as on 31.03.05

Liabilities Amount Assets Amount


Rs Rs
Sundry creditors 567,000 Goodwill 456,300
Bank overdraft 606,450 Plant and machinery 607,500
Joint life policy reserve 265,500 Furniture 64,650
Loan form Mrs. Ram 150,000 Stock 236,700
Capital accounts: Sundry debtors 534,000
Ram 420,000 Joint life policy 265,500
Shyam 225,000 Commission receivable 140,550
Pitamber 120,000 Cash on hand 48,750
Total 2,353,950 2,353,950

© The Institute of Chartered Accountants of Nepal 374


CAP II Paper 1: Advanced Accounting

The following details are relevant for the dissolution:


a. Sundry debtors were assigned to firm’s creditors in full satisfaction of their claims.
b. Ram took over goodwill and plant and machinery for Rs.900, 000. He also agreed to
discharge bank overdraft and loan from his wife.
c. Furniture and stock were divided equally between Ram and Shyam at an agreed
valuation of Rs.360, 000.
d. Commission receivable was received in time.
e. A bill discounted was dishonored and thereafter it proved valueless, amounting to
Rs.30, 750(including notice charge of Rs.500).
f. The joint policy was surrendered for Rs.232, 500.
g. Ram paid Rs.18, 000 towards the expenses of dissolution.
h. Pitamber agreed to receive Rs.150, 000 in full satisfaction of his rights, title and interest
in the firm.

You are required to show the accounts relation to closing of the books on dissolution of the
firm. (Inter Jun. 2005 Q1-20 Marks)

Answer:
Realization Account
To Goodwill a/c 456,300 By Sundry Creditors 567,000
To Plant and Machinery a/c 607,500 By Joint Life Policy Reserve 265,500
To Furniture 64,650 By Cash:
- Joint Life Policy 232,500
To Stock 236,700 By Ram’s Capital
To Sundry Debtors 534,000 - Goodwill, plant and machinery 900,000
- Furniture and stocks 180,000
To Joint Life Policy 265,500 By Shyam’s Capital:
To Ram’s Capital - Furniture and stocks 180,000
- Dissolution Expenses 18,000
To Cash-bills dishonored 30,750
To Partners’ Capital
(Profit on Realization)
- Ram 55,800
- Shyam 37,200
- Pitamber 18,600

2,325,000 2,325,000

Capital Accounts
Ram Rs. Shyam Pitamber Ram Shyam Pitamber
Rs Rs Rs. Rs Rs
To Realization: 900,000 By Balance b/d 420,000 225,000 12,000
Goodwill, plant By Bank b/d 606,450
and machinery

© The Institute of Chartered Accountants of Nepal 375


CAP II Paper 1: Advanced Accounting

Furniture, 180,000 180,000 By loan from Mr. 150,000


stocks Ram
6,840 4,560 By Realization:
To pitamber’s Diss. Expenses 180,000
capital 150,000 By Realization:
To cash Profit on 55,800 37,200 18,600
To cash 163,410 77,640 realization
(bal. figure) By partners’
capitals:
Ram 6.840
Shyam 4,560

1,250,250 262,200 150,000 1,250,250 262,200 150,000

Cash Account
Rs Rs
To balance b/d 48,750 By Realization- bill dishonored 30,750
To Realization: Joint Life Policy 232,500 By Partners’ Capital
To Commission Receivable 140,550 - Ram 163,410
- Shyam 77,640
- Pitamber 150,000
421,800 421,800

It must be noted that the amount of Rs.11, 400 representing excess payment of Pitamber
(Rs.150, 000 – 138,600) has been debited to Ram and Shyam in the ratio of 3:2.

6. A, B and C are partners in a firm sharing profit and losses in the ratio of 3:2:1 respectively.
They kept their books on single entry system and their Statement of Affairs as at 31.12.2004
showed their position as follows:

Statement of Affairs of M\s A,B and C as at 31.12.2004


Liabilities Rs. Assets Rs.
Sundry Creditors 25,200 Cash in hand 2,110
Bills Payable Cash at bank 11,400
Capital accounts Sundry debtors 50,400
A 60,000 Stock in trade 40,200
B 40,000 Furniture and fittings 12,000
Plant and Machinery 48,000
C 20,000 120,000 Current accounts

Current accounts
A 1,920 C 1,170

© The Institute of Chartered Accountants of Nepal 376


CAP II Paper 1: Advanced Accounting

B 1,360
3,280
165,280 165,280

On 31.12.2005, the assets and liabilities of the firm were as follows:


Cash in hand Rs. 1,700,stock in trade Rs.61,200, Plant and Machinery Rs. 75,000, Bills
Payable Rs.14,400, sundry debtors Rs.57,600, furniture and fittings Rs.12,000, sundry
Creditors Rs.20,400 and Bank overdraft Rs.18,000.

You are required to ascertain the profit or loss made by the firm in the year 2005 and show the
statement of Affairs as at that date after taking into consideration the following additional
information:
i. Plant and machinery is to be reduced by 10%
ii. Stock is to be reduced to Rs. 50,000
iii. A reserve for bad debts to be raised at 3% on sundry debtors.
iv. Interest on partners’ capital is to be allowed at 5%p.a
v. During the year 2005, the monthly drawings of the partners were A-Rs.1600
B- Rs.1,200 and C-Rs.1,000 respectively.
vi. Partners’ capital should be treated as fluctuating capital only for the purpose of allowing
interest on capital.
vii. Interest on Partners capital is to be allowed on opening balances. Calculation to be made to
nearest rupees. (Inter Jun. 2005 Q5a-12 Marks)

Answer
Statement showing the profit or loss of M/S A, B and C for the year ended on 31.12.2005

Particulars Rs. Rs.


Combined closing capital( Working note 1) 1,54,700
Add: Drawings during the year :
A 19,200
B 14,400
C 12,000 45,600
Less: Combined opening capital (Working note 2) 200,300
Profit before adjustment 122,110
Less Adjustment 78,190
Reduction in the value of Plant and Machinery 7,500
Reduction in the value of stock 11,200
Reserve for Bad debts 1,728 20,428
Net Profit for the year 57,762
Less: Interest on Partners’ Capital (Working note 2)
A 3,096
B 2,068
C 942 6,106

© The Institute of Chartered Accountants of Nepal 377


CAP II Paper 1: Advanced Accounting

Net profit for the year after charging interest on partners’ 51.656
capital.

Statement of Affairs of M/S A, B and C as at 31.12.2005

Liabilities Rs. Assets Rs.


Capital Accounts Furniture and Fittings 12,000
A 60,000 Plant and Machinery 67,500
B 40,000 Stock in trade 50,000
C 20,000 120,000 Sundry Debtors 55,872
Current Accounts Cash in hand 1,700
A 11,644 Current Accounts:
B 6,247 17,891 C 3,619
Bank overdraft 18,000
Sundry creditors 20,400
Bills payable 14,400

190,691 190,691

Working note 1

Computation of combined closing capital before adjustments


Statement of Affairs as at 31.12.2005 before adjustments

Liabilities Rs Assets Rs.


Bank overdraft 18, 000 Furniture and Fittings 12,000
Sundry creditors 20, 400 Plant and Machinery 75,000
Bills payable 14,400 Stock in trade 61,200
Capital(balancing figure) 154,700 Sundry Debtors 57,600
Cash in hand 1,700
Total 207,500 207,500

Working note 2

Computation of combined Opening capital and interest on partners’ capital

Particulars A B C Total
Capital account as on 01.01.2004 60,000 40,000 20,000 120,000
Current account as on 01.01.2004 1,920 1,360 (1,170) 2,110
Total opening capital 61,920 41,360 18,830 122,110
Interest @ 5% p.a 2,068 942 6,106

© The Institute of Chartered Accountants of Nepal 378


CAP II Paper 1: Advanced Accounting

Working note 3
Partners’ Current Accounts
Particulars A B C Total
Capital account as on 01.01.2004 1,920 1,360 (1,170) 2,110
Adjustments
Drawing (19,200) (14,400) (12,000) (45,600)
Interest on capital 3,096 2,068 942 6,106
Share of profits in the ratio of 25,828 17,219 8,609 51,656
3:2:1 11,644 6,247 (3,619) 14,247
Current account as on 31.12.2005

7. A, B and C are in partnership for a number of years. Profits and losses were shared in the ratio
of 3 : 4 : 3.
On 30th April, 2007 it was decided to dissolve the partnership. The balance sheet of the firm
as at that date stood as below:
Balance Sheet
As on 30th April, 2007

Liabilities Rs. Assets Rs.


Partners’ capital accounts: Fixed Assets 78,000
A 30,000 Debtors 52,500
B 90,000 Stock 147,000
C 120,000 Cash 37,500
Trade creditors 75,000
315,000 315,000

During the dissolution, the following cash and other transactions arose:
2007
3 May A agreed to settle a hire purchase debt outstanding on a motor car. The amount was
Rs. 6,750 and is to be adjusted in his capital account.
3 May Debtors were assigned to C for the agreed sum of Rs. 45,000.
3 May C settled one creditor for Rs. 3,000 by giving him one of the private paintings
3 May The fixed assets, apart from the car which had a book value of Rs. 26,250, were
sold at auction for Rs. 102,000. This car is to be taken at book value by C.
Adjustment is to be made in his capital account.
1 June Realization expenses of Rs. 5,250 were paid.
10 June Cash transfers among partners were completed.
16 June The remaining creditors were paid.

On 1st May 2007, B and C formed a new partnership by merging with another firm. The
partners in the other firm, X and Y, shared profits and losses equally. The new amalgamated
firm will be called Chalibe & Co. and will take over the stocks of both firms.

© The Institute of Chartered Accountants of Nepal 379


CAP II Paper 1: Advanced Accounting

All partners have agreed to the following value for assets which will be taken over
B and C X and Y
Rs Rs.
Stocks 73,500 93,000
Furniture 12,000
Motor Vehicles 22,500
Goodwill 25,200 67,500

The partners also agreed that profits and losses would be shared:

B- 20%, C- 15%, X- 30% and Y- 35%.

Capital is to be contributed in the same proportions after allowing Rs. 30,000 for working
capital. Goodwill is not to appear in any set of books.

You are required to:


a. Show the accounts closing the books of the A, B and C partnership.
b. Prepare a Balance Sheet for Chalibe & Co. as at 1 May 2007.
(Inter Jun. 2007 Q2-15 Marks)
Answer
In the books of M/s A, B and C
Realization Account
Rs. Rs.
To Fixed Assets 78,000 By C A/c
To Debtors 52,500 - Debtors Assigned 45,000
To Stock 1,47,000 By Cash A/c-fixed assets sold 1,02,000
To Cash A/c-Realization Expenses 5,250 By C A/c-car taken over 26,250
By Partner’s Capital A/c-Stock
taken over
B Rs. 42,000
C Rs. 31,500 73,500

By Partner’s Capital A/cs-


Loss on Realization
A Rs. 10,800
B Rs. 14,400
C Rs. 10,800 36,000
2,82,750 2,82,750

Partners’ Capital Accounts


A Rs. B Rs. C Rs. A Rs. B Rs. C Rs.
To Realization A/c - - 45,000 By Balance b/d 30,000 90,000 1,20,000
To Realization A/c - - 26,250 By Trade 6,750 - 3,000
creditors A/c

© The Institute of Chartered Accountants of Nepal 380


CAP II Paper 1: Advanced Accounting

To Realization A/c - 42,000 31,500 By Goodwill 7,560 10,080 7,560


To Goodwill- - 14,400 10,800
Written off
To Realization A/c 10,800 14,400 10,800
To Cash A/c-
Balance Paid 33,510 29,280 6,210
44,310 1,00,080 13,0560 44,310 1,00,080 1,30,560

Cash Account

Rs. Rs.
To Balance b/d 37,500 By Trade Creditors A/c 65,250
To Realization A/c 1,02,000 By Realization A/c 5,250
By Partner’s Capital A/cs-
A Rs. 33,510
B Rs. 29,280
C Rs. 6,210 69,000
1,39,500 1,39,500

Trade Creditors Account

Rs. Rs.
To Hire Purchase Debt Settled 6,750 By Balance b/d 75,000
To Private Paintings given 3,000
To Cash A/c 65,250
75,000 75,000

Balance Sheet of Chalibe & Co.


As on 1st May, 2007

Liabilities Rs. Assets Rs.


Partners’ Capital Accounts: Motor vehicles 22,500
B 46,200 Furniture 12,000
C 34,650 Stock 1,66,500
X 69,300 Cash 30,000
Y 80,850
2,31,000 2,31,000

Workings: Assets Introduced:

B Rs. C Rs. X Rs. Y Rs.


Stock 42,000 31,500 46,500 46,500
Furniture - - 6,000 6,000
Motor Vehicle - - 11,250 11,250
Goodwill 14,400 10,800 33,750 33,750
56,400 42,300 97,500 97,500

© The Institute of Chartered Accountants of Nepal 381


CAP II Paper 1: Advanced Accounting

Less: Goodwill written off 18,540 13,905 27,810 32,445


37,860 28,395 69,690 65,055

Total Assets of the new firm


[Rs. 2,01,000 + working capital (30000)
= Rs. 2,31,000]

Total Assets distributed in the new


profit showing ratio 46,200 34,650 69,300 80,850
Cash to be introduced 8,340 6,255 - 15,795
Cash to be withdrawn 390

8. Bramha, Bishnu and Mahesh were partners sharing profit and losses in the ratio of 2:2:1. Their
Balance sheet as on 1 Shrawan 2063 stood as follows:
Liabilities Rs Rs Assets Rs Rs
Capital Accounts Fixed Assets 1,00,000
Bramha 50,000 Stock 25,000
40,000 Debtors 35,000
30,000 1,20,000 Cash & Bank 10,000
Reserves 10,000
Creditors 40,000
1,70,000 1,70,000

The firm had taken the Joint Life Policy for Rs. 1,00,000, the premium amounts on which
were charged to the profit & loss account. On 1 Magh 2063, Mahesh died of Heart Attack.
His legal representatives agreed that:
i) Goodwill of the firm be valued at Rs. 50,000
ii) Fixed Assets be written down by Rs. 10,000.
iii) In lieu of profits up to 1 Magh 2063, Mahesh should be paid at the rate of 25 %
per annum on his capital as on 1-4-2063; and
iv) Profit after 1 Magh 2063, Mahesh’s legal heirs should be paid on the basis of the
proportionate capital as on the date of death.

The policy money was received on 32 Ashadh 2064 and Mahesh’s heirs were paid the total
amount due on the same day. Current Year’s (2063-64) profit after charging depreciation
of Rs. 9,500 (Rs. 5,000 related to the first half) was Rs. 40,500. The year-end figures of
Stock, Debtors and creditors, and Cash and Bank Balances were respectively Rs. 33,000,
Rs. 29,000, Rs. 35,000 and Rs. 66,217. The particulars regarding their drawings are given
below.

Up to Poush end 2063 After Magh 1, 2063


Rs. Rs.
Bramha 4,125 5,000

© The Institute of Chartered Accountants of Nepal 382


CAP II Paper 1: Advanced Accounting

Bishnu 4,125 5,000


Mahesh 1,750 -
Prepare the balance sheet of the firm as on 32 Ashadh, 2064 assuming that the
remaining partners did not retain Goodwill in their books.
(Inter Dec. 2007 Q2-15 Marks)
Answer:

Books of Bramha & Bishnu


Balance Sheet As on 32 Ashadh 2064
Liabilities Rs. Rs. Assets Rs. Rs.
Fixed Assets 100,000
Capital
Accounts: Less: Written Down 10,000
Bramha 91,858 90,000
Bishnu 81,859 173,717 Less: Depreciation 9,500 80,500
Stock 33,000
Creditors 35,000 Debtors 29,000
Cash & Bank 66,217
Total 208,717 Total 208,717

Working Notes:
Rs.
a) Profit after depreciation 40,500
Add: Depreciation 9,500
Profit before depreciation 50,000
Profit for the first half (Assuming profit earned evenly during
the year) 25,000
Less: Depreciation for the first half 5,000
Profit for the first half after depreciation 20,000
Profit for the second half (Assuming profit earned evenly
during the year) 25000
Less Depreciation for the second half 4,500
Profit for the second half after depreciation 20,500
Profit and Loss Appropriation Account
(For the first half)
Dr. Cr.
To Interest on Mahesh
Capital 3,750 By Profit 20,000
(30000 * 25 % for 6
months )
To Capital Accounts
Bramha 8,125
Bishnu 8,125

© The Institute of Chartered Accountants of Nepal 383


CAP II Paper 1: Advanced Accounting

Total 20,000 Total 20,000

Capital Accounts as on 1 Magh 2063


Dr Cr

Bramha Bishnu Mahesh Bramha Bishnu Mahesh


To Revaluation By Balance
loss of fixed assets 4,000 4,000 2,000 b/d 50,000 40,000 30,000

To Drawings 4,125 4,125 1,750 By Reserves 4,000 4,000 2,000

To Executors A/c 42,000 By Goodwill 20,000 20,000 10,000


To Balance c/d. 74,000 64000 By P & L
Appropriation
A/c 8,125 8,125 3,750

Total 82,125 72,125 45,750 Total 82,125 72,125 45,750

Capital Accounts Balance as on 1 Magh 2063


Bramha 74,000
Bishnu 64,000
Mahesh 42,000
Total 180,000
Profit earned during second half by Mahesh's Executors
=20,500 X 42,000/1,80,000
=4,783

Profit and Loss Appropriation Account


(For the second half)
Dr Cr
To Mahesh's Executors
A/c 4,783 By Profit 20,500
To Capital
Accounts
Bramha 7,858
Bishnu 7,859

Total 20,500 Total 20,500

Capital Accounts as on 32 Ashadh 2064


Dr. Cr.
Bramha Bishnu Bramha Bishnu

© The Institute of Chartered Accountants of Nepal 384


CAP II Paper 1: Advanced Accounting

By Balance b/d 74,000 64,000


To Drawings 5,000 5,000 By P & L
To Goodwill
Written off 25,000 25,000 Appropriation A/c 7,858 7,859
To Balance C/d. 91,858 81,859 By Joint Life Policy 40,000 40,000
Total 1,21,858 1,11,859 Total 1,21,858 1,11,,859

Mahesh's Executor's Account


Dr Cr
To Bank 66,783 By Mahesh Capital 42,000
By Profit & Loss
Appropriation A/c 4,783
By Joint Life Policy 20,000
66,783 66,783

9. Ram, Shaym and Hari were trading in partnership sharing profits and losses equally. The
accounts of the firm are closed on 31st Ashadh every year.

The partnership deed provided that:


On the death of a partner the goodwill was to be valued at 3 years’ purchase of the average
profit of past 3 years up to the date of death after deducting interest @ 10% on capital
employed and a fair remuneration of each partner. The profits are assumed to be earned
evenly throughout the year.
Following were the profits for different years before charging interest on capital employed:
Year Profit (Rs)
2062/63 67,200
2063/64 75,600
2064/65 72,000
2065/66 62,400

On 30th Poush 2065 Ram died due to an accident and it was agreed on his death to adjust
goodwill in the capital accounts without showing any amount of goodwill in the Balance
Sheet.

It was agreed for the purpose of valuation of goodwill that the fair remuneration for work
done by each partner would be Rs.15,000 per annum and that the capital employed is Rs.
1,50,000.Shyam and Hari were to continue the partnership, sharing profit and losses in a
ratio of 2:1 after the death of Ram.
Compute the value of the goodwill and show adjustment entries in the books of the firm.
(Inter Dec. 2009 Q2a-12 Marks)

© The Institute of Chartered Accountants of Nepal 385


CAP II Paper 1: Advanced Accounting

Answer
a) Computation of the value of goodwill:
(i) Average Profit for the year ending 30th Poush; before death of Ram:

Year ending 30th Poush, 2063: Rs. Rs.


½ of 2062/63 profits 33,600
½ of 2063/64 profits 37,800 71,400

Year ending 30th Poush, 2064:


½ of 2063/64 profits 37,800
½ of 2064/65 profits 36,000 73,800

Year ending 30th Poush, 2065:


½ of 2064/65 profits 36,000
½ of 2065/66 profit 31,200
67,200
Total profit for 3 years
2,12,400
Average Profit
70,800

(ii) Profit after adjustment of remuneration and interest on capital :


Rs
Average Profit 70800
Less: Remuneration (15000*3) 45000
25800
Less: Interest on Capital 15000
10800

(iii) Goodwill at 3 three years’ purchase =10,800*3=32,400

Adjustment entries for Goodwill


Journal
Goodwill Account 32,400
To Capital Accounts
Ram 10,800
Shyam 10,800
Hari 10,800
(Goodwill, valued @ Rs. 32,400 adjusted in the capital accounts of partners
on the death of Mr. Ram in the old profit sharing ratio)

Shyam’s Capital Account 21,600

© The Institute of Chartered Accountants of Nepal 386


CAP II Paper 1: Advanced Accounting

Hari’s Capital Account 10,800


To Goodwill Account 32,400
(Goodwill written off between continuing partners in the new profits sharing
ratio)

10. A & B are in partnership, sharing profits & losses in the ratio of two third and one third
respectively. The books are kept on single entry system; and their statement of affairs dated
31st Ashadh 2066 showed their position as follows:

Statement of Affairs of A & B as at 31st Ashadh 2066 Amount in Rs.

Fixed Assets
Building 60,000
Machinery 20,000
Furniture & Fixtures 5,000 85,000

Working Capital:
Current Assets:
Debtors 60,000
Stock 40,000
Cash & Bank 5,000
Bills Receivables 15,000 120,000
Current Liabilities:
Loan 10,000
Creditors 30,000
Bills Payable 25,000 65,000 55,000

Totals 140,000

Represented by
Capital:
A 1,00,000
B 40,000 1,40,000

Additional information are as follows:


On 32nd Ashadh 2067, the books disclosed following information:
Debtors Rs.80, 000 Creditors on open accounts Rs. 85,000
Cash Rs.8, 000 Creditors for Loan Rs.16,000

The stock was valued at Rs. 42,000 and the bills receivable amounted Rs. 14,000. An
examination of cash book showed that during the year, A had drawn on account of profit Rs.
15,000 and B Rs. 6,000. A had, in addition, withdrawn Rs.20,000 from his capital account
on 30th Poush 2066.

© The Institute of Chartered Accountants of Nepal 387


CAP II Paper 1: Advanced Accounting

The partners agreed to reduce the existing valuation of Machinery by 5 % and the Furniture
& Fixtures by 10% by way of depreciation. They also agreed to charge 5 % by way of
interest on capital.
Required:
a) Prepare statement of profit, dividing the balance between A and B for the year ended
32nd Ashadh 2067; and
b) Prepare statement of affairs showing the position as at 32nd Ashadh 2067.
(CAP Dec. 2010 Q1-20 Marks)
Answer

Statement of Profit & Loss of A and B for the year ended 31stAshad 2067

Figures in
Rs.
Combined Closing Capital ( WN 1) 128,000
Add: Combined drawing during the year ( A Rs 3,5000 + B Rs 41,000
6000) 169,000
140,000
Less: Combined opening Capital ( Rs 100,000 + Rs 40,000) 29,000
Profit before adjustments
Less: Adjustments; 1000
Depreciation on Machinery 500 1,500
Depreciation on Furniture & Fixtures 27,500
Net Profit 6,500
Less : Interest on capital ( A Rs 4,500 + B Rs 2,000) 21,000
Divisible Profit 14,000
A’s Share Two third 7,000
B’s Share One third

Statement of Affairs as at 31stAshad 2067

Fixed Assets
Building 60,000
Machinery 20,000
Less: depreciation 1000 19,000
Furniture & Fixtures 5,000
Less: depreciation 500 4,500 83,500

Working Capital:
Current Assets:
Debtors 80,000
Stock 42,000
Cash & Bank 8,000

© The Institute of Chartered Accountants of Nepal 388


CAP II Paper 1: Advanced Accounting

Bills Receivables 14,000 144,000


Current Liabilities:
Loan 16,000
Creditors 85,000 1,01,000 43,000

Total 126,500

Represented by
Capital:
A ( WN 2) 83,500
B ( WN 43,000 126,500
126,5002)

Working Note 1:

Ascertainment of Combined Closing Capital (before adjustments)

Fixed Assets
Building 60,000
Machinery 20,000
Furniture & Fixtures 5,000 85,000

Working Capital:
Current Assets:
Debtors 80,000
Stock 42,000
Cash & Bank 8,000
Bills Receivables 14,000 144,000
Current Liabilities:
Loan 16,000
Creditors 85,000 1,01,000 43,000

Totals 128,000

Represented by
Capital ( Balance figure) 128,000

Working Note: 2
Partners’ Capital A/C
Particulars A (Rs.) B (Rs.) Particulars A (Rs.) B (Rs.)
To Bank (Capital 20,000 - By Balance b/d 100,000 40,000
drawn) By Interest on 4,500 2,000
15,000 6,000 Capital

© The Institute of Chartered Accountants of Nepal 389


CAP II Paper 1: Advanced Accounting

To Bank 83,500 43,000 By Share on Profit 14,000 7,000


(Drawings) 118,500 49,000 118,500 49,000
To Bank c/d

Working Note:3
Interest on capital:
A: On Rs.100,000 @ 5% for ½ years = Rs. 2,500 and on Rs 80,000 @5% for ½ years=
Rs 2,000
B: On Rs. 40,000 @ 5% for 1 years = Rs 2,000

11. Laxman and Hari are partners of the firm LH & Co., from 1.4.2007. Initially both of them
contributed Rs. 1,00,000 each as capital. They did not contribute any capital thereafter. They
maintain accounts of the firm on mercantile basis. They were sharing profits and losses in the
ratio of 5:4. After the accounts for the year ended 31.3.2011 were finalized, the partners decided
to share profits and losses equally with effect from 1.4.2007. It was also discovered that in
ascertaining the results in the earlier years certain adjustments, details of which are given
below, were not taken into account.

Year ended 31st March 2008 2009 2010 2011


Profit as per accounts prepared and 140,000 260,000 320,000 360,000
finalized
Expenses not provided for (as at 31st 30,000 20,000 36,000 24,000
March)
Incomes not taken into account (as at 31st 18,000 15,000 12,000 21,000
March)

The partners decided to admit Chakra as a partner with effect from 1.4.2011. It was decided
that Chakra would be allotted 20% share in the firm and he must bring 20% of the combined
capital of Laxman and Hari.

Following is the Balance sheet of the firm as on 31.3.2011 before admission of Chakra and
before adjustment of revised profits between Laxman and Hari.
Liabilities Assets
Capital Accounts Plant and machinery 60,000
Laxman 2,11,500 Cash on hand 10,000
Hari 1,51,500 Cash at bank 5,000
Sundry creditors 2,27,000 Stock in trade 3,10,000
Sundry debtors 2,05,000
5,90,000 5,90,000

You are required to prepare:


a) Profit and Loss Adjustment account;
b) Capital accounts of the partners; and

© The Institute of Chartered Accountants of Nepal 390


CAP II Paper 1: Advanced Accounting

c) Balance Sheet of the firm after the admission of Chakra.


(CAP Dec. 2011 Q1-20 Marks)

Answer:
(i) Profit and Loss Adjustment Account
Particulars Rs. Particulars Rs.
To Expenses not provided for: By Income not considered for:
Year ended Year ended
30,000 18,000
31.03.2008 31.03.2008
Year ended Year ended
20,000 15,000
31.03.2009 31.03.2009
Year ended Year ended
36,000 12,000
31.03.2010 31.03.2010
Year ended Year ended
24,000 110,000 21,000 66,000
31.03.2011 31.03.2011
By Partners capital account:
Laxman 22,000
Hari 22,000 44,000
110,000 110,000
(ii) Partners’ Capital Accounts
Laxman Hari Chakra Laxman Hari Chakra
Particulars Particulars
Rs. Rs. Rs. Rs. Rs. Rs.
To P / L Adj. A/c By Balance
22,000 22,000 - 211,500 151,500 -
- Loss b/d
To Hari 60,000 - - By Laxman - 60,000 -
To Balance c/d 129,500 189,500 63,800 By Cash/Bank - - 63,800
211,500 211,500 63,800 211,500 211,500 63,800
(iii) Balance Sheet of LH & Co. as on 1.4.2011
(After admission of Chakra)
Liabilities Rs. Assets Rs.
Capital Accounts: Plant & Machinery 60,000
129,50
Laxman Stock in Trade 310,000
0
189,50
Hari Sundry Debtors 205,000
0
Chakra 63,800 382,800 Accrued Income 66,000
Sundry Creditors 227,000 Cash at Bank 5,000
Outstanding Expenses 110,000 Cash in Hand 73,800
719,800 719,800
Working Notes
1. Computation of Profit and Loss distributed among partners:
Rs.
Profit for the year ended 31.3.2008 140,000

© The Institute of Chartered Accountants of Nepal 391


CAP II Paper 1: Advanced Accounting

31.3.2009 260,000
31.3.2010 320,000
31.3.2011 360,000
Total Profit 1,080,000

Laxman Hari Total


Rs. Rs. Rs.
Profit shared in old ratio i.e. 5: 4 600,000 480,000 1,080,000
Profit to be shared as per new ratio i.e. 1 :
540,000 540,000 1,080,000
1
Excess / (Deficit) Profit 60,000 (60,000) -
Laxman to be debited by Rs. 60,000 and Hari to be credited by Rs. 60,000

2. Capital brought in by Chakra – 20% of the combined capital of Laxman & Hari:

Particulars Rs.
Capital of Laxman (As per Capital Account above) 129,500
Capital of Hari (As per Capital Account above) 189,500
Combined Capital 319,000
20% of Rs. 319,000 63,800
3. Cash in hand:
Particulars Rs.
Cash balance as per Balance Sheet 10,000
Add: Cash brought in by Chakra 63,800
Total 73,800

12. A, B and C carried on business in partnership, sharing profits and losses in the ratio of 1:2:3.
They decided to form a private limited company, AB (P) Ltd. and C is not interested to take
over the shares in AB (P) Ltd. The authorized share capital of the company is Rs. 1,200,000
divided into 12,000 ordinary shares of Rs. 100 each.
The company was incorporated and took over goodwill as valued and certain assets of the
partnership firm on 31-03-2012. The Balance Sheet of the partnership firm on that date was as
follows:
______________________________________________________________
Liabilities Rs. Assets Rs.
Capital Accounts: PPE:
A 100,000 Machinery 120,000
B 200,000 Land 174,000
C 300,000 Motorcycles 30,000
Current Accounts: Furniture & fittings 11,000
A 39,420 Current Assets:
B 60,580 Stock 235,000
A’s Loan A/c 28,000 Debtors 43,000

© The Institute of Chartered Accountants of Nepal 392


CAP II Paper 1: Advanced Accounting

Add: Interest accrued 2,000 30,000 Cash in hand 87,000


Current Liability: C’s overdrawn 100,000
Creditors 70,000
Total 800,000 Total 800,000

C, who retired was presented by the other partners (A and B) with one motorcycle valued in
the books of the firm Rs. 9,000. The remaining motorcycles were sold in the open market for
Rs. 13,000. C also received certain furniture for which he was charged Rs. 2,000. The debtors
which were all considered good, were taken over by C for Rs. 40,000. A and B were charged
in their profit sharing ratio for the book value of motorcycle presented by them to C.

It was agreed that C who is not willing to take the shares in AB (P) Ltd. was discharged first
by providing necessary cash. A and B should bring cash, if necessary.

AB (P) Ltd. took over the remaining furniture and fittings at a price of Rs. 13,000, the
machinery for Rs. 125,000, the stock at an agreed value of Rs. 200,000 and the land at its book
value. The value of the goodwill of the partnership firm was agreed at Rs. 88,000. The
creditors of the firm were settled by the firm for Rs. 70,000. A’s loan account together with
interest accrued was transferred to his capital account.

The purchase consideration was discharged by the company by the issue of equal number of
fully paid up equity shares at par to A and B.
Prepare realization account, capital account of the partners and cash account. Also draw the
balance sheet of AB (P) Ltd.
(CAP Dec. 2012 Q1-20 Marks)

Answer:
Dr. Realization Account Cr.
Particulars (Rs.) Particulars (Rs.)-
___________________________________________________________________________
__
To Machinery 1,20,000 By Creditors 70,000
To Land 1,74,000 By AB (P) Ltd.-Purchase
Consideration 6,00,000
(Refer Working Note)
To Motorcycles 30,000 By A’s Capital A/c 3,000
To Furniture & fittings 11,000 By B’s Capital A/c 6,000
To Stock 235,000 By C’s Capital A/c 42,000
(2000 + 40,000)
To Debtors 43,000 By Cash (Sale of Motorcycle) 13,000
To Cash (payment of creditors) 70,000
To Profit transferred to:
A’s Capital A/c 8,500
B’s Capital A/c 17,000

© The Institute of Chartered Accountants of Nepal 393


CAP II Paper 1: Advanced Accounting

C’s Capital A/c 25,500

Total 7,34,000 Total 7,34,000

Dr. Partners’ Capital Accounts Cr.


-
___________________________________________________________________________
__________
A B C A B
C

To Current A/c - - 100,000 By Balance b/d 100,000


200,000 3,00,000
To Realization A/c 3,000 6,000 42,000 By Current A/c 39,420
60,580 -
(Assets taken over)

To Equity Shares in
AB (P) Ltd. 300,000 300,000 - By A’s Loan 30,000
- -
To Cash - - 183.500 By Realization A/c 8,500
17,000 25,500
(Realization Profit)
By Cash A/c 125,080
28,420 -
Total 303,000 306,000 3,25,500 Total 303,000
306,000 325,500

Dr. Cash Account Cr.


Particulars (Rs.) Particulars (Rs.)-
___________________________________________________________________________
______
To Balance b/d 87,000 By Realization A/c 70,000
To Realization A/c 13,000 By C’s Capital A/c 183,500
To A’s Capital A/c 125,080
To B’s Capital A/c 28,420
Total 253,500 Total 2,53,500

Balance Sheet of AB (P) Ltd.


___________________________________________________________________________
__
Liabilities Rs. Assets Rs.

© The Institute of Chartered Accountants of Nepal 394


CAP II Paper 1: Advanced Accounting

Authorized Share Capital PPE:


12,000 Equity Shares of Rs. 100
each 1,200,000 Goodwill 88,000
Issued, Subscribed & Paid up: Land
174,000
6,000 Equity Shares of Rs. 100 Machinery
125,000
each fully paid up (shares were Furniture &Fixture
13,000
issued for consideration otherwise Current Assets:
than for cash) Stock 200,000
600,000
Total 600,000 Total
600,000

Working Note:

Calculation of Purchase Consideration

Assets taken over by AB (P) Ltd. (Rs.)


Machinery 125,000
Furniture & fittings 13,000
Land 174,000
Stock 200,000
Goodwill 88,000
Purchase Consideration 600,000

Purchase consideration is discharged by the issue of equal number of equity shares of Rs. 100
each (3,000 shares) at par to A & B.

13. Anil and Mohan who are equal partners carrying on manufacturing business as manufacture.
Their balance sheet as on 31st Ashadh 2069 was as follows:

Liabilities Rs. Assets Rs.


Sundry Creditors 55,500 Book Debts 60,000
Bank Overdraft 28,500 Stock in Trade 42,000
Bills Payable 10,500 Joint Life Policy-
surrender value 7,500
Anil's Capital A/C 33,000 Office Furniture 1,500
Mohan's Capital 52,500 Machinery & Plant 18,000
Leasehold Property 22,500
Anil's Drawing 9,000
Mohan's Drawing 3,000

© The Institute of Chartered Accountants of Nepal 395


CAP II Paper 1: Advanced Accounting

Profit & Loss A/C 16,500


Total 180,000 180,000

The business is carried on till 30th Poush 2069, by which time a net profit of Rs. 12,300
has been made for the half year after depreciation @ 10% per annum has been written off on
Leasehold Property. Meanwhile, Sundry Creditors have been reduced by Rs. 12,000, Bills
Payable by Rs. 2,925 and Bank Overdraft by Rs. 3,000. Partners' drawings for six months
amounted to Rs. 3,000 each. Stock in trade stood at Rs. 45,300 and book debts at Rs. 46,200
on 30th Poush 2069, other assets subject to any necessary adjustment, stood at the figure as on
31st Ashadh 2069.

In Chaitra 2069 the firm agrees to sell the business to Govind Ltd. on the basis that the stock
shall be taken over at a discount of 5% and book debts at a discount of 2.5% as on 30th Poush
2069. The company pays Rs. 7,500 for profits in the interval up to Chaitra 2069 subject to
drawings of Rs. 2,000 by Anil and Rs. 1,000 by Mohan during three months to Chaitra 2069;
the company doesn't take over the Joint Life Policy. The company takes over all other assets
and liabilities paying Rs. 37,500 for goodwill. The consideration in discharge by payment of
Rs. 60,000 in cash and the balance in preference shares of Rs. 10 each.
Prepare necessary accounts to close the books of the firm.
(CAP Dec. 2013 Q1-20 Marks)
Answer
Realization Account
Dr. Cr.
Chaitra Rs. Chaitra Rs.
2069 To Sundry Assets 2069 By Sundry Creditors 43,500
Stock 45,300 By Bank Overdraft 25,500
Machinery 18,000 By Bills Payable 7,575
Furniture 1,500 By Govind Ltd.-
Book Debts 46,200 consideration 94,380
JLP 7,500 By Bank (Joint Life
Leasehold Pro 21,375 Policy realized) 7,500
Misc. Assets 4,500 144,375
To Profit transferred to-
Anil's Cap A/C 17,040
Mohan's Cap A/C 17,040 34,080
178,455 178,455

© The Institute of Chartered Accountants of Nepal 396


CAP II Paper 1: Advanced Accounting

Bank Account
Dr. Cr.
Chaitra Rs. Chaitra Rs.
2069 2069
To Realization Account- 25,500 By Balance b/f 25,500
Transfer By Anil's Capital A/c 24,970
To Realization Account- 7,500 By Mohan's Capital 42,530
Joint Life Policy realized A/c
To Govind Ltd- Cash 60,000
received 93,000 93,000
Capital Accounts
Dr. Cr.
Date Particulars Anil Mohan Date Particulars Anil Mohan
2069 2069
Shrawan To Drawings 9,000 3,000 Shrawan By Balance b/d 33,000 52,500
1 To P/L 8,250 8,250 1 By Profit & loss App 6,150 6,150
Shrawan Account 3,000 3,000 Poush 30 A/C
1 To Drawings 2,000 1,000 Chaitra By Profit for three 3,750 3,750
Poush 30 To Drawings 37,690 64,190 31 months up Chaitra 31 17,040 17,040
Chaitra To Balance c/d By Realization A/C-
31 Profit
Chaitra
31
Total 59,940 79,440 Total 59,940 79,440
Chaitra To Preference Chaitra By Balance b/d 37,690 64,190
31 Shares in 31
Govind Ltd. 12,720 21,660
Chaitra To Bank 24,970 42,530
31
Total 37,690 64,190 Total 37,690 64,190

Working Note:-
Balance Sheet of the Firm as on 30th Poush 2069

Liabilities Rs Rs Assets Rs Rs
Sundry Creditors 43,500 Book Debts 46,200
Bank Overdraft 25,500 Stock in Trade 45,300
Bills Payable 7,575 JLP Surrender 7,500
Value
Anil's Capital Office Furniture 1,500
Balance as on 1st Shrawan 33,000 Machinery & Plant 18,000
2069

© The Institute of Chartered Accountants of Nepal 397


CAP II Paper 1: Advanced Accounting

Less: Loss up to 30th 12,000 Leasehold 22,500


Poush 2069 Property
Less: Loss up to 30th 2,100 18,900 Less: 10% Dep for 1,125 21.375
Poush 2069 6month
Mohan's Capital`
Balance as on 1st Shrawan 52,500
2069
Less: Drawings 6,000
Less: Loss up to 30th 2,100 44,400
Poush 2069
139,875 139,875

Note: Loss up to 31st Ashadh 2069 was Rs. 16,500: after that there has been a net profit of Rs.
12,300 reducing the loss to Rs. 4,200. This has been deducted from the partner's capital
accounts in the profit sharing ratio.

Govind Ltd., will pay Rs. 94,380 calculated as under:


Assets taken over:
Book Debts 2.5% less 45,045
Stock 5% less 43,035
Office Furniture 1,500
Machinery & Plant 18,000
Leasehold Property 21,375
Goodwill 37,500
Miscellaneous assets for profit up to 31st Chaitra 2069
Less: drawings (7,500- 2,000- 1,000) 4,500
Total 170,955
Less: Liabilities taken over
Sundry Creditors 43,500
Bank Overdraft 25,500
Bills Payable 7,575
Total 76,575
Net Assets to be paid for 94,380

Govind Ltd will pay Rs. 60,000 in cash and Rs. 34,380 in preference shares of Rs. 10 each,
the number of shares being 3438.
The preference shares 3,438 in number have been divided between Anil and Mohan in the
ratio of their final claims i.e., Anil Rs. 37,690 and Mohan Rs. 64,190.
Anil= (37,690 x 3,438)/101,880= 1,272= Rs. 12,720
Mohan= (64,190 x 3,438)/101,880= 2,166= Rs. 21,660
The balance due to them has been paid in cash

© The Institute of Chartered Accountants of Nepal 398


CAP II Paper 1: Advanced Accounting

14. Gold, Silver and Diamond are partners in a firm sharing profits and losses in the ratio of 5:3:2.
Gold retires on 32.03.2071 on which date the balance sheet of the firm stood as under:

Liabilities Amount (Rs.) Assets Amount (Rs.)


Capital Accounts: Goodwill 18,000
Gold 190,000 Furniture 45,000
Silver 120,000 Office Equipments 138,000
Diamond 100,000 410,000 Building 202,000
General Reserve 68,000 Stock 176,000
Sundry Creditors 182,000 Sundry Debtors 78,000
Bills Payable 14,000 Cash 17,000
674,000 674,000

It is provided in the deed of the partnership that in the event of the death or retirement of a
partner, goodwill is to be valued at 2 years’ purchase of the Average Profit of the last 4
years. The profits for the last 4 years are:

(Rs.)
• 2070-71: 46,000
• 2069-70: 40,000
• 2068-69: 10,000
• 2067-68: (28,000)
Furniture and Building are revalued at Rs. 40,000 and Rs. 225,000 respectively, stock is
overvalued by 10%. It was decided that Gold should be paid Rs. 140,000 immediately on
retirement and balance on his Capital account is to be treated as a loan to the firm. Silver
and Diamond contributed necessary sums in equal proportions for payment to Gold and to
leave Rs. 33,000 cash as working capital.

Assuming that above mentioned arrangements is given effect to; you are required to show
the Revaluation Account, Cash Account and Capital Account of the partners
(CAP Dec. 2014 Q 2a-10 Marks; CAP Jun. 2015 Q2a- 10 Marks)
Answer
In the Books of the Firm
Revaluation A/c
Dr. Cr.
Particulars Amount (Rs.) Particulars Amount (Rs.)
To Furniture A/c 5,000.00 By Building A/c 23,000.00
To Stock A/c 16,000.00
To Partners Capital A/c:
Gold 1,000.00
Silver 600.00
Diamond 400.00 2,000.00
23,000.00 23,000.00

© The Institute of Chartered Accountants of Nepal 399


CAP II Paper 1: Advanced Accounting

Cash A/c
Dr Cr.
Particulars Amount (Rs.) Particulars Amount (Rs.)
To Balance b/d 17,000.00 By Gold Capital A/c 140,000.00
To Partners Capital A/c:
(Bal Figure)
Silver 78,000.00
Diamond 78,000.00 156,000.00 By Balance c/d 33,000.00
173,000.00 173,000.00

Partners’ Capital A/c


Dr Cr
Particulars Gold Silver Diamond Particulars Gold Silver Diamond
To Bank A/c 140,000 By Balance b/d 190,000 120,000 100,000
To Gold 93,000 By General 34,000 20,400 13,600
Loan A/c Reserve A/c
To Goodwill 20,400 13,600 By Goodwill 8,000 4,800 3,200
A/c A/c
By Revaluation 1,000 600 400
A/c
To Balance 203,400 181,600 By Cash A/c 78,000 78,000
c/d
233,000 223,800 195,200 233,000 223,800 195,200

Calculation of Goodwill and its Treatment

2067-68 (28,000.00)
2068-69 10,000.00
2069-70 40,000.00
2070-71 46,000.00
Total 68,000.00
Average Profit of the last 4 years 17,000.00
Goodwill 2 years Average Profit 34,000.00

Goodwill already in the books of 18,000.00


accounts
Additional Goodwill 16,000.00

Goodwill Account has already been raised by Rs. 18,000 therefore before Gold’s retirement it
should be raised by Rs. 16,000(34,000-18,000) in the current profit sharing ratio 5:3:2.
Gold 5/10 X 16,000=8,000
Silver 3/10 X 16,000=4,800

© The Institute of Chartered Accountants of Nepal 400


CAP II Paper 1: Advanced Accounting

Diamond 2/10 X 16,000=3,200


Total 16,000

Therefore Goodwill is to be written off in the new profit sharing Ratio 3:2
Silver 3/5 X 34,000=20,400
Diamond 2/5 X 16,000 =13,600
Total 34,000

Profit Sharing Ratio


Old new
Gold 5
Silver 3 3
Diamond 2 2
Total 10 5

Actual Value of Stock


Value of Stock=Rs 17600 x 100 = Rs. 160000
110
Therefore it is Overvalued By (176,000-160,000) = Rs. 16,000

15. Amar, Binod, Chandra and Dev were in partnership sharing profits and losses in the ratio
3:3:2:2. The Balance Sheet of the partners as on 31st Ashadh 2072 was as under:

Capital and Liabilities Amount Assets Amount


Capital accounts Cash at bank 2,000
Amar 20,000 Sundry debtors 16000
Binod 15,000 35,000 Less: Provision for
Amar’s loan 10,000 bad debts 500 15,500
Sundry creditors 15,500 Stock 10,000
Furniture and fittings 4,000
Trade marks 7,000
Capital accounts
Chandra 16,000
Dev 6,000 22,000
60,500 60,500

On 31st Ashadh 2072, the partnership firm was dissolved and Binod was appointed to realize
the assets and pay off the liabilities. He was entitled to commission @ 5% on amounts finally
paid to other partners as capital. He was also to bear the expenses on realization. The assets
realized as follows:
Sundry debtors 11,000
Stock 8,000

© The Institute of Chartered Accountants of Nepal 401


CAP II Paper 1: Advanced Accounting

Furniture and fittings 1,000


Trade Marks 4,000

Creditors were paid in full; in addition, a contingent liability for bills receivable discounted
materialized to the extent of Rs. 2,500. Also there was a joint life policy of Rs.30,000 which
was surrendered for Rs.3,000. Expenses of realization amounted to Rs.500. Chandra was
insolvent but a sum of Rs.3,700 was recovered from his estate.

You are required to write up the Cash Account, Realization Account and Partners Capital
accounts to close the books of the firm. (CAP Dec. 2015 Q2b)

Answer
Cash Book
Date Particulars Amount Date Particulars Amount
2072 2072
31/3 To balance b/d 2,000 31/3 By Sundry Creditors 15,500
31/3 To Realization A/c 27,000 31/3 By Realization A/c 2,500
Assets realized 31/3 By Amar’s Loan A/c 10,000
31/3 To Chandra's Capital A/c 3,700 31/3 By Amar’s Capital A/c 11,219
31/3 To Binod’s Capital A/c 3,600 31/3 By Binod’s Capital A/c 9,081
31/3 To Amar’s Capital A/c 3,600
31/3 To Dev’s Capital A/c 8,400

48,300 48,300

Realization Account

Date Particulars Amount Date Particulars Amount


2072 2072
31/3 To Sundry Assets 31/3 By Provision for Bad dets 500
Sundry Debtors 16,000 31/3 By Cash
Stock 10,000 31/3 Sundry Debtors 11,000
Furniture fittings 4,000 Stock 8,000
Trade Marks 7,000 37,000 Furnitures etc. 1,000
Trade Marks 4,000
31/3 To Cash (Bill Discounted) 2,500 Joint Life Policy 3,000 27,000
By Net Loss Transferred to:
Amar's Capital A/c 3,600
Binod's Capital A/c 3,600
Chandra's Capital A/c 2,400
Dev’s Capital A/c 2,400 12,000
39,500 39,500

© The Institute of Chartered Accountants of Nepal 402


CAP II Paper 1: Advanced Accounting

Partners' Capital Account

Particulars Amar Binod Particulars Amar Binod


To Realization A/c 3,600 3,600 By balance b/fd 20,000 15,000
To Chandra’s Capital A/c 8,400 6,300 By Cash A/c 3,600 3,600
(Deficiency written off) By Amar’s Capital A/c - 381
To Binod’s Capital A/c 381 - (5% Commission on
(Commission) Rs.7,619)
To Cash 11,219 9,081
23,600 18,981 23,600 18,981

Partners' Capital Account

Particulars Chandra Dev Particulars Chandra Dev


To balance b/fwd 16,000 6,000 By Cash 3,700 8,400
To Realization 2,400 2,400 By Amar’s Capital A/c 8,400 -
By Binod’s Capital A/c 6,300 -

18,400 8,400 18,400 8,400

Working Notes
1. Since Dev has a debit balance in his capital account on the date of dissolution, he is not
required to bear any proportion of loss due to Chandra’s insolvency. Only Amar and
Binod has to bear the loss in the ratio of their capital i.e. 4:3.
2. The commission payable to Binod is 5% on the amount paid to the partners as capital.
Only Amar receives such an amount. That comes to Rs.8,000 not counting the
commission payable to Binod. The commission is 8,000X5/105 i.e. Rs.381. or
11600*5/105
3. The actual expenses of realization have been assumed to be paid by Binod privately. If
the firm pays the amount, the amount will be debited to Binod’s capital and credited to
cash. In such a case, the amount finally paid to Binod comes to Rs.8,581.

16. Eddi and Freddi carrying on business in partnership sharing Profit and Losses equally, wished
to dissolve the firm and sell the business to Sreddi Ltd. on 31.03.2074, when the firm’s position
was as follows:

Balance Sheet as at 31.03.2074


Liabilities Amount (Rs.) Assets Amount (Rs.)
Eddi’s Capital 175,000 Land and Building 125,000
Freddi’s Capital 130,000 Furniture 52,500
Sundry Creditors 80,000 Stock 112,500

© The Institute of Chartered Accountants of Nepal 403


CAP II Paper 1: Advanced Accounting

Debtors 91,000
Cash 4,000
Total 385,000 Total 385,000

The arrangement with Sreddi Limited Company was as follows:


i) Land and Building was purchased at 20% more than the book value.
ii) Furniture and stock were purchased at book values less 15%.
iii) The goodwill of the firm was valued at Rs. 40,000.
iv) The firm’s debtors, cash and creditors were not to be taken over, but the company agreed
to collect the book debts of the firm and discharge the creditors of the firm as an agent,
for which services, the company was to be paid 5% on all collections from the firm’s
debtors and 3% on cash paid to firm’s creditors.
v) The purchase price was to be discharged by the company in fully paid equity shares of Rs.
10 each at a premium of Rs. 2 per share.

The company collected all the amounts from debtors. The creditors were paid off less by Rs.
1,000 allowed by them as discount. The company paid the balance due to the vendors in cash.

Prepare the Realization Account, the Capital Accounts of the partners and the Cash Account
in the books of partnership firm. (CAP Dec. 2017 Q2a-10 Marks)

Answer
Amount in NPR
Dr Realization A/c Cr
Particulars Amount Particulars Amount
To Land & Building 125,000 By Sundry Creditors 80,000
To Furniture 52,500 By Sreddi Ltd Co- 330,250
Purchase consideration
(W.N-1)
To Stock 112,500 By Sreddi Ltd Co-
Sundry Debtors
91,000
To Debtors 91,000 Less- Commission 86,450
To Sreddi Ltd Co- Sundry Creditors 79,000 (5% on 91,000)
To Sreddi Ltd Co- Commission 2,370 4,550
(3% on 79,000)
To Profit transferred to Eddi’s 34,330
Capital 17,165
Freddi’s Capital 17,165
Total 496,700 Total 496,700

Amount in NPR
Dr Capital Account Cr

© The Institute of Chartered Accountants of Nepal 404


CAP II Paper 1: Advanced Accounting

Particulars Eddi Freddi Particulars Eddi Freddi


To Shares in Sreddi 187,023 143,227 By Balance b/d 175,000 130,000
Ltd (W.N-2)
To Cash –Final 5,142 3,938 By Realization A/c 17,165 17,165
Payment
Total 192,165 147,165 Total 192,165 147,165

Amount in NPR
Dr Cash Account Cr
Particulars Amount Particulars Amount
To Balance b/d 4,000 By Eddi’s Capital – Final 5,142
payment
To Sreddi Ltd Co- 5,080 By Freddi’s Capital – 3,938
(Amount realized from Debtors less Final payment
amount paid to creditors)
( W.N-3)
Total 9,080 Total 9,080

Working Notes
1. Computation of Purchase Consideration
Particulars Amount
Land & Building (125,000+20% of 125,000) 150,000
Furniture (52,500-15% of 52,500) 44,625
Stock (112,500-15 % of 112,500) 95,625
Goodwill ( given) 40,000
Total 330,250

2. The shares received from the company have been distributed between the two partners
Eddi & Freddi in the ratio of their final claims i.e., 192,165: 147,165
330,250
No of shares received from the company= = 27,521
12
192,165
Eddi gets (27,521 330,250 ) = 15,585 Shares valued at 15,585*12 =NPR.187, 023.
Freddi gets the remaining 11,936 shares valued at 11,936*12 = NPR.143, 227
3. Calculation of net amount received from Sreddi Ltd on account of amount realized
from debtors less amount paid to creditors.
Particulars Amount
Amount realized from debtors 91,000
Less: Commission for realization from debtors (5% on 91,000) 4,550
86,450
Less: Amount paid to creditors 79,000
7,450
Less: Commission for cash paid to creditors (3% on 79,000) 2,370
Total 5,080

© The Institute of Chartered Accountants of Nepal 405


CAP II Paper 1: Advanced Accounting

17. M, N and O were in partnership with a capital of Rs. 30,000 originally contributed in the
proportions of 1/2, 1/3 and 1/6 respectively and sharing profits and losses in the same
proportions. The partnership was dissolved on March 31, the balance sheet on which date was
as follows:

Liabilities Rs. Assets Rs.


Capitals - M 20,000 Cash 4,000
N 10,000 Debtors 16,000
O 2,000 Stock 42,000
Loans - M 6,000
N 4,000
Creditors 20,000
62,000 62,000

It was agreed that the net realizations should be distributed in their due order at the end of
each calendar month. The realizations and expenses were as follows:
Debtors Stock Expenses
April 4,000 8,000 1,000
May 1,000 10,000 500
June 6,000 11,000 1,000
July 1,000 10,000 400
August 3,000 2,000 500
The stock having been completely disposed off, it was agreed that O should take over the
remaining debts at Rs.600.You are required to prepare a statement showing distribution of cash
under Highest Relative Capital Method. (Inter Dec. 2008 Q 3-15 Marks)

Answer
Statement showing distribution of Cash
Creditors M's N's M's N's O's
Loan Loan Capital Capital Capital
(Rs.) (Rs.) (Rs.) (Rs.) (Rs.) (Rs.)
Balance due 20,000 6,000 4,000 20,000 10,000 2,000
Cash in hand - paid to creditors 4,000 - - - - -
Balance due 16,000 6,000 4,000 20,000 10,000 2,000

April
Balance available Rs. 11,000 (WN.1)
Amount paid to creditors 11,000 - - - - -
Balance due 5,000 6,000 4,000 20,000 10,000 2,000

May
Balance available Rs 10,500 (WN.1)
Amount paid:

© The Institute of Chartered Accountants of Nepal 406


CAP II Paper 1: Advanced Accounting

a. Rs. 5,000 paid to creditors 5,000 - - - - -


b. Rs. 5,500 to M's or N's Loan
in the ratio of their loan(6:4) - 3,300 2,200 - - -
Balance due - 2,700 1,800 20,000 10,000 2,000

June
Balance available Rs. 16,000 (WN.1)
Amount paid:
a. Rs. 2,700 to M's Loan & Rs.
1,800 to N's Loan - 2,700 1,800 - - -
b. Rs. 5,000 to M's capital (WN.2) - - - 5,000 - -
c. Balance Rs. 6,500 to M's or N's capital
in their profit sharing ratio (3:2) (WN.2) - - - 3,900 2,600 -
Balance due - - - 11,100 7,400 2,000

July
Balance available Rs. 10,600 (WN1)
Amount paid:
a. Rs. 5,100 to M's capital or Rs. 3,400 to N's capital - - - 5,100 3,400-
b. Balance Rs. 2,100 to three partners in their
profit sharing ratio - - - 1,050 700 350
Balance due - - - 4,950 3,300 1,650

August
Amount available Cash Rs 4500 (WN.1)
and debtor Rs 600 paid to three partners
in profit sharing ratio - - - 2,550 1,700 850
Balance unpaid or loss - - - 2,400 1,600 800

Working Notes:

1. The cash available at the end of each month is calculated as follows:

i.e. Debtors + Stock - Expenses = Cash available

April = 4,000 + 8,000 - 1,000 = 11,000


May = 1,000 + 10,000 - 500 = 10,500
June = 6,000 + 11,000 - 1,000 = 16,000
July = 1,000 + 10,000 - 400 = 10,600
August = 3,000 + 2,000 - 500 = 4,500
At the end of August, the total amount to be distributed is Rs. 4,500 cash + Rs. 600 debtors,
i.e. Rs. 5,100

2. Highest Relative Capital Basis

© The Institute of Chartered Accountants of Nepal 407


CAP II Paper 1: Advanced Accounting

M (Rs.) N (Rs.) O (Rs.)


Balance of Capital Account 20,000 10,000 2,000
Profit sharing ratio 3 2 1
Proportionate capital (taking O's Capital as base) 6,000 4,000 2,000
Excess capitals 14,000 6,000 -
Proportionate capital between M & N
(Taking N's Capital as base) 15,000 10,000 -
M's excess actual over proportionate 5,000

18. Mohan, Rohan and Sohan were partners sharing profits and losses at the ratio of 2:2:1. Sohan
wants to retire on 31.03.2066. Given below is the Balance Sheet of the partnership as well as
other information:
Balance Sheet as on 31.03.2066
Liabilities Rs. Assets Rs.
Capital Accounts: Sundry Fixed Assets 1,50,000
Mohan 1,20,000 Stocks 50,000
Rohan 80,000 Debtors 50,000
Sohan 60,000 Bills receivables 20,000
Reserve 10,000 Bank Balances 50,000
Sundry Creditors 50,000
3,20,000 3,20,000

Mohan and Rohan agree to share profits and losses at the ratio of 3:2 in the future. Value of
goodwill is taken to be Rs. 50,000. Sundry Fixed Assets are revalued upward by Rs. 30,000
and stocks by Rs. 10,000. Bills receivable dishonored Rs. 5,000 on 31.03.2066 but not
recorded in the books. It was due to insolvency of a customer. Mohan and Rohan agree to
bring sufficient cash to discharge the claim of Sohan and to make their capital proportionate.
They also wanted to maintain Rs. 75,000 bank balance for working capital requirements.
However, they did not want to show goodwill in the books of accounts. Pass necessary
journal entries to give effect for the retirement and draft the Balance Sheet of M/S Mohan
and Rohan. (Inter Dec. 2009 Q3-15 Marks)

Answer
Journal Entries
Particulars Dr Cr
Rs. Rs.
1. Goodwill A/c 50,000
To Mohan's Capital A/c 20,000
To Rohan's Capital A/c 20,000
To Sohan's Capital A/c 10,000

© The Institute of Chartered Accountants of Nepal 408


CAP II Paper 1: Advanced Accounting

(Being the goodwill raised on Sohan's retirement)


2. Reserve A/c 10,000 4,000
To Mohan's Capital A/c 4,000
To Rohan's Capital A/c 2,000
To Sohan's Capital A/c
(Being transfer of reserve to Partner's Capital A/cs )
3. Sundry Fixed Assets A/c 30,000
Stock A/C 10,000
To Profit and Loss Adj. A/c 40,000
(Being upward revaluation of Assets recorded)
4. Profit and Loss Adj. A/c 5,000
To Bills Receivable A/c 5,000
(Being loss arising due to Bill dishonored recorded )
5. Profit and Loss Adj. A/c 35,000
To Mohan's Capital A/c 14,000
To Rohan's Capital A/c 14,000
To Sohan's Capital A/c 7,000
(Being Profit on revaluation transferred to Partner's
Capital A/cs )
6. Mohan's Capital A/c 30,000
Rohan's Capital A/c 20,000
To Goodwill A/c 50,000
(Being Goodwill written off in the new profit sharing
ratio)
7. Bank A/c 1,04,000
To Mohan's Capital A/c 70,000
To Rohan's Capital A/c 34,000
(Being cash brought in by Mohan and Rohan)
8. Sohan's Capital A/c 79,000
To Bank A/c 79,000
(Being final payment made to Sohan on his retirement)

M/S Mohan and Rohan


Balance Sheet as at 01.04.2066
Liabilities Rs. Assets Rs.
Capital Accounts: Sundry Fixed Assets 1,80,000
Mohan 1,98,000 Stock 60,000
Rohan 1,32,000 Debtors 50,000
Sundry Creditors 50,000 Bills receivable 15,000
Bank Balances 75,000
3,80,000 3,80,000

© The Institute of Chartered Accountants of Nepal 409


CAP II Paper 1: Advanced Accounting

Working notes:
1. Partner's Capital A/cs:
Particulars Mohan Rohan Sohan Particulars Mohan Rohan Sohan
Rs. Rs. Rs. Rs. Rs. Rs.
To Goodwill 30,000 20,000 - By Bal.b/d 1,20,000 80,000 60,000
To Bal. c/d 1,28,000 98,000 79,000 By Goodwill 20,000 20,000 10,000
By P&L Adj. 14,000 14,000 7,000
By Reserve 4,000 4,000 2,000
1,58,000 1,18,000 79,000 1,58,000 1,18,000 79,000
To Bank - - 79,000 By Bal. b/d 1,28,000 98,000 79,000
To Bal. c/d 1,98,000 1,32,000 - By Bank 70,000 34,000 -
1,98,000 1,32,000 79,000 1,98,000 1,32,000 79,000

2. Bank A/c:
Particulars Rs. Particulars Rs.
To Balance b/d 50,000 By Sohan's Capital A/c 79,000
To Mohan's Capital A/c 70,000 By Balance b/d 75,000
To Mohan's Capital A/c 34,000
1,54,000 1,54,000

3. Calculation of continuing Partner's Capital:


Rs.
Sundry Fixed Assets (Rs.1,50,000 + Rs.30,000) 1,80,000
Stock (Rs.50,000 + Rs.10,000) 60,000
Debtors 50,000
Bills Receivable (Rs.20,000 – Rs.5,000) 15,000
Bank Balances 75,000
3,80,000
Less: Sundry Creditors 50,000
3,30,000
Mohan's Share (Rs.3,30,000 × 3/5) 1,98,000
Rohan's Share (Rs.3,30,000 × 2/5) 1,32,000

19. Suman and Samir are chartered accountants, practicing independently at Kathmandu and
Pokhara respectively and decide to, with effect from 1.4.2066 to form a partnership in which
each of them will have an equal share in profits and Losses. They decide to bring in to the Firm
all their assets and Liabilities of their proprietary practices except the outstanding due on
Government Audit work done prior to 31.3.2066, which will be to the benefit of the individuals
concerned.
It was also decided that a sum of goodwill, equivalent to 1.5 times the value of Net Assets
taken over will be credited to the partners Account.

Details of Assets and liabilities as 31.3.2066, were as under –

© The Institute of Chartered Accountants of Nepal 410


CAP II Paper 1: Advanced Accounting

Particulars Suman Samir


Cash on Hand/ Bank 16,000 12,000
Fixed Deposits (out of Professional Receipts) 40,000 26,000
Amount Billed and Receivable 38,000 32,000
Bills still to be raised ( including Rs.24000 due to Suman on 160,000 82,000
Govt Audit work)
Office Furniture, Fixtures, Telephone Lines etc. 31,000 18,000
Amount due to Articled and Audit Clerks stipends 8,400 12,000
Rent Deposit 18,000 18,000
Rent Due 1,500 1,500
Office Salaries Due 6,000 4,500
Expenses to be paid 800 -

For the year 2066-67, the analysis of Bank / Cash account revealed the following –

Particular Rs. Particular Rs.


Gross Fee Received ( including Rs. 442,000 Stipends paid 60,000
18000 due to Suman Government Audit
work)
Interest on Fixed Deposits credited 6,600 Salaries to staff 82,000
Rent 36,000 Suman Drawings 50,000
Printing and Stationery 40,000 Samir Drawings 36,000
Office Expenses 18,800

Other information as on 31.3.2067 –


(a) Bills due by clients – Rs. 144,000;
(b) Bills still to be raised on clients – Rs.60,000; and
(c) Salaries Due – Rs. 9,600.

There were no outstanding bills for collection as on 31.3.2067 that related to earlier years.
The partners decided that from 1.4.2066, they will adopt the mercantile system of accounts
for all items except professional receipts which will be accounted on cash basis. Interest of
Fixed Deposits is to be shared in the ratio of holdings at the time of formation of the firm.
Depreciate Furniture and Fixture at 10%.
Prepare the profit and Loss A/c of the Firm for the year ended 31.3.2067, and also a Balance
Sheet as on that date. (Inter Dec. 2010 Q1-20 Marks)
Answer
1. Valuation of Goodwill

Particulars Suman Samir


Assets:
Cash In hand / bank 16,000 12,000

© The Institute of Chartered Accountants of Nepal 411


CAP II Paper 1: Advanced Accounting

Fixed Deposits 40,000 26,000


Sundry Debtors :
- Bills raised but due 38,000 32,000
- Bills not yet raised excluding Gov. audit 136,000 82,000
Rent deposits 18,000 18,000
Furniture 31,000 18,000
Total Assets 279,000 188,000
Less : Outstanding
- Stipend 8,400 12000
- Salaries 6,000 4500
- Rent 1,500 1500
- Expenses 800 (16,700) - (18,000)
Net Assets taken over 262,300 170,000
Goodwill (1.5 Times of net assets taken 393,450 255,000
over)

2 Opening Balance of Capital Account

Particulars Suman Samir


Net Assets Taken Over 262,300 170,000
Add: Goodwill 393,450 255,000
Less : Sundry Debtors ( See Notes) (144,000) (144,000)
Opening Balance of Capital Account 511,750 281,000

Notes:
➢ Total Debtors (other than those relating to Government audit) is Rs. 288,000 i.e. Suman
Rs. 38,000+136,000 and Samir Rs. 32,000+ Rs. 82,000.
➢ Professional Receipts are accounted for on Receipt Basis by the Firm and therefore
when the above debtors are realized, the partnership will sustain and therefore, they will
be shared equally by the partners.
➢ Consequently, the amount due from the above audit work for Rs. 288,000 is reduced
from the partner's capital Accounts.

3 Profit and loss Account for the year ended 31.3.2067

Particulars Rs. Rs. Particulars Rs. Rs.


To office Expenses 18,800 By Fees 442,000
Less: Previous year (800) 18,000 Less: (18,000) 424,000
outstanding Sunman's
Govt. Audit
To Rent 36,000
Less: Previous year (3,000) 33,000
outstanding

© The Institute of Chartered Accountants of Nepal 412


CAP II Paper 1: Advanced Accounting

To Printing and stationery 40,000


To Stipends 60,000
Less Previous year (20,400) 39,600
outstanding
To Salaries to Staff 82,000
Less Previous year (10,500)
outstanding
Add: Current year outstanding 9,600 81,100
To Depreciation on furniture 4,900
and fittings ( 10% on 49000)
To Profit transferred to Capital
A/c
- Suman 103,700
- Samir 103,700 207,400
Total 424,000 Total 424,000

4 Cash Account

Particulars Rs Particulars Rs
To balance b/d 28,000 By Office Expenses 18,800
To Professional Charges 442,000 By Rent 36,000
Received
To interest on Fixed Deposits 6,600 By Printing and Stationery 40,000
By Stipends 60,000
By Salaries 82,000
By Drawing :
- Suman 50,000
- Samir 36,000
By balance c/d 153,800
Total 476,600 Total 476,600
5. Balance Sheet of Suman and Samir as on 31.3.2067

Equity and Liabilities Rs Assets Rs


Capital A//C : Goodwill 648,450
- Suman 587,450 Furniture, Telephone 49000
- Samir 351,300 Less: Depreciation (4900) 44,100
Fixed deposits 66,000
Rent Deposits 36,000
Outstanding Salaries 9,600 Cash and Bank 153,800
Total 948,350 Total 948,350
6 Partner's Capital Account

© The Institute of Chartered Accountants of Nepal 413


CAP II Paper 1: Advanced Accounting

Particulars Suman Samir Particulars Suman Samir


To Drawing 50,000 36,000 By balance b/d 511,750 281,000
To balance c/d 587,450 351,300 By Fees 18,000 -
By Interest on Fixed de3p. 4,000 2,600
(20:13)
By Profit and Loss Account 103,700 103,700
Total 637,450 387,300 Total 673,450 387,300

Alternative Solution:

It can be assumed that the outstanding Fees at the beginning of the year was not brought to
profit and Loss Account, but taken directly to the partner's Capital Account. In such an event,
the year's receipts will be lower by R. 288,000 and consequently there will be loss of Rs.
80600 i.e. Present Profit Rs. 207,400 Less by Professional Receipts credited to Capital
Account Rs. 288000. The partner's Capital Account will be as under:

Particulars Suman Samir Particulars Suman Samir


To Drawing 50,000 36,000 By balance b/d 655,750 425,000
To Profit and 40,300 40,300 By Fees 18,000 -
Loss Account
To balance c/d 587,450 351,300 By Interest on FD ( 20:13) 4,000 2,600
Total 637,450 387,300 Total 637,450 387,300

20. Anil, Sunil and Rahim carried on a manufacturing business in partnership sharing profits and
losses in the ratio of 5:3:2. It was agreed that Sunil should retire from the partnership on 31st
March, 2011 and a private limited company, SR Ltd, was formed to take over the business and
certain assets of the partnership on that date.
The authorized capital of SR Ltd. was Rs. 700,000 divided into 10,000 12% preference shares
of Rs. 20 each and 25,000 ordinary shares of Rs. 20 each.
The firm's Balance Sheet as on 31st March, 2011 was as follows:
Capital & Liabilities Rs. Rs. Assets Rs. Rs.
Capital Accounts: Fixed assets less depreciation:
Anil 200,000 Machinery 160,000
Sunil 184,000 Motor cars 52,000
Rahim 96,000 480,000 Furniture 28,000 240,000
Current Accounts: Goodwill 40,000
Anil 90,000 Life assurance policy 70,000
Sunil (10,000) Current assets:
Rahim 40,000 120,000 Stock 180,000
Loan Account – Anil 120,000 Debtors 116,000
Creditors 70,000 Bank balance 144,000 440,000
790,000 790,000

© The Institute of Chartered Accountants of Nepal 414


CAP II Paper 1: Advanced Accounting

Sunil owned the freehold premises occupied by the firm and he agreed to sell them to the
company in return for issue to him of 2,000 preference shares at par. The company agreed to
discharge Anil's loan by the issue to him of 6,000 preference shares at par.
The life assurance policy had been taken out on the life of Sunil and was surrendered for
Rs. 84,000, which was received by the partners on 1st April, 2011.
On his retirement Anil took over one of the cars at the book value of Rs. 14,000. The
furniture in his private office was his own property, for which the company paid him
Rs. 5,000. He agreed further to take over the partnership debtors at the balance sheet
value less 5%, and to pay the creditors. SR Ltd. agreed to purchase the remaining assets,
including the bank balance for Rs. 520,000, the purchase consideration being discharged
by the payment of Rs. 80,000 in cash and the issue of 22,000 ordinary shares to Anil,
Sunil, and Rahim in the proportion in which they shared profits and losses.
Formation expenses of Rs. 11,000 were paid by the company. For the purpose of opening
entries in the company's book the assets taken over from partnership were valued: Machinery
Rs. 140,000; Motor cars Rs. 40,000; Furniture Rs. 20,000; and Stock Rs. 150,000.

You are required to prepare:


g) The realization account of partnership.
h) The partners' combined capital and current account showing the dissolution of the
partnership.
i) The opening balance sheet of SR Ltd. as on 1st April, 2011.
(Inter Dec. 2011 Q1-20 Marks)
Answer:
Realization Account
Particulars Rs. Rs. Particulars Rs.
To Sundry assets at book value 790,000 By Creditors 70,000
To Anil's capital – creditors 70,000 By Cash – surrender of policy 84,000
To Partner's capital – profit: By Anil's capital – motor car 14,000
Anil (1/2) 29,100 By Anil's capital – debtors* 110,200
Sunil (3/10) 17,460 By SR Ltd – consideration 640,000
Rahim (1/5) 11,640 58,200
918,200 918,200
* Rs. 116,000 less 5%.
Anil's Capital & Current Account
Particulars Rs. Particulars Rs.
To Realization A/c – motor cars 14,000 By Balance b/d 290,000
To Realization A/c – debtors 110,200 By Realization A/c – creditors 70,000
To Shares in SR Ltd. 220,000 By Realization A/c – profit 29,100
To Cash 44,900
389,100 389,100
Sunil's Capital & Current Account
Particulars Rs. Particulars Rs.
To Shares in SR Ltd. 132,000 By Balance b/d 174,000

© The Institute of Chartered Accountants of Nepal 415


CAP II Paper 1: Advanced Accounting

To Cash 59,460 By Realization A/c – profit 17,460


191,460 191,460
Rahim's Capital & Current Account
Particulars Rs. Particulars Rs.
To Shares in SR Ltd. 88,000 By Balance b/d 136,000
To Cash 59,640 By Realization A/c – profit 11,640
147,640 147,640

SR Ltd.
Balance Sheet as on 1st April, 2011
Liabilities Rs. Assets Rs. Rs.
Share capital: Fixed assets:
Authorized: Goodwill 146,000
10,000 12% pref. shares of Rs. 20 each 200,000 Premises 40,000
25,000 ordinary shares of Rs. 20 each 500,000 Machinery 140,000
700,000
Issued and paid up: Motor cars 40,000
8,000 12% pref. shares of Rs. 20 each 160,000 Furniture 25,000 391,000
22,000 ordinary shares of Rs. 20 each 440,000 Current assets:
Stock 150,000
Bank balance 48,000 198,000
Preliminary expenses 11,000
600,000 600,000

Working notes:
(i) Purchase consideration:
Rs.
Cash 80,000
Ordinary shares (Rs. 520,000 – Rs. 80,000) 440,000
Preference shares – discharge of Anil's loan 120,000
Total 640,000
(ii) Computation of goodwill:
Rs. Rs.
Purchase consideration 640,000
Less: Assets taken over:
Machinery 140,000
Motor cars 40,000
Furniture 20,000
Stock 150,000
Bank balance 144,000 494,000
Goodwill 146,000
(iii) Bank balance:

© The Institute of Chartered Accountants of Nepal 416


CAP II Paper 1: Advanced Accounting

Rs. Rs.
Bank balance taken over 144,000
Less: Purchase consideration discharged 80,000
Purchase of furniture from Anil 5,000
Formation expenses 11,000 96,000
Balance 48,000
(iv) Preference shares issued:
Rs.
For discharging Anil's loan (Rs. 20 × 6,000) 120,000
For purchasing premises owned by Anil (Rs. 20 × 2,000) 40,000
Total 160,000
(v) Furniture:
Rs.
Taken over from partnership 20,000
Separately purchased Anil's personal furniture 5,000
Total 25,000

21. A’ and ‘B’ are in partnership sharing profits and losses equally. They keep their books by single
entry system. The following balances are available from their books as on 31.3.2006 and
31.3.2007:
31.3.2006 31.3.2007
NRs. NRs.
Building 1,50,000 1,50,000
Equipments 2,40,000 2,72,000
Furniture 25,000 25,000
Debtors ? 1,00,000
Creditors 65,000 ?
Stock ? 70,000
Bank loan 45,000 35,000
Cash 60,000 ?

The transactions during the year ended 31.3.2007 were the following:
NRs.
Collection from debtors 3,80,000
Payment to creditors 2,50,000
Cash purchases 65,000
Expenses paid 40,000
Drawings by ‘A’ 30,000

• On 1.4.2006 an equipment of book value NRs.20,000 was sold for NRs.15,000. On


1.10.2006, some equipments were purchased.
• Cash sales amounted to 10% of sales.
• Credit sales amounted to NRs.4,50,000.

© The Institute of Chartered Accountants of Nepal 417


CAP II Paper 1: Advanced Accounting

• Credit purchases were 80% of total purchases.


• The firm sells goods at cost plus 25%.
• Discount allowed NRs.5,500 during the year.
• Discount earned NRs.4,800 during the year.
• Outstanding expenses NRs.3,000 as on 31.3.2007.
• Capital of ‘A’ as on 31.3.2006 was NRs.15,000 more than the capital of ‘B’,
equipments and furniture to be depreciated at 10% p.a. and building @ 2%
p.a.

You are required to prepare:


j) Trading and Profit and Loss account for the year ended 31.3.2007 and
k) The Balance Sheet as on that date. (CAP Jun. 2009 Q1- 20 Marks)

Answer:
a)Trading and Profit and Loss A/c
for the year ended 31.3.2007
Debit Credit
Particulars NRs. Particulars NRs.
To Opening stock (W.N.3) 145,000 By Sales- Cash (W.N.1) 50,000
To Purchases-Cash 65,000 Credit 4,50,000 500,000
Credit (W.N.2) 2,60,000 325,000 By Closing stock 70,000
To Gross profit c/d 100,000
Total 570,000 Total 570,000
To Loss on sale of equip By Gross profit b/d 100,000
(20,000-15,000) 5,000
To Depreciation By Discount received 4,800
Building 3,000
Furniture 2,500
Equipment (W.N.4) 24,600 30,100
To Expenses paid 40,000
Add : Outstanding exp. 3,000 43,000
To Discount allowed 5,500
To Net profit transferred to:
A’s capital A/c 10,600
B’s capital A/c 10,600 21,200
Total 104,800 Total 104,800

b) Balance Sheet as on 31-3-2007


Equity and Liabilities NRs. Assets NRs.
A’s capital (W.N.7) 280,250 Building 1,50,000
Less: Drawings (30,000) Less: Depreciation 3,000 147,000
Add: Net profit 10,600 260,850 Equipments 2,72,000
B’s capital (W.N.7) 265,250 Less: Depreciation 24,600 247,400

© The Institute of Chartered Accountants of Nepal 418


CAP II Paper 1: Advanced Accounting

Add: Net profit 10,600 275,850 Furniture 25,000


Sundry creditors (W.N.5) 70,200 Less: Depreciation 2,500 22,500
Bank loan 35,000 Debtors 100,000
Outstanding expenses 3,000 Stock 70,000
Cash balance (W.N.8) 58,000
Total 644,900 Total 644,900

Working Notes:
W.N. 1: Calculation of total sales and cost of goods sold
Cash sales = 10% of total sales
Credit sales = 90% of total sales = NRs.450,000
450,000
Total sales = 100 = 5,00,000
90
Cash sales = 10% of 500,000 = NRs.50,000

W.N. 2: Calculation of total purchases and credit purchases


Cash purchases = NRs.65,000
Credit purchases = 80% of total purchases
Cash purchases = 20% of total purchases
65,000
Total purchases = 100 = Rs.3,25,000
20
Credit purchases = 325,000 – 65,000 = NRs.260,000

W.N. 3: Calculation of opening stock

Debit Stock Account Credit


Particulars NRs. Particulars NRs.
To Balance b/d (Bal. Fig.) 145,000 By Cost of goods sold
500,000
 100
125 400,000
To Total purchases (W.N.2) 325,000 By Balance c/d 70,000
Total 470,000 Total 470,000

W.N. 4: Purchase of equipment & depreciation on equipments

Debit Equipment Account Credit


Particulars NRs. Particulars NRs.
To Balance b/d 240,000 By Cash -equipment sold 15,000
To Cash-purchase (Bal. Fig.) 52,000 By P/L Account ( Loss on sale) 5,000
By Balance c/d 272,000
Total 292,000 Total 292,000

Depreciation on equipment:

© The Institute of Chartered Accountants of Nepal 419


CAP II Paper 1: Advanced Accounting

Particulars NRs.
@ 10% p.a. on NRs.2,20,000 (i.e. NRs.240,000 – NRs.20,000) = 22,000
@ 10% p.a. on NRs.52,000 for 6 months (i.e. during the year) = 2,600
Total 24,600

W.N. 5: Calculation of closing balance of creditors

Debit Creditors Account Credit


Particulars NRs. Particulars NRs.
To Cash 250,000 By Balance b/d 65,000
To Discount received 4,800 By Credit purchases (W.N.2) 260,000
To Balance c/d (Bal. Fig.) 70,200
Total 325,000 Total 325,000
W.N. 6: Calculation of opening balance of debtors

Debit Debtors Account Credit


Particulars NRs. Particulars NRs.
To Balance b/d (Bal. Fig.) 35,500 By Cash 380,000
To Sales (Credit) 450,000 By Discount allowed 5,500
By Balance c/d 100,000
Total 485,500 Total 485,500

W.N. 7: Calculation of capital accounts of A & B as on 31.3.2006

Balance Sheet as on 31.3.2006


Equity & Liabilities NRs. Assets NRs.
Combined Capital Accounts of
A & B (Bal. Fig.) 545,500 Building 150,000
Creditors 65,000 Equipments 240,000
Bank Loan 45,000 Furniture 25,000
Debtors (W.N.6) 35,500
Stock (W.N.3) 145,000
Cash balance 60,000
Total 655,500 Total 655,500

Particulars NRs.
Combined Capitals of A & B 545,500
Less: Difference in capitals of A and B 15,000
Total 530,500

530,500
A’s Capital as on 31.3.2006 = = 265,250 +15,000 = NRs.280,250
2

© The Institute of Chartered Accountants of Nepal 420


CAP II Paper 1: Advanced Accounting

530,500
B’s Capital as on 31.3.2006 = = NRs.265,250
2

W.N. 8:

Debit Cash Account Credit


Particulars NRs. Particulars NRs.
To Balance b/d 60,000 By Creditors 250,000
To Debtors 380,000 By Purchases 65,000
To Equipment (sales) 15,000 By Expenses 40,000
To Cash sales (W.N.1) 50,000 By A’s drawings 30,000
By Bank loan paid (45,000-35,000) 10,000
By Equipment purchased (W.N.4) 52,000
By Balance c/d (Bal. Fig.) 58,000
Total 505,000 Total 505,000

22. A, B and C were partners sharing profit and losses in the ratio of 5:3:2 respectively. The trial
balance of the firm on 31st March 2010 was the following:
Particulars Dr. Amount (Rs.) Cr. Amount (Rs.)
------------- -------------------- --------------------
Machinery at Cost 1,00,000
Stock 68,700
Sundry Debtors 62,000
Sundry Creditors 64,700
Bills Payable 20,000
Capital A/c
A 68,000
B 45,000
C 23,000

Drawings
A 25,000
B 23,000
C 17,000
Depreciation on Machinery 40,000
Profit for the year ended 31.03.2010 1,24,300
Cash at Bank 89,300
------------ ------------
3,85,000 3,85,000
----------- ----------

© The Institute of Chartered Accountants of Nepal 421


CAP II Paper 1: Advanced Accounting

Interest on capital account at 10% p.a. on the amount standing to the credit of the partners’
capital account at the beginning of the year was not provided before preparing the above trail
balance.

On 1st April, 2010 they formed ABC Private Ltd. with an authorized share capital of Rs.
2,00,000 in shares of Rs. 10 each to be divided in different classes to take over the business of
partnership.

You are informed as under;


• Machinery is to be transferred at Rs. 70,000.
• Shares in the company are to be issued to the partners, at par in such numbers and in such
classes as will give the partners, by reason of their shareholdings alone, the same rights as
regards interest on capital and the sharing of profit and losses as they had in the partnership.
• Before transferring the business, the partners wish to draw from the partnership profits
to such an extent that the bank balance is reduced to Rs. 50,000. For this purpose
sufficient profit of the year are to be retained in profit sharing ratio.
• All assets and liabilities except machinery and bank are to be transferred
at their book value as on 31.03.2010

You are required to prepare:


a) Capital Account showing all adjustments required to dissolve the partnership.
b) Statement showing the workings of the number of shares of each class to be issued by
the Company to each of the partners and a statement of additional drawings in cash.
c) The Balance Sheet of the company, immediately after acquiring the business of the
partnership and issuing of shares. (Inter Jun. 2010 Q1- 20 Marks)

Answer
Working Number: 1
Profit & Loss Appropriation Account
Date Particular Amount Date Particular Amount
31.3.2010 To Interest on Capital 31.3.2010 Balance b/d 124,300
A 6800
B 4500
C 2300 13,600
31.3.2010 To Partners Capital
A/C
Being share of profit
A 55350 110,700
B 33210
C 22140
124,300 124,300

Working Number: 2

© The Institute of Chartered Accountants of Nepal 422


CAP II Paper 1: Advanced Accounting

Statement showing Additional Drawings


Particulars Total A B C
Total Profit Allocation 124,300 62,150 37,710 24,440
Drawing Total
(Reduced in Bank Balance) (89300 - 50000) 39,300
Drawings by Partners 65,000
Total 104,300
Less : Retained Earnings (Retained) 20,000
Allocation of Retention (in profit sharing Ratio
5:3:2 10,000 6,000 4,000
)
Total Drawing (After Retention ) 104,300 52,150 31,710 20,440
Already Drawn 65,000 25,000 23,000 17,000
Additional Drawing 39,300 27,150 8,710 3,440

Working Number: 3
Revised Bank Balance
Date Particular Amount
31.3.2010 as per Trial 89,300
(-) Additional drawing (39,300)
Revised Bank Balance 50,000

Working Number: 4
Realization Account
Date Particular Amount Date Particular Amount
31.3.2010 To Machinery 100,000 31.3.2010 By Prov for depreciation 40,000
'' To Stock 98,700 '' By Bills payable 20,000
'' To Sundry Debtors 62,000 '' By Sundry Creditors 64,700
'' To Cash at Bank 50,000 '' By Purchase Consideration 166,000
To Partners Capital A/C
Being Realization gain.
'' A 5000 10,000 ''
B 3000
C 2000
290,700 290,700

Working Number: 5
Calculation of Purchase Consideration

Machinery 70,000
Stock 68,700
Sundry Debtors 62,000

© The Institute of Chartered Accountants of Nepal 423


CAP II Paper 1: Advanced Accounting

Cash at bank 50,000


Total 25,0700
Less: Sundry creditors 64,700
84,700
Bills Payable 20,000
Net Purchase Consideration 166,000

A) Partner Capital Account


Particulars A B C Particulars A B C
To Drawing A/c 52,150 31,710 20,440 By Balance b/d 68,000 45,000 23,000
To Equity shares in By Interest on
72,500 43,500 29,000 6,800 4,500 2,300
ABC Pvt. Ltd. A/c Capital A/c
To Preference
By Profit & Loss
Shares in ABC Pvt. 10,500 10,500 - 55,350 33,210 22,140
Appropriation A/c
Ltd. A/c
By Realization A/c 5,000 3,000 2,000
135,150 85,710 49,440 135,150 85,710 49,440

B) Statement Showing Number of Shares to be issued to Partners


Particulars A B C
Outstanding Balance (Net capital) 83,000 54,000 29,000
Profit sharing Ratio 5 3 2
Equated capital (Outstanding balance/ Profit sharing ratio) 16,600 18,000 14,500
Since C has Lowest capital balance it is considered as base capital for calculating the equity capital of A
and B
Equity Shares to be issued 72,500 43,500 29,000
29,000 ÷ 2 × 5 29,000 ÷ 2 × 3
Preference shares to be issued 10,500 10,500 -
83,000 54,000 29,000

Balance Sheet of ABC Private Ltd


As at 1.4.2010

Equity and Liabilities Amount Asset Amount


Equity share capital 145,000 Machinery 70,000
Preference share capital 21,000 Inventories 68,700
Bills payable 20,000 Debtors 62,000
Sundry Creditors 64,700 Bank 50,000
250,700 250,700

© The Institute of Chartered Accountants of Nepal 424


CAP II Paper 1: Advanced Accounting

23. Gopi and Hari are in partnership sharing profits and losses in the ratio of 3:2. The following
information relates to the year to 31st Ashadh 2066:

Particulars Dr. Rs.’000 Cr. Rs.’000


Opening balance on Capital A/c and Drawing A/c
Gopi 40 300
Hari 40 100
Cash at bank 5
Creditors and accruals 25
Debtors and prepayments 18
Fixed Asset 300
Accumulated Depreciation 100
Salary to Hari 10
Net profit for the year 60
Closing Stock at cost 90
Trade Creditor 141
Trade Debtors 223
726 726

Consider the following points:


• Gopi made his drawing on 30th Poush 2065 and Hari on 30th Aswin 2065.
• The partnership agreement allows for Hari to be paid a salary of Rs.20,000 p.a., and for
interest of 5% p.a. to be paid on the partners’ capital account balances as at 1st Shrawan
each year. Interest at a rate of 10% p.a. is charged on the partners’ drawings.
• The partners decided to dissolve the partnership as at 31st Ashadh 2066, and the
business was then sold to Valley Ltd. The purchase consideration was to be 4000
Rs.100 equity shares in Valley at 25% premium per share. The shares were to be issued
to the partners on 31st Ashadh 2066 and they were to be shared between them in their
profit-sharing ratio.
• The sale agreement allowed Gopi to take over one of the business motorcycles at an
agreed valuation of Rs.10,000.
• Apart from the motorcycle and the cash and bank balances, the company took over all
the other partnership assets and liabilities at their book values as at 31st Ashadh 2066.
• Matters relating to the appropriation of profit for the year to 31st Ashadh 2066 are to
be dealt with in the partners’ capital accounts, including any arrear of salary owing to
Hari.

Required:
a) Prepare profit and loss appropriation account for the year.
b) Prepare partners’ capital accounts
c) Prepare realization account
d) Prepare Valley’s Balance sheet as at 1st Shrawan 2066
(Inter Jun. 2011 Q 1- 20 Marks)

© The Institute of Chartered Accountants of Nepal 425


CAP II Paper 1: Advanced Accounting

Answer:
(i)
Profit & Loss Appropriation Account
of Gopi and Hari for the year ended 31st Ashadh 2066
Dr. Cr.
Particulars Rs. Particulars Rs.
To Salary to Hari 10,000 By Net profit for the year 60,000
To Hari's Capital a/c (arrear salary) 10,000 By Drawing a/c (interest charged)
To Capital a/c (interest on capital) Gopi (40,000 x 10% x 6/12) 2,000
Gopi (300,000 x 5%) 15,000 Hari (40,000 x 10% x 9/12) 3,000
Hari (100,000 x 5%) 5,000
To Capital a/c
(appropriation of balance profit)
Gopi (25,000 x 3/5) 15,000
Hari (25,000 x 2/5) 10,000
65,000 65,000

(ii)
Capital Account of Gopi and Hari
for the financial year 2065-66
Dr. Cr.
Gopi Hari Gopi Hari
Particulars (Rs.) (Rs.) Particulars (Rs.) (Rs.)
To Drawing a/c (WN 1) 42,000 43,000 By Balance c/d 300,000 100,000
To Shares in Valley Ltd. 300,000 200,000 By Profit & Loss App. a/c - 10,000
To Realization a/c (Arrear of salary to
(Motorcycle) 10,000 - Hari)
To Cash a/c (balance)
(WN 2) 65,000 By Profit & Loss App. a/c 15,000 5,000
(Interest on capital)
By Profit & Loss App. a/c 15,000 10,000
(appropriation of profit)
By Realization a/c 87,000 58,000
By Cash a/c (deficiency)
(WN 2) - 60,000
417,000 243,000 417,000 243,000

(iii)
Realization Account
Dr. Cr.
Particulars Rs. Particulars Rs.
To Debtors and pre-payments 18,000 By Creditors and accrual 25,000
To Fixed assets 300,000 By Accumulated depreciation 100,000

© The Institute of Chartered Accountants of Nepal 426


CAP II Paper 1: Advanced Accounting

To Closing stock 90,000 By Trade creditors 141,000


By Gopi's Capital a/c
To Trade debtors 223,000 (Motorcycle) 10,000
To Capital a/c (realization
profit) By Shares in Valley Ltd. 500,000
Gopi (145,000 x 3/5) 87,000
Hari (145,000 x 2/5) 58,000

776,000 776,000

Note: It is assumed that the agreed value of motorcycle taken over by Gopi is its book
value.

(iv)
Balance Sheet of Valley Ltd.
as at 1st Shrawan 2066
Liabilities Rs. Assets Rs.
Share Capital 400,000 Fixed Assets (300-100-10) 190,000
Share Premium 100,000 Debtors and pre-payments 18,000
Creditors and accruals 25,000 Trade Debtors 223,000
Trade Creditors 141,000 Closing Stock 90,000
Goodwill 145,000
666,000 666,000

Working Notes:
(1)
Drawing Account of Gopi and Hari
for the financial year 2065-66
Dr. Cr.
Particulars Gopi (Rs.) Hari (Rs.) Particulars Gopi (Rs.) Hari (Rs.)
To Balance c/d 40,000 40,000
To Profit & Loss App. a/c 2,000 3,000 By Capital a/c 42,000 43,000
(Interest on drawing)
42,000 43,000 42,000 43,000

(2)
Cash Account
Dr. Cr.
Particulars Rs. Particulars Rs.
To Balance b/d 5,000 By Gopi's Capital a/c 65,000
To Hari's Capital a/c 60,000
(Deficiency)
65,000 65,000

© The Institute of Chartered Accountants of Nepal 427


CAP II Paper 1: Advanced Accounting

(3)
Calculation of Value of Goodwill to be appeared in the balance sheet of Valley Ltd
Assets taken over at book value:
Fixed Assets (300-100-10) 190000
Debtors and pre-payments 18000
Trade Debtors 223000
Closing Stock 90000 521000
Less: Liabilities assumed:
Creditors and accruals 25000
Trade Creditors 141000 166000
Net Assets taken over (A) 355000
Purchase consideration paid:
4000 equity shares of Rs.100 each at 25% premium (B) 500000
Goodwill (B-A) 145000

24. Krishna, Danfey and Bagmati were in partnership sharing and losses in the ratio of 9:4:2.
Danfey retired from the partnership on 31st March, when the firm's balance sheet was as under
:
Rs. in 000's
Liabilities Rs. Assets Rs.
Sundry Creditors 600 Cash and Bank 284
Capital Accounts : Sundry Debtors 400
Krishna 2,700 Stock 800
Danfey 1,200 Furniture 266
Bagmati 600 Plant 850
Land and Building 2,500

Total 5,100 Total 5,100


Danfey's share in goodwill and capital was acquired by Krishna and Bagmati in the ratio
of 1: 3, the continuing partners bringing in the necessary finance to pay off Danfey. The
partnership deed provides that on retirement or admission of a Partner, the goodwill of the
firm is to be valued at three times the average annual Profits of the firm for the four years
ended on the date of retirement or admission. The profits of the firm (Rs. in 000's) during
the four previous financial years are Rs. 450; Rs. 250; Rs. 600 and Rs. 700 respectively.

The deed further provided that goodwill account is not to appear in the books of account at
all. The continuing partners agreed that with effect from 1st April (immediately after
retirement), Ganpati, son of Krishna is to be admitted as a partner with 25% Share of
Profits. Krishna gifts to Ganpati, by transfer from his capital account, an amount sufficient
to cover up 12.5% of capital and goodwill requirement. The balance 12.5% of capital and
goodwill requirement is purchased by Ganpati from Krishna and Bagmati in the ratio of
2:1.

© The Institute of Chartered Accountants of Nepal 428


CAP II Paper 1: Advanced Accounting

You are required to:


a) Prepare a statement showing the continuing partners' shares.
b) Pass journal entries including for bank transactions; and
j) Prepare the balance sheet of the firm after Ganpati's admission.
(Inter Jun. 2012 Q1- 20 Marks)
Answer:

a) Computation of Partners' Shares consequent to Retirement and Admission


Particulars Krishna Danfey Bagmati Ganpati
Ratio before retirement of Danfey 9/15 4/15 2/15
Adjustment on retirement 4/15 X - 4/15 X 3/4=
1/4=1/15 3/15
New Ratio before admission of 10/15 5/15
Ganpati (2:1)
Add/ Less : On admission of
Ganpati : (1/8) 1/8
- Gift by A (12.5/100) (2/24) (1/24) 3/24
- Purc. from Krishna &
Bagmati
New Ratio 11/24 7/24 6/24
Note :- Purchase from Krishna = 2/3 X 1/8 = 2/24; while Purchase from Bagmati = 1/3 X
1/8 = 1/24 .

b) Journal Entries In books of the firm (Amount in Rs. 000's)


S.No, Particulars Dr Cr
1 Krishna's Capital A/c 100
Bagmati's Capital A/c 300
To Danfey's Capital Aa/c 400
(Being purchase of Goodwill from Danfey by Krishna and
Bagmati in the ratio 1 : 3)
2 Krishna's Capital A/c 750
To Ganpati's Capital A/c 750
(Being gift made by Krishna to Ganpati)
3 Bank A/ c 3100
To Krishna's Capital A/c 775
To Bagmati's Capital A/c 1387.5
To Ganpati's Capital A/c 937.5
(Being capital brought in by the partners based on capital
requirement as per profit sharing ratio)
4 Danfey's Capital A/c (1200 + 400) 1600
To Bank A/c 1600
Being final payment made to Danfey on retirement )

© The Institute of Chartered Accountants of Nepal 429


CAP II Paper 1: Advanced Accounting

5 Ganpati's Capital A/c 187.5


To Krishna's Capital A/c 125
To Bagmati's Capital A/c 62.5
(Being Goodwill Adjusted in the accounts ) refer WN 4

C) Balance Sheet as on 1st April (after Retirement and Admission) (Amount in Rs. 000's)
Liabilities Rs. Assets Rs.
Sundry Creditors 600 Cash and Bank 1784
Capital Sundry Debtors 400
- Krishna 2750 Stock 800
- Bagmati 1750 Furniture 266
- Ganpati 1500 Plant 850
Land and Building 2500
Total 6600 Total 6600

Working Note 1 Adjustment of Goodwill on Danfey Ltd.'s Retirement

(a) Average Profits of past four years = (450+250+600+700) ÷ 4 years Rs. 500
(b) Goodwill = 3 years Purchase of Average Profits = 3 X Rs. 500 Rs. 1500
(c) Danfey's Share in above Goodwill Of the Firm at 4/15th Share = Rs 400
th
1500 X 4/15
Since this Goodwill is to be adjusted in the Continuing Partners' Share in the ratio of their
taking over, i.e. 1:3, the amount to debited to Krishna and Bagmati are Rs. 400 as 1:3 =
Rs. 100 & Rs. 300

Working Note 2 Computation of Total Capital and Gift by Krishna to Ganpati

(a) Danfey Share of Capital and Goodwill = 1200 + 400 = Rs. 1600
(b) Since Danfey's Share is 4/15th, Rs.1600 represents 4/15th of the Total Rs. 6000
Capital & Goodwill of the Firm. Hence, Total Capital and Goodwill of the firm
= Rs. 1600 ÷ 4/15th =
(C) Apportionment of Total Capital to Continuing Partners' Account in new
ratio (11:7:6)
- Krishna's Share= Rs. 6000 X 11/24th = Rs.2750
- Bagmati's Share = 6000 X 7/24th = Rs. 1750
- Ganpati's Share = Rs, 6000 X 6/24th Rs. 1500
(d) Gift by Krishna to Ganpati = 12.5 % of Total i.e. 12.5% X Rs.6000= Rs. 750

Working Note 3 Ganpati's Purchase of Goodwill Share in the ratio 2 : 1 from Krishna
and Bagmati

(a) Total Goodwill of the Firm as computed above Rs. 1500

© The Institute of Chartered Accountants of Nepal 430


CAP II Paper 1: Advanced Accounting

(b) Ganpati's Share of Goodwill to be brought in = 6/24th = 1/4th of Rs. 375


Rs. 1500
(c) Amount gifted by Krishna = 12.5% of Total = 12.5% X Rs. 1500 = Rs.
187.50
This Rs. 187.50 is included in the total gift of Rs. 750 computed in WN
3 above.
(d) Balance Purchased from Krishna and Bagmati in the ratio 2:1 Rs.
187.50
(e) Amount to be adjusted in the Capitals of Krishna and Bagmati in the Rs. 125 &
ratio 2:1 Rs. 62.50

Working Note 4 Partner's Capital Account ( Amount in Rs. 000's)

Particulars(G Kris Danf Bag Ganpa Particular's


Kris Dan Bagmat Ganpa
oodwill) hna ey mati ti hna fey i ti
To Danfey 100 0 300 0 By Balance 270 120 600 0
b/d 0 0
To Ganpati 750 0 0 0 By Krishna 0 400 0 0
A/c - Gift of and
12.5% Bagmati
(Goodwill)
To Krishna 0 0 0 187.5 By Cash & 775 0 1387.5 937.5
and Bagmati Bank (bal.
figure)
To Cash & 0 1600 0 0 By Krishna 0 0 0 750
bank A/c
Total balance 2750 0 1750 1500 By Ganpati 125 0 62.5 0
c/d A/c
Total 3600 1600 2050 1687.5 Total 360 160 2050 1687.5
0 0

Working Note 5. Computation of Cash and Bank Balances after the above settlement

Cash and Bank prior to Retirement and Admission 284


Add : Amount brought in partners as per Capital A/c above ( 775 + 3100
1387.5 +937.5)
3384
Less : Settlement to Retiring partner – Danfey 1600
Closing Balance after the above transactions 1784
Note : The above increase in cash and Bank Balance represents the Goodwill valuation
Rs. 1500.

© The Institute of Chartered Accountants of Nepal 431


CAP II Paper 1: Advanced Accounting

25. Sima, Rima and Hima are running a Beauty Parlor in the form of a partnership firm. They share
profits and losses in the proportion of 1:4:5. Sima received an offer for foreign employment,
and retires from the partnership. However, existing partners are not able to pay her share of
capital at the time of retirement. Adjusted capital at the time of retirement of Sima was as
follows:
Sima: Rs. 100,000
Rima: Rs. 250,000
Hima: Rs. 300,000

Remaining two partners continued to carry on the business of partnership and they are in a
position to settle Sima’s share of capital after 6 months. During the period the Parlor made a
net profit of Rs. 36,000. Sima borrowed loan of Rs. 120,000 from a bank at an interest rate of
10% at the time of going for foreign employment.

There was a dispute among the partners regarding sharing of profits after separation of Sima.
However, existing partners agreed either to pay bank interest on borrowed fund or to share
profits according to Capital share at the time of separation.

Required:
Suggest Sima of the option that is more beneficial for her.
(CAP Jun. 2010 Q4a- 5 Marks)
Answer:
As Sima is already retired, she is entitled a profit share in proportion to her capital investment
and previous profit sharing ratio is irrelevant in this situation.
Total profit for 6 month’s period = Rs 36,000
Sima’s Share = 100,000/(100,000+250,000+300,000) X 36,000= Rs.5,538.46
Interest on unpaid Capital=100,000 X10%X6/12=Rs.5,000

Since share of profit will be more beneficial to Sima, she has to opt for the option of sharing
profits in the proportion of capital at the time of separation.

26. Ramesh and Suraj decided to work in partnership in the following scheme, agreeing to share
profit as under:
Ranesh 3/4th share
Suraj 1/4th share

They guaranteed the subscription at par of 10,00,000 shares of Rs.10 each in Sheetal ltd and to
pay all expenses up to allotment in consideration of Sheetal Ltd. furthermore 50,000 Share of
Rs. 10 each fully paid (other than 10,00,000 Share issued to public) was issued to them.
Ramesh introduced cash in to the business to meet the following expenses:

Registration fees Rs. 48,500


Advertising charges Rs 28,500
Printing charges of Memorandum of Association and

© The Institute of Chartered Accountants of Nepal 432


CAP II Paper 1: Advanced Accounting

Articles of Association and Prospectus etc. Rs. 23,000

Suraj introduced cash to meet following expenses:


Rent Rs. 37,000
Legal fees Rs. 13,000

Application fell short of the 10,00,000 shares by 30,000 shares. Suraj introduced further cash
on joint account for the said 30,000 shares. This amount was utilized to subscribe the said
30,000 shares and paid to the company.

The guarantee having been fulfilled, Sheetal Ltd. Handed over the Ramesh and Suraj 50,000
shares.
The partnership sold all shares. Suraj received the sale proceed of 20,000 shares amounting to
Rs. 1,80,000 and Ramesh of the remaining 60,000 shares amounting Rs. 5,00,000.
Give necessary accounts in the books of both parties.
(CAP Jun. 2011 Q2b- 10 Marks)
Answer:
Memorandum Joint Venture Account

To Ramesh: Rs. Rs. By Ramesh; Rs


Registration 48,500 Sale proceeds of
charges 28,500 60,000 shares 5,00,000
Advertising 23,000 1,00,000
Printing
To Suraj: By Suraj:
Rent 37,000 Sale proceeds of
Legal fees 13,000 50,000 20,000 shares 1,80,000
To Suraj: subscription 3,00,000
of
Unapplied for
shares
To Ramesh (3/4th 1,72,500
profit)
To Suraj ( 1/4th profit) 57,500
Total 6,80,000 Total 6,80,000
Student should be careful to make no entry for the shares received from the company as
consideration for the guarantee.

In the book of Ramesh, the account with Suraj will be as follows:


Suraj in Joint Venture Account
To Bank: Rs. By Bank: Rs.
Registration charges 48,500 Sale proceed of 60,000
Advertising 28,500 shares 5,00,000

© The Institute of Chartered Accountants of Nepal 433


CAP II Paper 1: Advanced Accounting

Printing 23,000
To Profit & Loss Account
2/3rd share of profit 1,72,500
To Bank 2,27,500
Total 5,00,000 Total 5,00,000

In the book of Suraj, the account with Ramesh will be as follows:


Ramesh in Joint Venture Account
To Bank: Rs. By Bank: Rs.
Rent 37,000 Sale proceed of 20,000
Legal 13,000 shares 1,80,000

To Bank: Subscription of By Bank: Settlement 2,27,500


Unapplied for shares 3,00,000
To Profit & Loss Account:
1/4th share of profit 57,500
Total 4,07,500 Total 4,07,500

27. Ram, Bharat, Laxman and Shatrughan are partners in a firm sharing profits and losses in the
ratio of 4:1:2:3. The following is their Balance Sheet as at 16th July, 2011:

Liabilities Rs. Assets Rs.


Sundry Creditors 3,00,000 Sundry Debtors 3,50,000
Capita Accounts: Less: Doubtful Debts 50,000
Ram 7,00,000 3,00,000
Shatrughan 3,00,000 10,00,000
Cash in hand 1,40,000
Stocks 2,00,000
Other Assets 3,10,000
Capital Accounts :
Bharat 2,00,000
Laxman 1,50,000 3,50,000
13,00,000 13,00,000

On 16th July 2011, the firm is dissolved and the following points are agreed upon:

i) Ram is to take over sundry debtors at 80% of book value.


ii) Shatrughan is to take over the stocks at 95% of the value and
iii) Laxman is to discharge sundry creditors.

Other assets realize Rs. 3,00,000 and the expenses of realization come to Rs. 30,000.
Bharat is found insolvent and Rs. 21,900 is realized from his estate.

© The Institute of Chartered Accountants of Nepal 434


CAP II Paper 1: Advanced Accounting

Prepare Realization Account and Capital Accounts of the partners. The loss arising out of
capital deficiency may be distributed following the decision in Garner vs. Murray.
(CAP Jun. 2011 Q3b-8 Marks)
Answer:
Realization Account
Particulars Rs. Particulars Rs.
To Sundry debtors 350,000 By Sundry creditors 300,000
To Stock 200,000 By Provision for doubtful debts 50,000
To Other assets 310,000 By Ram's capital – Debtors 280,000
To Laxman's capital – Creditors 300,000 By Shatrughan's capital – Stock 190,000
To Cash – Expenses 30,000 By Cash – Other assets 300,000
By Ram's capital 28,000
By Bharat's capital 7,000
By Laxman's capital 14,000
By Shatrughan's capital 21,000
1,190,000 1,190,000

Ram's Capital Accounts


Particulars Rs. Particulars Rs.
To Realization A/c – Debtors 280,000 By Balance b/d 700,000
To Realization A/c – Loss 28,000
To Bharat's capital 129,570
To Cash 262,430
392,000 392,000

Bharat's Capital Accounts


Particulars Rs. Particulars Rs.
To Balance b/d 200,000 By Cash 21,900
To Realization A/c – Loss 7,000 By Ram's capital 129,570
By Shatrughan's capital 55,530
207,000 207,000

Laxman's Capital Accounts


Particulars Rs. Particulars Rs.
To Balance b/d 150,000 By Realization A/c – 300,000
To Realization A/c – Loss 14,000 Creditors
To Cash 136,000
300,000 300,000

Shatrughan's Capital Account


Particulars Rs. Particulars Rs.
To Realization A/c – Stocks 190,000 By Balance b/d 300,000
To Realization A/c – Loss 21,000

© The Institute of Chartered Accountants of Nepal 435


CAP II Paper 1: Advanced Accounting

To Bharat's capital 55,530


To Cash 33,470
300,000 300,000

Note:
Bharat’s deficiency will be borne by Ram and Shatrughan in the ratio of 7 : 3 i.e. on opening
capitals of Rs. 7,00,000 and Rs. 3,00,000. Laxman will not bear any portion of the loss since
at the time of dissolution he had a debit balance in his capital account.

28. Radha, Kishan, Hari and Shyam were partners sharing Profit & Losses in the ratio of 3:3:2:2.
Following was the Balance Sheet as on 31st Ashadh, 069:

Liabilities Rs.Rs. Assets Rs. Rs.


Trade Creditors 23,250 Trade Debtors 24,000
Less: Provision for
Kamal’s Loan 15,000 Bad Debts (750) 23,250
Capital Accounts: Inventories 15,000
Radha 30,000 Cash at Bank 3,000
Kishan 22,500 52,500 Furniture & Fixtures 6,000
Trade Marks 10,500
Capital Accounts:
Hari 24,000
Shyam 9,000 33,000
90,750 90,750
st
On 31 Ashadh, 2069, the partnership was dissolved. The assets realized were as follows:

Trade Debtors Rs. 16,500; Inventories Rs. 12,000; Furniture & Fixtures Rs. 1500; Trade Mark
Rs. 6,000; Trade Creditors were settled at Rs. 23,000; also there was a joint life insurance
policy for Rs. 45,000; this was surrendered for Rs. 4,500. Expenses of realization amounted
to Rs. 750. Hari was insolvent, but Rs. 5,500 was recovered from his estate.
Show the following accounts in the books of the partnership firm:
(CAP Jun. 2013 Q1– 20 Marks)
Answer:

Realization Account as on Ashadh 2069


Particulars Amount Amount Particulars Amount Amount
By Provision for
To Sundry Assets: Doubtful Debts 750
Furniture & Fixtures 6000 By Trade Creditors 23250
Trade Marks 10500 By Cash:
Trade Debtors 24000 Furniture & Fixtures 1500
Inventories 15000 55500 Trade Marks 6000

© The Institute of Chartered Accountants of Nepal 436


CAP II Paper 1: Advanced Accounting

To Cash Trade Debtors 16500


(Payment to Creditors) 23000 Inventories 12000
Surrender Value of
To Cash Policy 4500 40500
By Partners Capital
(Realization Expense) 750 Account
(Loss on Realization)
Radha 4425
Kishan 4425
Hari 2950
Shyam 2950 14750

79250 79250

Cash Account
Particulars Amount Amount Particulars Amount Amount
To Balance B/d 3,000 By Realization Account
To Realization 40,500 By Trade Creditors 23,000
Account
(Assets Realized) Realization Expenses 750 23,750
To Partners Capital By Kamal's Loan 15,000
Radha 4,425 To Partners Capital:
Kishan 4,425 Final Payment
Shyam 2,950 11,800 Radha's Account 17,743
To Hari Capital 5,500 Kishan's Account 13,307 31,050
To Shyam Capital 9,000

69,800 69,800

Working Notes

© The Institute of Chartered Accountants of Nepal 437


a) Partners’ Capital
CAP II Account Paper 1: Advanced Accounting
Particulars Radha Kishan Hari Shyam Particulars Radha Kishan Hari Shyam
To balance B/d - - 24,000 9,000 By balance B/d 30,000 22,500 - -
To Realization
Account (Loss) 4,425 4,425 2,950 2,950 By Cash 4,425 4,425 - 2,950
To Hari Capital 12,257 9,193 - - By Cash - - 5,500 9,000
(Loss of Capital By Radha
Written off) Capital - - 12,257 -
By Kishan
Capital - - 9,193 -
To Cash A/C 17,743 13,307 - -
34,425 26,925 26,950 11,950 34,425 26,925 26,950 11,950
1. There was a debit balance of Rs 9,000 in the Shyam’s Capital and Shyam is a Solvent
Partner; hence, he must bring cash for balance capital
2. Hari is insolvent therefore he is not able to bring cash. The deficiency in his account
is borne by Radha & Kishan in the ratio 4:3 (Capital Ratio) as per Garner & Murray.
Deficiency in Hari Capital =Rs 24000+Rs. 2950-Rs. 5500=Rs. 21450
Borne By Radha=4/7*21450=12257
Borne by Kishan=3/7*21450=9193
Mr. Kamal Loan is paid off in Cash

29. The following is the trial balance as on 31st March 2013 of Amar, Bisnhu and Chetan who
had branches at different places and who shared profits & losses in the ratio of 2:1:2
respectively,
Rs. Rs.
Dr. Cr.
st
Capitals, 1 April 2012
Amar 140,000
Bishnu 140,000
Chetan 20,000
Drawings
Amar 22,720
Bishnu 19,920
Chetan 9,160

Patents & Trade Marks 24,000


Sundry Creditors 69,800
Profit & Loss account for the year (before allowing interest @ 10%
on capital, as a charge) 25,000

Sundry Debtors-
Dhankuta 67,500
Pokhara 32,500
Kailali 15,000

© The Institute of Chartered Accountants of Nepal 438


CAP II Paper 1: Advanced Accounting

Furniture-
Dhankuta 10,000
Pokhara 5,000
Kailali 6,000

Stock-
Dhankuta 46,000
Pokhara 34,000
Kailali 53,000
_____ ________
369,800 369,800

The firm was dissolved on that date. Amar took over Dhankuta Branch, and Bisnu took over
Pokhara Branch; the assets being taken at 10% less than the book values. Patents and Trade
Marks were found valueless. The business at Kailali was sold to a limited company which
allotted 6,000 equity shares of Rs. 10 each credited as fully paid. To pay the liabilities, Amar
and Bisnhu introduced cash in the profit sharing ratio. The cost of winding up came to Rs.
4,000 for which Amar advanced cash. Chetan is insolvent and can pay nothing. Bishnu received
all the shares.
Prepare necessary accounts to close the books of the firm; partners give effect to the correct
position without having to make up losses in cash. Assume the capitals to be fluctuating. Amar
and Bishnu settles accounts themselves. (CAP Jun. 2014 Q1- 20 Marks)

Answer:

Dr. Profit & Loss Account Cr.


2013 NPR 2013 NPR
March To Loss for the year 25,000 March By Loss transferred to
31 To Interest on Capitals 31 Amar's Capital 22,000
Amar's Capital 14,000 Bishnu's Capital 11,000
Bishnu's Capital 14,000 Chetan's Capital 22,000 55,000
Chetan's Capital 2,000 30,000

55,000 55,000

Dr. Realization Account Cr.


2013 NPR 2013 NPR
March To Patents & 24,000 March By Amar's Capital-
31 Trade Marks 31 Debtors at Dhankuta 67,500
To Sundry 115,000 Furniture at Dhankuta 10,000
Debtors 21,000 Stock at Dhankuta 46,000
To Furniture 133,000 Total 123,500
To Stock 4,000 Less : 10% 12,350 111,150

© The Institute of Chartered Accountants of Nepal 439


CAP II Paper 1: Advanced Accounting

To Cash By Bishnu's Capital


(Expenses) Debtors at Pokhara 32,500
Furniture at Pokhara 5,000
Stock at Pokhara 34,000
Total 71,500
Less : 10% 7,150 64,350

By Shares in New Company Ltd 60,000

By Loss transferred to:


Amar's Capital 24,600
Bishnu's Capital 12,300
Chetan's Capital 24,600 61,500
297,000 297,000

Dr. Sundry Creditors Account Cr.


2013 NPR 2013 NPR
March 31 To Cash 69,800 March 31 By Balance b/fd 69,800

69,800 69,800

Dr. Cash Account . Dr


2013 NPR 2013 NPR
March 31 To Amar's Capital A/C 46,533 March 31 By Realization A/c 4,000
(Expenses) 69,800
To Bishnu's Capital 23,267 By Sundry Creditors
A/C
(Cash advance to pay
creditors) 4,000
To Amar's Capital A/C
(Cash paid for
expenses)

73,800 73,800

Dr. Shares in New Company Ltd. Cr.


2013 NPR 2013 NPR
March 31 To Realization A/c 60,000 March 31 By Bishnu's Capital A/c 60,000

60,000 60,000

© The Institute of Chartered Accountants of Nepal 440


CAP II Paper 1: Advanced Accounting

Dr. Capital Account Cr.


Particulars Amar Bishnu Chetan Particulars Amar Bishnu Chetan

To Drawings 22,720 19,920 9,160 By Balance b/d 140,000 140,000 20,000


To Profit & 22,000 11,000 22,000 By Interest on 14,000 14,000 2,000
Loss A/c 109,280 123,080 - Capital - - 9,160
To Balance c/d By Balance c/d
154,000 154,000 31,160 154,000 154,000 31,160
To Balance b/d 9,160 By Balance b/d 109,280 123,080
To Realization 111,150 64,350 By Cash 46,533 23,267
A/c (Assets 24,600 12,300 24,600 By Cash 4,000
taken over) Expenses 15,880
To Realization 15,880 17,880 By Amar's 17,880
(loss) 60,000 Capital A/c 8,183
To Chetan's By Bishnu
Capital A/c 8,183 Capital A/c
To Shares in By Bishnu's
New Co. Ltd. Capital A/c
To Amar's (Cash paid to
Capital ( Cash Amar)
received from
Bishnu)
159,813 154,530 33,760 159,813 154,530 33,760
Note: Chetan's loss has been divided between Amar and Bishnu in the ratio of 109,280 :
123,080 respectively.

30. A and B are partners sharing profits and losses in the ratio 5:3. C is admitted as a partner getting
3/10 of the profit share. If C acquires 4/20 of his share from A and 2/20 from B, what would
be the new profit sharing ratio? (CAP Jun. 2014 Q3d- 2 Marks)

Answer:
Calculation of new profit sharing ratio
A: 5/8-4/20 = 25-8/40 =17/40
B: 3/8-2/20 = 15-4/40 =11/40
C: 3/10 or 12/40
Thus new profit sharing ratio would be 17:11:12

31. A, B and C are partners of M/s Sun Trading sharing profits and losses in the ratio of 3:3:2.
Their balances as on July 16, 2015 is as under:

© The Institute of Chartered Accountants of Nepal 441


CAP II Paper 1: Advanced Accounting

Particulars Amount (Rs.) Amount (Rs.)


Fixed Assets:
Cost 360,000
Less: Depreciation 216,000 144,000
Investment: Cost (Market Value Rs. 250,000) 120,000
Working Capital 400,000
Total 664,000
Financed by:
Loans from D 180,000
E 120,000 300,000
Reserves 264,000
Capitals B 240,000
C 120,000
360,000
Less: A 260,000 100,000
Total 664,000

On that day, A retired from business. B and C decided to admit D as a partner and E, who
is a minor, to benefits of partnership. B, C, D and E are to share profits in the ratio of
3:3:2:2. Losses, if any, are to be borne by B, C and D in the ratio of 3:3:2.

For the purpose of above retirement-cum admission, it is decided that:


a) Goodwill of the firm is valued at Rs. 250,000. However, no account is to appear for
goodwill. Treatment of goodwill in relation to minor is to be deferred to the date when
he opts to be a partner on attaining majority, when the value of goodwill will be
deemed, for this purpose, to remain unchanged at Rs. 250,000.
b) A takes over: (i) Fixed Assets (cost Rs. 125,000; WDV Rs.55,000) for Rs.100,000; and
(ii) Investments (cost Rs. 15,000) at their market value of Rs. 12,500.
c) The continuing partners take over the remaining assets at their book values, except
investments which are taken over at their market values.
d) Loans are to be transferred to capital accounts.
e) The amount due to/from A is to be settled by Cheque immediately.
You are required to prepare: (a) the Revaluation / Adjustment Account; (b) the Capital
Accounts; and, (c) the Balance Sheet of the firm after giving effect to all the above points of
agreement. (CAP Jun. 2016 Q1- 20 Marks)

Answer:
Revaluation Account:
Particulars Amount Particulars Amount
To Investment A/c 2,500 By Fixed Assets A/c 45,000
To Partners Capital A/c By Investment A/c 132,500
A 65,625
B 65,625

© The Institute of Chartered Accountants of Nepal 442


CAP II Paper 1: Advanced Accounting

C 43,750
177,500 177,500

Balance Sheet of the new firm as on 16.07.2015.


Liabilities Amount Assets Amount
Capital Accounts Fixed Assets (Cost Less Dep) 89,000
B 404,625 Investment (Market Value) 237,500
C 198,500 Working Capital 514,125
D 117,500
E 120,000
840,625 840,625

Partners’ Capital Account:


Particulars A B C D E Particulars A B C D E
To Balance By Balance
B/d 260 c/d 240 120
By C's 31.
To A's Capital 31.25 62.50 Capital 250
To Fixed By D's 62.
Assets 100 Capital 500
To Investment 12.50 By Reserves 99 99 66
To Balance 404. By 65. 65.6 43.
c/d - 625 198.50 117.50 120 Revaluation 625 25 750
By Loan 12
A/c - - - 180 0
114
By Bank .12
A/c 5
404. 372 404. 229 12
372.5 625 229.75 180 120 .5 625 .75 180 0

Working Notes:
1. Adjustment of Goodwill:

Goodwill Value: Rs. 250,000


Particulars A B C D
A. Credited in the old Profit Sharing ratio (Rs.) 93,750 93,750 62,500
B. Debited in the new profit sharing ratio (Rs.) - 93,750 93,750 62,500
Required Adjustment (A-B) 93,750 - (31,250) (62,500)
Note: E’s Account cannot be debited for writing – off goodwill since he is a minor.

2. Provision for Depreciation Account:

© The Institute of Chartered Accountants of Nepal 443


CAP II Paper 1: Advanced Accounting

Particulars Amount Particulars Amount


To Assets Account 70,000.00 By Balance b/d 216,000.00
To Balance c/d 146,000.00
216,000.00 216,000.00
3. Fixed Assets Account:
Particulars Amount Particulars Amount
To Balance b/d 360,000.00 By Prov for Depreciation 70,000.00
To Revaluation A/c 45,000.00 By A's Capital A/c 100,000.00
By Balance c/d 235,000.00
405,000.00 405,000.00
4. Investment Account:
Particulars Amount Particulars Amount
To Balance B/d 120,000.00 By A's Capital 12,500.00
To Revaluation A/c 132,500.00 By Revaluation A/c 2,500.00
By Balance c/d 237,500.00
252,500.00 252,500.00

5. Ascertainment of Working Capital:


Opening working Capital: Rs. 400,000
Add: Cheque by A: Rs. 114,125
Closing Working Capital: Rs. 514,125

32. A and B were carrying on business sharing profits and losses equally. The firm's Balance
Sheet as at 31.03.2074 was:

Liabilities Rs. Assets Rs.


Capital Accounts: Plant 1,60,000
A 1,50,000
B 1,30,000 2,80,000
Sundry Creditors 80,000 Building 48,000
Bank Overdraft 45,000 Debtors 75,000
Stock 70,000
Joint Life Policy 6,000
Profit & Loss A/c 30,000
Drawing Account: 16,000
A 9,000
B 7,000
Total 4,05,000 Total 4,05,000

The operations of the business were carried on till 30.09.2074. A and B both withdrew in equal
amount half the amount of profit made during the current period of six months after charging
depreciation at 10% per annum on plant and after writing off 5% on building . During the

© The Institute of Chartered Accountants of Nepal 444


CAP II Paper 1: Advanced Accounting

current period of six months, creditors were reduced by Rs. 20,000 and bank overdraft by Rs.
5,000.
The life policy was surrendered for Rs. 6,000 before 30th Poush, 2074. Stock was valued at Rs.
84,000 and debtors at Rs. 68,000 on 30th Poush, 2074. The other items remained the same as
31.03.2074.
On 30.09.2074, the firm sold its business to AB Ltd. The value of goodwill was estimated at
Rs. 1,30,000 and the remaining assets were valued on the basis of the balance sheet as on
30.09.2074.

AB Ltd. paid the purchase consideration in equity shares of Rs. 10 each.


You are required to prepare;
a) Balance Sheet of the firm as at 30.09.2074,
b) Realization account,
c) Partners' Capital Accounts showing the final settlement between them
(CAP Jun. 2018 Q1-20 Marks)
Answer:

a) Balance Sheet of the firm as at 30.09.2074


Liabilities Rs. Assets Rs.
Capital Account : Plant :
A's Capital 1,33,800 Opening Balance 1,60,000
B's Capital 1,15,800 Less: Depreciation @ 10% 8,000 1,52,000
Creditors 60,000 Building :
Bank Overdraft 40,000 Opening Balance 48,000
Less: Written-off @ 5% 2,400 45,600
Debtors 68,000
Stock 84,000
Total 3,49,600 Total 3,49,600

b) Realization Account
Dr.
Cr.
Particular Amount Particular Amount
To Sundry Assets: By Creditors 60,000
Plant 1,52,000 By Bank Overdraft 40,000
Building 45,600
Stock 84,000 By AB Ltd A/c 3,79,600
Debtors 68,000 (working note 2 )
To Profit:
A's Capital A/c 65,000
B's Capital A/c 65,000
------------- ---------------
4,79,600 4,79,600

© The Institute of Chartered Accountants of Nepal 445


CAP II Paper 1: Advanced Accounting

b) Partner's Capital Accounts

Date Particular A (Rs.) B (Rs.) Date Particular A (Rs.) B (Rs.)


01.04.74 To Profit & 01.04.74 By balance b/d 1,50,000 1,30,000
Loss A/c 15,000 15,000
01.04.74 To Drawing A/c 9,000 7,000 30.09.74 By Profit 15,600 15600
30.09.74 To Drawing A/c 7,800 7,800 (W.N.1)
(W. N.1)
30.09.74 To Balance c/d 1,33,800 1,15,800 ---------- ----------
- --
Total 1,65,600 1,45,600 Total 1,65,000 1,45,600
30.09.74 By Balance b/d 1,33,800 1,15800

30.09.74 To shares in AB 1,98,800 1,80,800 30.09.74 By Realization 65,000 65,000


Ltd. A/c (Profit)
1,98,800 1,80,000 1,98,800 1,80,000

Working Notes:
(1) Ascertainment of profit for the period of 6 Months ended 30.09.2074
Amount (Rs)
Closing Assets :
Stock 84,000
Debtors 68,000
Plants less Depreciation 1,52,000
Building Less Written off 45,600
Total 3,49,600
Less: Closing Liabilities :
Creditors 60,000
Bank Overdraft 40,000 1,00,000
Closing Net Assets 2,49,600
Less: Opening adjusted Capitals
A(Rs. 1,50,000 - Rs. 15,000 - Rs. 9,000) 1,26,000
B(Rs. 1,30,000 - Rs. 15,000 - Rs. 7,000) 1,08,000 2,34,000
Profit Net of drawings 15,600
Actual Profit for six months before drawings (half of profit )=15,600x2 31,200
Combined Drawing during six months (half of profit) 15,600

(2) Ascertainment of purchase consideration


Rs.

© The Institute of Chartered Accountants of Nepal 446


CAP II Paper 1: Advanced Accounting

Closing Net Assets (As above) 2,49,600


Add: Goodwill 1,30,000
Total Purchase consideration 3,79,600

33. Pokhrel and Paudel were carrying on hosiery business sharing profits and losses equally.
The firm’s Balance Sheet as at 32nd Ashadh 2075 was as follows:
Liabilities Amount (Rs.) Assets Amount (Rs.)
Sundry Creditors 6,000,000 Stock 6,000,000
Bank Overdraft 3,500,000 Machinery 15,000,000
Capital Accounts: Debtors 7,000,000
Pokharel 14,000,000 Joint Life Policy 900,000
Paudel 13,000,000 Leasehold Premises 3,400,000
Profit and Loss Account 2,600,000
Drawing Accounts:
Pokharel 1,000,000
Paudel 600,000
Total 36,500,000 Total 36,500,000
The business was carried on till Poush end 2075. The partners withdrew in equal amounts half
the amount of profits made during the period of six months after charging depreciation at 10%
p.a. on machinery and after writing off 5% on leasehold premises. In the half year, sundry
creditors were reduced by Rs. 1,000,000 and bank overdraft by Rs. 1,500,000.

On Poush end 2075, stock was revalued at Rs. 7,500,000 and debtors at Rs. 6,000,000, the
joint life policy had been surrendered for Rs. 900,000 before Poush end 2074 and other items
remained the same as at 32nd Ashadh 2075.

On Poush end 2075, the firm sold the business to Global Apparels Ltd. by fixing value of
goodwill at Rs. 10,000,000 and the rest of the assets were valued on the basis of the Balance
Sheet as at Poush end 2075.
Global Apparels Ltd. paid the purchase consideration in equity shares of Rs. 10 each.
You are required to prepare:
i) Balance Sheet of the firm as at Poush end 2075.
ii) Partners’ Capital Account showing the final settlement between them.
(CAP Jun. 2019 Q 2a-10 Marks)

Answer:
a) M/S Pokhrel and Paudel
Balance Sheet
As at Poush end 2075
Liabilities Amount Assets Amount
Rs. Rs.
Sundry Creditors 5,000,000 Stock 7,500,000
Bank Overdraft 2,000,000 Machinery 15,000,000
Capital Accounts: Less: Dep. 750,000 14,250,000
Pokharel 12,290,000 23,980,000 Debtors 6,000,000
Paudel 11,690,000 Leasehold Premises 3,400,000

© The Institute of Chartered Accountants of Nepal 447


CAP II Paper 1: Advanced Accounting

Less:Dep. 170,000 3,230,000


30,980,000 30,980,000

Pokharel = 14,000,000 -1,000,000-1,300,000+1,180,000-590,000 = 12,290,000


Paudel= 13,000,000-600,000-1,300,000 +1,180,000-590,000 = 11,690,000

Partners’ Capital Account


Particulars Pokhrel Paudel Particulars Pokhrel Paudel
To P/L A/C 1,300,000 1,300,000 By Balance b/d 14,000,000 13,000,000
To Drawing 1,000,000 600,000 By P/L A/C 1,180,000 1,180,000
To Drawing (Profit) 5,000,000 5,000,000
(Out of Profit) 590,000 590,000 By Realization - 300,000
To Equity Shares in 16,990,000 16,990,000 A/C
Global Apparel ltd. - By Cash
To Cash 300,000
20,180,000 19,480,000 20,180,000 19,480,000

Working Notes:
Computation of Profit earned:
Balance Sheet as at Poush end 2075
Liabilities Amount Rs. Assets Amount Rs.
Sundry Creditors 5,000,000 Stock 7,500,000
Bank Overdraft 2,000,000 Machinery 15,000,000
Capital Accounts:# Less: Dep. 750,000 14,250,000
Pokharel 11,700,000 Debtors 6,000,000
Paudel 11,100,000 Leasehold Premises 3,400,000
Retained Earnings (BF)* 1,180,000 Less: Dep. 170,000 3,230,000
Total 30,980,000 Total 30,980,000
• Since half the profit was withdrawn, total profit for the six month was Rs. 2,360,000.
Capital Accounts:#
Pokharel = 14,000,000 -1,000,000-1,300,000 = 11,700,000
Paudel= 13,000,000-600,000-1,300,000 = 11,100,000

Realization Account
To Assets (Transfer) By Sundry Creditors 5,000,000
Stock 7,500,000 By Bank Overdraft 2,000,000
Machinery 14,250,000 By Global Apparel Ltd 33,980,000
Debtors 6,000,000 (Purchase Consideration)
Premises 3,230,000
To Capital A/C
(Profit on Realization)
Pokhrel 5,000,000 10,000,000
Paudel5,000,000
40,980,000 40,980,000

© The Institute of Chartered Accountants of Nepal 448


CAP II Paper 1: Advanced Accounting

CHAPTER 5 PREPARATION & PRESENTATION of FINANCIAL


STATEMENTS of SPECIAL ORGANIZATIONS

5.1 Banking & Financial Institutions

A. Theoretical Questions

1. Explain the rationale of linking Risk Weighted Assets with Capital of a bank or financial
institution. (CAP Dec. 2009 Q 3a(i) -3 Marks)

Answer
It is an inbuilt mechanism by which total risk exposure of financial institutions is made directly
proportional to its capital. If total risk exposure is increased or in other words if the institution
intends to expose itself to more risky portfolios, it has to increase its capital. Ultimately, it plays
as a cushion for protecting depositors’ interest. Owners/Shareholders should have adequate
capital so that they can continue managing public deposits in the form of different assets.

2. What are the classifications of Loans/Advances as per Nepal Rastra Bank Directives
(CAP Dec. 2011 Q 4c-5 Marks)
Answer:
Entire loans and advances extended by a licensed institution have to be classified as follows
based on expiry of the deadline of repayment of the principal and interest of such
loans/advances:
1. Pass: Loans/advances which have not overdue and which are overdue by a period up to
three months.
2. Sub-Standard: Loans/advances which are overdue by a period from three months to a
maximum period of six months.
3. Doubtful: Loans/advances which are overdue by a period from six-months to a
maximum period of one year.
4. Loss: Loans/advances which are overdue by a period of more than one year.
The loans which are in pass class and which have been rescheduled/restructured are called as
"the performing loan, and the sub-standard, doubtful and loss categories are called non-
performing loans.
Note: Loans/advances also include bills purchased and discounted.

3. Non-performing assets
(CAP Dec. 2012 Q6c-2.5 Marks, CAP Dec. 2014 Q 3b-3 Marks; Inter Dec. 2008 Q6a-4
Marks; CAP Jun. 2015 Q3b- 3 Marks; Inter Jun. 2003 Q9b -3 Marks)

Answer:
While preparing financial statements of a bank, it is necessary to identify non-performing assets
mostly based on statutory/regulatory norms. An asset becomes non-performing when
installment of matured principal and or income from it is not received by the bank for a certain

© The Institute of Chartered Accountants of Nepal 449


CAP II Paper 1: Advanced Accounting

period. Income from non-performing assets can only be accounted for as and when it is actually
received. Nepal Rastra Bank has issued directives for the classification of loans and advances.
Necessary provision should be made for non-performing assets classifying as sub-standard,
doubtful or loss assets as the case may be as per the rate prescribed by Nepal Rastra Bank.
NPA is a classification used by financial institutions that refer to loans that are in jeopardy of
default. Once the borrower has failed to make interest or principle payments for 90 days the
loan is considered to be a non-performing asset. Non-performing assets are problematic for
financial institutions since they depend on interest payments for income. Troublesome pressure
from the economy can lead to a sharp increase in non-performing loans and often results in
massive write-downs.
The nonperforming asset is not yielding any income to the lender in the form of principal and
interest payments.

4. Accounting for Non-Banking assets as per NRB Directives.


(CAP Dec. 2013 Q6b-2. 5 Marks)
Answer:
Banks and financial institutions are primarily engaged in lending activities. No bank or
financial institution lends without getting adequate level of collateral. All the debts of banks
are securitized as there is adequate level of collateral back up. In case where there are least
possibilities of recovery of loans and advances, recovery procedures are initiated and the assets
taken as collaterals are taken over by banks as Non-Banking Assets. The Non-banking assets
are to be taken over lower of the following values:
1. Fair Valuation of collateral at the time of taking over as non-banking assets, or
2. Principal and interest outstanding on the day immediately preceding the day of taking over
as NBA.
In case if Fair Valuation of collateral is less than outstanding loans and advances on the day
immediately preceding the day of taking over as NBA, the balance amount should be written
off to profit and loss account.

5. Capital adequacy ratio. (CAP Dec. 2015 Q 6b)

Answer:
Capital adequacy ratio is a measure of a bank’s capital. It is expressed as a percentage of a
bank’s risk weighted credit exposure. It is also known as “Capital to Risk Weighted Assets
Ratio (CAAR). This ratio is used to protect depositors and promote the stability and
efficiency of financial system.
It is calculated as:
CAR = Tier I capital + Tier II Capital
Risk weighted assets
• Tier I capital is core capital which includes equity capital and disclosed reserves.
• Tier II capital is supplementary capital which includes items such as revaluation
reserves, undisclosed reserves, hybrid instruments and subordinated term debts.

© The Institute of Chartered Accountants of Nepal 450


CAP II Paper 1: Advanced Accounting

• Risk weighted assets is banks assets or off balance sheet exposure weighted according
to risk.

6. Exchange equalization fund (CAP Dec. 2015 Q6d; CAP Dec. 2017 Q6b-3 Marks)

Answer:
Section 45 the Bank and Financial Institution Act, 2063 requires that every Bank Financial
Institution which has obtained a license to deal in foreign exchange must maintain an Exchange
Equalization Fund. The bank and financial institution must transfer 25% of the revaluation
profit earned as a result of changes in the exchange rates of foreign currencies other than the
Indian currency every year at the end of the same fiscal year to the exchange equalization fund.
The amount credited to the Exchange Equalization Fund may not be spent or transferred for
any purpose other than the adjustment of losses resulting from the devaluation of foreign
currencies without the approval of the Nepal Rastra Bank.

7. General reserve fund (CAP Dec. 2016 Q6c-3 Marks)

Answer:
Section 44 of Bank and Financial Institution Act 2073, states that every Bank and Financial
Institution must maintain a general reserve fund and transfer at least 20 percent of the annual
net profit to such fund every year until the amount of such fund reaches an amount double the
paid-up capital. The amount credited to the general reserve fund of a licensed institution may
not be invested or transferred to any other head without the prior approval of the Nepal Rastra
Bank.

8. Write short notes on Rebate on Bills Discounted (Inter Jun. 2011 Q5c- 5 Marks)

Answer:
When a bank discounts a bill of exchange, the full amount of the discount earned is credited
to the Discount Account but some of the bills discounted may not mature for payment by the
close of the year; as a result' the amount of discount in respect of such bills would not have
been earned during the year. On this consideration, the unexpired portion of such discount is
carried forward to next accounting period by debiting the Discount Account and crediting the
Rebate on Bills Discounted Account. The later account is shown on the liabilities side of the
Balance Sheet as income received which had not accrued before the close of the year. At the
commencement of the period next following, the account is closed off by transferring the
amount to the Discount Account.

9. Write short notes on Rate of Loan Loss Provision for Performing & Non Performing Loan as
per NRB Directives (CAP Jun. 2012 Q6d- 2.5 Marks)

Answer:

© The Institute of Chartered Accountants of Nepal 451


CAP II Paper 1: Advanced Accounting

For the loans and bills purchase classified according to NRB Directives, the following
loan loss provision shall be maintained based on the remaining amount of principal:
Assets Rate of Provision
Pass / Good 1%
Sub-standard 25 %
Doubtful 50 %
Loss loan/the loan extended to blacklisted
persons, firms, company or corporate body 100%

10. Features of a bank’s book-keeping system


(Inter Jun. 2005 Q6d – 4 Marks; Inter Jun. 2007 Q5b-5 Marks)

Answer:
(i) Voucher posting: Entries in the personal ledgers are made directly from vouchers instead
of being posted from the books of prime entry.
(ii) Voucher summary sheets: The vouchers entered into different personal ledgers each day
are summarized on summary sheets. The totals of these summary sheets are posted to the
control accounts in the general ledger.
(iii) Daily trial balance: The general ledger trial balance is extracted and agreed every day.
(iv) Continuous checks: All entries in the detailed personal ledgers and summary sheets are
checked by persons other than those who have made the entries. This checking is done
every day. As a result most clerical mistakes are detected before another day begins.
(v) Control accounts: A trial balance of the detailed personal ledgers is prepared periodically,
usually every two weeks. It is agreed with general ledger control accounts.
(vi) Double voucher system: Two vouchers are prepared for every transaction not involving
cash - one debit voucher and another credit voucher.

11. Write short notes on Discounting and Collection of Bills (Inter Dec. 2004 Q 6c-4Marks)

Answer
Among the various function carried out by a bank, the two important ones are those relating to
discounting of bills and acceptance and endorsement of bills on behalf of customers.
A bank may either straight way purchase a bill or any other credit instrument from a customer
on may collect it on his behalf. If it purchase or discounts the bill the amount would be
immediately credited to the account of the customer less discount charges, and debited to
Discounted bills Account. This account is an asset. If, on the other hand, a bill is to be collected
for a customer, the particulars of the bill would be recorded in a special book known as “Bill
for collection Register”. Any book entry would not be required until the bill is collected. On
collection, cash will be debited with the full amount received and the customer’s account will
be credited after deducting the amount of the commission. Bills held for collection are to be
shown by way of note below the Balance Sheet as per requirement of New forms.

© The Institute of Chartered Accountants of Nepal 452


CAP II Paper 1: Advanced Accounting

B. Practical Questions

1. Following are the extraction of the Financial Statements of a Financial Institution for the
Year ending 2008/09:

Rs. In lakhs
Cash in Hand 300
Cash at Bank 500
Loan and Advances
Gross- 10,000
Accumulated Provisions 200 9,800
Fixed assets 150
Other Assets 200

Following further information is available:


Risk Weightage given for above assets is 0%,20%,100%,100% and 100%
respectively. Other assets include Rs.130 Lakhs for expenditures not yet written off.
The financial institution has managed its total Capital up to 11% of the total Risk
Weighted Assets.

Find out the amount of Reserve and Surplus if the paid up capital is Rs. 900 Lakhs.
(CAP Dec. 2009 Q3a ii-7 Marks)
Answer:

Calculation of Risk Weighted Assets


Amount Rs. In Lacs
Assets Amount Risk Weight (in %) RWA Amount
Cash in Hand 300 0 0
Cash at Bank 500 20 100
Loan And Advances 10000 100 10000
Fixed Assets 150 100 150
*Other Assets 70 100 70
Total 11020 10320

*Expenses not written off is excluded from RWA as it is to be deducted from its core capital

As the Institution has just maintained its capital of 11 %, Total Capital including Reserve and
Surplus comes to- 11% of 10320,i.e. Rs 1135.2 Lacs

However, there are fictitious assets not written off to the extent of Rs 130 Lacs which is already
deducted while arriving at above total capital.

Hence, total capital comes to Rs (1135.2+130)=Rs. 1265.20 Lacs

© The Institute of Chartered Accountants of Nepal 453


CAP II Paper 1: Advanced Accounting

Total Capital Comprises of Paid up Capital and Reserve and Surplus and paid up capital is Rs
900 Lacs.

So, total Reserve and Surplus comes to Rs.(1265.20-900)= Rs 365.20 Lacs

2. Following facts have been extracted from the record of Adarsha Bank Ltd. in respect of the
year ending Ashadh 32, 2067:
On 1-4-2066 Bills for collection were Rs.700,000. During 2066/067 bill received for collection
amounted to Rs. 6,450,000, bills collected were Rs. 4,700,000 and bills dishonored and
returned were Rs.550,500.
Required:
Prepare Bills for Collection (Assets) A/C and Bills for Collection (Liability) A/C.
(CAP Dec. 2010 Q 4b-5 Marks)
Answer:
Bills for Collection (Assets) A/c
Date Particulars Rs. Date Particulars Rs.
01/04/2066 To balance b/d 700,000 2066/067 By Bills for collection
2066/067 To Bills for collection (Liabilities) A/c 4,700,000
( Liabilities) A/c 6,450,000 By Bills for collection
(Liabilities) A/c 550,500
32/3/2067 By Balance c/d 1,899,500

7,150,000 7,150,000

Bills for Collection (Liabilities) A/c


Date Particulars Rs. Date Particulars Rs.
2066/067 To Bills for collection 01/04/2066 By Balance b/d 700,000
( Assets) A/c 4,700,000 2066/067 By Bills for collection
To Bills for collection (Assets ) A/c 6,450,000
( Assets) A/c 550,500
32/3/2067 To Balance c/d 1,899,500

7,150,000 7,150,000

3. Following are the extracts from the record of Alpha Bank Ltd. in respect of the year ending
Ashadh 31, 2069:
On Shrawan 01, 2068; bills for collection were Rs. 400,000. During 2068-069 bill received for
collection amounted to Rs. 6,050,000, bills collected were Rs. 4,500,000 and bills dishonored
and returned were Rs. 300,000.
Prepare bills for collection (Assets) account and bills for collection (Liability) account.
(CAP Dec. 2012 Q4b-5 Marks)

© The Institute of Chartered Accountants of Nepal 454


CAP II Paper 1: Advanced Accounting

Answer:

Bills for Collection (Assets) A/c


Date Particulars Rs. Date Particulars Rs.
By Bills for
1/4/2068 To balance b/d 400,000 2068/069 collection
To Bills for
2068/069 collection (Liabilities) A/c 4,500,000
By Bills for
( Liabilities) A/c 6,050,000 collection
(Liabilities) A/c 300,000
31/3/2069 By Balance c/d 1,650,000
6,450,000 6,450,000

Bills for Collection (Liabilities) A/c


Date Particulars Rs. Date Particulars Rs.
To Bills for
2068/069 collection 1/4/2068 By Balance b/d 400,000
By Bills for
( Assets) A/c 4,500,000 2068/069 collection
To Bills for
collection (Assets ) A/c 6,050,000
( Assets) A/c 300,000
31/3/2069 To Balance c/d 1,650,000

6,450,000 6,450,000

4. Great Bank Limited has the following status of Loans and Advances as on 31st Ashadh
2070:
Category Amount (Rs.)
Good 3,309,000
Rescheduled /Restructured -
Substandard 292,000
Doubtful 665,000
Bad 99,000

You are required to calculate the loan loss provision as per the directive issued by Nepal
Rastra Bank and find the percentage of total Non-performing assets.
(CAP Dec. 2013 Q3b-2 Marks)
Answer:

Calculation of Loan Loss Provisioning and Non-Performing Loans

© The Institute of Chartered Accountants of Nepal 455


CAP II Paper 1: Advanced Accounting

Provision
Category Weight Amount Amount
1. Good 1% 3,309,000 33,090
2. Rescheduled /Restructured 12.50% - 0
3. Substandard 25% 292,000 73,000
4. Doubtful 50% 665,000 332,500
5. Bad 100% 99,000 99,000
TOTAL 4,365,000 537,590
Non-Performing Loan (NPL) 1,056,000
NPL/Total Loan 24.19

5. Following information as at second quarter ending 2071 were drawn from the records of
Shivam Bank Limited as under:

Loans outstanding for: Amount (Rs.)


Up to 3 months 1,868,420
More than 3 months but not more than 6 months 11,720
More than 6 months but not more than 12 months 262
More than 12 months 2,032
Total 1,882,434
Following additional information relating to previous quarter ending were extracted from the
records of the bank:

Paid up equity share capital 161,016


Statutory general reserve 163,632
Retained earnings 96,657
General loan loss provisions 18,497
Exchange equalization reserve 25,919
Un-audited current year cumulative profit 33,185
Deferred revenue expenses 2,668

The bank is in the process of preparing the documents for quarterly reporting. As a reporting
and compliance officer of the bank you are required to calculate movement in loan loss
provision amount. The bank has also provided a Term loan of Rs. 125,000 to a Party during
the period under review.
(CAP Dec. 2015 Q2a)
Answer:
As per provisions of the Directive 3 of Unified Directives, a bank can provide a credit up to
25% of its core capital to single party. This limit is called as Single Obligor Limit (SOL). While
calculating the SOL core capital of previous quarter shall be taken as base. In case any excess
credit has been provided by the bank, additional 100% provision shall be made for such excess
credit amount. Before calculating the provision amount, SOL of the bank shall be tested upon.

© The Institute of Chartered Accountants of Nepal 456


CAP II Paper 1: Advanced Accounting

Computation of SOL and credit amount in excess of SOL


Particulars Amount Rs.
Core Capital:
Paid up Equity Share Capital 161,016
Statutory General Reserve 163,632
Retained Earnings 96,657
Un-audited current year cumulative profit 33,185
Less: Deferred Revenue Expenses (2,668)
Total Core Capital 451,822
Single Obligor Limit (25% of Core Capital) 112,956
Loan to a Single Party 125,000
Loan in Excess of SOL 12,044
Now, we compute the loan loss provision amount as per provisions of the Directive 2 of
the NRB.
Computation of Loan Loss Provision Amount
Due for Categories Loan Provision Provision
Amount Rate Amount
Not due or <=3 Months Pass 1,868,420 1% 18,684
>3 Months but <=6 Months Substandard 11,720 25% 2,930
>6 Months but <=1 Year Doubtful 262 50% 131
>1 Year Loss 2032 100% 2,032
Total 1,882,434 23,777
Additional Provision for 12,044
Loan in excess of SOL
Total Provision Amount 35,821

Shivam Bank Limited


Movement in Provision Amount
For Second Quarter of Fiscal Year 2071/72

Particulars Amount
Opening Provision Amount 18,497
Closing Provision Amount 35,821
Movement (addition during the Quarter) in Provision Amount 17,324

6. Following information as at second quarter ending 2073 were drawn from the records of
Minamma Bank Limited as under:

Loan outstanding for Amount (Rs.)


Up to 3 months 1,673,000
More than 3 months but not more than 6 months 13,612
More than 6 months but not more than 12 months 782

© The Institute of Chartered Accountants of Nepal 457


CAP II Paper 1: Advanced Accounting

More than 12 months 2,198


Total 1,689,592
The bank has not restructured or rescheduled any of its credit.
Following additional information relating to previous quarter ending were
extracted from the records of the bank:

Particulars Amount (Rs.)


Paid up equity share capital 171,010
Statutory general reserve 155,432
Retained earnings 87,886
General loan loss provision 16,983
Exchange equalization reserve 22,313
Un-audited current year cumulative profit 31,991
Deferred revenue expenses 2,884

The bank is in the process of preparing the documents for quarterly reporting. The bank has
also provided a term loan of Rs. 125,000 to a single party during the period under review. As
a reporting and compliance officer of the bank you are required to calculate movement in loan
loss provision amount.
(CAP Dec. 2017 Q2b-10 Marks)
Answer:

As per the provision of the Directive 3 of unified Directives, a bank can provide a credit up to
25% of its core capital to a single party. This limit is called the single obligatory limit (SOL).
While calculating the SOL, core capital of previous quarter shall be taken as base. In case any
excess credit than SOL has been provided by the bank, additional 100% provision shall be
made for such excess credit amount.
Before calculating the provision amount, SOL of the bank shall be tested upon.

Computation of SOL and credit amount in excess of SOL


Particulars Amount
Core Capital
Paid up Equity Share Capital 171,010
Statutory General reserve 155,432
Retained earnings 87,886
Un-audited current year cumulative profit 31,991
Less: Deferred Revenue expenses (2,884)
Total Core capital 443,435
Single obligatory limit ( 25% of the core capital) 110,859
Loan to single party 125,000
Loan in excess of SOL 14,141

Computation of loan loss provision amount as per provisions of the Directive 2 of the NRB

© The Institute of Chartered Accountants of Nepal 458


CAP II Paper 1: Advanced Accounting

Computation of Loan Loss Provision amount


Categories Loan Amt Provision Provision
Rate Amount
Not due or <=3 months Pass 1,673,000 1% 16,730
>3 months <= 6 mths Sub-standard 13,612 25% 3,403
>6 months <= 12 mths Doubtful 782 50% 391
>12 months Loss 2,198 100% 2,198
Total 1,689,592 22,722
Additional provision for loan in excess of SOL 14,141
Total Provision amount 36,863

Minamma Bank Ltd


Movement in Provision Amount
For Second Quarter of Fiscal Year 2073/74

Amount in NPR
Particulars Amount
Opening Provision amount 16,983
Closing Provision amount 36,863
Movement in provision amount (addition during the quarter) 19,880

7. From the following information, find out the risk weighted exposure for operational risk
assuming 11% is the capital adequacy requirement.
(Rs.)
Particulars Year I Year II Year III
Interest Income (Rs.)
1,466,454 1,626,474 1,775,583
Interest Expenses 561,964 648,842 767,411
Commission and Discount Income 132,816 165,448 193,224
Other Operating Income 137,301 198,130 151,637
Exchange Fluctuation Income 41,301 52,325 40,329
Addition in interest Suspense Account 25,693 34,376 36,711
Non-Operating and Extraordinary Income 2,795 1,887 100,257
(CAP Dec. 2017 Q5a -5 Marks)
Answer:
Net Interest Income 904,490.00 977,632.00 1,008,172.00
Commission and Discount Income 132,816.00 165,448.00 193,224.00
Other Operating Income 137,301.00 198,130.00 151,637.00
Exchange Fluctuation Income 41,301.00 52,325.00 40,329.00
Addition/Deduction in Interest Suspense during the
period 25,693.00 34,376.00 36,711.00
Gross Income( a) 1,241,601.00 1,427,911.00 1,430,073.00
Alfa (b) 15% 15% 15%

© The Institute of Chartered Accountants of Nepal 459


CAP II Paper 1: Advanced Accounting

Fixed percentage of Gross Income © 186,240.15 214,186.65 214,510.95


Capital Requirement for operational risk (d )
(average of C) 204,979.25
Risk weight in times(e) 11
Equivalent Risk Weight Exposure (f=d*e) 225,4771.75

Net Interest Income


Year I Year II Year III
Interest Income 1,466,454.00 1,626,474.00 1,775,583.00
Interest Expenses 561,964.00 648,842.00 767,411.00
Net Interest Income 904,490.00 977,632.00 1,008,172.00

8. Following is the information related to loan loss provisions (LLP) of Big Bank Ltd. as on
32.03.2075.

Classification LLP (in lakhs)


Pass loan 2,500
Watchlist 500
Re-scheduled 350
Sub-standard 150
Doubtful 400
Bad 1,000

Sub-standard loan is fully insured with DCG Fund. Find out the level of Non-Performing Loan
(NPL) as on 32.03.2075 of the banks.
(CAP Dec. 2018 Q 4b-5 Marks)
Answer:
Statement of calculation of NPA as on 31.03.2075.
(Amount in Lakhs)

Classification LLP LLP rate Loan amount


Pass loan 2,500 1% 250,000
Watchlist 500 5% 10,000
Re-scheduled 350 12.50% 2,800
Sub standard 150 6.25%(insured so 25% only required) 2,400
Doubtful 400 50% 800
Bad 1,000 100% 1,000
Total 267,000
NPA 7,000
NPA% 2.62

9. Following figures are extracted from the financial statements of a Commercial Bank named
Naya Nepal Bank Ltd.:

© The Institute of Chartered Accountants of Nepal 460


CAP II Paper 1: Advanced Accounting

(Rs. in Lakh.)
Particulars Rs. Particulars Rs.
Paid up Capital 84,44 Exchange equalization Fund 2,59
General Reserve Fund 36,24 Subordinated Term Debt 12,90
Retained Earnings 3,46 General Loan Loss Provision 14,72
Contingent Reserves 2,00 Specific Loan Loss Provision 15,12
Other Free Reserves 11,64 Risk Weighted Exposure:
Proposed Dividend: For Credit Risk 16,02,50
Stock 33,78 For Market Risk 1,06,23
Cash 6,22 For Operational Risk 8,02
Fictitious Assets 1,21

Calculate the Capital Adequacy Ratios (Core or Primary Capital and Capital Fund Ratio) of the
Bank as per NRB Directives.
(Inter Jun. 2010 Q5d- 8 Marks)
Answer:
Commercial Banks in Nepal are required to maintain certain level of capital as minimum capital
as per Capital Adequacy Framework issued by Nepal Rastra Bank. The Capital Adequacy
Framework issued by NRB clearly spells out as to what constitutes Capital – both Primary
(Core) Capital and Supplementary Capital. As per the framework, Primary Capital and
Supplementary Capital both put together constitute Total Capital Fund. The framework also
provides the methods for calculating Capital Adequacy Ratios. As such, Primary (Core) Capital
Ratio as well as Total Capital Fund Ratio is expressed as a percentage of Total Risk Weighted
Exposure of the bank.
As per the Capital Adequacy Framework issued by NRB, the Capital Adequacy Ratios of the
Naya Nepal Bank Limited can be calculated as follows:
Naya Nepal Bank Limited
Capital Adequacy Table
(Rs. In Lakh)

Particulars Rs.
A. Risk Weighted Exposures 17,16,75
Risk Weighted Exposure for Credit Risk 16,02,50
Risk Weighted Exposure for Operational Risk 1,06,23
Risk Weighted Exposure for Market Risk 8,02
B. Capital
1. Core or Primary Capital 1,68,35
Paid up Capital 84,44
General Reserve Fund 36,24
Proposed Stock Dividend 33,78
Retained Earnings 3,46
Other Free Reserves 11,64
Less: Fictitious Assets (1,21)

© The Institute of Chartered Accountants of Nepal 461


CAP II Paper 1: Advanced Accounting

2. Supplementary Capital 32,21


General Loan Loss Provision 14,72
Subordinated Term Debt 12,90
Exchange Equalization Fund 2,59
Contingent Reserves 2,00
3. Total Capital Fund 2,00,56
C. Capital Adequacy Ratios
Core Capital to Total Risk Weighted Exposures 9.81 %
Total Capital Fund to Total Risk Weighted Exposures 11.68 %

10. Find out the income to be recognized in the case of Good Faith Bank Limited for the year
ended 31st Ashadh, 2067 as per the directives issued by NRB.

(Rs. in Lakhs)
Performing Assets Non-performing Assets
Interest Realized in Interest Realized in
accrued cash accrued cash
Term Loans 540 400 450 20
Consumer Loan 250 205 360 16
(CAP Jun. 2011 Q3a i- 2 Marks)
Answer:
Calculation of interest income of Good Faith Bank Ltd. to be recognized for the year ended
31st Ashadh, 2067.
Term Loans Rs. in Lakhs
Interest realized in cash regarding performing assets 400
Interest realized in cash regarding non-performing assets 20 420
Consumer Loan
Interest realized in cash regarding performing assets 205
Interest realized in cash regarding non-performing assets 16 221
Total Interest to be recognized 641

11. Following are the extracts of financial statements of a commercial bank:


Particulars Rs. '000
Total risk weighted exposure 62,704,174
Paid up equity share capital 3,012,924
Share Premium 11,849
Proposed bonus equity shares 753,231
Statutory general reserves 1,532,353
Retained earning 19,635
Debenture redemption reserve 630,364

© The Institute of Chartered Accountants of Nepal 462


CAP II Paper 1: Advanced Accounting

Investment in equity of institutions with financial interests 101,500


Subordinated term debt 1,050,000
General loan loss provision 414,741
Exchange equalization reserve 50,633
Investment adjustment reserve 38,920

Required:

i) Amount of core capital (Tier I)


ii)Amount of supplementary capital (Tier II)
iii)
Amount of total capital fund
iv)Capital adequacy ratios with regard to:
(a) Core capital
(b) Total capital fund
v) Explain whether the bank meets standard capital adequacy ratios requirement.
(CAP Jun. 2013 Q 3a-10 Marks)
Answer:

i) Calculation of amount of Core Capital (Tier I)

Particulars Rs.'000
Paid up Equity Share Capital 3,012,924
Share Premium 11,849
Proposed Bonus Equity Shares 753,231
Statutory General Reserves 1,532,353
Retained Earning 19,635
Debenture Redemption Reserve 630,364
less: Investment in equity of institutions with financial interests (101,500)
Total Core Capital 5,858,856

ii) Calculation of amount of Supplementary Capital (Tier II)


Particulars Rs.'000
Subordinated Term Debt 1,050,000
General loan loss provision 414,741
Exchange Equalization Reserve 50,633
Investment Adjustment Reserve 38,920
Total Supplementary Capital 1,554,294

iii) Amount of total capital fund (Rs.'000) = Core Capital + Supplementary Capital
= 5,858,856 + 1,554,294
= 7,413,150

iv) Capital Adequacy Ratios

© The Institute of Chartered Accountants of Nepal 463


CAP II Paper 1: Advanced Accounting

Core Capital to Total Risk weighted exposure = Core Capital/ Total RWE
= 5,858,856/ 62,704,174
= 0.0934
= 9.34%

Total capital fund to Total Risk weighted exposure = Total capital fund/ Total RWE
= 7,413,150/ 62,704,174
= 0.1182
= 11.82%

v) As per NRB requirement, the core capital to total RWE should be at least 6%
and total capital fund to total RWE should be at least 10%. From the above calculation,
it is evident that the capital adequacy ratios of the bank meet the standard requirement.

12. BN Bank Limited provides you the following information regarding Loan Loss
provisioning as on 31st Ashadh 2069:

Category Amount Rs.


1. Good 5000,000
2. Rescheduled /Restructured 210,000
3. Substandard 500,000
4. Doubtful 300,000
5. Bad 500,000
During financial year 2069-70 additional loans amounting to Rs. 3,000,000 were
disbursed. The Bad loans amounting to Rs. 200,000 was written off during the year. Loans
amounting to Rs. 150,000 were shifted from Doubtful category to Bad category. Similarly,
Loans amounting to Rs. 500,000 shifted from Good category to substandard category and
Substandard Loans amounting to Rs. 200,000 were rescheduled during the year.
From the above information, you are required to calculate the loan loss provision and pass
necessary journal entries in the books of BN Bank Limited as per the directive issued by Nepal
Rastra Bank and find the percentage of total Non-Performing assets.
(CAP Jun. 2014 Q 3c- 8 Marks)
Answer:

(Rs.)
Calculation of Loan Loss Provisioning
Provision
Loan as on Amount
Amount 31st 31st 31st
Ashadh Ashadh Additional Ashadh Rate of Total
Category 2069 2069 Loans Shifting 2070 Provision Provision
1. Good 5,000,000 50,000 3,000,000 (500,000) 7,500,000 0.01 75,000

© The Institute of Chartered Accountants of Nepal 464


CAP II Paper 1: Advanced Accounting

2. Rescheduled
/Restructured 210,000 26,250 200,000 410,000 0.125 51,250
3. Substandard 500,000 125,000 300,000 800,000 0.25 200,000
4. Doubtful 300,000 150,000 (150,000) 150,000 0.5 75,000
5. Bad 500,000 500,000 (50,000) 450,000 1 450,000

Total 6,510,000 851,250 9,310,000 851,250


Total NPA 1,400,000
NPA/Total
Loans 15.038

Journal Entries
In the books of BN Bank Limited

Loans and Advances Dr.


To Cash A/c 30,00,000
(Being Loans given during the year. Assumed that all the loans 30,00,000
are taken in cash)
Profit and Loss A/c Dr.
Restructured Loans Dr. 2,00,000
Substandard Loans Dr. 2,00,000
To Good Loans 3,00,000
To Doubtful Loans 5,00,000
To Bad loans 1,50,000
(Being changes of in various categories of loans and advances 50,000
during the year.)
Profit and Loss A/c Dr.
To LLP Good Loans 1,25,000
To LLP Restructured Loans 25,000
To LLP Substandard 25,000
(Being changes of in various categories of loans loss provisioning 75,000
during the year.)
LLP Doubtful Loans Dr.
LLP Bad Loans Dr. 75,000
To Profit and Loss A/c 50,000
(Being changes of in various categories of loans and advances 1,25,000
during the year.)
Note:
Though no additional entry is required for the amount of LLP as the amount of LLP last year
and this year is same but individual category wise LLP has changed thus requiring the above
journal entries.

© The Institute of Chartered Accountants of Nepal 465


CAP II Paper 1: Advanced Accounting

13. A Commercial Bank has the following capital funds and assets. Segregate the capital funds
into core capital (Tier I) and supplementary capital (Tier II). Find out the risk weighted
asset and capital fund ratio.

Particulars (Rs. in crores)


Equity share capital 500.00
Statutory reserve 270.00
Capital reserve (of which Rs. 16 crores were due to revaluation of
assets and the balance due to sale of capital asset) 78.00
Assets:
Cash balance with Nepal Rastra Bank (NRB) 10.00
Balance with other banks 18.00
Other investments 36.00
Loans and advances:
(i) Guaranteed by the Government 16.50
(ii) Others 5,675.00
Premises, furniture and fixtures 78.00
Off-Balance Sheet items:
(i) Advance Payment Guarantee 800.00
(ii) Contingent Liability in respect of Income Tax 4,800.00
(CAP Jun. 2016 Q 2a- 10 Marks)
Answer:
(i) Rs in crores Rs in crores
Core Capital- Tier I
Equity share capital 500
Statutory reserve 270
Capital reserve (arising of sale of assets) (78-16) 62
832
Supplementary Capital – Tier II
(Capital Reserve (arising out of revaluation of assets) 16 0.32
As per NRB Directive- Capital Reserve relates with
revaluation reserve of assets, up to 2% of total
Supplementary Capital only allowed for the computation)
Total Capital Fund 832.32

Rs in crores % of weight Rs in crores


(ii) Risk Weighted Assets
Funded Risk Assets
Cash balance with NRB 10 0 0
Balance with other banks 18 20 3.60
Other investments 36 100 36
Loans and advances:

© The Institute of Chartered Accountants of Nepal 466


CAP II Paper 1: Advanced Accounting

i) Guaranteed by the government 16.5 0 0


ii) Others 5,675 100 5,675
Premises, furniture and fixtures 78 100 78
5,792.60
Rs in crores Credit
conversion
factor
Off-Balance Sheet items:
Advance Payment Guarantees 800 100 800
Contingent Liability in respect of Income 4,800 100 4,800
Tax 11,392.60

Capital Fund Ratio:


Capital fund x100
Risk weighted assets
(832.32/11,392.60) x 100 =7.31%

14. Following balances relating to loan and advances are extracted from records of a ‘A’ class
Commercial Bank.

Amount in lakhs
Categories of Loan Insured Amount Other Amount Total Amount
Pass 3,335 161,528 164,863
Restructured/Rescheduled 775 4,464 5,239
Substandard 138 706 844
Doubtful 217 1,927 2,144
Loss 662 3,539 4,201
Total 5,127 172,164 177,291

While scrutinizing the records of the bank, it was found that the bank has provided a credit of
Rs. 5,560 lakhs in excess of SOL. The bank has categorized this loan as Pass. Find out the
amount of loan loss provisions as per NRB Directives.
(CAP Jun. 2018 Q3b-5 Marks)
Answer:

Computation of Loan Provisions Amount


(Amount in Lakhs)
Categories Insured Rate Provision Other Rate
Provision Total
of Loan Amount Amount Amount Provision
Amount
Pass 3,335 0.25 8 161,528 1% 1615 1624
Restructured 775 3.13 24 4,464 12.5% 558 582

© The Institute of Chartered Accountants of Nepal 467


CAP II Paper 1: Advanced Accounting

/Rescheduled
Substandard 138 6.25 9 706 25% 177 185
Doubtful 217 12.5 27 1,927 50% 964 991
Loss 662 25 166 3,539 100% 3539 3705
Total 5,127 234 172,164 6852 7086

Additional 100% Provision for Credit Provided in Excess of SOL (B) =5560
Total Loan Loss Provisions (A+B) = 12646
Working Note:
Provisioning Rate for Insured Loan i.e. = 25% of prescribed rate
1×0.25=0.25%
12.5×0.25=3.125
20×0.25=6.25%
50×0.25=12.5%
100×0.25=25%

15. The following trial balance was extracted from the books of Paschimanchal Bank Limited,
as on March 31,2004.

Particulars Dr Cr
Share Capital 3,00,000
Cash in hand and with NRB 46,350
Investments in Govt. Bonds 1,94,370
Other investments 1,55,630
Gold Bullion 15,130
Interest accrued on investments 24,620
Security deposits of employees 15,000
Saving account balances 7,420
Current ledger control account 97,000
Fixed deposits 23,050
Share Premium account 90,000
Statutory reserves 1,40,000
Silver Bullion 2,000
Constituent liability for acceptance and endorsement 56,500
Buildings 65,000
Furniture 5,000
Borrowing from banks 77,230
Money at call and short notice 26,000
Advances 2,00,000
Profit and loss Account 6,500
Bills discounted and purchased 12,500
Bills for collection 43,500
Acceptance and endorsements 56,500
Interest 7,950 72,000

© The Institute of Chartered Accountants of Nepal 468


CAP II Paper 1: Advanced Accounting

Commissions and Brokerage 25,300


Discounts 42,000
Bills receivable being bills for collection 43,500
Audit fees 5,000
Loss on sale of furniture 1,000
Director’s fees 1,200
Salaries 21,200
Postage 50
Rent 600
Profit on bullion 1,200
Managing director’s remuneration 12,000
Miscellaneous income 2,700
Loss on sale of investments 30,000
Deposits with other banks 75,000
Branch adjustments 20,000
Depreciation provisions on building 20,000
Total 10,20,000 10,20,000

The following further information is furnished to you:

(a) Provide rebate on bills discounted Rs. 5,000.


(b) A scrutiny of current account ledger reveals that there are accounts overdrawn to
the extent of Rs.25, 000 and the total of the credit balance is Rs.1, 22,000.
(c) Claims by employees for bonus amounting to Rs.15, 000 is pending award of
arbitration.
(d) Depreciation on building for the year is calculated at Rs.5, 000.
(e) It is proposed to transfer 20% to general reserves.
(f) The directors have decided to propose a dividend of 8% for the current year.

You are required to prepare the Balance sheet as on March 31, 2004 and a Profit and Loss
Account for the year ending as on that date, together with the necessary schedules and
workings.
(Inter Dec. 2004 Q 2)
Answer:

Pashcimanchal Bank Ltd.


Balance Sheet at 31st March, 2004
Schedule As on As on 31.3.2003(
No. 31.3.2004 Previous Year)
Capital and Liabilities
Capital 1 3,00,000
Reserve & Surplus 2 2,67,900
Deposits 3 1,52,470
Borrowings 4 77,230
Other liabilities and Provisions 5 44,000

© The Institute of Chartered Accountants of Nepal 469


CAP II Paper 1: Advanced Accounting

Total 8,41,600

Assets
Cash and Balance with NRB 6 47,350
Balances with banks and money at call and 7 1,01,000
short notice
Investments 8 3,65,130
Advances 9 2,37,500
Fixed assets 10 45,000
Other assets 11 46,620
Total 8,41,600

Contingent liabilities 12 71,500


Bills for collection 43,500

Pashcimanchal Bank Ltd.


PL A/c for the year ended 31st March, 2004
Schedule As on As on 31.3.2003
No. 31.3.2004 (Previous Year)
I. Income
Interest earned 13 1,09,000
Other Income 14 -1,200
Total 1,07,800

II. Expenditure
Interest expended 15 7,950
Operating expenses 16 44,450
Provisions and contingencies (Note 1) -
Total
52,400
III. Profit/Loss
Net Profit/Loss(-) for the year
Profit/Loss (-) brought forward. 55,400
Total 55,400
IV. Appropriations
Transfer to statutory reserves (20% of Net
Profit) 11,080
Transfer to other reserves -
Proposed dividend 24,000
(8% of Rs.3,00,000)
Balance carried over to balance sheet. 20,320
Total
55,400

Schedule 1: Capital Rs.


Authorized Capital. Shares of Rs…. each

© The Institute of Chartered Accountants of Nepal 470


CAP II Paper 1: Advanced Accounting

Issued Capital …shares of Rs…. Each


Subscribed Capital …shares of Rs…. Each
Called – up Capital …shares of Rs…. Each 3,00,000
Less : Calls unpaid -
Add : Forfeited shares -
Total 3,00,000

Schedule 2: Reserves & Surplus Rs


I. Statutory Reserves
Opening Balance 1,40,000
Addition during the year 11,080 1,51,080

II. Capital Reserves


Opening balance -
Addition during the year -
Deduction during the year 3,00,000
III. Share Premium
Opening balance - -
Addition during the year - -
Deduction during the year 90,000
IV. Revenue & Other Reserves
Opening balance -
Addition during the year -
Deduction during the year -
V. Balance in Profit & Loss Account
Last year 6,500
Current year 20,320 26,820
Total (I+II+III+IV+V) 2,67,900

Schedule 3: Deposits Rs.


A 1. Demand Deposits
(i) From Banks (Rs. 97,000 + Rs. 25,000) 1,22,000
(ii) From others 11,080 -
II. Saving Bank Deposit 7,420
III. Term Deposits
(i) From Banks
(ii) From others 23,050
Total 1,52,470
B.(i) Deposits of branches in Nepal 1,52,470
(ii) Deposit of branches outside Nepal - -
Total 1,52,470

Schedule 4 Borrowings Rs.


I. Borrowings in Nepal -
(i) NRB
(ii) Other Banks 77,230

© The Institute of Chartered Accountants of Nepal 471


CAP II Paper 1: Advanced Accounting

(iii) Other Institutions & Agencies -


II. Borrowing Outside Nepal
Total ( I + II ) 77,230
Secured Borrowing in I & II above 77,230

Schedule 5: Other Liabilities & Provisions Rs.


i. Bills payable
ii. Inter office adjustments (Net)
iii.Interest accrued
iv. Other ( including provisions)
Proposed Dividend 24,000
Provision for Rebate 5,000
Security Deposit of Employees 15,000 44,000
Total (I+II+III+IV) 44,000

Schedule 6: Cash and Balance with NRB Rs.


I. Cash in hand (including foreign currency notes) 46,350
II. Balance with NRB in :
i. Current Account
ii. Other Account
Total (I+II) 46,350

Schedule 7: Balance with Bank and Money at call and Short Notes Rs.
I. In Nepal
i. Balance with Banks. 75,000
a. in Current Accounts -
b. in other Deposits Accounts
ii. Money at Call & Short Notice 26,000

a. with Banks
b. with other institutions
Total (I + II) 1,01,000
II. Outside Nepal
i. in current Account -
ii. in Other Deposits Accounts -
iii. Money at call and short notice -
Total -
Grand Total ( I+II ) 1,01,000

Schedule 8: Investments Rs.


I. Investment in Nepal (Note 1)
Govt. Securities 1,94,370
Debentured and Bonds 1,55,630
Gold 15,130
II. Investment outside Nepal -
Grand Total (I+II) 3,63,130

© The Institute of Chartered Accountants of Nepal 472


CAP II Paper 1: Advanced Accounting

Schedule 9: Advances Rs.


A. (i) Bills Discounted and purchased 12,500
(ii) Cash credits, overdrafts & Loans payable on demand 2,25,000
( Rs.2,00,000 + Rs.25,000)
Total 2,37,500

Schedule 10: Fixed Assets Rs.


I. Premises :
At cost and as on 31st march of the preceding year 65,000
Additions during the year -
Deductions during the year -
Depreciation to date 25,000
40,400
II. Other Fixed Assets (including Furniture & Fixture)
At cost and as on 31st March of the preceding year 5,000
Additions during the year
Deductions during the year
Depreciation to date
Total (I+II) 45,400

Schedule 11 : Other Assets Rs.


I. Inter – office Adjustments (net) 20,000
II. Interest Accrued on Investments 24,620
III. Tax paid in Advance/ Tax Deducted at source
IV. Stationery and Stamps
V. Non – banking assets acquired in satisfaction of claims
VI. Other (silver) 2,000
Total 46,620

Schedule 12: Contingent Liabilities Rs.


I. Claims against the Bank not acknowledge as debts 15,000
II. Liability for partly paid investments
III. Liability of account of outstanding forward exchange
IV. Guarantees given on behalf of constituents
i. In India
ii. Outside Nepal
V. Acceptances, Endorsements and other obligations
VI. Other items for which the bank is contingently liable 56,000

Total 71,500
* Claim by employees for bonus for Rs.15,000 i.e. pending award of
arbitration

Schedule 13 ; Interest Earned Rs.

© The Institute of Chartered Accountants of Nepal 473


CAP II Paper 1: Advanced Accounting

I. Interest and discount receive


Add : Opening Bal. of rebate on bill discounted 1,14,000
Less: Closing Bal.of rebate on bill discounted
II. Interest on balance with NRB and other inter – bank funds -5000 1,09,000
Others
Total 1,09,000

Schedule 14 : Other Income Rs.


I. Commission exchange and brokerage. 25,300
II. Rent received 600
III. Net Profit on Sale of Investments 1,200
Less : Net Loss on Sale of investment 30,000 -28000
IV. Net profit on revaluation of investments
V. Net profit on sale of land, building and other assets -
Less: Net Loss on Sale of furniture -1000
VI. Net profit on exchange transactions -
VII. Income earned by way of dividends etc. from
Subsidiaries\Joint ventures setup abroad\ in Nepal -
VIII. Miscellaneous Income 2,700
Total -1,200

Schedule 15: Interest Expended Rs


I. Interest on deposits
II. Interest on NRB/Inter – bank borrowings 7,950
III. Others
Total 7,950
Schedule 16: Operating Expenses Rs
I. Payment to and provision for employees 32,200
II. Rent, taxes and lighting -
III. Printing and stationery -
IV. Advertisement and Publicity -
V. Depreciation on Banker’s Property 5,000
VI. Director’s fees and allowances 1,200
VII. Auditor’s fees and allowances 5,000
VIII. Law charges -
IX. Postage and telegram 50
X. Repairs and maintenance -
XI. Insurance -
XII. Other expenditure
Total 44,450
* ( Rs. 21,200 + 12,000 )

16. The following balances pertaining to Vijaya Bank Limited are presented to you. You are
required to prepare the Profit and Loss Account of the bank for the year ended on March
31, 2002. (All working should form part of your answer)

© The Institute of Chartered Accountants of Nepal 474


CAP II Paper 1: Advanced Accounting

Rs. '000s
i. Interest on loans 300
ii. Interest on fixed deposits 275
iii. Commission 10
iv. Exchange and brokerage 20
v. Salaries and allowances 150
vi. Discount on bills (gross) 152
vii. Interest on cash credits 240
viii. Interest on temporary overdrafts in current account 30
ix. Interest on saving bank deposits 87
x. Postage, telegram and stamps 10
xi. Printing and stationery 20
xii. Sundry expenses 10
xiii. Rent 15
xiv. Taxes and license renewals 10
xv. Audit fees 10
xvi. Additional information:
Rebate on bills discounted 30
Salary of managing director 30
Bad debts 40
xvii. Provision for income tax is to be made @33%(round off to the nearest
thousand)
xviii. Interest of Rs. 4,000 on doubtful debts was wrongly credited to interest on loans
account
(Inter Jun. 2002 Q 5 – 15 Marks)
Answer:
Vijaya Bank Limited
Profit & Loss A/C for the year ended Mar 31, 2002

Particulars Sch. No. Amt. (Rs.) ('000)


I. Income
Interest Earned 13 718
Other Income 14 30
------
748
II. Expenditure
Interest Expended 15 362
Operating Expenses 16 225
Provision & Contingencies 100

687
III. Profit/ Loss

© The Institute of Chartered Accountants of Nepal 475


CAP II Paper 1: Advanced Accounting

Net Profit for the year 61


Profit brought forward –
61
IV. Appropriation
Transfer to Statutory Reserve (20% of C. yrs. profit i.e. 12
20% of 41)
Balance Carried over the B/S 49
61

Schedule 13: Interest Earned


Interest on Loans (300-4) 296
Discount on bills 152
Interest on cash credits 240
Interest on temporary ODS in current A/c 30
-----
718
------
Schedule 14: Other Income
Commission 10
Exchange & brokerage 20
-----
30
------
Schedule 15: Interest Expenses
Interest on fixed deposits 275
Interest on saving bank deposits 87
-----
362
------
Schedule 16: Operating Expenses
i. Payments to and provision for employees (150-30) 120
ii. Rent, taxes and lighting (15+10) 25
iii. Printing and Stationery 20
iv. Directors', fees, allowances and expenses 30
v. Auditors' fees and expenses 10
vi. Postage, telegram and stamps 10
vii. Sundry expenses 10
------
225
=====
Provisions & Contingencies
Rebate on bills discounted 30
Bad debts 40

© The Institute of Chartered Accountants of Nepal 476


CAP II Paper 1: Advanced Accounting

Tax Provisions (WN 1) 30


-----
100
====
Working Note 1
Tax Provision:
As per schedules 13 & 14 748
Less: As per schedules 15 + 16 (587)
Less: Rebate on bills discount (30)
Less: Bad debts (40)
-------
91
=====

 33% of 91 = 30.03 = 30

17. The details of bills discounted in case of a bank are as under:


Amount Rs. Due Date Rate of Discount
2,00,000 4th June, 2000 12½%
5,50,000 12th June, 2000 12½%
3,20,000 30th June, 2000 12½%
4,50,000 9th July, 2000 15%
Calculate the rebate on bills discounted for the year ending 31st March, 2000.

Unexpired rebate on 31st March, 1999 was Rs. 40,000. Discount received during the year was
Rs. 1,35,000. Find out the discount to be credited to Profit and Loss Account and give the
necessary Journal entries.
(Inter Jun. 2001 Q 5b – 8 Marks)
Answer
Rebate on Bills discounted
Rs.
Bill due on 4th June i.e. i.e. 65 days from 31/3/2000
= 4,452
i.e. 2,00,00 65/365  12.5/100
Bill due on 12th June i.e. i.e. 73 days from 31/3/2000
= 13,750
i.e. 5,50,000  73/365  12.5/100
Bill due on 30th June i.e. i.e. 91 days from 31/3/2000
= 9,973
i.e. 3,20,000  91/365  12.5/100

Bill due on 9th July i.e. i.e. 100 days from 31/3/2000
=` 18,493
i.e. 4,50,000  100/365  15/100
46,668

Discount to be credited to Profit and Loss Account will be:

© The Institute of Chartered Accountants of Nepal 477


CAP II Paper 1: Advanced Accounting

Rs.
Opening Balance 31/3/1999 40,000
Add: Received 1,35,000
1,75,000
Less rebate on 31/03/2000 46,668
1,28,332

Journal Entries
Dr. (Rs.) Cr. (Rs.)

(i) Rebate on bills discounted 40,000


To Discount on Bills 40,000
(Balance on former a/c as on 31/3/99 adjusted)

(ii) Discount on Bills 46,668


To Rebate on bills discounted 46,668
(Rebate as on 31/3/2000 as per working
adjusted)

(iii) Discount on bills a/c 1,28,332


To Profit & Loss a/c 1,28,332
(Balance on former a/c/ transferred)

18. Following particulars are extracted from the books of Big Bank Ltd. As the Head of
Finance Department of the bank, you are required to report to the CEO the level of Capital
Adequacy Ratios (Tier I and Total Capital Ratio) for the third quarter ending on Chaitra
2075.
Particulars Amount (Rs. 000’)
Paid up Equity Share Capital 2,409,000
Gross income 6,067,000
Net Interest Income 4,550,000
Securities Premium 11,000
Exchange Equalization Reserve 37,000
Statutory General Reserve 1,324,000
Investment in equity arising out of underwriting commitment 58,000
Subordinated term debt 1,050,000
Retained Earning up to previous year 255,000
Current Year cumulative profit (Unaudited) 294,000
Discount Value of 5 year Debenture (337,000)
General Loan Loss Provision 421,000
Debenture redemption reserve 480,000

© The Institute of Chartered Accountants of Nepal 478


CAP II Paper 1: Advanced Accounting

Investment in equity of institutions having financial interest 78,000


Cumulative Redeemable preference shares 256,000
Irredeemable non-cumulative Preference Shares 263,000
Fictitious Assets 16,000
Other reserve 31,000
Risk Weighted Exposures:
For Credit Risk 54,416,000
For Operational Risk 3,860,000
For Market Risk 368,000
Supervisory Response:
During onsite inspection of the bank, Nepal Rastra Bank was not satisfied with the quality of
risk management and control procedures and instructed the bank management to adjust the risk
weighted exposures as follows:
(a) Asset liability management practices to effectively manage market risk of the bank is not
satisfactory and an additional risk weight of 1% of Net Interest Income shall be added to
the risk weight for market risk.
(b) The bank has not adopted sound practice of operational risk management, therefore, an
additional capital charge of 2% of Gross Income shall be charged for operational risks.
(c) Bank has not made desired level of disclosure requirements; the total risk weighted
exposures of the bank shall be increased up to 3%.
(CAP Jun. 2019 Q2b – 10 Marks)
Answer:
As per the Capital Adequacy Framework issued by Nepal Rastra Bank, the capital adequacy
ratios of the Big Bank Ltd. can be calculated as follows:

Big Bank Ltd.


Capital Adequacy Table
For the 3rd Quarter ending on Chaitra 2075
Amount
Particulars (Rs. in 000’)
Core Capital (Tier I)
Paid up Equity Share Capital 2,409,000
Irredeemable Non-cumulative Preference Shares 263,000
Securities Premium 11,000
Statutory General Reserve 1,324,000
Retained Earning up to previous year 255,000
Current Year profit 294,000
Debenture Redemption Reserve 480,000
Less:
Investment in equity of institutions having financial interest (78,000)
Fictitious Assets (16,000)
Investment in equity arising out of underwriting commitment (58,000)
Total Core Capital 4,884,000
Supplementary Capital (Tier II)

© The Institute of Chartered Accountants of Nepal 479


CAP II Paper 1: Advanced Accounting

Cumulative and redeemable Preference Share 256,000


Subordinated Term Debt 1,050,000
Less: 20% discount value to 5 year Debenture (337,000)
General Loan Loss Provision 421,000
Exchange Equalization Reserve 37,000
Other Reserve 31,000
Total Supplementary Capital 1,458,000
Total Capital Fund (Tier I + Tier II) 6,342,000
Risk Weighted Exposure
Risk Weighted Exposure for Credit Risk 54,416,000
Risk Weighted Exposure for Operational Risk 3,860,000
Risk Weighted Exposure for Market Risk 368,000
Risk weighted Exposure before Supervisory Adjustment 58,644,000
Add: 3% of the total RWE due to non-compliance to disclosure 1,759,320
2% Capital Charge for operational risk 121,340
1% Capital Charge for NII 45,500
Total Risk Weighted Exposure (After Supervisory Adj. of Pillar II) 60,570,160
Core Capital to Total Risk Weighted Exposure 8.06%
Total Capital to Risk Weighted Exposure 10.47%

© The Institute of Chartered Accountants of Nepal 480


CAP II Paper 1: Advanced Accounting

5.2 Cooperatives

A. Theoretical Questions

1. Write short notes on: PEARLS System of Monitoring the Cooperative.


(June 2018, No. 6b-3 Marks)
Answer:

PEARLS is defined as a set of financial ratios to monitor the financial stability of the credit
unions within World Council of Credit Union's developing movement projects. These ratios
provide credit unions, project staff, national federations and regulators with essential tools
for monitoring, planning, standardizing, ranking and facilitating supervisory control in
credit unions. Each letter in the word PEARLS measures the key areas of credit union
operations: Protection, Effective financial structure, Asset quality, Rate of return and Costs
and Liquidity and Sign of growth. PEARLS system is designed as a management tool that
goes beyond the simple identification of problems. It helps managers find meaningful
solutions to serious institutional deficiencies. Use of the system permits managers quickly
and accurately pinpoint troubled areas, and to make the necessary adjustments before
problems become serious. In essence, PEARLS is an "early warning system" that generates
invaluable management information.

© The Institute of Chartered Accountants of Nepal 481


CAP II Paper 1: Advanced Accounting

5.3 Insurance Companies

A. Theory Questions

1. State how the Premium Income, Claim Expense and Commission Expense are computed in
case of insurance company.
(CAP Dec. 2010 Q4a-5 Marks; Inter Jun. 2012 Q6a – 5 Marks; CAP Dec. 2015 Q3d)

Answer
Premium Income:
Premium income should be recognized on cash basis. If the date of premium payment is later
than cash received, the income should be recognized on later date.
Premium on re-insurance accepted may be recognized on accrual basis.
If premium amount is received for more than one year, the amount relating to first year should
be recognized as income and the remaining amount should be treated as deposit. The amount
of deposit should be proportionately treated as income in later years. But, in case of one time
premium, the total amount should be recognized as income. The amount of premium income
to be credited to revenue account for a year may be computed as:

Premium Received on risks undertaken during the year


Direct Insurance
Yearly premium xxxxx
Renewal premium xxxxx
One-time premium xxxxx xxxxx
Re-insurance accepted xxxxx
xxxxx
Less: Premium on re-insurance ceded xxxxx
Premium Income xxxxx

Note:
The above computational and revenue recognition procedure is same for both cases, i. e. for
life insurance business and for non-life insurance business.

Claim Expense:
A claim occurs when a policy falls due for payment. In the case of life insurance business, it
will arise either on death or maturity of policy that is on the expiry of the specified term of
years. In the case of general insurance business, a claim arises only when the loss occurs or the
liability arises. The amount of claim to be charged to revenue account may be worked out as
under:

Life Insurance
Claims settled during the year (including legal fees, survey, consultancy charges etc.)
Death claim xxxxx

© The Institute of Chartered Accountants of Nepal 482


CAP II Paper 1: Advanced Accounting

Maturity claim xxxxx


Partial maturity claim xxxxx
Surrender claim xxxxx
Other claim xxxxx
Total claim paid xxxxx
Adjustment : Share of re-insurance in total claim xxxxx
Net claim expenses xxxxx

Non-life insurance
Claims settled during the year (including legal fees, survey, consultancy charges etc.)
Claim of current year xxxxx
Claim prior to one f.y. xxxxx
Claim prior to two f.y. xxxxx
Claim prior to three f.y. xxxxx
Claim prior to four or more f.y. xxxxx
Total claim paid xxxxx
Adjustment : Share of re-insurance in total claim xxxxx
Net claim expenses xxxxx

Commission Expenses: In insurance business received through agents, the agency


commission expenses should be booked in the year in which the related premium income is
recognized. Commission expense to be charged to revenue account is computed as follows:
Commission on first year premium xxxxx
Commission on Renewal premium xxxxx
Commission on One-time premium xxxxx
Commission expenses xxxxx

2. Re-Insurance
(CAP Dec. 2011 6c-2.5 Marks; Inter Jun. 2008 Q6 i-4 Marks; Inter Jun. 2011 Q6b – 5
Marks; Inter Jun. 2009 Q6d - 5 Marks)

Answer
If an insurer does not wish to bear the whole risk of policy written by him, he may reinsure a
part of the risk with some other insurer. In such a case the insurer is said to have ceded a part
of his business to other insurer. The reinsurance transaction may thus be defined as an
agreement between a ‘ceding company’ and ‘reinsurer’ whereby the former agreed to ‘cede’
and the latter agrees to accept a certain specified share of risk or liability upon terms as set out
in the agreement. A ‘ceding company’ is the original insurance company which has accepted
the risk and has agreed to ‘cede’ or pass on that risk to another insurance company or a
reinsurance company. It may however be emphasized that the original insured does not acquire
any right under a reinsurance contract against the reinsurer. In the event of loss, therefore, the
insured’s claim for full amount is against the original insurer. The original insurer has to claim
the proportionate amount from the reinsurer. There are two types of reinsurance contracts,
namely, facultative reinsurance and treaty reinsurance. Under facultative reinsurance each

© The Institute of Chartered Accountants of Nepal 483


CAP II Paper 1: Advanced Accounting

transaction has to be negotiated individually and each party to the transaction has a free choice,
i.e., for the ceding company to offer and the reinsurer to accept. Under treaty reinsurance a
treaty agreement is entered into between ceding company and the reinsurer whereby the volume
of the reinsurance transactions remain within the limits of the treaty.

3. Write short note about unexpired risk reserve in Insurance Company


(CAP Dec. 2014 3d-3 Marks; CAP Dec. 2018 Q6a-3 Marks; CAP Jun. 2015 Q3d- 2 Marks;
Inter Dec. 2005-5 Marks, Q 6c-5 Marks; Inter Dec. 2007 Q 6a-5 Marks)

Answer
As per Rule 15 of the Insurance Regulation 2049, every insurance Business operating, Non-
Life Insurance Business shall transfer an amount not less than fifty percent of the Net Insurance
Premium shown in Revenue Account to the ‘Unexpired Risk Reserve account. Such amount
shall be allocated for every category of Insurance the Insures operating e.g. an insures operating
non-life business and accepting risk for Fire Insurance, Marine Insurance Motor insurance and
aviation insurance, then the insurer shall maintain the Unexpired Risk Reserve for each of the
fire, marine, motor and aviation insurance.
Such Unexpired Risk reserve shall be recognized as income in the next year except the
Unexpired Risk reserve maintained for Marine Insurance. In case of marine insurance,
unexpired risk reserve maintained for it shall not be recognized as income for at least three
year.

4. Ceding Company (Inter Jun. 2008 Q6 e -4 Marks)

Answer
Ceding Company is the original insurance company which had accepted the risk and has agreed
to cede or pass on that risk to another insurance company or a reinsurance company. It may
however be emphasized that the original insured does not acquire any right under a reinsurance
contract. In the event of loss, therefore, the insured's claim for full amount is against the original
insurer.

5. Floating Policy (CAP Jun. 2010 6c – 2.5 Marks)

Answer
It is some sense which resembles blanket policy of insurance. It is a policy which covers several
types of goods for one lump sum premium. The goods may be lying at different locations and
may be exposed to different level of risk.
The premium, normally charged under this policy, should nearly resemble the sum of premium
charged under specific policy.
The benefit of this policy is that insured need not take many specific policies for goods lying
in different locations and for different types thereon.

6. Reserve for Outstanding Insurance Claim Payables (CAP Jun. 2016 Q6b – 3 Marks)

© The Institute of Chartered Accountants of Nepal 484


CAP II Paper 1: Advanced Accounting

Answer
As Per Rule15 of the Insurance Regulation 2049, every insurer shall provide an amount of one
hundred fifteen percent (115%) of the remaining amounts of the payment against the claim
made by the Insurer before the expiry of each fiscal year. Such amount shall be recognized as
income in next year.

7. Life Insurance Fund (CAP Jun. 2018 Q6a-3 Marks)

Answer:
As per section 22 of the Insurance Act, 2049, every Insurer shall maintain a reserve fund as
specified by the Insurance Board for the liability relating to its Insurance Business within the
state of Nepal. Further, section 21 of the Act states that an Insurer shall maintain separate fund
for each category of Insurance Business and amount to be received from each Insurance
Business shall be deposited in the concerned fund. The fund maintained for one category of
Insurance Business shall not be utilized to bear the liabilities relating to other category of
Insurance Business.
The following amount shall be transferred to the Life Insurance Fund:
• An amount not less than the total liability as specified by the Actuary on the basis of the
Insurance Policies published by the Insurer of the Life Insurance Business within the state
of Nepal.
• In case of fiscal year where no Actuary Valuation is done, the whole surplus amount of
Revenue Account; and
• Amount specified by Insurance Board for Solvency Margin from time to time.
The amount kept under Life Insurance Fund shall be distributed as per Actuary Valuation
Report following the directives of the Insurance Board on this regard.

8. Explain ‘Valuation Balance Sheet’ as prepared by a life insurance company.


(Inter Jun. 2006 Q 6b – 5 Marks)

Answer:
The nature of the business of life insurance is quite different from the nature of business from
the point of view of determination of profit earned over a particular period. In life insurance
business, we cannot determine the profit/loss by preparing profit and loss account. A life
insurance company cannot be said to have made profit unless it has reserves equal to the net
liability likely to arise in future in respect of policies already issued and still in force. The
process of ascertaining the liability is known as valuation and is done by an actuary. In a life
insurance business, profit is determined by comparing the life assurance fund on a particular
date with the net liability disclosed by valuation on the date. There will be profit if the life
assurance fund exceeds the net liability. The comparison is made in a statement which is called
as “Valuation Balance Sheet”.

9. Co-Insurance (Inter Jun. 2007 Q 5a-5 Marks)

© The Institute of Chartered Accountants of Nepal 485


CAP II Paper 1: Advanced Accounting

In cases of large risks the business is shared between more than one insurer under co-insurance
arrangements at agreed percentages. The leading insurer issues the documents, collects
premium and settles claims. The leading insurer renders statements of Account to the other co-
insurers. Accounting for premium, claims etc. under co-insurance is done in the same manner
as that of the direct business except in respect of the following peculiar features.

Incoming co-insurance
(i) Premium: The co-insurer books the premium based on the statement received from the
leading insurer usually by issuing dummy documents. Entries are made in the Premium
Register from which the Premium Account is credited and the Leading Insurer Company’s
Account debited. In case the statement is not received, the premium is accounted for on
the basis of advices to ensure that all premium in respect of risk assumed in any year is
booked in the same year; share of premium relatable to further extension/endorsements on
policies by the leading insurer are also accounted for on the basis of subsequent advices.
Reference to the relevant communications should be made from the concerned companies
to ensure that premium collected by them and attributable to the company is recorded.

(ii) Claims Paid: Normally, on the basis of claims paid, advices received from the leading
insurer, the Claims Paid Account is debited with a credit to the co-insurer. All such advices
are entered into the Claims Paid Register. It is a practice to treat all claims paid advices
relating to the accounting year received up to 31st January of the subsequent year from
leading insurer as claims paid.

Outgoing co-insurance: The share of the insurer only for both premium and claims has to be
accounted under respective accounts. The share of other co-insurers is credited or debited, as
the case may be, to their personal accounts and not routed through revenue accounts.

© The Institute of Chartered Accountants of Nepal 486


CAP II Paper 1: Advanced Accounting

B. Practical Questions

1. From the following information of Reliable Marine Insurance Ltd. for the year ending
st
31 Ashadh, 2074 find out the (CAP Dec. 2017 Q4a-10 Marks)
i) Net premiums earned
ii) Net claims incurred
Particulars Direct Business (Rs.) Re-insurance (Rs.)
Premium:
Received 88,00,000 7,52,000
Receivable– 01.04.2073 4,39,000 36,000
Receivable– 31.03.2074 3,77,000 32,000
Paid 6,09,000
Payable– 01.04.2073 27,000
Payable– 31.03.2074 18,000
Claims:
Paid 69,00,000 5,54,000
Payable– 01.04.2073 89,000 15,000
Payable– 31.03.2074 95,000 12,000
Received 2,01,000
Receivable– 01.04.2073 40,000
Receivable– 31.03.2074 38,000

Answer
Net Premium earned
Particulars Rs. Rs.
Premium from direct business received 88,00,000

Add: Premium on re-insurance accepted 7,52,000


Add :Receivable as on 31.03.2074 32,000
Less: Receivable as on 01.04.2073 (36,000) 7,48,000
95,48,000
Less: Premium on re-insurance ceded 6,09,000
Add: Payable ason31.03.2074 18,000
Less :Payable as on 01.04.2073 (27,000) (6,00,000)
Net Premium earned 89,48,000

Net Claims incurred


Rs.
Claims paid on direct business 69,00,000
Add: Re-insurance 5,54,000
Add: Outstanding as on 31.3.2073 12,000
Less: Outstanding as on 1.4.2074 (15,000) 5,51,000
74,51,000
Less: Claims received from re-insurance 2,01,000

© The Institute of Chartered Accountants of Nepal 487


CAP II Paper 1: Advanced Accounting

Add: Outstanding as on 31.3.2074 38,000


Less: Outstanding as on 1.4.2073 (40,000) (1,99,000)
72,52,000
Add: Outstanding direct claims at the end of the year 95,000
73,47,000
Less :Outstanding claims at the beginning of the year (89,000)
Net claims incurred 72,58,000

2. You are given the following balances of Nepal Insurance Co. Ltd. as of 31st December 2007:
(Rs in ’000)
Capital 32,000
Balances of funds as of 1.1.07
Fire Insurance 80,000
Marine Insurance 95,000
Miscellaneous Insurance 21,865
Unclaimed Dividends 850
Amount due to other Insurance Companies 3,450
Sundry Creditors 7,250
Deposit and Suspense Account (Cr.) 2,280
Profit and Loss Account as on 31.12.06 8,040
Agents Balances (Dr.) 13,500
Interest accrued but not due (Dr.) 2,250
Amount due from other Insurance Companies 6,450
Cash on Hand 350
Balance at Bank 7,480
Furniture & Fixtures WDV (Original cost 10000) as on 31.12.06 5,800
Stationery Stock 140
Expenses of Management
Fire Insurance 28,000
Marine Insurance 16,000
Misc. Insurance 4,000
Others 3,000
Foreign Taxes – Marine 800
Outstanding Premium 8,200
Donations to Flood Relief Funds (IT deductible) 1,000
Transfer fees 100
VAT refund received during the year 3,000
Reserve for Bad Debts 1,170
Income Tax paid 12,000
Mortgage Loan (Dr.) 97,500
Sundry Debtors 2,500
Government securities (102000) 105,700
Debentures 46,550

© The Institute of Chartered Accountants of Nepal 488


CAP II Paper 1: Advanced Accounting

Equity Shares of Joint Stock Companies 22,500


Claims less Re-insurance
Fire 45,000
Marine 35,890
Misc. 6,800
Premium less Re-insurance
Fire 176,250
Marine 102,250
Misc. 26,225
Interest and dividends received on Investments 5,850
Tax Deducted at Source 1,170
Commission
Fire 50,000
Marine 35,000
Misc. 8,000

You are required to make the following provisions:


Depreciation on Furniture – 10% of Original Cost
Depreciation on Investments of Joint Stock Companies shares 1,000
Transfer to General Reserve 1,000
Outstanding claims as of 31.12.07
Fire 20,000
Marine 5,000
Misc. 3,250

Provision for tax @ 50%. Proposed dividends @ 20%. Provision for unexpired risks is to
be made as follows:

On Marine Policies 100% Premium less reinsurance


On Other Policies 50% Premium less reinsurance

You are required to prepare the revenue and profit and loss account for year ended 31st
December 2007 and balance sheet as of that date of the company.
(Inter Dec. 2008 Q1-20 Marks)
Answer

Nepal Insurance Co. Ltd.


Revenue Account for the year ending 31 Dec 2007
Rs. in '000' Rs. in '000'
Particulars Fire Marine Misc. Particulars Fire Marine Misc.
To Claims less reinsurance 45,000 35,890 6,800 By Balance of funds 80,000 95,000 21,865
Add Outstanding claim as By Premium less
on 31.12.07 20,000 5,000 3,250 reinsurance 176,250 102,250 26,225
To Commission 50,000 35,000 8,000

© The Institute of Chartered Accountants of Nepal 489


CAP II Paper 1: Advanced Accounting

To Expenses of
Management 28,000 16,000 4,000
To Foreign Taxes 800
To Balance of funds as on
31.12.07 (WN.1) 88,125 102,250 13,113
To Profit transferred to P&L
A/C 25,125 2,310 12,927
Total 256,250 197,250 48,090 Total 256,250 197,250 48,090

Profit and Loss Account


for the year ended 31st Dec 2007 Rs. in '000'

Particulars Amount Amount Particulars Amount Amount


To Expenses of Management 3,000 By Revenue A/C
To Donations 1,000 Fire 25,125
To Depreciation Marine 2,310
Furniture (100010%) 1,000 Misc. 12,927 40,362
Investments 1,000 2,000 By Interest & Dividend 5,850
To Provision for Income Tax
(WN-2) 20,156 By Transfer Fees 100
To Balance carried to P&L
Appropriation A/C 20,156
Total 46,312 Total 46,312

Profit & Loss Appropriation A/C Rs. in '000'


Particulars Amount Particulars Amount
To Transfer to General
Reserve 1,000 By Balance b/d 8,040
To Proposed Dividend
(3200020%) 6,400 By VAT refund 3,000
To Balance carried to Balance Sheet 23,796 By Transfer from P&L A/C 20,156
Total 31,196 Total 31,196

Balance Sheet as of 31st Dec 2007


Liabilities Amount Amount Assets Amount Amount
Shareholder's Funds Fixed Assets:
Share Capital 32,000 Furniture 10,000
Less: Depreciation up to
Reserves 31.3.06 4,200
General Reserve 1,000 Less: Depreciation for the year 1,000 4,800
Investment Reserve 1,000 97,500

© The Institute of Chartered Accountants of Nepal 490


CAP II Paper 1: Advanced Accounting

Mortgage Loan

Reserve for Bad Debt 1,170 Investment


Profit & Loss A/c 23,796 26,966 Govt. Securities 105,700

Balance of Funds Debentures 46,550


Fire 88,125 Equity Shares 22,500 174,750

Current Assets
Marine 102,250 Agent's Balances 13,500
Misc. 13,113 203,488 Outstanding Premiums 8,200
Current Liabilities: Interest accrued but not due 2,250
Due from other insurance
Claims outstanding 28,250 companies 6,450
Amount due to other
Insurance companies 3,450 Sundry Debtors 2,500
Sundry Creditors 7,250 Cash in hand 350
Deposit & Suspense A/C 2,280 Cash at Bank 7,480
Unclaimed Dividend 850 Stationary Stock 140
Provision for Taxation
(WN. 3) 6,986 40,870
Proposed Dividend 6,400 55,466
Total 317,920 Total 317,920

Working Notes:
1. Calculation of Balance of fund as on 31.12.07
Rs in '000'
Particulars Fire Marine Misc.
Premium less Re-issuance 176,250 102,250 26,225
Provision rate for unexpired risk
(as a percentage of Premium less reinsurance) 50% 100% 50%

Reserve for unexpired risk 88,125 102,250 13,113

2. Calculation of Provision for Income Tax


Rs. in '000'
Net profit of the year before tax 40,312
(as per Profit and loss A/C)
Income Tax rate 50%
Provision for income tax/ 20,156

3. Computation of balance of Provision for taxation

© The Institute of Chartered Accountants of Nepal 491


CAP II Paper 1: Advanced Accounting

Particulars Amount Particulars Amt


To Bank 12,000 By Profit & Loss A/C 20,156
To Tax deducted at source 1,170
To Balance carried forward 6,986
Total 20,156 Total 20,156

3. Nepal Insurance Co. Ltd. received Rs. 5,90,000 as premium on new policies and Rs. 1,20,000
as renewal premium. The company received Rs. 90,000 towards reinsurance accepted and paid
Rs. 70,000 towards reinsurance ceded. How much will be credited to Revenue Account
towards premium? (CAP Jun. 2012 3a (i) - 2Marks)

Answer
Computation of premium to be credited to 'Revenue Account'
Rs.
Premium received in respect of new policies 590,000
Add: Renewal premium received 120,000
Add: Premium received on reinsurance accepted 90,000
Total premium received 800,000
Less: Premium paid on reinsurance ceded 70,000
Premium to be credited to Revenue Account 730,000

4. From the following information relating to M/s Genuine General Insurance find out the
revenue profit for the period ended 31.12.2074.
Amount in Lakhs
Gross premium (unrealized Rs. 120) 8,000
Claim paid (including cheque issued but not collected Rs. 95) 1,680
Other administrative expenses 1,455
Unexpired risk reserve (opening) 2,450
Unexpired risk reserve (closing) 2,760
Outstanding claim liability (opening) 1,440
Outstanding claim liability (closing) 1,230
As per company's strategy 35% business is covered under reinsurance treaty, which provides
average 6.5% commission
(CAP Jun. 2018 Q4b-5 Marks)
Answer
Statement of calculation of Revenue Profit
For the period ended 31.12.2074

Gross premium (less unrealized Rs. 120) 7,880.00


Less: reinsurance portion (35%) (2,758.00)
Reinsurance commission (6.5% of Rs. 2,758) 179.27
Claim paid (1,680.00)
Other administrative expenses (1,455.00)

© The Institute of Chartered Accountants of Nepal 492


CAP II Paper 1: Advanced Accounting

Unexpired risk reserve (opening) 2,450.00


Unexpired risk reserve (closing) (2,760.00)
Outstanding claim liability (opening) 1,440.00
Outstanding claim liability (closing) (1,230.00)
Revenue Profit for the period 2,066.27

Alternate Solution

Statement of calculation of Revenue Profit


For the period ended 31.12.2074
in Lakhs
Gross premium (less unrealized Rs. 120) 7,880.00
Less: reinsurance portion (35%) (2,758.00)
Reinsurance commission (6.5% of Rs. 2,758) 179.27
Claim paid (1,680.00)
Other administrative expenses (1,455.00)
Unexpired risk reserve (opening) 2,450.00
Unexpired risk reserve (closing) (2,760.00)
Outstanding claim liability (opening) 115% 1,656.00
Outstanding claim liability (closing) 115% (1,414.50)
Revenue Profit for the period 2,097.77

5. The Revenue Account of Kathmandu Life Insurance Company Limited shows the Life
Assurance fund on 31st December, 2002 at Rs. 62,21,310 before taking into account the
following items: (i) Claims covered under reinsurance Rs. 12,000; (ii) Bonus utilized in
reduction of Life Insurance Premiums Rs. 4,500; (iii) Interest accrued on securities Rs. 8,260;
(iv) Outstanding Premium Rs. 5,410; (v) Claims intimated but not admitted Rs. 26,500.

What is the Life Assurance fund after taking into account the above items?
(Inter Jun. 2003 Q 4 -8 Marks)

Answer
Kathmandu Life Insurance Co. Ltd.
Calculation of Life Assurance Fund

Rs. Rs.
Balance of Life Fund 31st December, 2002 62,21,310
Add: Bonus utilized in reduction of Premium 4,500
Add: Interest accrued on securities 8,260
Add: Premium outstanding 5,410
Less: Claim outstanding 26,500 18,170
Less: Cover under reinsurance 12,000 14,500 62,39,480
Less: Bonus in reduction of premium 4,500

© The Institute of Chartered Accountants of Nepal 493


CAP II Paper 1: Advanced Accounting

19,000
Balance of correct Life Assurance Fund 62,20,480

6. Excellent Life Assurance Company Limited had a paid up capital of Rs. 100 crores divided
into 10 lakhs equity shares of Rs. 100 each. Its net liability on all contracts in force as on
31.3.2001 was Rs. 21 lakhs. From the following figures extracted from its books for the year
ended 31.3.2001. you are required to prepare the Revenue Account for the year ended
31.3.2001 and a valuation Balance Sheet as on that date:

Rs. in lakhs

Life Assurance Fund on 1.4.2000 25


Premiums received during the year ended 31.3.2001 14
Interest, dividends and rents 8
Bonus in cash 2
Fines and fees 0.5
Management expenses 2
Bonus in reduction of premium 0.25
Commission 1.20
Surrenders 1.00
Claims by death 14
Claims by maturity 5
Consideration for Annuities granted 0.5
Annuities paid 1

The company has paid interim Bonus of Rs. 0.10 lakh during the valuation period.
Ascertain the Surplus and prepare a Statement to allocate such surplus as detailed below:
25% of such surplus to be allocated to Equity Shareholders, 70% to policyholders and
the balance to be carried forward.
(Inter Dec. 2001 Q4-5 Marks)
Answer:

Excellent Life Assurance Company Limited


Revenue Account for the year ended 31.3.2001

Rs. in Rs. In
Lakh Lakh
To Claims Paid: By Assurance fund at the beginning 25.00
By Death 14 By Premiums 14,00
By Maturity 5_ 19.00 By Consideration for Annuities granted 0.50
To Annuities 1.00 By Interest, Dividends and Rents 8.00
To Surrenders 1.00 By Fines and Fees 0.50
To Bonus in cash 2.00
To Bonus in reduction of premium 0.25
To Management expenses 2.00
To Commission 1.20
To Life assurance fund at the end 21.55
48.00 48.00

© The Institute of Chartered Accountants of Nepal 494


CAP II Paper 1: Advanced Accounting

7. From the following information as on 32nd Ashadh, 2075 of Xeta General Insurance Co. Ltd.,
prepare the Revenue Account, reserving 50% of the net premiums for unexpired risk and an
additional reserve of Rs. 700,000.
Particular Amount (Rs.)
Reserve for unexpired risk on 31st Ashadh, 2074 1,500,000
Additional reserve on 31st Ashadh, 2074 300,000
Claims paid 1,920,000
Estimated liability in respect of outstanding claims on 31st
Ashadh, 2074 195,000
Estimated liability in respect of outstanding claims on 32nd
Ashadh, 2075 270,000
Expenses of management (including Rs. 90,000 incurred in
connection with claims) 840,000
Re-Insurance premium paid 225,000
Re- Insurance recoveries 60,000
Premiums 3,360,000
Interest and dividend (gross before TDS) 150,000
Profit of sale of investments 30,000
Commission 50,000
(CAP Jun. 2019 Q3b – 5 Marks)
Answer:
Xeta General Insurance Company Limited
Revenue Account for the year ended 32nd Ashadh, 2075
Particulars Schedule Amount Rs.
Premium earned (Net) 1 26,67,500
Profit on sale of investment 30,000
Others -
Interest and dividend (gross) 1,50,000
Total (A) 28,47,500
Claims incurred (Net) 2 20,25,000
Commission 3 50,000
Operating expenses related to insurance 4 7,50,000
Total (B) 28,25,000
Operating profit from insurance business (A)-(B) 22,500

Schedule-1 Premium earned (net)


Rs
Premium received 33,60,000
Less: Premium on reinsurance ceded (2,25,000)
Net premium 31,35,000
Less: Adjustment for change in Reserve for Unexpired risk (as per W.N.) (4,67,500)
Total premium earned 26,67,500

Schedule-2 Claims incurred (Net)


Rs
Claims paid 19,20,000
Add: Expenses regarding claims 90,000

© The Institute of Chartered Accountants of Nepal 495


CAP II Paper 1: Advanced Accounting

20,10,000
Less: Re-insurance recoveries (60,000)
19,50,000
Add: Claims outstanding as on 32nd Ashadh, 2075 2,70,000
22,20,000
Less: Claims outstanding as on 31st Ashadh, 2074 (1,95,000)
20,25,000

Schedule -3 Commission
Rs
Commission paid 50,000

Schedule -4 Operating expenses related to Insurance Business


Rs
Expenses of management (Rs 8,40,000-Rs 90,000) 7,50,000

Working Note:
Calculation for change in Reserve for Unexpired risk:
Rs.
Reserve for Unexpired Risk as on 32nd Ashadh, 2075 15,67,500
Additional Reserve as on 32nd Ashadh, 2075 7,00,000 22,67,500
Less: Reserve for Unexpired Risk as on 31st Ashadh, 2074 1500,000
Additional Reserve as on 31st Ashadh, 2074 3,00,000 (18,00,000
)
4,67,500

© The Institute of Chartered Accountants of Nepal 496


CAP II Paper 1: Advanced Accounting

5.4 Agriculture Accounting

A. Theory Questions

1. Mention a few peculiar features of Farm Accounting. Explain why accounting in agricultural
operations is not popular. (Inter Jun. 2003 Q 8-8 Marks; CAP Dec. 2017 Q6d-3 Marks)
Answer:
The following are the few peculiar features noticed in the case of Farm Accounting:
1. Agricultural farms are tradition based.
2. They are family-oriented.
3. A large chunk of the farm produce is appropriated towards consumption by the family
members.
4. The entire work of farming in developing and under developed countries is subject to the
vagaries of the nature.
5. Another feature is that there are many divisions in farm accounting and the finished
products of one become the raw material of another.
6. Due to the intrinsic nature, there are difficulties in ascertaining the value of standing crops,
poultry, livestock and other agricultural produces.
7. The input in the form of agricultural labor is mostly provided by the family members who
devote their time on other works also.
8. Agriculture in many cases is not a full-time occupation. Many agriculturists have other
occupations also.

The following important reasons can be cited for the unpopularity of accounting of agricultural
operations:
1. Agriculture in our country is largely an unorganized sector.
2. Fragmentation of holdings and domination by small farmers.
3. Farmers think that they cannot afford to bear the charges of accountings services and they
are not aware of the benefits.
4. There are no specific laws requiring account keeping of agricultural operations

2. Records required for the compilation of accounting information for agricultural farm.
(Inter Jun. 2006 Q 7c-3 Marks)
Answer:
Agricultural activities are carried on mostly in an unorganized manner. The farmer has no
office and does not find time for day-by-day record keeping. The transactions and events are
also not supported by vouchers or other documents in most of the cases. So it is desirable to
maintain a Diary to record happenings of the day. This Diary becomes the source document for
record keeping.
Seven registers are required for running the accounting system.
1. Cash Book: to record cash transactions.
2. Fixed Assets Register: to record details of fixed assets – description of assets, cost of
purchases/construction/generation, disposal, depreciation and balance.

© The Institute of Chartered Accountants of Nepal 497


CAP II Paper 1: Advanced Accounting

3. Loan Register: to record borrowings from bank, co-operatives and other agencies, trade
creditors along with interest paid or payable.
4. Stock Register: to record details of input, output and by-products – receipts, utilization,
wastage and balance.
5. Debtors and Creditors Register: to record credit transactions classified by parties involved.
6. Register for Notional Transactions: to record transactions between farm and farm
household.
7. Cost Analysis Register: to record crop wise input and output inclusive of apportionment of
common costs and finding out crop profit.

3. Common costs of agricultural farm and the basis of their apportionment


(Inter Dec. 2010 Q6b-5 Marks)
Answer
The costs which cannot be identified crop-wise are common costs of the agricultural farms.
Costs of irrigation, services of agricultural machinery, depreciation, animal or other power,
interest etc. can be classified as common costs. Common costs should be apportioned among
the crop enterprises on the basis of usage, wherever use of assets can be quantified. In other
cases, length of crop season can be used. Following are the examples:
Cost Elements Bases of
apportionment
Interest on fixed capital Length of crop season
Interest on working capital Working capital
investment
Maintenance of farm shed and depreciation Length of crop season
Maintenance of agricultural machinery Machine hours used
Depreciation of agricultural machinery Machine hours used
Maintenance of draught animal and depreciation Animal base

4. What is the information that are extracted from the well-designed accounting system in
Agricultural Farm? (CAP Jun. 2009 6a- 4 Marks)

Answer
A well designed accounting system can be used for extracting the following information:
i) Crop-wise performance and overall performance of the agricultural enterprise.
ii) Comprehensive information regarding yield, revenue, input and cost of the
enterprise.
iii) Financial state of affairs i.e. assets and liabilities of the farm at a particular point of
time.
iv) Profit/Loss of the farm during a year.
v) Data base for other decisions, namely (a) acquire assets or hire services for
ploughing, irrigation, weeding, threshing etc.; (b) replacement of draught animal,
machinery and farming technique; (c) selection of crop-mix; (d) choosing farm size;
(e) farm diversification, for example adding crop and non-farming activities like

© The Institute of Chartered Accountants of Nepal 498


CAP II Paper 1: Advanced Accounting

processing of agricultural products; (f) divestment decisions whether to discontinue


agricultural operations.
vi) Supporting data to the lenders including banks and co-operative societies to assess
farm’s financial requirements as well as debt servicing ability.
vii) Reliable data for farm management survey.
viii) Reliable data for assessment of agricultural income tax.

5. Compilation of accounting information for agricultural farm


(Inter Jun. 2012 Q6c- 5 Marks)
Answer
Agricultural activities are carried on mostly in an unorganized manner. Generally, the farmer
does not have office and also does not find time for day to day record keeping. The transactions
and events of such agricultural activities are also not supported by vouchers or other documents
in most of the cases. Therefore, it is essential to maintain a Diary to record happenings of the
day. This Diary becomes the source document for record keeping. The following registers are
required for compilation of the accounting information of agricultural activities:
i. Cash Book: to record cash transactions.
ii. Fixed Assets Register: to record details of fixed assets such as description of assets, cost of
purchases/construction/generation, disposal, depreciation and balance.
iii. Loan Register: to record borrowings from bank, cooperatives and other agencies trade creditors
along with interest paid or payable.
iv. Stock Register: to record details of input, output and by-product – receipts, utilization, wastage
and balance.
v. Debtors and Creditors Register: to record credit transactions classified by parties involved.
vi. Register for National Transactions: to record transactions between farm and farm household.
vii. Cost Analysis Register: to record crop-wise input and output inclusive of apportionment of
common costs and finding out crop profit.

6. Challenges of farm accounting in Nepal (CAP Jun. 2017 Q6d-3 Marks)


Answer
There are many challenges of farm accounting in Nepal. Some of the challenges are as follows:
1) Agriculture sector in Nepal is unorganized. It is dominated by small farmers and most of these
small famers hold only few acres of land. The average size of holding of land is very small for
commercial farming.
2) Most of the small farmers are illiterate and very poor. Hence, they are neither aware about the
necessity of accounting nor they can afford the expenses of employing someone to maintain the
account.
3) Even big farmers who own considerable land do not keep any accounting to ascertain the profit
from the operations or to take any economic decisions.

© The Institute of Chartered Accountants of Nepal 499


CAP II Paper 1: Advanced Accounting

CHAPTER 6 PREPARATION of FINANCIAL STATEMENTS


FROM INCOMPLETE RECORDS

A. Theory Questions

1. Self-Balancing Ledger System (Inter Dec. 2010 Q6a-5 Marks)

Answer
A) Meaning: It is the system of maintaining books of accounts used in large Firms
wherein
a) Ledgers are classified into-
(i) General/ Nominal Ledger (for all impersonal accounts,)
(ii) Debtors / Sales Ledger (for all Debtors Accounts) and (iii) Creditors/ Purchase /
Bought Ledger (for all creditors Accounts), and
b) Each part is made to "tally" independently of the other parts.
B) Purpose: In case of larger businesses and voluminous transactions, where accounts are
maintained in a manual system, the task of locating errors in the books of accounts would be
difficult due to a Large number of Ledger Accounts and entries. The Self-Balancing System
has been designed primarily for this purpose, i.e. to make the task of locating errors simpler.

2. Difference between Single Account and Double Account System.


(Inter Dec. 2011 Q5c-5 Marks)
Answer
The following are the main differences between the single account system and double
account system:
1. Under single account system only one balance sheet is presented in its usual form
of a statement of assets and liabilities but under double account system the balance
sheet is prepared and presented in two parts i.e., capital account and general balance
sheet.
2. The main purpose of preparing the balance sheet under single account system is to
show the financial position of the concern on a particular date but in case of double
account system the main purpose is to show the amount of capital raised and how
the same has been spent on the acquisition of fixed assets.
3. Under single account system assets are shown in the balance sheet after deduction
of depreciation from the concerned assets but under double account system assets
are shown at their original value in the capital and depreciation fund created for
charging depreciation is shown on the liabilities side of the general balance sheet.
Profit and Loss Account and Profit and Loss Appropriation Account under single
account system take the form of Revenue Account and Net Revenue Account in
case of double account system.

© The Institute of Chartered Accountants of Nepal 500


CAP II Paper 1: Advanced Accounting

3. Write short notes on Debtors and Creditors Suspense Accounts:


(Inter Jun. 2001 Q. 6c – 4 Marks)

Answer:
Sometimes the buyers of an existing business may also agree to collect its debts and to pay off
the creditors on behalf of the vendors.
The debtors and creditors of the vendors will be included in the accents of the company by
debit or credit to separate Total Accounts in the general ledger and center entries will be made
in corresponding Suspense Accounts. Details of debtors and creditors balances will be kept in
separate ledgers. The following entry would be passed in case of debtors taken over.
i) Sundry Debtors a/c Dr.
To Debtors Suspense a/c
In respect of cash received from debtors and payable to vendors the following
entry will be passed
ii) Debtors Suspense a/c Dr.
To Vendors a/c

B. Practical Questions

1. Mr. E deals in wholesale business of food products in the name of Excel Trading House. He
has employed an assistant to help him in collecting cash from his customers and depositing
them into bank account. He presented the following information of last financial year and
available documents and information relating to the transactions of current financial year:
i) As at the end of last financial year his capital was Rs.2,585,000 and he owed Rs. 620,000
to his suppliers. He had shown Rs.360,000 as value of furniture, Rs.1,470,000 as stock in
trade, Rs.895,000 as receivable from customers, Rs.435,000 as balance with bank and
Rs.45,000 as cash in vault.
ii) From the memoranda books, it is seen that he has made total cash sales of Rs.827,000
during the current financial year but the credit sales are yet to be compiled.
iii) He informs you that all his sales are made with a mark-up of 25 percent on cost.
iv) During this financial year, his assistant had collected cash of Rs.300,000 and cheques
amounting to Rs.1,540,000 from his credit customers, and Mr. E had issued cheques
amounting to Rs.1,705,000 to his suppliers.
v) Bank statement showed a total of Rs.1,000,000 of cash lodged into bank in various date
during this financial year. Similarly, he had drawn Rs.70,000 from bank for his personal
use, had paid Rs.214,000 for sundry expenses and Rs.60000 for rent to landlord.
vi) Mr. E had paid Rs.95,000 in cash for purchases.
vii) His cash in vault was Rs.35,000 at the end of current financial year.
viii) At the end of the current financial year, he owed Rs.640,000 to his suppliers and found
Rs.930,000 as receivable from his customers.
ix) The stock was not taken at the end of the year.
Required:
Prepare Statement of Profit or Loss and Financial Position of Mr. E for this financial year.
(Inter Jun. 2011 Q3a- 10 Marks)

© The Institute of Chartered Accountants of Nepal 501


CAP II Paper 1: Advanced Accounting

Answer
a)
(i) Statement of Profit or Loss of Mr. E for the current financial year
Particulars (Rs.) Particulars (Rs.)
opening stock 1470000 Sales (WN 2) 2702000
purchase (WN3) 1820000 Closing stock 1128400
gross profit (WN4) 540400 (balancing figure)
3830400 3830400
sundry expenses 214000 Gross profit 540400
rent 60000
cash shortage 42000
net profit 224400
540400 540400

Financial Position of Mr. E as at the end of current financial year


Capital & Liabilities (Rs.) (Rs.) Assets (Rs.) (Rs.)
Capital 2585000 Furniture 360000
Net profit 224400 Stock in trade 1128400
Drawings -70000 2739400 Debtors 930000
Creditors 640000 Cash at bank (WN 5) 926000
Cash in hand (WN 5) 35000
3379400 3379400

Working Notes:
(1) Financial Position of Mr. E as at the end of last financial year
Capital & Liabilities (Rs.) Assets (Rs.)
Capital 2585000 Furnitures 360000
Creditors 620000 Stock in trade 1470000
Debtors 895000
Cash at bank 435000
Cash in hand 45000
3205000 3205000

(2) Calculation of Sales for current financial year (Rs.)


Cash Sales 827000
Collection from customers:
in cheques 1540000
in cash 300000
Add closing balance of debtor 930000
Less opening balance of debtor -895000
Credit Sales 1875000
Total Sales 2702000
(3) Calculation of Purchase for current financial year (Rs.)

© The Institute of Chartered Accountants of Nepal 502


CAP II Paper 1: Advanced Accounting

Cash Purchase 95000


Paid to suppliers:
in cheques 1705000
Add closing balance of creditor 640000
Less opening balance of creditor -620000
Credit Purchase 1725000
Total Purchase 1820000
(4) Calculation of Gross Profit (Rs.)
Gross Profit= (2702000*25/125) 540400

(5) Cash Book for the current financial year


Cash Bank Cash Bank
Particulars (Rs.) (Rs.) Particulars (Rs.) (Rs.)
To balance b/f 45000 435000 By purchase 95000
To sales 827000 By creditors 1705000
To debtors 300000 1540000 By drawings 70000
To cash 1000000 By sundry expenses 214000
By rent 60000
By bank 1000000
By cash shortage (assumed
as defalcation by assistant) 42000
By balance c/f 35000 926000
1172000 2975000 1172000 2975000

2. The following is the Financial Position of Manoj Shrestha, a proprietor, as on 1st April, 2011:

Liabilities Rs. Assets Rs


Capital Account 252,500 Machinery 120,000
Sundry Creditors for 45,000 Furniture 20,000
Purchases Stock 33,000
Debtors 100,000
Cash in Hand 8,000
Cash at Bank 16,500
Total 297,500 Total 297,500
st
Riots occurred and fire broke out on the evening of 31 March 2012 destroying the books of
account and furniture. The cashier was grievously hurt and the cash available in the cash box
was stolen.

Manoj Shrestha gives you the following information:


a) Sales are effected as 25% for cash and the balance on credit. His total sales for the year
ended 31st March 2012 were 20% higher than the previous year. All the sales and purchase
of goods were evenly spread throughout the year (as also in the last year).
b) Terms of credit : debtor – 2 months : creditors – 1 month.

© The Institute of Chartered Accountants of Nepal 503


CAP II Paper 1: Advanced Accounting

c) Stock level was maintained at Rs. 33,000 as throughout the year.


d) A steady gross profit rate of 25% on the turnover was maintained throughout. Creditors are
paid by cheque only, except for cash purchase of Rs. 50,000.
e) His private records and the bank pass book disclose the following transactions for the year:
• Miscellaneous business expenses : Rs. 157,500 (including Rs. 5,000
paid by cheque and Rs. 7,500 was outstanding as at the year-end)
• Repairs : 3,500 (paid by Cash )
• Addition to machinery : Rs. 60,000 (paid by cheque)
• Private drawing : Rs. 30,000 (paid by cash)
• Traveling expenses : Rs. 18,000 (paid by cash)
• Introduction of additional capital by depositing into the bank : Rs. 5,000
f) Collection from debtors was all through cheques.
g) Depreciation on machinery is to be provided at 15% on the closing book value.
h) The cash stolen is to be charged to the profit and loss A/c.
i) Loss of furniture is to be adjusted from the capital A/c.

Prepare Statement of Profit or Loss for the year ended 31st March 2012 and Financial Position
as on that date. Make appropriate assumptions wherever necessary.
(Inter Jun. 2012 Q2- 15 Marks)

Answer
Computation of Sale during the year and Debtors at year – end
Particular Rs.
Debtors as at the beginning (relating to Sales of last year) 100,000
Average Credit period for the last year 2 months
Hence, Total Credit Sales in last year (Rs.100000 * 12 months / Credit Period of 2 600,000
months) 200,000
Add : Cash Sales (= 1/3 of Credit Sales i.e. ¼ of the Total Sales) 800,000
Total Sales during last year 160,000
Add : Increase in Sales during the current year at 20%
Total Sales during the current year 960,000
Cash Sales : ¼ * Total Sales of Rs. 960000 240,000
Credit Sales : ¾ * Total Sales of Rs. 960000 720,000
Debtors at the end of current year = 2 months Credit Sales = Rs. 720000 * 120,000
2/12

Computation of Purchases during the year


Particulars Rs.
Sales for the current year (as per WN 1) 960,000
Less :- Gross Profit at 25% of Sales (Rs. 960000 * 25%) 240,000
Cost of Goods Sold 720,000
Since there are no changes in Stock Level, Opening Stock = Closing Stock. So, 720,000
Purchases = COGS

© The Institute of Chartered Accountants of Nepal 504


CAP II Paper 1: Advanced Accounting

Credit Purchases = Total Purchases = Rs. 720000 – Rs. 50000 Cash Purchases (given) 670,000
Creditors at year – end = 1 month’s Credit Purchases = 670000 * 1/12 = 55,833

Sundry Debtors Account (to compute Collections from Debtors)


Particular Rs. Particulars Rs.
To balance b/d 100000 By Bank – colln from Debtors (bal. figure) 700000
To Credit Sales – (as per WN 720000 By balance c/d – (as per WN 1) 120000
1)
Total 820000 Total 820000

Sundry Creditors Account (to find out Payments to Creditors )


Particulars Rs. Particulars Rs.
To Bank – pymt to Creditors (bal. 659167 By balance b/d 45000
figure)
To balance c/d – (as per WN 2) 55833 By Credit Purchases (as per WN 670000
2)
Total 715000 Total 715000

Cash and Bank Account (to find out Cash Lost and Closing Bank Balance)
Particular Cash Bank Particular Cash Bank
To Balance C/d 8000 16500 By Creditors – Payment 50000 659167
made
To Debtors – Collections - 700000 By Miscellaneous 145000 5000
recd Expenses
To Cash Sales 240000 - By Repairs 3500 -
To Additional Capital - 5000 By Addition to Machinery - 60000
To balance c/d (bal. figure) - 2667 By Travelling Expenses 18000 -
(Overdraft Balance) By Private Drawings 30000 -
By Cash lost by theft (Bal. 1500 -
Figure)
Total 248000 724167 Total 248000 724167

Statement of Profit or Loss of Manoj Shrestha for the year ended 31st March
Particulars Rs. Particulars Rs.
Opening Stock 33000 Sales 960000
Purchases 720000 Closing Stock 33000
Gross Profit 240000
Total 993000 Total 993000
Business Expenses 157500 Gross Profit 240000
Repairs 3500
Depreciation 27000
Travelling Expenses 18000

© The Institute of Chartered Accountants of Nepal 505


CAP II Paper 1: Advanced Accounting

Loss by Theft – Cash 1500


Loss by Fire – Furniture 20000
Net Profit 12500
Total 240000 Total 240000

Financial Position of Manoj Shrestha as at 31st March


Liabilities Rs. Rs. Assets Rs. Rs.
Capital 252500 Machinery 180000
Add: Additional 5000 Less :Accumulated 27000 153000
Capital Depreciation
Add: Profit 12,500
Drawings (30000) 240000 Stock in Trade 33000
Bank Overdraft 2667 Sundry Debtors 120000
Sundry Creditors 55833
Outstanding Expenses 7500
Total 306000 Total 306000

3. Z maintains Accounts under Single Entry system and furnishes the following information for
the year ending 31st December, 2000:

1.1.2000 31.12.2000
Rs Rs.
Bank Balance 28,000
Stock 30,000 40,000
Debtors 45,000 33,000
Furniture (After depreciation) 15,000 18,000
Building 1,50,000
Creditors 32,000 36,000

Cost of Goods sold during the year was Rs. 3,60,000, which constituted 75% of the sales for
the year. The rate of Gross Profit is 25% on sales.
All purchases and sales are on credit and amounts received from customers and payments to
suppliers are by cheque.
X realized Rs. 10,000 in cash on the sale of scrap from which he paid Rs. 6,000 as freight on
purchases and the balance was retained for his personal use.

Details of his other transactions with the bank are as under:


Rs.
Receipts
Sales of Private Investments brought in as Capital 150,000
Sale of Office Furniture (Book value Rs. 1,000) 800
Payments

© The Institute of Chartered Accountants of Nepal 506


CAP II Paper 1: Advanced Accounting

10% Govt. Bonds (Face value Rs. 1,40,000 purchased on 150,000


1.7.2000)
Salaries 60,000
Taxes (11 months ending 30.11.2000) 11,000
Printing and Stationery` 7800
Miscellaneous Expenses 12,000
Drawings 26,000

Bad Debts written off during the year were Rs. 7,000. Furniture has been depreciated by 10%
and Building is to be depreciated by 2%.
The shop assistant is entitled to a commission of 10% of net profit after charging his
commission.

Prepare the Statement of Profit or Loss of his business for the year ending 31st December, 2000
and the Financial Position as on that date.
(Inter Jun. 2001 Q 1 – 20 Marks)

Answer:
Financial Position as at 1/1/2000

Liabilities & Equity Rs. Assets Rs.


Capital (Balancing Figure) 2,36,000 Building 1,50,000
Creditors 32,000 Furniture 15,000
Stock 30,000
Debtors 45,000
Bank Balance 28,000
2,68,000 2,68,000

Statement of Profit or Loss


of Z for the year ending 31.12.200

Dr. Rs. Cr. Rs.


Opening Stock 30,000 Sales 4,80,000
Purchases (Balancing Figure) 3,64,000 100
3,60,000  75
Freight on Purchases 6,000 Closing Stock 40,000
Gross Profit (25% of Sales) 1,20,000
Salaries 60,000
Taxes 11,000 Sale of Scrap 10,000

Outstanding 1,000 Interest on Bonds


12,000 (1,40,000  10/100  7,000
6/12)

© The Institute of Chartered Accountants of Nepal 507


CAP II Paper 1: Advanced Accounting

Printing & Stationery 7,800


Misc. Expenses 12,000
Loss on Sale of Furniture 200
Bad Debts 7,000
Depreciation
i) Furniture 2,000
ii) Building 3,000
Commission
(10/100  33,000) 3,000
Net Profit 30,000
1,37,000 1,37,000

Working Notes:
Debtors A/c
Dr. Rs. Cr. Rs.
To Balance B/d 45,000 By Bad Debts 7,000
To Sales 4,80,000 By Bank (Balancing 4,85,000
Figure)
By Balance c/f 33,000
5,25,000 5,25,000

Creditors A/c
Dr. Rs. Cr. Rs.
To Bank (Balancing Figure) 3,60,000 By Balance b/d 32,000
To Balance c/d 36,000 By Purchase 3,64,000
3,96,000 3,96,000

Bank A/c
Dr. Rs. Cr. Rs.
To Balance b/f 28,000 By 10% Govt. Bonds 1,50,000
To Debtors 4,85,000 By Creditors 3,60,000
To Capital (Sales of Investments) 1,50,000 By Furniture 6,000
To Furniture 800 By Salaries 60,000
By Taxes 11,000
By Printing & 7,800
Stationery
By Miscellaneous 12,000
By Drawings 26,000
By Balance c/f 31,000
6,63,800 6,63,800

© The Institute of Chartered Accountants of Nepal 508


CAP II Paper 1: Advanced Accounting

Furniture A/c
Dr. Rs. Cr. Rs.
To Balance b/f 15,000 By Bank 800
To Bank 6,000 By Loss on Sale 200
By Balance c/f 20,000
21,000 21,000

Financial Position as on 31.12.2000


Liabilities Rs. Assets Rs.
Capital 2,36,000 Building 1,50,000
Brought in + 1,50,000 Less: Depreciation 3,000
Net Profit + 30,000 1,47,000
4,16,000 Furniture 18,000
Less: Drawings Investment
(26,000+4,000) 30,000 (FV 1,40,000) 1,50,000
3,86,000 Add Interest Due 7,000
Creditors 36,000 1,57,000
Outstanding Expenses Stock 40,000
Taxes 1000 Debtors 33,000
Commission 4000 4,000 Bank Balance 31,000
4,26,000 4,26,000

4. Dear Mr. A K Sharma,


I am most relieved that you have agreed to assist me by sorting out the financial affairs of my
second-hand business, which, as you know, commenced on 1 January 2005. Such records as I
have kept are, unfortunately, to be found on now rather scruffy scraps of paper stored in a large
cardboard box. Doubtless, you will want to examine these records for yourself, but I thought it
might assist you if I were to summarize my business dealings up to 31st December 2005 as I
recall them.
In December 2004, I was luckily enough to win Rs. 5,000 on the football pools, and this,
together with Rs. 1,000 loaned to me by a friend – 1 agreed, incidentally, to pay him 10% per
year interest – formed the initial capital of Rs. 6,000. I put Rs. 5,500 into the bank immediately,
in a separate business account, I needed a lorry to enable me to collect and deliver the second-
hand goods, a dealer was asking Rs. 1,300 for a second-hand lorry, but I beat him down to Rs.
840 I’ve only paid by cheque Rs. 200 of this so far, but as I will finish paying the full Rs. 840
in three years, it will be mine before it falls to pieces in another five years from now.
I rent some business premises, and, as they are fairly falling down, I only pay Rs. 350 a year.
I’ve paid by cheque this year’s rent and also Rs. 50 in respect of next year.
My first bit of business was to buy a job lot of 2,000 pairs of jeans for Rs. 6,000. I’ve paid a
cheque for Rs. 4,000 so far, and my supplier is pressing me for the balance. To date, I’ve sold
1,500 pairs and received Rs. 5,800 but I reckon I am still owned Rs. 500, most of which I
should be able to collect. I promptly banked the Rs. 5,800 as it was all in cheques.

© The Institute of Chartered Accountants of Nepal 509


CAP II Paper 1: Advanced Accounting

I brought 800 T-shirt for Rs. 1,200 out of my Bank Account. I have sold 700 of these for cash
– Rs. 1,500 in all – but as the remainder have got damaged, I will be lucky if I get Rs. 50 for
them.
I managed to get some pocket calculator cheaply. 50 of them only cost me Rs. 400, but I’m
rather pleased I haven’t paid for them yet, as I think there is something wrong with them. My
supplier has indicated that he will in fact accept Rs. 200 for them, and I intend take up his offer,
as I reckon I can repair them for Re. 1 each and sell them at Rs. 8 a time – a good profit.
I haven’t paid my cash into the bank at all, as the cash I got for the T-shirts and my initial float
enabled me to pay for my petrol Rs. 400 and odd expenses Rs. 250. Also, it enabled me to draw
Rs. 20 per week for myself. As I have done so well, I also took my wife on holiday; it made a
bit of a hole in the Bank Account but it was worth all Rs. 600 of it.
Perhaps, from what I have told you, you can work out what profit I have made, only keep as
small as possible as I don’t want to pay too much tax.
Yours sincerely,
Chandra Raj Dhakal
Required: From the data provided by Mr. Chandra Raj Dhakal prepare a business Statement of
Profit or Loss for the period ended 31st December 2005 and a Financial Position as on that date.
Show clearly all your workings and assumptions as notes to the Accounts.
(Inter Jun. 2006 Q2-15 Marks)
Answer:

Chandra Raj Dhakal


Statement of Profit or Loss
for the year ended 31.12.2005
Dr. Cr.
Particulars Rs. Particulars Rs.
Purchase Sales
6,30
Jeans 6,000 Jeans
0
1,50
T-Shirts 1,200 T-Shirts 7,800
0
Calculators 200 7,400 Closing Stock
1,50
Gross Profit c/d 2,150 Jens
0
T Shirts 50
Calculators 200 1,750
9,550 9,550
provision for doubtful debts 50
interest on loan 100
Dep. on lorry 140
Rent 350
Petrol 400
Sundry Exp. 250

© The Institute of Chartered Accountants of Nepal 510


CAP II Paper 1: Advanced Accounting

Net Profit 860


2,150 2,150

Financial Position of Chandra Raj Dhakal


as on 31.12.2005

Liabilities Rs. Assets Rs.


Capital: Lorry at cost 840
Opening 5,000 Less: Depreciation 140 700
Add: Profit 860 Debtors 500
5,860 Less: Provision for 50 450
bad debts
Less: Drawings 1,640 4,220 Closing stock 1,750
Loan 1,000 Cash at bank 4,900
Interest on loan 100 Cash in hand 310
Creditors for lorry 640 Prepaid rent 50
Creditors for goods 2,200
8,160 8,160

Assumptions:
1. 52 weeks in a year
2. Provision for doubtful debts 10%
3. Depreciation on lorry 840 / 6 = Rs. 140 (2005 + 5 years)

Working Notes:
Cash Account

Particulars Rs. Particulars Rs.


To Capital 5,000 By Bank 5,500
To Loan 1,000 By Petrol 400
To Sales 1,500 By Sundry expenses 250
By Drawings 1,040
By Balance c/d 310
7,500 7,500

Bank Account

Particulars Rs. Particulars Rs.


To Cash 5,500 By Lorry 200
To Debtors 5,800 By Rent 350
By Advance rent 50
By Creditors 4,000
By Purchase 1,200

© The Institute of Chartered Accountants of Nepal 511


CAP II Paper 1: Advanced Accounting

By Drawings 600
By Balance c/d 4,900
11,300 11,300

5. Arvind operates a warehouse selling wall papers direct do the public on strictly cash basis. On
1st April 2005, a serious fire at his premises destroyed or damaged all his stock and most of his
accounting and stock records. He has asked you to calculate the cost of stocks so that he can
make an insurance claim. He has also asked you to establish whether he has trading profitably
in the period form 1st October 2004 to the date of the fire, so that he can decide whether to start
trading again in this line.
You obtain the following information in connection with his trading activities:
i. Arvind obtains supplies of wall papers from two companies only Tee Ltd. and Dee Ltd.
Both the companies supply Arvind with goods at recommended retail price (R.R.P) less
1
33 %. A cash discount of 10% is given on this net price for payment within two weeks,
3
which Arvind always takes.
ii. Both suppliers give an extra bulk rebate based on the value of goods purchased over the
winter months from 1st October to 31st March. The rebate from Tee Ltd. amount to 5%
R.R.P for goods purchased excluding the first Rs. 25,000 in the winter months. Dee Ltd.
gives rebate of 8% on R.R.P for goods purchased excluding the first Rs. 37,500 in those
months.
iii. The bulk rebate for the six months to 31st March, 2005 was received in April 2005 and
amounted to Rs. 4,100 in respect of purchases from Tee Ltd. and Rs. 3,800 in respect of
purchases from Dee Ltd.
iv. Arvind sells all goods at a price, which gives a gross profit equal to 25% of the cost of
goods, before deducting either the cash discount or bulk rebate.
v. General expenses paid out of cash sales prior to banking are estimated at following
monthly amounts:
Wage & salaries 1,470
Motor expenses 218
Sundry expenses 105
Drawing by Arvind 150
vi. Information obtained from paid cheques and bank statements showed bank deposits from
sales of Rs. 1,62,362, general overheads of Rs. 27,452 and two quarterly rent payments
of Rs. 1,500 each.
vii. Fixtures which cost Rs. 8,000 and vehicles which cost Rs. 7,600 are to be depreciated at
the rate of 5% and 25% per annum respectively.
viii. In January 2005 Arvind used wall paper, which cost before deducting either the bulk
rebate or cash discount Rs. 180 in decorating his own house.
ix. The stock held by Arvind on 30th September 2004 has a cost before deduction of any
rebates or discounts of Rs. 56,807.

You are required to:

© The Institute of Chartered Accountants of Nepal 512


CAP II Paper 1: Advanced Accounting

Calculate the cost before deduction of any rebate or discounts of Arvind’s stock on 31.3.2005.
Prepare a Statement of Profit or Loss for the six months to 31.3.2005.
(Inter Jun. 2006 Q 4-15 Marks)
Answer:

Arvind
Statement of Profit or Loss
For the period ended 31.03.2005

Dr. Cr.
Amount Amount
Particulars Particulars
(Rs.) (Rs)
Opening Stock 56,807 Sales (W.N. 1) 1,74,020
Purchases 1,28,000 Goods taken by proprietor 180
Gross Profit 34,804 Closing Stock
(20% on sales) (balancing figure) 45,411
Wage & Salaries (1470 × 6) 8,820
Motor Expenses (218 × 6) 1,308 Discount received
(1,28,000 × 10%) 12,800
Sundry Expenses (105 × 6) 630 Bulk rebate
Tee Ltd. 4,10
Dee Ltd. 0 7,900
3,80
0
General Overhead 27,452
Rent (1,500 × 2) 3,000
Depreciation:
Fixture 600
Vehicles 950 1,550
Net Profit 12,744

Working Notes:
1. Computation of Purchases

Particulars Tee Ltd. Dee Ltd.


Rs. Rs.
Bulk Rebate 4,100 3,800
Rebate Rate (On MRP) 5% 8%
Bulk Purchases 82,000 47,500
Basic Purchase 25,000 37,500
Total Purchase 1,07,000 85,000

© The Institute of Chartered Accountants of Nepal 513


CAP II Paper 1: Advanced Accounting

2. Computation of Cost of Purchases


Total Purchases at R.R.P. 1,92,000
Less: Trade Discount (1/3) 64,000
Cost of purchases 1,28,000

3. Computation of Cash Sales


Amount deposited in the bank 1,62,362
Wages & Salaries 8,820
Motor Car expenses 1,308
Sundry expenses 630
Drawings 900
1,74,020

6. The following is the Financial Position of the retail business of Mr. Pradeep as at 31st Dec,
2009:
Equity and Liabilities Rs. Assets Rs.
Pradeep's Capital 250,000 Furniture & Fittings 50,000
Creditors for Goods 60,000 Stock 1,50,000
Outstanding Rent 2,000 Sundry Debtors 40,000
Cash at Bank 70,000
Cash in Hand 2,000
312,000 3,12,000
You are furnished with the following information:
i) Mr. Pradeep always sells his goods at a profit of 25% on sales.
ii) Goods are sold for cash and credit. Credit customers pay by cheque only.
iii) Payments for purchases are always made by cheque.
iv) It is the practice of Mr. Pradeep to send to the bank every week-end the collections of the
week after paying every week salaries of Rs. 500 to the clerk, sundry expenses of Rs. 100
and personal expenses of Rs. 200.
Analysis of the bank pass book for the period ending 31st March, 2010 disclosed the
following:
Particulars Rs.
Payments to Creditors 150,000
Payment of rent 8,000
Amount remitted to the bank including cheques for Rs. 20,000
received from customers to whom goods were sold on credit 270,000
st
The following are the balances on 31 March, 2010:
Particulars Rs.
Stocks 65,000
Creditors for Goods 65,000
Sundry Debtors 60,000
st
On the evening of 31 March, 2010, the cashier absconded with the available cash in the cash
box.

© The Institute of Chartered Accountants of Nepal 514


CAP II Paper 1: Advanced Accounting

You are required to prepare a statement showing the amount of cash defalcated by the cashier
and also a Statement of Profit or Loss for the period ended 31st March, 2010 and a Financial
Position as on that date.
(Inter Dec. 2010 Q3-15 Marks)

Answer
Mr. Pradeep
Statement of Profit or Loss
For the 3 months ending 31st March, 2010
Expenses Rs. Income Rs.
Opening Sck 150,000 Sales:
Purchases 155,000 Cash 280,000
Credit 40,000 320,000
Gross Profit c/d 80,000 Closing Stock 65,000
385,000 3,85,000
Salaries [4] 6,500 Gross Profit b/d 80,000
Rent 6,000
Sundry Expenses [4] 1,300
Net Profit 66,200
80,000 80,000

Mr. Pradeep's Financial Position


As on 31st March, 2010
Capital & Liabilities Rs. Assets Rs.
Mr. Pradeep's Capital 250,000 Furniture & Fittings 50,000
Add: Net Profit 66,200 Stock 65,000
316,200 Sundry Debtors 60,000
Less: Drawings [4] 2,600 313,600 Cash at Bank 182,000
Sundry Creditors 65,000 Cash Recoverable from cashier 21,600
378,600 378,600

Working Notes:
1. Total Debtors Account
Particulars Rs. Particulars Rs.
To Balance b/d 40,000 By Bank 20,000
To Credit Sales – bal. fig. 40,000 By Balance c/d 60,000
80,000 80,000

2. Total Creditors Account


Particulars Rs. Particulars Rs.
To Bank 150,000 By Balance b/d 60,000
To Balance c/d 65,000 By Credit Purchases – bal. fig. 155,000
215,000 215,000

© The Institute of Chartered Accountants of Nepal 515


CAP II Paper 1: Advanced Accounting

3. Bank Account
Particulars Rs. Particulars Rs.
To Balance b/d 70,000 By Sundry Creditors 150,000
To Sundry Debtors 20,000 By Rent 8,000
To Cash Remitted 250,000 By Balance c/d 182,000
340,000 340,000

4. Cash Account
Particulars Rs. Particulars Rs.
To Balance b/d 2,000 By Salaries [Rs. 500 × 13] 6,500
By Sundry Exp. [Rs. 100 ×
To Cash Sales 280,000 1,300
13]
By Drawings [Rs. 200 × 13] 2,600
By Bank – Remitted 250,000
By Balance – Defalcated Cash 21,600
282,000 282,000

5. Cash Sales:
Particulars Rs.
Opening Stock 150,000
Add: Purchases 155,000
305,000
Less: Closing Stock 65,000
Cost of Sales 240,000
Sales [Rs. 2,40,000 × 100/75] 320,000
Less: Credit Sales 40,000
Cash Sales 280,000
52
6. No. of weeks in 3 month period = 12 × 3 = 13 Weeks.

7. Lal Prasad does not maintain proper books of accounts. However, he maintains a record of his
bank transactions and also is able to give the following information:

1.1.2004 31.12.2004
Rs Rs
Debtors 1, 02,500
Creditors 46,000
Stock 50,000 62,500
Bank Balance 50,000
Fixed Assets 7,500 9,000

Details of his bank transactions were as follows:-

© The Institute of Chartered Accountants of Nepal 516


CAP II Paper 1: Advanced Accounting

Rs.
Received from debtors 3,40,000
Additional capital brought in` 5,000
Sale of fixed assets (book value Rs.2, 500) 1,750
Paid to creditors 2,80,000
Expenses paid 49,250
Personal drawings 25,000
Purchase of fixed assets 5,000

No cash transaction took place during the year. Goods are sold at cost plus 25%. Cost of goods
sold was Rs.2, 60,000. Prepare his final accounts for the year 2004.
(Inter Dec. 2005 Q 4-15 Marks)

Answer:
Statement of Profit or Loss
for the year ended 31st December, 2004

Amount Amount
(Rs) (Rs)
Opening stock 50,000 Sale 325,000
Purchase (balancing figure 272,500 (Rs.260,000*125/100) 62,500
Gross Profit c/d Closing Stock
(Rs.260,000 * 25/100) 65,000
387,500
Expenses 49,250
Loss on Sale of fixed Assets 750
depreciation on fixed Assets 1,000
(W.N.1)
Net Profit 14,000
65,000 65,000

Equity & Liabilities Amount Assets Amount


Rs. Rs.
Capital 1,69,000 Non-current assets 9,000
Add: Additional Capital 5,000 Debtors(W.N.3) 87,500
Net Profit 14,000 Stock 62,500
Bank Balance 50,000
1,88,000
Less Drawings 25,000
1,63,000
Creditors 46,000
209,000 209,000

© The Institute of Chartered Accountants of Nepal 517


CAP II Paper 1: Advanced Accounting

Working Notes:

1. Non-current assets Account:


Dr Cr
Rs Rs
To Balance b/d 7,500 By Bank (sale) 1,750
To Bank 5,000 By Loss on sale of assets. 750
By Depreciation (balancing 1,000
figure) 9,000
By Balance c/d

12,500 12,500

2. Bank Account
Dr Cr
Rs. Rs.
To Balance b/d(balancing 62,500 By creditors 2,80,000
figure) 3,40,000 By expenses 49,250
To Debtors 5,000 By drawings 25,000
To Capital 1,750 By fixed assets 5,000
To Sale of Fixed Assets By balance c/d 50,000

4,09,250 4,09,250

3. Debtors Account
Dr Cr
Rs. Rs.
To Balance b/d 1,02,500 By Bank 3,40,000
To Sales 3,25,000 By Balance c/d 87,500
(Rs. 2,60,000*1.25) (Balancing figure)
4,27,500 4,27,500

4. Creditors Account
Dr Cr
Rs. Rs.
To Bank 280,000 By Balance b/d (balancing 53,500
To Balance c/d 46,000 figure) 2,72,500
By Purchases( from
trading account)
3,26,000 3,26,000

© The Institute of Chartered Accountants of Nepal 518


CAP II Paper 1: Advanced Accounting

5. Financial Position as on 1st January, 2004

Liabilities Amount Assets Amount


Rs. Rs.
Creditors (W.N.4) 53,500 Non-current assets 7,500
Capital (balancing figure) 1,69,000 Debtors 1,02,500
Stock 50,000
Bank Balance(W.N.2) 62,500

2,22,500 2,22,500

© The Institute of Chartered Accountants of Nepal 519


CAP II Paper 1: Advanced Accounting

CHAPTER 8 ANALYSIS & INTERPRETATION OF FINANCIAL


STATEMENTS

A. Theory Questions

1. Prediction of insolvency on the basis of ratios (CAP Dec. 2009 6c -2.5 Marks)

Answer
The relevance of the ratios in predicting insolvency can be elaborated with the help of the following
illustrative ratios as below:
Working capital to total assets indicates the liquidity position of the firm. If the ratio is too low it
indicates inability of the firm to carry on its day to day activities. If it is negative, the firm will not have
fund for its day to operations. If such situation continues, the firm may be forced to suspend its
operations and it may result insolvency in the long run.

Similarly, ratio of sales to total assets indicates the utilization of its assets to generate sales which
ultimately generates surplus for the firm. If it is too low, it indicates that the firm is keeping idle assets
which in long run may result to insolvency.

Another example can be given of retained earnings to total assets. Retained earnings are cushion for
firm’s health. So if it is too thin it may indicate that firm has very low leverage and is posed to
insolvency earlier.

2. What is Debt Service Coverage Ratio? (CAP Dec. 2010 3a i-2 Marks; CAP Dec. 2018 Q6d-
3 Marks; Inter Jun. 2009 Q6b- 5 Marks)

Answer
The ratio is a key financial ratio for the lenders.
• Debt servicing means timely payment of principal amount of
instalments plus interest.
• Borrower should be able to service the debt out of the profits. Profit means the profit
available for debt servicing.
• This ratio is calculated as:
Profit available for Debt Servicing
Loan instalments +Interest

• This ratio normally should be 1.33 but a higher coverage is of advantage to the
business as it improves its strength to service the debts promptly

3. Price earnings ratio (CAP Dec. 2010 6b-2 Marks)

Answer
Price earnings ratio indicates sensitivity of market price of a share with its earnings per share.
Generally, low P/E ratio indicates the possibility of share price increase in future as far as the
effect of earning is considered and vice versa. It is calculated as price per share divided by

© The Institute of Chartered Accountants of Nepal 520


CAP II Paper 1: Advanced Accounting

earnings per share. The price per share (numerator) is the market price of a single share of the
stock. The Earnings per share (denominator) is the Net Income of the company for the most
recent 12 month period, divided by number of shares outstanding.

4. Capital Gearing Ratio (Inter Dec. 2008 Q6c-4 Marks)

Answer
Capital gearing ratio gives the proportion of interest-bearing fund to non-interest-bearing fund.
Preference share capital is fixed dividend bearing fund. Likewise, other long term and short term
borrowed funds carry fixed interest. Equity share capital, reserve and surplus do not carry fixed
dividend. The word 'gear' is used to indicate the proportion of fixed interest /dividend bearing fund.
Higher the proportion of fixed interest/dividend bearing fund to non interest/dividend bearing fund,
higher is the gearing and as a consequence commitment to pay fixed interest/dividend out of profit is
higher. The capital gearing ratio is calculated from the following formula;
Capital gearing ratio = (Preference share capital + Debt)/(Equity - Preference share Capital)

5. Importance of Ratios analysis (CAP Jun. 2012 Q6b – 2.5 Marks)

Answer
The following are the important managerial use of ratio analysis:
1. Helpful in assessing operating efficiency of the business.
2. Helpful in measuring financial solvency.
3. Helpful in future forecasting.
4. Helpful in decision making.
5. Helpful as corrective measures.
6. Helpful in comparing inter firm performance.
7. Helpful in cost control.

6. ‘Current Ratio’ vs. ‘Quick Ratio’ (CAP Jun. 2016 6d - 3 Marks)


a) Current Ratio:
The current ratio is a financial ratio that investors and analysts use to examine the liquidity of
a company and its ability to pay short-term liabilities (debt and payables) with its short-term
assets (cash, inventory, receivables).
Current Ratio = Current Assets / Current Liabilities.
b) Quick Ratio:
The quick ratio, on the other hand, is a liquidity indicator that filters the current ratio by
measuring the amount of the most liquid current assets there are to cover current liabilities (you
can think of the “quick” part as meaning assets that can be liquidated fast). The quick ratio,
also called the “acid-test ratio,” is calculated by adding cash & equivalents, marketable
investments and accounts receivables, and dividing that sum by current liabilities.
Quick Ratio = (Current Assets – Inventories) / Current Liabilities.

The main difference between the current ratio and the quick ratio is that the latter offers a more
conservative view of the company’s ability to meets its short-term liabilities with its short-term
assets because it does not include inventory and other current assets that are more difficult to

© The Institute of Chartered Accountants of Nepal 521


CAP II Paper 1: Advanced Accounting

liquidate (i.e., turn into cash). By excluding inventory (and other less liquid assets) the quick
ratio focuses on the company’s more liquid assets.

7. Financial Leverage Multiplier (FLM)


(CAP Jun. 2019 Q6a-3 Marks)
Answer
Financial Leverage Multiplier (FLM) is the connection between return on assets and return on
equity. The FLM is one of the several ways of looking at the relative amounts of debt and
equity the organization is using to finance its assets. An important feature of FLM is the
relationship:
ROA*FLM=ROE
That implies that if ROE is important to investors in an organization, then the relative level of
ROE can be managed by changes in the FLM once ROA results can be anticipated.
FLM= Total Assets/ Equity
Alternatively
FLM= (Debt+Equity)/Equity

B. Practical Questions

1. You are given the following particulars:


Current Ratio 2
Working Capital Rs. 400,000
Capital Block to Current Asset 3:2
Fixed Assets to Turnover 1:3
Sales Cash /Credit 1:2
Creditors Velocity 2 Months
Stock Velocity 2 Months
Debtors Velocity 3 Months
Net Profit 10% of Turnover
Reserve 2.5% of Turnover
Debentures/Shares Capital 1:2
Gross Profit 25%
Required:
Prepare the Financial Position on the basis of above information. (CAP Dec. 2010 3a ii-8 Marks)

Answer
Financial Position
Capital & Liabilities Rs. Assets Rs.
Share Capital 600,000 Non-Current (Fixed) Assets 800,000
Net Profit (P/L) 240,000 Stock 300,000
Reserves 60,000 Debtors 400,000
Debenture 300,000 Other Current Assets 100,000
Sundry Creditors 300,000
Other Current Liabilities 100,000
1,600,000 1,600,000

© The Institute of Chartered Accountants of Nepal 522


CAP II Paper 1: Advanced Accounting

Working Notes:

Current Assets (CA)


1. Current Ratio =
Current Liabilities (CL)

Or, 2 = CA/ CL
Or, CA = 2 CL
Again,
WC = CA- CL
Or, 400,000= 2CL- CL
Or, CL = Rs. 400,000
& CA = 2* Rs.400,000
CA = Rs.800,000

2. Capital Block: Current Assets = 3:2


If Current Assets = 8,00,000
Capital Block = 12,00,000

3. Debenture: Share Capital = 1:2


Debentures = Share Capital/2

4. Capital Block = Share Capital + Profit + Reserve + Debenture


Let Share Capital be X
Let Sales be Y
Net Profit = 10% of Y = 0.1Y
Reserve = 2.5% of Y = 0.025Y
Capital Block = X+0.1Y+0.025Y+0.5X
or, 1.5X+0.125Y=12,00,000-------------------(I)

5. Fixed Assets : Turnover= 1:3


Turnover
or, Fixed Assets =
3

Y
Fixed Assets =
3
We know, Capital funds - fixed assets = working capital and
Capital funds = Capital Block
or, 1.5X+0.125Y- Y/3 = 4,00,000
or, 4.5X -0.625Y= 12,00,000 ------------------(ii)
Solving the equation (i) and (ii) we get
X= Share Capital = 6,00,000
Y= Sales = 24,00,000

6 Y 2,400,000
a. Fixed Assets = .= .= 800,000
3 3

© The Institute of Chartered Accountants of Nepal 523


CAP II Paper 1: Advanced Accounting

1
b. Cash Sales = X 2,400,000 .= 800,000
3
2
c. Credit Sales = X 2,400,000 .= 1,600,000
3
10
d. Net Profit = X 2,400,000 .= 240,000
100
2.5
e. Reserve = X 2,400,000 .= 60,000
100
25
f. Gross Profit = X 2,400,000 .= 600,000
100
Share Capital
g. Debentures = .= 300,000
2
Cost of Goods Sold = 2,400,000- 600,000 = 1,800,000

Average Stock
7. Stock Velocity = x 12 = 2
Cost of goods sold
therefore, Closing Stock = 300,000
In the absence of information here it is assumed that average stock = closing stock

Average Debtors
8. Debtor Velocity = x 12 = 3
Credit Sales
therefore, Closing debtors = 400,000
In the absence of information here it is assumed that average debtors = closing debtors

Other current Assets = 800,000-400,000 -300,000 = 100,000

Average Creditors
9. Creditors Velocity= x 12 = 2
Credit Purchases
therefore, Closing Creditors = 300,000
In the absence of information regarding cash and credit purchase and opening and closing
stock, cost of goods sold is considered.
Other current liabilities = 400,000-300,000 = 100,000

2. A company sold 25% of the goods on cash basis and the balance on credit basis. Debtors are
allowed 2 months credit and their balance as on 31.3.2011 is Rs. 140,000. Assume that the sale
is uniform throughout the year. Calculate the total sales of the company for the year ended
31.3.2011. (CAP Dec. 2011 Q4b-5 Marks)

Answer
Calculation of Total Sales for the year ended 31st March, 2011
Since debtors are allowed two months credit term, debtors as on 31st March (Rs. 140,000)
represents two months (February & March) credit sales. Therefore, monthly credit sales are Rs.
70,000. As per question, cash sales is 25% of total sales and sales being uniform throughout
the year, total sales will be calculated as follows:

© The Institute of Chartered Accountants of Nepal 524


CAP II Paper 1: Advanced Accounting

Rs.
Total Credit Sales for the year [Rs. 70,000 × 12] 840,000
Add: Cash Sales – Being 25% of total sales [Rs. 840,000 × 25 / 75] 280,000
Total Sales 1,120,000

3. From the Following Financial Statement, calculate the given ratios:


Summarized Accumulated reserves
Particulars Amount Rs. (In '000)
Revenue 3,029
Gross Profit 179
Net Profit 45
Summarized Financial Positions
Particulars Amount Rs. (In '000)
Fixed Assets at Book Value 257
Stock 236
Debtors 9
Other Assets 66
Total 568
Share Holders’ Equity 320
Long Term Liabilities 64
Current Liabilities 184
Total 568
Ratios:
i) Net Profit Percentage, ii) Total Asset Turnover, iii) Rate of Return on Gross Assets, iv) Liquidity
Ratio and v) Stock Turnover Ratio (CAP Dec. 2013 3d-3 Marks)

Answer
i) Net Profit Percentage: Net Profit/Sales=45/3029=1.49%
ii) Total Asset Turnover: Sales/Total Assets=3029/568=5.33
iii) Rate of Return on Gross Assets: Net Profit/Total Assets=45/568=7.92%
iv) Liquidity Ratio: Current Assets /Current Liabilities=245/184=1.33:1
v) Stock Turnover Ratio: Cost of Sales/Average Inventory=2850/236=12.08

4. The Management of KAS Ltd. appointed a new young sales manager towards the end of
Ashadh 2070. The new manager devised a plan to increase sales and profit by reducing selling
price and extending credit period to customers.

The extracts of the operational result are enlisted here below:


2069-70 2070-71
Particulars (Amount in (Rs.) Lakhs) (Amount in (Rs.) Lakhs)
Sales 1,200 1,800
Gross Profit 150 162
Net Profit 60 63
Equity Share Capital 600 800

© The Institute of Chartered Accountants of Nepal 525


CAP II Paper 1: Advanced Accounting

Explain whether you believe that the profitability for the year ended 2071 has improved as a result of
the new policies adopted by the new manager. You should support your answer with appropriate ratios.
(CAP Dec. 2014 4a -5 Marks; CAP Jun. 2015 Q4a- 5 Marks)

Answer
Ratios Formula 2069-70 2070-71
Gross Profit Margin =Gross Profit / Sales X 100% 12.50% 9.00%
Net Profit Margin =Net Profit / Sales X 100% 5.00% 3.50%
Return on Equity =Net Profit / Equity X 100% 10.00% 7.88%

• With the extracts it could be seen that there has been a rocket increase in the sales which
has marked a growth of 50%
• However, the Net Profit has marginally increased from Rs. 60 lacs to Rs. 63 Lacs in
absolute terms.
• The company’s Gross Profit Ratio has been reduced from 12.50% to 9.00%.
• The company’s Net Profit Ratio has been reduced from 5.00% to 3.50%; indicating
hard time for shareholders in future.
• The company’s Return on Equity has reduced from 10% to 7.88%.
• The new credit policy of the Management may force the company to hold huge stocks
as well as the figures of the debtors could be increased, likely resulting to a higher risk
of Bad Debts.
Conclusion
The company should review the new Sales Policy so that profitability in relative terms can also be
increased.

5. Following are the ratios relating to the trading activities of an organization:


Debtor's velocity 3 month
Stock Velocity 6 months
Creditor's velocity 2 months
Gross Profit Ratio 20%
Gross profit for the year ended 31st March 2015 was Rs. 5,00,000. Stock as on 31st March 2015 was Rs.
20,000 more than it was on 1st April 2014. Bills payable and receivable were Rs. 36,667 and Rs. 60,000
respectively.
Ascertain the figures of:
i) Sales
ii) Debtors
iii) Creditors; and
iv) Stock
(CAP Dec. 2015 Q3c)
Answer
Rs.
000
1) Gross profit 500
Rate of Gross profit
20%

© The Institute of Chartered Accountants of Nepal 526


CAP II Paper 1: Advanced Accounting

Sales Rs.500 x 100/20


2,500

2) Sales
2,500
Debtor's velocity 3
months
Accounts receivable as on 31st March 2015
Rs. 2,500/4 625
Less: Bills receivable 60
Trade Debtors 565

3) Purchases:
Cost of sales = (Sales- Gross profit) = ( Rs. 2,500 – Rs. 500 )
2,000
Increase in stock 20
Purchase for the year
2,020

Accounts payable as on 31st March 2015 = Rs. 2,020 /6 =


336.667
Less: Bills payable
36.667
Creditors 300

4) Cost of goods sold


2,000
Stock Velocity 6
months
Average stock = Rs. 2,000/2 =
1,000
Let opening stock be = x
Then, closing stock = x + 20
Twice the average stock or Rs. 1,000 X 2 = 2x + 20
Or 2x = 1,980 = x= 990
Opening Stock = Rs. 990
Closing Stock = Rs. 990 + 20 = Rs. 1,010

6. Working capital of R.S. Trading Ltd. is Rs. 1,35,000 and current ratio is 2.5, Liquid ratio is 1.5
and the proprietary ratio 0.75, bank overdraft is Rs. 30,000. There are no long term loans and
fictions assets. Reserve and surplus amount to Rs. 90,000 and gearing ratio (Equity capital/
preference capital) is 2.

© The Institute of Chartered Accountants of Nepal 527


CAP II Paper 1: Advanced Accounting

Required: From the above ascertain current assets, current liabilities, net block, proprietary
fund, quick liabilities, quick assets, stock and preference and equity capital.
(CAP Dec. 2016 3b-5 Marks)
Answer
Current liabilities and current Assets
C.R. =CA =2.5
CL
Or, CA – 2.5CL =0 ………I
Or, CA – CL = Rs. 135,000 ……II
Subtracting Eq. II for Eq. I
1.5 CL = Rs. 135000

CL = Rs. 135000 = Rs. 90,000


1.5
CA = Rs. Rs. 90,000 x 2.5 = Rs. 225,000

Quick liabilities, Quick Assets and stock


Quick Liabilities = Current liabilities – Bank overdraft
= Rs. 90,000 – Rs. 30,000 = Rs. 60,000
Liquid Ratio = Quick Assets = 1.5
Quick Liabilities
= Quick Assets =1.5
Rs. 60000
Or, Quick Assets = Rs. 90,000
Stock = Current Assets – Quick Assets
= Rs. 225,000 – Rs. 90,000
= Rs. 135000

Net Fixed Assets and Proprietary Fund


Proprietary Ratio = Proprietary Fund
Total Assets
= Proprietary Fund =0.75
Net Fixed Assets + Current Assets
Proprietary Fund = 0.75 Net Fixed Assets + 168,750
i.e. since there is no long term loan,
Proprietary = Net Fixed Assets + Working Capital
= Net Fixed Assets + 168,750

So, Net Fixed Assets + 135,000 = 0.75 Net Block + 168,750


Or, 0.25 Net Fixed Assets = 33,750
Net Fixed Assets = 135,000
Proprietary Fund = Rs. 135,000 x .75 + Rs. 168,750
= Rs. 270,000

Preference and Equity Capital


Preference and Equity Capital = Proprietary Fund – Reserve & Surplus
= Rs. 270,000 – 90,000

© The Institute of Chartered Accountants of Nepal 528


CAP II Paper 1: Advanced Accounting

= Rs. 180,000
Equity Capital = 2 Preference Capital
Or, Equity Capital – 2 Preference Capital = 0 ………..I
Equity Capital + Preference Capital = 180,000 ……….II
Subtracting Eq. I from Eq. II
Preference Capital = Rs. 180,000
Preference Capital = 180000
3
= Rs. 60,000
Equity Capital = Rs. 60,000 x 2
= Rs. 120,000

7. You are the Financial Accountant for Kathmandu Ltd., a company that manufactures household
furniture. Kathmandu Ltd. has experienced both a reduction in sales revenue and cash flow
during the last financial period. You are provided with the following information regarding
Kathmandu Ltd. for the years 2072/73 and 2073/74:
Statement of Profit or Loss 2072/73 2073/74
(Rs.'000) (Rs.'000)
Revenue 500 700
Cost of sales (300) (350)
Gross profit 200 350
Operating expenses (75) (140)
Operating profit 125 210
Interest on debentures (30) (50)
Profit before tax 95 160
Tax (12) (20)
Profit after tax 83 140
Statement of Financial Position
Ashadh End 2073 Ashadh End 2074
Non-Current Assets: (Rs.'000) (Rs.'000)
Property, Plant and Equipment 1,160 1,200
Intangible assets 650 400
1,810 1,600
Current Assets:
Inventory 41 39
Trade Receivables 69 67
Bank - 150
110 256
Total Assets 1,920 1,856
Equity & Liabilities:
Issued Share Capital 800 800
Retained Earnings 612 529
1,412 1,329
Non-Current Liabilities:
10% Debenture 300 500
Current Liabilities:
Bank Overdraft 171 -

© The Institute of Chartered Accountants of Nepal 529


CAP II Paper 1: Advanced Accounting

Trade Payables 37 27
508 527
1,920 1,856

Required: (CAP Dec. 2018 Q4a -10 Marks)


Calculate the following ratios for both years:
i) Operating profit margin
ii) Current ratio
iii) Acid test ratio
iv) Inventory days
v) Receivable days
vi) Payable days
vii) Return on capital employed
viii) Gearing Ratio

Answer

i) Operating Profit Margin 125/500*100=25%


210/700*100=30%
=Operating Profit/Sales*100

ii) Current Ratio 110:208= 0.53:1 256:27=9.48:1


=Current Assets: Current Liabilities

iii) Acid Test Ratio (110-41):208=0.33:1 (256-


39):27=8.04:1
=(Current Assets-Inventories): Liabilities

iv) Inventory Days 41/300*365= 50 Days 39/350*365= 41


Days
Inventory/Cost of Sales*365 Days

v) Receivable Days 69/500*365= 50 Days 67/700*365=35


Days
Receivable/Revenue*365 Days

vi) Payable Days 37/300*365= 45 Days 27/350*365=28


Days
Payable/Revenue*365 Days

vii) Return on capital employed 125/1712*100=7.3% 210/1829*100=11.48


=Operating Profit/(Total Assets-Current Liabilities)

© The Institute of Chartered Accountants of Nepal 530


CAP II Paper 1: Advanced Accounting

viii) Gearing Ratio 300/1712*100=17.52%


500/1829*100=27.34%
Or Debt Equity Ratio
= Long term liabilities/(Total Assets- Current Liabilities)

8. ABC Ltd. has produced its annual report and accounts for the year ended 31st Ashadh 2066.
Following are the extracts:

Accumulated reserves for the year ended


31st Ashadh 2066 31st Ashadh 065
Rs. (in millions) Rs. (in millions)
Turnover 830.4 746.5
Cost of Sales 646.2 577.8
Gross Profit 184.2 168.7
Distribution Costs (20.6) (19.7)
Administrative Expenses (56.8) (41.2)
Operating Profit 106.8 107.8
Net Income from Investments 14.0 13.5
Profit on ordinary activities before taxation 120.8 121.3
Taxation (37.4) (33.7)
Profit on ordinary activities after taxation 83.4 87.6
Dividends (17.2) (24.1)
Retained Profits 66.2 63.5

Financial Position as at 31st Ashadh


2066 2065
Rs. (in millions) Rs. (in millions)
Non-current (Fixed) Assets 237.4 176.6
Current Assets
Inventory 104.7 87.0
Debtors 43.2 48.4
Cash at Bank 271.6 156.2
419.5 291.6
Creditors: Amounts falling due within 1 year (346.2) (234.2)
Net Current Assets 73.3 57.4
Total Assets less current Liabilities 310.7 234.0
Creditors: Amounts falling due after more than
1year (77.5) (67.0)
233.2 167.0
Capital and Reserve
Called-up Share Capital 45.2 45.2
Share Premium 2.8 2.8
Accumulated reserves 185.2 119.0
233.2 167.0

© The Institute of Chartered Accountants of Nepal 531


CAP II Paper 1: Advanced Accounting

Share capital comprises of 4,52,000 ordinary shares of Rs.100 each. Following the publication
of these results, the market price stands currently at Rs.580 per share, compared to Rs.450 a
year ago.
Mr. X has a holding of 1,000 ordinary shares in ABC Ltd. He is disappointed with the lack of
growth in operating profits, together with the large cut in dividends. He asks your advice on
the strength of the company in comparison with Ashadh end 2065 results.

Required:
d) Carry out a full ratio analysis on the accounts of ABC Ltd., paying particular attention to
the areas of liquidity, profitability and efficiency. Provide a commentary on the ratios you
calculate.
e) Conclude your analysis by advising Mr. X on the key stock market ratios, outlining
your overall impression of the company’s performance.
(Inter Dec. 2009 Q1-20 Marks)
Answer
F.Y. 2065/66 F.Y. 2064/65
A. Liquidity Ratios
i. Current Ratio
(Current Assets/ Current Liabilities) 419.5/ 346.2 291.6/ 234.2
= 1.21 1.25

ii. Quick Ratio


{(Current Assets-Stock)/ Current Liabilities)} (419.5-104.7)/ 346.2 (291.6-87)/ 234.2
= 0.91 0.87
Comments:

The company has maintained a solid current ratio in F.Y.2065/66, boosted by a significant increase in
cash balances which have offset the rise in creditors. Although the current ratio is lower than the “rule
of thumb” yardstick of two, the size of the cash balances means that, even at 1.21, it reflects a
comfortable working capital position.

There may be some concern about the growth in cash levels, which seems to have occurred at the
expense of creditors. It may be asked whether creditors are being over-stretched and whether cash can
be better utilized in other, more productive directions, for example in fixed assets. The quick ratio has
improved marginally, buoyed by the higher cash balances. At 0.91, it is close to the yardstick figure for
the quick ratio. With the high proportion of cash, it represents a healthy liquidity position.

F.Y. 2065/66 F.Y. 2064/65


B. Profitability Ratios
i. Profit Margin
(PAT/ Sales) 83.4/ 830.4 87.6/ 746.5
=10.04% 11.73%

ii. Return on Total Assets


(PAT/ Total Assets) 83.4/ (237.4+419.5) 87.6/ (176.6+291.6)
=12.77% 18.71%

© The Institute of Chartered Accountants of Nepal 532


CAP II Paper 1: Advanced Accounting

iii. Gross Profit Margin


(GP/ Sales) 184.2/ 830.4 168.7/ 746.5
=22.18% 22.6%

iv. Return on Equity


(PAT/ Capital Employed 83.4/ 233.2 87.6/ 167.0
=35.77% 52.46%

v. Distribution Cost/ Sales 20.6/ 830.4 19.7/ 746.5


=2.48% 2.64%

vi. vi. Administrative Exp./ Sales 56.8/ 830.4 41.2/ 746.5


= 6.84% 5.52%
Comments:
Although gross profit margins have held up fairly well at 22.18%, there are clear signs of concern with
declining overall profitability. Both return on total assets and return on equity have fallen sharply in
F.Y. 2065/66. Investigation must focus on the increase in administrative expenses, significantly up from
5.52% to 6.84% of turnover.

F.Y. 2065/66 F.Y. 2064/65


C. Efficiency Ratios
i. Inventory Turnover
(Sales/ Inventory) 830.4/ 104.7 746.5/ 87.0
= 7.93 times 8.58 times
ii. Average Collection Period
(Debtor/ Sales Per Day) 43.2/ (830.4/365) 48.4/ (746.5/365)
= 19 days 24 days
iii. Fixed Assets Turnover
(Sales/ Fixed Assets) 830.4/ 237.4 746.5/ 176.6
= 3.5 times 4.2 times

Comments:
The decline in the inventory turnover ratio reflects some genuine concern about rising inventory levels
in the Financial Position. Attention must be focused on the realizable value of this inventory in the
future.

There are, however, positive signs with a substantial improvement in the average collection period from
24 days to 19 days, indicative of a more effective credit control function.
On the other hand, there has been a disappointing fall in the fixed asset turnover ratio from 4.2 times to
3.5 times. The additional investment in fixed assets has not produced equivalent benefits in higher
turnover.

F.Y. 2065/66 F.Y. 2064/65


Stock Market Ratios
i. Earnings Per Share (EPS)
(PAT/ No. of ordinary shares) 83.4/ 0.452 87.6/ 0.452
= Rs. 184.51 193.81
ii. Price Earnings Ratio

© The Institute of Chartered Accountants of Nepal 533


CAP II Paper 1: Advanced Accounting

(Market Price/ EPS) 580/ 184.51 450/ 193.81


= 3.14 2.32
iii. Dividend Yield
(DPS/ MP) (17.2/0.452)/ 580 (24.1/0.452)/ 450
= 6.56% 11.85%
iv. Dividend Cover
(PAT/ Dividend) 83.4/ 17.2 87.6/ 24.1
= 4.85 times 3.63 times
Comments:
EPS are slightly down, reflecting the fall in profitability. Despite this fact, the P/ E Ratio has risen,
possibly due to other factors. With dividend yield also declining, it may well be the right time to advise
the disposal of shares in this company. It seems that the company has decided to restrict its dividend
payout, perhaps to conserve cash for the future, although its cash balances are strong.

9. Using the following data, complete the Financial Position given below:
Gross Profit (20 % on sales) Rs. 60,000
Credit sales to Total sales 80 %
Total Assets to turnover 1:3
Inventory turnover (to cost of sales) 8 times
Average collection period (360 days a year) 18 days
Current ratio 1.6
Long Term Debt to Equity 40 %
(Inter Jun. 2008 Q3- 15 Marks)
Answer:
Financial Position of PQR Ltd.

As on 31 March 2006

Liabilities Rs. Assets Rs.

Shareholders’ Equity 50,000 Cash ……..


Long Term Debt …….. Debtors ……..
Creditors …….. Inventory ……..
Non-current (Fixed) Assets 52,000
-------- --------
Total ______ Total ______

Answer:
Financial Position of PQR Ltd.
As on 31-March 2005

Liabilities Rs. Assets Rs.


Shareholders’ Equity 50,000 Cash (WN - 7) 6,000
Long Term Debt (WN - 3) 20,000 Debtors (WN - 6) 12,000
Creditors (WN - 4) 30,000 Inventory (WN - 5) 30,000
Non-current (Fixed) Assets 52,000

© The Institute of Chartered Accountants of Nepal 534


CAP II Paper 1: Advanced Accounting

100,000 100,000

Working Notes:

WN 1
Gross Profit = Rs. 60,000
Rate of gross profit = 20%
Total Sales = 60,000*100/20
Total sales = Rs. 300,000
Cost of Goods sold = 80% of sales
Cost of Goods sold = Rs. 240,000

Credit sales to Total sales = 80%


Credit sales = Rs. 240,000

WN 2
Total Assets to Turnover = 1/3
Total Assets = Turnover/3
= 300,000/3
Total Assets = Rs. 100,000

Total Assets = Total Liabilities


Total Liabilities = Rs. 100,000

WN 3
Shareholders Equity = Rs. 50,000
Long term Debt = 40% of Shareholders Equity
= 40% of Rs. 50,000
= Rs. 20,000

WN 4
Total Liabilities = Shareholders Equity + Long Term Debt + Creditors
Creditors = 100,000 – 50,000 -20,000
= Rs. 30,000

WN 5
Inventory Turnover = 8 times
Cost of goods sold = 240,000
Closing inventory = Cost of goods sold/inventory turnover
= 240,000/8
= Rs. 30,000

WN 6
Average collection period = 18 Days

© The Institute of Chartered Accountants of Nepal 535


CAP II Paper 1: Advanced Accounting

Debtors turnover = 360/18 times


= 20 times

Debtors = Credit sales / Debtors turnover


= Rs. 12,000

WN 7
Current Ratio = 1.6
𝐶𝑢𝑟𝑟𝑒𝑛𝑡𝐴𝑠𝑠𝑒𝑡𝑠
or, 𝐶𝑢𝑟𝑟𝑒𝑛𝑡𝐿𝑖𝑎𝑏𝑖𝑙𝑖𝑡𝑖𝑒𝑠 = 1.6
or, Current Assets = 1.6 current liability (i.e. Sundry Creditor)
= 1.6 × 30,000
= 48,000

Current Assets = Cash + Debtors + Inventory = 48,000


or, Cash + 12,000 + 30,000 = 48,000
or, Cash = 48,000 -12,000 - 30,000
= Rs. 6,000

WN 8
Total Assets = Fixed Assets + Current Assets = 100,000
or, Fixed Assets + 48,000 = 100,000
or, Fixed Assets = 100,000 – 48,000
= 52,000

10. Lumbini Books Limited is considering the acquisition of a bookbinding company to expand
its publishing interests. Its chief executive has highlighted two possible targets, Siddhartha
Binding Limited and Gautam Publishing Services Limited.
The following information has been obtained for their most recent financial years:

Siddhartha Binding Gautam Publishing


Accumulated reserves Rs.’000 Rs.’000 Rs.’000 Rs.’000
Sales 68 000 61 500
Cost of sales 30 000 38 000
Gross profit margin 38 000 23 500
Selling & distribution costs 16 000 10 000
Administration costs 10 000 26 000 6 500 16 500
Profit before tax 12 000 7 000
Taxation 3 000 1 750
Profit after tax 9 000 5 250

Financial Position
Non-Current (Fixed) assets 10 600 8 300
Current assets
Inventory 18 000 8 000
Debtors 28 000 10 000

© The Institute of Chartered Accountants of Nepal 536


CAP II Paper 1: Advanced Accounting

Cash – 2 000
46 000 20 000
Current liabilities
Creditors 26 000 6 000
Overdraft 5 000 –
31 000 6 000
Net current assets 15 000 14 000
25 600 22 300
Capital and reserves
Ordinary share capital 10000 12000
Retained profits 5600 10300
15600 22300
Long-term loan 10000 -
25600 22300

The Chief Executive favors Siddhartha Binding ahead of Gautam Publishing, since it earned higher
profits and generated higher sales in the past financial year.

Required:
As the company’s financial adviser, you have been asked for your assessment of the two
companies, from the particular viewpoints of:
a) liquidity (Current and Quick ratio);
b) profitability (Profit margin, Return on total assets and Return on owners’
equity);
c) capital structure (Fixed to current assets and Debt ratio);
d) efficiency (Inventory turnover, Average collection period, Fixed assets turnover)
(Inter Jun. 2011 Q4- 10 Marks)
Answer
i. Liquidity
(a) Current ratio = Current assets/ Current liabilities

Siddhartha= Rs.46 000/ Rs.31 000 = 1.48 times


Gautam = Rs.20 000/ Rs.6 000 = 3.33 times

(b) Quick ratio = Current assets (− Inventory)/ Current liabilities

Siddhartha = Rs.28 000/ Rs.31 000 = 0.9 times


Gautam = Rs.12 000/ Rs.6 000 = 2.0 times

Comment
Gautam is significantly stronger in both ratios than Siddhartha, which seems to be carrying too much
inventory. Whereas Gautam has plenty in reserve to meet its short-term obligations, Siddhartha is more
likely to be exposed to cash flow difficulties.

ii. Profitability ratios


(a) Profit margin ratio = Profit after tax/ Sales

© The Institute of Chartered Accountants of Nepal 537


CAP II Paper 1: Advanced Accounting

Siddhartha = Rs.9 000/ Rs.68 000 = 0.1324 = 13.24%


Gautam = Rs.5 250/ Rs.61 500 =0.0854 = 8.54%

(b) Return on total assets = Profit after tax/ Total assets


Siddhartha = Rs.9 000/ Rs. 56 600 = 0.1590 = 15.90%
Gautam = Rs.5 250/ Rs. 28 300 =0.1855 = 18.55%
Where, Total assets = Fixed assets + Current assets

(c) Return on owner’s equity = Profit after tax/ Owner’s equity


Siddhartha = Rs.9 000/ Rs. 15 600 = 0.5769 = 57.69%
Gautam = Rs.5 250/ Rs. 22 300 =0.2354 = 23.54%

Comment
The Chief Executive’s view on Siddhartha’s superiority is well justified in terms of profitability
assessment. Only in terms of the return on total assets is it inferior to Gautam.

iii. Capital structure

(a) Fixed to current asset ratio = Fixed assets/ Current assets


Siddhartha = Rs. 10 600/ Rs. 46 000 = 0.2304 = 23.04%
Gautam = Rs. 8 300/ Rs. 20 000 =0.4150 = 41.50%

(b) Debt ratio = All creditors/ Total assets


Siddhartha = Rs. 41 000/ Rs. 56 600 = 0.7244 = 72.44%
Gautam = Rs. 6 000/ Rs. 28 300 =0.2120 = 21.20%

Comment
Both companies are operating with low levels of fixed assets, which may be indicative of a lack of new
investment in recent years. Siddhartha’s gearing ratio is high, whereas Gautam’s is on the low side –
which will prevent the possibility of some borrowed funds being injected into Siddhartha to fund any
new investment without seriously damaging the debt ratio.

iv. Efficiency

(a) Inventory turnover = Cost of sales/ Inventory


Siddhartha= Rs. 30 000/ Rs. 18 000 = 1.67 times
Gautam = Rs. 38 000/ Rs. 8 000 = 4.75 times

(b) Average collection period = Debtors/ Sales per day


Siddhartha = Rs.28 000/ (Rs.68 000 / 365) = Rs.28 000/ Rs.186.30 = 151 days
Gautam = Rs.10 000/ (Rs.61 500 / 365) = Rs.10 000/ Rs.168.493 = 60 days

(c) Fixed asset turnover = Sales/ Fixed assets


Siddhartha = Rs.68 000/ Rs.10 600 = 6.42 times
Gautam = Rs.61 500/ Rs.8 300 = 7.41 times

Comment

© The Institute of Chartered Accountants of Nepal 538


CAP II Paper 1: Advanced Accounting

In terms of the efficient organization and control of its assets, it is evident that Gautam is being managed
much better than Siddhartha, which would seem to have too much slow-moving inventory and poor
credit control resulting in high debt.

Note to the solution: All Rs. are in thousands.

11. Pashupati asked you to prepare his Financial Position from the particulars furnished
hereunder:
Gross Profit margin: 10%
Stock velocity: 12
Capital turnover ratio: 2
Fixed assets turnover ratio: 5
Debt collection period: 1 month
Creditor’s payment period: 73 days
Gross profit: Rs. 100,000
Excess of closing stock over opening stock: Rs. 30,000
Make suitable assumptions wherever necessary.
(CAP Jun. 2010 3b (i) – 7 Marks)
Answer
Financial Position of Mr. Pashupati

Liabilities Rs. Assets Rs.


Capital 500,000 Non-current (Fixed) Assets 200,000
Creditors 1,86,000 Stock 90,000
Debtors 83,333
*Bank balance 312,667
686,000 686,000

*taken to be balancing figure


Working Note
(i) Gross profit= Rs. 100,000
Gross profit margin= 10%
Hence, Sales= Rs. 100,000 x 100/10= Rs. 1,000,000
Cost of goods sold = Sales – Gross profit
= Rs.( 1000,000-100,000)
= Rs. 900,000
Purchase = Cost of goods sold + Increase in stock
= Rs. ( 900,000+30,000) = Rs. 930,000

Average stock= Cost of goods sold/stock velocity


= Rs. 900,000/12=75000

(ii) Capital:
Capital turnover ratio = 2
Sales/Capital=2

© The Institute of Chartered Accountants of Nepal 539


CAP II Paper 1: Advanced Accounting

Hence, Capital= Rs. 1,000,000/2 = Rs. 500,000

(iii)Creditors:
Creditors payment period = 73 days
Hence, Creditors= Purchases x 73/365 =Rs. 930,000 x 1/5 = Rs. 186,000

(iv) Fixed assets:


Fixed assets turnover ratio = 5
Hence, Fixed Assets = Sales/5= Rs. 1,000,000/5= Rs. 200,000

(v) Closing stock:


Closing stock is Rs. 30,000 more than opening stock; it is Rs. 15000 more than
average stock.
Hence, closing stock= Average stock + Rs. 15000
= Rs. 75,000+Rs.15,000= Rs. 90,000

(vi) Debtors:
Debt collection period = 1 month
Hence, debtors= Sales x 1/12 = Rs. 1,000,000 x 1/12 = Rs. 83,333.33
It is assumed that there is no change in capital during the period.

Note: Alternate solutions are possible.

12. From the following data, prepare the Financial Statements of the Everest Company Ltd. for
the year ended on 31st March:
Gross Profit Ratio 40% on Sales Net profit Ratio 10% on Sales
Debtors Turnover Ratio 2 Months Other Expenses (Administrative) Rs. 25 Lakhs
Creditors Turnover Ratio 1.5 Months Depreciation Rs. 25 Lakhs
Inventory Turnover Ratio 2 Months Debentures to Equity Share Capital 10%
Current Ratio 2.5 Months

Opening Stock was less than the Closing Stock by Rs. 4 Lakhs. The ratio of Cash Sales to
Credit Sales was 16:9. Depreciation was charged on Non-current (Fixed) Assets at 20%. Other
Expenses include the payment of Interest on Debentures. No dividends were declared during
the year. Ignore taxation.
(Inter Dec. 2011 Q5b-10 Marks)
Answer
A. Application of Ratios for computing missing figures

1. Sales Since GP Ratio and NP Ratio are 40% and 10%


Of Sales respectively, Other Expenses debited
to P/L= 40% - 10% = 30% of Sales.
Since Other Expenses + Depreciation charged to
P/L = Rs. 25 Lakhs + Rs. 5 Lakhs = Rs.

© The Institute of Chartered Accountants of Nepal 540


CAP II Paper 1: Advanced Accounting

30 Lakhs, Sales = 30÷ 30% = Rs. 100 Lakhs

2. Gross Profit = 40% of Sales = Rs. 40Lakhs


3. Net Profit = 10% of Sales = Rs. 10 Lakhs
4. Credit Sales Cash Sales to Credit Sales = 16:9.
Hence, Credit Sales = Total Sales × 9/25= 100x9/25 Rs. 36 Lakhs
5. Debtors = Credit Sales × 2 months / 12 months = Rs. 36×2/12= Rs. 6 Lakhs
6. Average Stock = COGS× 2 months ÷ 12 months = (Sales – GP)
× 2/12= 60×2/12 Rs. 10 Lakhs
7 Closing Stock Average Stock = (Opening Stock + Closing
Stock) ÷ 2= 10 Lakhs. Opening Stock =
Closing Stock – 4 Lakhs. On substituting,
(Closing Stock - 4 + Closing Stock) ÷ 2 = 10;
Hence, Closing Stock = Rs.12 Lakhs
Therefore Opening Stock = 12 – 4 = Rs. 8 Lakhs

8. Purchases COGS = Opening Stock + Purchase – Closing


Stock .Since COGS = Sales – GP = 100 – 40
= 60, Opening and Closing Stock are known, on
Substitution, Purchase will be the bal. figure = Rs. 64 Lakhs
9. Creditors = Credit Purchase ×1.5 months ÷12 months
= 64 × 1.5 / 12 Rs. 8 Lakhs
10. Current Assets Current Ratio = 2.5; CA ÷ CL = 2.5; Hence,
CA= 2.5 CL. Since CL= Creditors = Rs.8 Lakhs,
On substitution, CA = 2.5×8 Lakhs = Rs. 20 Lakhs
11. Fixed Assets = Depreciation ÷ Deprn. Rate = Rs.5 Lakhs ÷20%=Rs.25Lakhs

12. Net Block = Gross Block – Depreciation = Rs. 25 Lakhs


– Rs. 5 Lakhs = Rs. 20 Lakhs
13. Cap. Employed = Fixed Assets + Net Working Capital =20+ (20-8) =Rs.32Lakhs

14. Debentures Capital Employed= Debt. +Equity= Debentures+(Capital + R & S)


= Rs. 32 Lakhs, of which P&L = Rs. 10 Lakhs.
Hence, Debentures + Share Capital = Rs. 22 Lakhs.
Since Debentures to Share Capital = 10%, Debentures
= Rs. 22× 10/ 110 = Rs. 2 Lakhs

1. Profit/Loss for the year ended 31st March


Particulars Rs. Lakhs Particulars Rs. Lakhs
To Opening Stock 8 By Sales 100
To Purchases 64 By Closing Stock 12
To Gross Profit c/d 40
Total 112 Total 112

2. Financial Position as on 31st March

© The Institute of Chartered Accountants of Nepal 541


CAP II Paper 1: Advanced Accounting

Liabilities Rs. Lakhs Assets Rs. Lakhs


Equity Share Capital 20 Non-current (Fixed) Assets 20
Accumulated Profit 10 Current Assets
Debentures 2 Debtors 6
Creditors 8 Stock 12
Cash – bal. figure 2
Total 40 Total 40

13. The following information is extracted from the annual report of a limited company for a year
end:
Issued and paid up equity shares of Rs. 100 each Rs. 12,000,000
Net working capital Rs. 6,060,000
Current ratio 1.75 : 1
Liquidity ratio 1.25 : 1
Net Non-current (fixed )assets to shareholders' equity 60%
Gross profit margin 20%
Net profit to share capital 16%
Inventory turnover 6.575 times
Average age of outstanding debtors 2 months
Make appropriate assumptions and prepare an Income Statement and Financial Position for the
company. (CAP Jun. 2013 Q3b -10 Marks)

Answer:
Income Statement
Particulars Rs. Rs.
Sales (WN 4) 26,563,000
Less: Cost of goods sold (WN 4) (21,250,400)
Gross profit 5,312,600
Other expenses (balancing figure) (3,392,600)
Net profit (WN 7) 1,920,000

Financial Position
Rs. Rs.
Share capital: 12,000,000
120,000 equity shares of Rs.100 each fully paid up
Reserve and Surplus: 3,150,000
Retained earnings (WN 9)
Total 15,150,000
Net Non-current (fixed) assets (WN 8) 9,090,000
Current assets:
Inventory (WN 3) 4,040,000
Debtors (WN 5) 4,427,167
Cash and bank balance (WN 6) 5,672,833
14,140,000
Less: Current liabilities (WN 1) (8,080,000) 6,060,000
Total 15,150,000

© The Institute of Chartered Accountants of Nepal 542


CAP II Paper 1: Advanced Accounting

Working Notes:
1) Calculation of current assets and current liabilities
Net working capital = Current assets – Current liabilities
or, CA – CL = Rs.6,060,000………………….. (i)
Besides,
Current ratio = 1.75 : 1
or, CA/ CL = 1.75/1
or, CA = 1.75CL
or, CA – 1.75CL = 0 …………………………(ii)

Subtracting eq. (ii) from eq. (i)


0.75CL = Rs.6,060,000
or, CL = Rs.6,060,000/ 0.75
or, CL = Rs.8,080,000
and,
CA = 1.75 x Rs.8,080,000
= Rs.14,140,000

2) Calculation of liquid assets


Liquid assets/ CL = 1.25/ 1
or, Liquid assets/ Rs.8,080,000 = 1.25/ 1
or, Liquid assets = 1.25 x Rs.8,080,000
or, Liquid assets = Rs.10,100,000

3) Calculation of inventory
Since liquid assets exclude inventory from total current assets,
Inventory = Current assets – Liquid assets
= Rs.14,140,000 - Rs.10,100,000
= Rs.4,040,000

4) Calculation of cost of goods sold, gross profit and sales


Inventory turnover = 6.575 times
or, Sales/ Inventory = 6.575
or, Sales = 6.575 x Rs.4,040,000
Sales = Rs,26,563,000

Gross profit = 20% x Rs,26,563,000 = Rs.5,312,600


Cost of goods sold = Rs,26,563,000 - Rs.5,312,600 = Rs.21,250,400

5) Calculation of Debtors
Assumed that all sales were on credit.

Debtors = Sales x Average collection period/ Month in a year


= Rs,26,563,000 x 2/ 12
= Rs.4,427,167

6) Calculation of cash and bank balance


Assumed that liquid assets consisted of debtors and cash and bank balance only.

© The Institute of Chartered Accountants of Nepal 543


CAP II Paper 1: Advanced Accounting

Cash and bank balance = Liquid assets – Debtors


= Rs.10,100,000 - Rs.4,427,167
= Rs.5,672,833

7) Calculation of Net profit

Net profit = 16% of Share capital


= 0.16 x Rs.12,000,000
= Rs.1,920,000

8) Calculation of Net fixed assets and shareholders' equity


Shareholders equity = Net fixed assets + Net working capital
If Net fixed assets are 60% (given) of shareholders' equity, Net working capital
is 40% of the same.
Therefore,
Net fixed assets = Net working capital x 60/40
= Rs.6,060,000 x 60/40
= Rs.9,090,000

Shareholders' equity = Net fixed assets/ 0.60


= Rs.9,090,000/ 0.60
= Rs.15,150,000

9) Calculation of Reserve and surplus


Shareholders' equity = Share capital + Reserve and surplus
Reserve and surplus = Rs.15,150,000 – Rs.12,000,000
= Rs.3,150,000

14. From the following particulars, prepare the Balance Sheet Green Limited, which has equity
shares of Rs. 10 each fully paid up:

i. Sales for the year Rs. 21,00,000


ii. Gross profit ratio 33 1/3%
iii.Stock turnover ratio = 14 (turnover represents cost of goods sold)
iv. Opening stock was Rs. 90000
v. Cost of sales to Fixed Assets = 1.4
vi. Fixed Assets to Net worth = 5/6
vii.General reserve to Equity Share Capital = 1/3
viii.
Debt collection period = 73 days
ix. Quick Assets ratio = 1.25
x. Current Assets ratio = 1.5
xi. 14% Secured (long-term) Debentures = Rs. 20,000
(Inter Dec. 2001 Q5-16 Marks)
Answer:

Given:
Sales 2,10,000
Less Gross Profit = 7,00,000 (33 1/3% on sales)

© The Institute of Chartered Accountants of Nepal 544


CAP II Paper 1: Advanced Accounting

Therefore Cost of goods Sold = 14,00,000

Stock turnover Ratio 14


Cost of goods sold / Average Stock 14

Therefore Average Stock = Cost of goods sold / 14


= 14,00,000/14
= Rs. 1,00,000

Therefore Total Stock = AV. Stock  2 = Rs. 2,00,000


= Op. Stock + Cl. Stock

Given Op. Stock = 90,000


Therefore, Closing Stock = 2,00,000 - 90,000 = Rs. 1,10,000

Fixed Assets to Net worth = 5/6


But Cost of sales to Fixed Assets = 1.4

Cost of sales / Fixed Assets = 1.4


14,00,000 / Fixed Assets = 1.4

Therefore Fixed Assets = 14,00,000 / 1.4 = Rs. 10,00,000

Therefore Net Worth = 10,00,000 / 5  6 = 12,00,000


Net worth = Equity share capital + general reserve

General Reserve to Equity share capital = 1/3 or 1:3


Therefore General reserve = 12,00,000/4 = Rs. 3,00,000
Equity share capital = Rs. 9,00,000

Debt Collection period = Debtors / Sales  365 = 73 days

Therefore Sundry Debtors = 21,00,000 / 365  73 = Rs. 4,20,000

Current Assets = 1.5 (Stock + S. Drs. + Cash)


Quick assets ratio = 1.25 (Sundry Drs. + Cash)

Closing Stock = 0.25 = Rs. 1,10,000


Therefore Current Assets = 110000 / 0.25  1.50 = 660000
Current Assets / Current Liabilities = 1.5 / 1
Therefore Current Liabilities = 660000 / 1.5  1 = 440000
C.A. = 660000 = S. Drs. + Stock + Cash
660000 = 420000 + 110000 + Cash
Therefore cash = 130000

Financial Position of Green Limited as at ...........

Liabilities Rs. Assets Rs.

90000 Eq. Sh. of Rs. 10 each 900000 Non-current (Fixed) Assets 1000000

© The Institute of Chartered Accountants of Nepal 545


CAP II Paper 1: Advanced Accounting

General Reserve 300000 Current Assets:


14% Secured Debentures 20000 Cash 130000
Current Liabilities 440000 Closing Stock 110000
Sundry Drs. 420000
1660000 1660000

15. The Following information regarding Nepal Terrytex Limited are supplied to you. You are
required to make out a statement of assets and liabilities with complete details.
i. Current ratio 2.5
ii. Liquid ratio 1.5
iii. Proprietary ratio (fixed assets/proprietary funds) 0.75
iv. Working capital Rs. 60,000
v. Reserves and surplus Rs. 40,000
vi. Bank overdraft Rs. 10,000
There are no long term loans or fictitious assets.

You can assume that the bank overdraft is a quick liability.


(Inter Jun. 2002, Q 3a – 15 Marks)
Answer:

Nepal Terrytex Limited


Statement of Assets & Liabilities

Liabilities Rs. Assets Rs.


Share Capital (WN 200,000 Non-current (Fixed) Assets 180,000
6) (WN 5)
Reserves & Surplus 40,000
Cost (WN 3) 30,000 Stock (WN 2) 40,000
B/L OD 10,000 Debtors (WN 7) 60,000
280,000 280,000

Working Notes:

1. CA - CL = WC, CA = 2.5, CL = 1.5, WC = Rs. 60,000


2.5 CL - CL = WC
or, 1.5 CL = 60,000
 CL = 40,000
CA = 2.5  40,000 = 1,00,000
CA - Stock
2. LR = CA/CL = CL
100000 - Stock
or, 1.5 = 40000

 Stock = 40,000

3. Bank Overdraft given = Rs. 10,000

© The Institute of Chartered Accountants of Nepal 546


CAP II Paper 1: Advanced Accounting

CL = 40,000
Hence creditors = 40,000 - 10,000 = 30,000

4. Since there are no long term loans,

Working Capital
Proprietary Funds = 0.25

60000
or, PF = 0.25, Hence PF = 2,40,000

5. Fixed Assets : 0.75  2,40,000 = 1,80,000

6. Proprietary Funds : 2,40,000


Reserves & Surplus given = 40,000
Hence Share Capital = 2,00,000

7. Total Current Assets = 1,00,000


Stock as per WN 2 = 40,000
Hence Debtors = 1,00,000 - 40,000 = 60,000
Assuming Cash balance is Nil as Bank overdraft is given Rs. 10,000

16. Prepare the Financial Position on the basis of following particulars.

Current Rate 2
Working Capital 4,00,000
Capital Block to Current Asset 3:2
Fixed Assets to turnover 1:3
Sales Cash/Credit 1:2
Creditors Velocity 2 months
Stock Velocity 2 months
Debtors Velocity 3 months
Net Profit 10% of turnover
Reserve 2½% of turnover
Debentures / Share capital 1:2
Gross Profit Rate 25%
(Inter Dec. 2003 Q 2b-8 Marks)
Answer:
Financial Position

Capital & Liabilities Rs. Assets Rs.

Share Capital 6,00,000 Non-current (Fixed) Assets 8,00,000


Net Profit 2,40,000 Stock 3,00,000
Reserves 60,000 Debtors 4,00,000
Debenture 3,00,000 Other Current Assets 1,00,000

© The Institute of Chartered Accountants of Nepal 547


CAP II Paper 1: Advanced Accounting

Sundry Creditors 3,00,000


Other Current Liabilities 1,00,000 ________
16,00,000 16,00,000

Working Notes:
Current Assets
i. Current Ratio = Current Liabilities
Current Assets = 2 Current Liabilities

a. Current Assets - Current Liabilities = Current Liabilities


Current Liabilities = 4,00,000
Current Assets = 8,00,000

ii. Capital Block : Current Assets = 3:2


If Current Assets = 8,00,000
Capital Block = 12,00,000

iii. Debentures: Share Capital = 1:2


 Debentures = 2 Share Capital

iv. Capital Block = Share Capital + Profit + Reserve + Debenture


Let Share Capital be x
Let Sales be y
Net Profit = 10% of y = 0.1y
Reserve = 2.5 y = 0.025y
Capital Block. = x + 0.1y + 0.025y + 0.5x
or, 1.5x + 0.125y = 12,00,000 -----------------(i)

v. Fixed Assets : Turnover = 1:3


Turnover
or, Fixed Assets = 3
y
Fixed Assets = 3
We know, Proprietor's funds - fixed assets = working capital and proprietor
funds = capital block
y
or, 1.5x + 0.125y - 3 = 4,00,000
or, 4.5x - 0.625y = 12,00,000 ------------------ (ii)
Solving the equation (i) and (ii) we get
x = Share Capital = 6,00,000
y = Sales = 24,00,000

y 2400000
vi. a. Fixed Assets = 3 = = 8,00,000
3
1
b. Cash Sales = 3  24,00,000 = 8,00,000
2
c. Credit Sales = 3  24,00,000 = 16,00,000

© The Institute of Chartered Accountants of Nepal 548


CAP II Paper 1: Advanced Accounting

10
d. Net Profit = 100  24,00,000 = 2,40,000
2.5
e. Reserve = 100  24,00,000 = 60,000
Share Capital
f. Debentures = = 3,00,000
2
25
g. GP = 100  24,00,000 = 6,00,000
Cost of goods sold = 24,00,000 - 6,00,000 = 18,00,000

Average Stock
vii. Stock Velocity = Cost of goods sold  12 = 2
In the absence of information here it is assumed that average stock - closing stock
Closing Stock
 1800000 = 3,00,000

Average debtors
viii. Debtor Velocity =  12 = 3
Credit Sales
In the absence of information, here it is assumed that average debtor = closing
debtors

Closing debtors
 12 = 3
1600000
 Closing debtors = 4,00,000
Other Current Assets = 8,00,000 - 4,00,000 - 3,00,000 = 1,00,000

Average Creditors
ix. Creditor Velocity = Credit Purchases  12 = 2
In the absence of information regarding cash and credit purchases and opening and
closing stock cost of goods sold is considered.
Closing Creditors
Cost of Goods Sold  12 = 2
2  1800000
a. Closing creditor = = 3,00,000
12
Other current liabilities = 4,00,000 - 3,00,000 = 1,00,000

17. From the following details, prepare the Financial Position of the firm concerned in the format
given below :

Stock Velocity 6
Capital Turnover ratio 2
Fixed assets turnover ratio 4
Gross Profit 20%
Debt. Collection period 2 months
Creditor’s payment Period 73 days

The gross Profit was Rs. 60,000.Closing Stock Was Rs. 5,000 in excess of the opening stock

© The Institute of Chartered Accountants of Nepal 549


CAP II Paper 1: Advanced Accounting

Financial Position
Liabilities Rs Assets Rs.
Capital Closing Stock
Creditor Debtors
Non-current (Fixed) Assets
Cash
Total Total
(Inter Dec. 2004 Q6)
Answer:

Working Notes:
(i) Gross Profit ratio = Gross Profit ÷ Sales x 100
20 = Rs.60, 000 ÷ Sales x 100
Sales = Rs. 300,000

Cost of goods sold = Sales – Gross Profit


= Rs.300,000 – Rs.60,000
= Rs. 240,000

(ii) Stock Velocity = Cost of goods sold ÷ Average Stock


6 = Rs. 240,000 ÷ Average Stock
Average Stock = Rs.240,000 ÷ 6 = Rs.40,000

(Opening stock + Closing stock) ÷ 2 = Rs. 40,000 (………………1)


Closing Stock – Opening Stock = Rs. 5,000 (…………..2)

Solving two equations simultaneously:


Closing Stock + Opening Stock = Rs.80,000 (3)
Closing Stock – Opening Stock = Rs.5,000 (2)

Subtracting equation 2 from equation 3,we have

2 Opening Stock = Rs.75,000


Opening Stock = Rs.37,500 (75,000/2)
Hence, Closing Stock = Rs.42,500 (37,500 + 5,000)

(iii) Capital Turnover ratio = Cost of Sales ÷ Capital


2 = Rs.240, 000 ÷ Capital
Capital = Rs.120, 000

(iv) Fixed assets turnover ratio = Cost of sales ÷ Fixed Assets


4 = Rs.240, 000 ÷ Fixed Assets
Fixed Assets = Rs.240, 000 ÷ 4
= Rs.60, 000

(v) Debt collection period = 2 months


Debtor turnover ratio = 12 months ÷ Debt Collection period
= 12 ÷ 2
= 6
Also debtors turnover ratio = Credit Sales ÷ Average Debtors

© The Institute of Chartered Accountants of Nepal 550


CAP II Paper 1: Advanced Accounting

Assuming sales to be credit sales and debtors turnover ratio is based on year – end figures, we
have :
6 = Rs.300, 000 ÷ 6
= Rs.50,000

(vi) Creditors payment period = 73 days


Creditors turnover ratio = 365 days ÷ Creditor’s payment period
= 365days ÷ 73
Assuming all purchases to be credit purchases, the amount of credit purchases is determined as
follows:

Cost of goods sold = Opening stock + Purchases – Closing stock


Rs.240,000 = Rs.37500 + Purchases – Closing stock
Rs. 240, 000 + 42500 -37500 = Purchases
Rs. 245, 000 = Purchases.

Assuming creditors turnover ratio is based on the year –end figure, the amount of creditors is
as follows :
Creditors turnover ratio = Credit Purchases ÷ Closing Creditors
5 = Rs. 245, 000 ÷ Closing Creditors
Creditors = Rs.49, 000

Financial Position
Liabilities Rs. Assets Rs.

Capital 120, 000 Closing stock 42,500


Creditors 49, 000 Debtors 50, 000
Non-current (Fixed) assets 60, 000
Cash (Balancing figure 16,500
Total 169, 000 Total 169, 000

18. Using the following data complete the balance sheet of ABC Ltd as on 31-3-2009.
i) Gross Profit 25% of sales
ii) Gross Profit NRs. 120,000
iii) Shareholder's equity NRs. 100,000
iv) Credit Sales to Total Sales 80%
v) Total Turnover to Fixed Assets 4 times
vi) Cost of sales to inventory 10 times
vii) Average Collection Period 5 days, assume 365 days in a year
viii) Long Term Debt ?
ix) Acid Test Ratio 1.2
x) Sundry Creditors NRs. 70,000

Financial Position of ABC Ltd as on 31-3-2009


Equity & Liabilities NRs. Assets NRs.
Share Capital ----- Fixed Assets ---
Long Term Debt ----- Inventory ---
Sundry Creditors ----- Sundry Debtors --
Cash ---

© The Institute of Chartered Accountants of Nepal 551


CAP II Paper 1: Advanced Accounting

Total ----- Total ----


(Inter Jun. 2009 Q5a- 10 Marks)
Answer
ABC Ltd
Financial Position as on 31-3-2009
Equity & Liabilities NRs. Ref. Assets NRs. Ref.
Share Capital 100,000 Given Non-current Assets 120,000 W.N. 1
Long Term Debt 70,000 Given Inventory 36,000 W.N. 2
Sundry Creditors 70,000 W.N. 5 Sundry Debtors 5,260 W.N. 3
Cash 78,740 W.N. 4
Total 240,000 Total 240,000

Working Notes
W.N. 1: Non-current Assets
Gross Profit =25% of Sales
Sales =Gross Profit/0.25

=NRs. 120,000/ 0.25


=NRs. 480,000

Credit Sales = 80%of Total Sales

= 480,000×80%

= NRs. 384,000

Total Turnover / Fixed Assets = 4 Times


Fixed Assets = Total turnover / 4
Fixed Assets = 480,000/ 4
= NRs. 120,000

W.N. 2: Inventory

Cost of sales to Inventory =10


Cost of sales to Inventory =75%of sales
so, Inventory =75%of sales /10
=75%×480,000/ 10
=NRs.36,000
W.N. 3: Sundry Debtors
Average Collection Period =5 days
Debtors =Credit Sales×5/ 365

=NRs. 384,000×5 /365


=NRs. 5260
W.N. 4: Cash
Acid Test Ratio =1.2
Let, cash be NRs. 'x'
Now, (x+5,260) / 70,000 =1.2

© The Institute of Chartered Accountants of Nepal 552


CAP II Paper 1: Advanced Accounting

or, x + 5,260 =1.2*70,000


=84,000
or, x =84,000-5,260
.* .cash = NRs. 78,740

W.N. 5: Long Term Debt


Long Term debt = (Sum of all the assets) – ( Equity + Creditors)
= 240,000-(100,000+70,000)
= NRs. 70,000

© The Institute of Chartered Accountants of Nepal 553


CAP II Paper 1: Advanced Accounting

CHAPTER 7 GOVERNMENT ACCOUNTING

A. Theory Questions

1. Government Accounting System in Nepal


(CAP Dec. 2009 6b-2.5 Marks)
Answer
Government Accounting System in Nepal is generally on Cash Basis. It has set chart of accounts
under which revenue and expenditure are accounted for. It follows double entry system; however, do
not follow the mercantile system of accounting. Government accounting system broadly classifies
expenditure into administrative and development expenses. Accounting system followed by the
government differentiates Capital expenditures and revenue expenditure in its subsidiary records.
Office of the Financial Comptroller General specifies the chart of accounts under which all the
government revenue and expenditure are to be accounted for.

2. Differentiate between Government Accounting and Business Accounting.


(CAP Jun. 2015 Q4b- 5 Marks; CAP Dec. 2014 Q4b-5 Marks)
Answer
1. Meaning
The accounting system maintained by the government offices is known as government
accounting.
The accounting system maintained by business organizations is known as business accounting.

2. Objective
Government accounting is maintained by the government offices to know the position of public
fund.
Business accounting is maintained by business organizations to know the profit or loss and the
financial position of the business.

3. Budget
Government accounting strictly follows the government budgeting system.
Business accounting does not follow the government budgeting system.

4. Basis
Government accounting is prepared on cash basis.
Business accounting is prepared on cash as well as accrual basis.

5. Level of Accounting
Government accounting has the system of central level and operating level accounting.
Business accounting has no provision of central level and operating level accounting.

6. Rules and Provisions:


Government accounting is strictly maintained by following the financial rules and
provisions of government.

© The Institute of Chartered Accountants of Nepal 554


CAP II Paper 1: Advanced Accounting

Business accounting is maintained by following the rules and principles of 'Generally


Accepted Accounting Principles'.

7. Information:
Government accounting provides information to the government about the receipts,
transfer and deposition of public funds.
Business accounting provides information to the concerned parties about the operating
result and financial position of the business.

3. Public Expenditure and Financial Accountability (PEFA)


(CAP Jun. 2019 Q6d-3 Marks)
Answer
PEFA is the framework for improving public expenditure system of a country. Since
public funds hold utmost importance, PEFA pays high priority in transparency and
accountability in managing the fund. PEFA initiatives has developed a robust tool for
measuring public financial management (PFM) performances and providing sound
assessment of the quality of PFM for countries of all income level. Nepal has assessed
PEFA indicators and adopted action plan that serve as the national policy for the
overall improvement in PFM system, processes and institutions. Effective
implementation of action plan contribute to improve PFM performances and
eventually help to achieve better service delivery and bringing efficiency in public
expenditure management.

© The Institute of Chartered Accountants of Nepal 555


CAP II Paper 1: Advanced Accounting

CHAPTER 8 ACCOUNTING IN COMPUTERIZED SYSTEM

A. Theoretical Questions

1. Discuss the meaning of Chart of Accounts in Computerized Accounting Environment and


explain how you can improve the exiting chart of accounts so that it matches the organizations’
requirement.
(CAP Jun. 2016 Q5b -5 Marks)
Answer
The chart of accounts is a listing of all accounts used in the general ledger of an organization.
The chart is used by the accounting software to aggregate information into an entity's financial
statements.
The chart is usually sorted in order by account number, to ease the task of locating specific
accounts. The accounts are usually numeric, but can also be alphabetic or alphanumeric.
Accounts are usually listed in order of their appearance in the financial statements, starting
with the balance sheet and continuing with the income statement. Thus, the chart of accounts
begins with cash, proceeds through liabilities and shareholders' equity, and then continues with
accounts for revenues and then expenses. Many organizations structure their chart of accounts
so that expense information is separately compiled by department; thus, the sales department,
engineering department, and accounting department all have the same set of expense accounts.
Typical accounts found in the chart of accounts are:
Assets: Liabilities: Stockholders' Revenue:
▪ Cash ▪ Accounts Equity: ▪ Revenue
▪ Marketable Payable ▪ Common Stock ▪ Sales returns
Securities ▪ Accrued ▪ Retained and allowances
▪ Accounts Liabilities Earnings (contra account)
Receivable ▪ Taxes Payable Expenses:
▪ Prepaid ▪ Wages Payable ▪ Cost of Goods
Expenses ▪ Notes Payable Sold
▪ Inventory ▪ Advertising
▪ Fixed Assets Expense
▪ Accumulated ▪ Bank Fees
Depreciation ▪ Depreciation
(contra account) Expense
▪ Other Assets ▪ Payroll Tax
Expense
▪ Rent Expense
▪ Supplies Expense
▪ Utilities Expense
▪ Wages Expense
▪ Other Expenses
The following points can improve the chart of accounts concept for a company:

© The Institute of Chartered Accountants of Nepal 556


CAP II Paper 1: Advanced Accounting

▪ Consistency. It is of some importance to initially create a chart of accounts that is


unlikely to change for several years, so that you can compare the results in the
same account over a multi-year period. If you start with a small number of
accounts and then gradually expand the number of accounts over time, it becomes
increasingly difficult to obtain comparable financial information for more than
the past year.
▪ Lock down. Do not allow subsidiaries to change the standard chart of accounts without a
very good reason, since having many versions in use makes it more difficult to consolidate
the results of the business.
▪ Size reduction. Periodically review the account list to see if any accounts contain relatively
immaterial amounts. If so, and if this information is not needed for special reports, shut
down these accounts and roll the stored information into a larger account. Doing this
periodically keeps the number of accounts down to a manageable level.

2. Advantages of customized accounting packages (CAP Jun. 2017 Qc6-3 Marks)

Answer
Following are the advantages of the customized accounting packages:
1. The input screens can be tailor made to match the input documents for ease of data entry.
2. The reports can be as per the specification of the organization. Many additional MIS
reports can be included in the list of reports.
3. Bar-codes canners can be used as input devices suitable for the specific need so fan
individual organization.
4. The system can suitably match with the organizational structure of the company.

© The Institute of Chartered Accountants of Nepal 557


CAP II Paper 1: Advanced Accounting

Chapter 9 Miscellaneous

A. Theoretical Question

1. Address commission and Primage


(Inter Dec. 2008 Q6e-4 Marks; Inter Jun. 2001 Q 6f -4 Marks)

Answer
Address commission is calculated as a percentage on freight including primage earned by
the shipping company, which is payable either to the agents of the charterers or the charterers
themselves. If a ship has not been chartered, the commission is payable by the shipping
company to the agents who book freight for it.

Primage is the additional freight collected as a percentage of the amount of freight. In the past
the amount represented the commission payable to the captain of the ship to interest him in the
safe delivery of the cargo. Nowadays, it is collected by the shipping company.

2. What is Profit or Loss prior to Incorporation? (CAP Dec. 2011 Q3a-2 Marks)

Answer
When a running business is taken over by the promoters of a company, from a date before the
company which is to manage and own the business is registered, the amount of profit or loss
of such a business for the period prior to the date the company came into existence is referred
to as pre incorporation profits or losses.

3. Write short notes on Pre-incorporation expenses


(Inter Jun. 2003 Q9a -3 Marks)
Answer
Pre-incorporation expenses are defined as expenses incurred by the promoters for the purposes
of the company before its incorporation. Broadly, these include expenses in connection with:
a. Preliminary study and analysis of the conceived idea.
b. Detailed investigation in terms of technical feasibility and commercial viability to
establish the soundness of the proposition.
c. Preparation of 'project report' or 'feasibility report' and its verification through
independent appraisal authority; and
d. Arrangements and organization of funds, properties and managerial ability and
assembling of other business elements.
Pre-incorporation expenses are charged off as expenses when incurred.

© The Institute of Chartered Accountants of Nepal 558


CAP II Paper 1: Advanced Accounting

4. Classification of Reserves (CAP Dec. 2011 Q6b-2.5 Marks)

Answer
Reserves are generally classified into: (a) Capital Reserves and (b) Revenue
Reserves.

1. Capital Reserves are those which are not distributed (expected for certain circumstances)
as profits as a matter of law, prudence or business policy. These reserves may or may not
involve any receipts of cash.

2. Revenue Reserves are any reserves which are not capital reserves and are available for
distribution as profits. These are created by retaining profits. Examples of revenue reserves
are credit balance of the Profit and Loss Account, general reserve, dividend equalization
reserves, investment fluctuation reserves etc. Revenue reserves can further be classified
into: (a) General Reserves and (b) Specific Reserves.

(i) General Reserves: These reserves are not created for any particular purpose. These
are created for safeguarding the business against unforeseen losses in the future or
with a view to planning for further development of the business. General reserves
are not earmarked against any particular asset.

(ii) Specific Reserves: These reserves are created for some specific purpose and are
utilized for these purposes only. These are generally earmarked against some
particular asset and expressed as ‘reserve fund’. An amount of the reserve created
is invested outside the business in securities for a specified period. At the end of
that specified period, all investments are sold away. The proceeds are utilized for
meeting that particular purpose for which the reserve was created.

5. Describe the term Capital Account in the context of accounting system.


(CAP Dec. 2015 4c)
Answer
Capital account is a general ledger account which shows owners investment plus net income
from the firms operations, less net losses (if any) from operations less withdrawals of funds by
the owners for personal use.
The components of the capital account vary as per the types of the business entity. The
component of capital account in case of proprietorship and partnership entity, are same as
defined above. However in case of company, the component of capital account includes:
• Funds contributed by shareholders
• Retained earnings
• Reserves representing appropriation of retained earnings.
The balance of capital accounts has generally a credit balance and is reported in as an
owner’s equity.

© The Institute of Chartered Accountants of Nepal 559


CAP II Paper 1: Advanced Accounting

6. Short working Clause (Inter Jun. 2008 Q6 ii-4 Marks)

Answer:
The short working clause in the royalty agreement enables the lessee to recoup out of the
royalties which may be payable in the future, the amount of minimum rent paid in excess of
that which would have been payable on the basis of actual output. If the actual output is not
sufficient to cover the minimum rent, the minimum rent will be payable but, in the subsequent
periods, the actual royalty due will be reduced to the extent of the unrecouped year which had
short workings in the past. The short working clause may permit the lessor.
• To carry forward the short working indefinitely
• To set off short workings within a certain number of years following the year in which the
short working occurs
• To set off the short workings only within a certain number of years after the commencement
of the lease.

7. What are the advantages of maintaining subsidiary books by a trading/manufacturing


organization? (CAP Jun. 2009 6d- 4 Marks)

Answer
Advantages of maintaining subsidiary books by a trading/manufacturing organization are:
i) Division of work: In place of one journal, there are many subsidiary books. The
accounting work can be divided amongst a number of people.
ii) Specialization and efficiency: As a person is handling only one type of work, he
acquires full knowledge and becomes efficient in handling the work. Accounting
work is done efficiently.
iii) Saving of time: Various accounting processes can be undertaken simultaneously
because of the use of a number of books. This results in quicker completion of
work.
iv) Availability of information: Since a separate register is kept for each class of
transactions, the information relating to each class of transaction is available at
one place.
v) Additional information for VAT, sales tax, excise etc., can also be compiled
from the appropriate columns in the purchases and sales registers.
vi) Facility in checking: When the trial balance does not agree, the location of errors
is facilitated by the existence of separate books. Similarly audit of the various
books of prime entry can be conducted simultaneously by a team of audit staff.

8. Perpetual and Periodic Inventory Systems. (Inter Jun. 2011 Q6a- 5 Marks)
There are two main systems for keeping merchandise inventory records known as perpetual
and periodic.

Perpetual inventory System

© The Institute of Chartered Accountants of Nepal 560


CAP II Paper 1: Advanced Accounting

The perpetual inventory system is a system that keeps a running, continuous record that tracks
inventories and the cost of goods sold on a day – to- day basis. It helps manager control
inventory levels and prepare interim financial statements. In addition to this continuous record-
keeping process, companies periodically physically count and value the inventory. A physical
count is the process of examining and identifying all items in inventory. We then value the
inventory by assigning a specific cost to each item. Companies should conduct a physical count
at least once a year to check on the accuracy of the continuous records, Accountants developed
the perpetual system to provide managers with information to aid in pricing or ordering. At
first it was extremely cumbersome and expensive to maintain constant records, but
computerized inventory systems and optical scanning equipment at checkout counters make
implementation of perpetual inventory systems inexpensive today in many entities.

Periodic inventory system


Periodic inventory system does not involve a day – to – day record of inventories or of the cost
of goods sold. Instead we compute the cost of goods sold and an updated inventory balance
only at the end of an accounting period, when we take a physical count of inventory. The
physical count allows management to remove damaged or obsolete goods from inventory and
thus helps reveal inventory shrinkage, which refers to losses of inventory from theft breakage,
and loss. Inventory shrinkage can be quite large in some businesses.

Under the periodic system, calculations for the cost of goods sold start with the cost of goods
available for sale. This is the sum of the opening inventory for the period plus purchases during
the period. We subtract the ending inventory from the cost of goods available for sale to
compute the cost of goods sold. Thus, the periodic system computes cost of goods sold as a
residual amount. Of course, practically speaking, if someone stole the item, its cost will also
be included in cost of sales.

Unlike the perpetual system, which computes cost of goods sold instantaneously as a company
sells goods, under the periodic system, we delay computing cost of goods sold until we make
a physical count

9. Accounting treatments leading to formation of secret reserve


(CAP Jun. 2013 Q6b-3 Marks)
Answer
Following accounting treatment may result in formation of secret reserves within an
organization:
i) Writing off excessive depreciation;
ii) Charging capital expenditures to profit & loss account;
iii) Undervaluation of closing stocks;
iv) Suppression of sales;
v) Showing a contingent liability as an actual liability;
vi) Showing an assets as contingent assets;
vii) Crediting revenue receipt to an assets account

© The Institute of Chartered Accountants of Nepal 561


CAP II Paper 1: Advanced Accounting

10. Mark-up and margin (CAP Jun. 2013 Q6c-3 Marks)

Answer
When gross profit is shown as a fraction or percentage of the cost price, it is termed as mark-
up. Besides, when gross profit is shown as a fraction or percentage of the selling price, it is
termed as margin. Both the terms refer to the same gross profit, but expressed as percentage of
different figures.

11. Write short notes on Letters of Credit and Travelers’ cheque.


(Inter Dec. 2004 Q 5d)
Answer:
These are issued as facility to travelers within the country or abroad. In either case, the person
desiring such instruments of credit, to be issued in his favor or some other party is made to
deposit the full value of the letter of credit or travelers’ cheques issued in his favor.
The amount deposited by the customers is placed to the credit of Letters of credit Accounts or
Travelers Cheques Accounts, as the case may be. When the bills of exchange drawn against
the letters of credit are received for payment , the amount is debited to the letter of credit
accounts, similarly travelers’ cheques, when presented are debited to the Travelers’ Cheque
Account. In the case of customer desiring travelers cheque in a foreign currency, the equivalent
value thereof in home currency is collected from them at the rate of exchange prevailing on the
date of issue of the travelers cheque and the bank either purchases immediately the amount of
foreign exchange equal to the value of travelers cheque issued, or transfers out of its balances
of foreign currency and amount equivalent to the value of travelers’ cheque to the Travelers’
cheque Account. In the case of letter of credit in foreign currency, the same procedure is
followed.

12. Reasonable return (Inter Dec. 2003 Q 6ii-4 Marks)

Answer
The annual rate of return on an investment, expressed as a percentage of the total amount
invested is a rate of return. This is also termed as return. The reasonable return is a return within
average limits. This is the return conforming to established standards. The reasonable return
may differ in different markets as it is decided after taking into consideration different factors.

13. Fund based accounting technique


(CAP Jun. 2019 Q6c-3 Marks)
Answer
Fund based accounting involves preparation of financial statements fund-wise. In case of
institutions like school, college, university etc. separate ledgers are maintained for each fund.
Fund may be created for purchase, acquisition, construction of fixed assets or for any specific
activities of the organization. For example, a building fund may be created with a view to
construct building. All receipts in connection with the construction of the building are
separated from the main transaction of the organization and shown under building fund.
Expenditures in construction of the building will be made out of this fund. When building is

© The Institute of Chartered Accountants of Nepal 562


CAP II Paper 1: Advanced Accounting

completed, building will be shown as an asset and consequently that portion of the building
fund which has been utilized for the construction of the building should be transferred to
general fund.

B. Practical Questions

1. The trial balance of Duplex Ltd. as at year ended on 31st March, 2010 shows the following
items:
Dr. (Rs.) Cr. (Rs.)
Advance payment of income-tax 220,000 -
Provision for income tax for the year ended 31.3.2009 - 120,000
The following further information are given :
iii) Advance payment of income tax includes Rs. 140,000 for the year 2008/09.
iv) Actual tax liability for the year 2008/09 amounts to Rs. 152,000 and no effect for the
same has so far been given in accounts.
v) Provision for income tax has to be made for the year 2009/10 for Rs. 160,000.

Required:
(a) Prepare provision for income-tax account,
(b) Prepare advance payment of income tax account,
(c) Prepare liabilities for taxation account and
(d) Show how the relevant items will appear in the statement of profit or loss and statement
of financial position of the Company. (CAP Dec. 2010 2b -10 Marks)

Answer
The requirements of a, b & c are not relevant as income tax is not a provision under NAS 37.
Also, NAS 12 requires current tax expense to be recognized in the profit or loss for the period
and only the amount that is unpaid in respect of current tax shall be recognized as liability,
therefore, though Duplex carrying both liability & assets in its books is permissible, whilst
preparing its financial statements, Duplex needs to offset these figures as it has the legal right
to settle amounts on a net basis and, due to lack of information on the contrary, assumed that it
intends to settle on a net basis as well. Duplex might need to restate the figures presented in
the financial statements for 2008/09 as well if this method of presentation was not followed in
those financial statements.

Journal Entries:
a. Provision for tax Dr. 120,000
Advance Tax A/c 120,000

b. Current Tax expense Dr. 32,000


Advance Tax A/c 32,000

c. Current Tax Expense Dr. 160,000


Advance Tax A/c 68,000

© The Institute of Chartered Accountants of Nepal 563


CAP II Paper 1: Advanced Accounting

Current Tax Liability A/c 92,000

Statement of Profit or Loss


for the year ended 31st March, 2010 (Extracts)
Rs. Rs.
Profit before Taxation -----
Less: Current Taxes
- Pertaining to current period 160,000
- Pertaining to previous period/s 32,000 192,000
Net Profit after tax -----

Statement of Financial Position


As at 31st March, 2010 (Extracts)
Equity & Liabilities Rs. Assets Rs.
Current Liabilities
Current Tax Liability 92,000

2. The following is the Financial Position of a concern on 32.3.2067:

Equity & Liabilities Rs. Assets Rs.


Capital 10,00,000 Equipments 4,00,000
Creditors (Trade) 1,40,000 Stock 3,00,000
Accumulated Profit 60,000 Debtors 1,50,000
Cash & Bank 3,50,000
12,00,000 12,00,000
The management estimates the purchases and sales for the year ended 32.3.2068 as under:
Up to Jestha 2068 Ashadh 2068
Rs. Rs.
Purchases 14,10,000 1,10,000
Sales 19,20,000 2,00,000
It was decided to invest Rs. 1,00,000 in purchases of Equipments, which are depreciated @
10% on cost.
The time lag for payment to Trade Creditors for purchase and receipt from Sales (Debtor's) is
one month. The business earns a gross profit of 30% on turnover. The expenses against gross
profit amount to 10% of the turnover. The amount of depreciation is not included in these
expenses. Draft a Balance Sheet as at 32.3.2068 assuming that creditors are all Trade Creditors
for purchases and Debtors for sales and there is no other item of current assets and liabilities
apart from stock and cash and bank balances. (CAP Dec. 2011 Q3 -10 Marks)
Answer
Projected Financial Position of......
As on 32.3.2068
Liabilities Rs. Assets Rs.
Capital 1,000,000 Equipments 450,000

© The Institute of Chartered Accountants of Nepal 564


CAP II Paper 1: Advanced Accounting

Accumulated Profit: Stock in Trade 336,000


01.04.2067 60,000 Sundry Debtors 200,000
Add: Profit for the year 374,000 434,000 Cash & Bank Balances 558,000
Trade Creditors 110,000
1,544,000 1,544,000
Working Notes:
1. Projected Statement of Profit or Loss
For the year ended 32.3.2068
Particulars Rs. Particulars Rs.
Opening Stock 300,000 Sales 2,120,000
Purchases 1,520,000 Closing Stock – Bal. fig. 336,000
Gross Profit c/d – 30% on Sales 636,000
Sundry Expenses – 10% on Sales 212,000
Depreciation 50,000
Net Profit 374,000
2. Cash and Bank Account
Particulars Rs. Particulars Rs.
To Balance b/d 350,000 By Sundry Creditors - Payment 1,550,000
To Sundry Debtors - Collection 2,070,000 By Expenses 212,000
By Equipments 100,000
By Balance c/d 558,000
2,420,000 2,420,000
3. Equipment and Depreciation:
Rs.
Opening Balance 400,000
Add: Purchases during the year 100,000
Less: Depreciation @ 10% 50,000
Closing Balance 450,000

4. Sundry Debtors A/c


Particulars Rs. Particulars Rs.
To Balance b/d 150,000 By Cash & Bank 2,070,000
To Sales 2,120,000 By Balance c/d 200,000
2,270,000 2,270,000

5. Sundry creditors A/c


Particulars Rs. Particulars Rs.
To Cash & Bank 1,550,000 By Balance b/d 140,000
To Balance c/d 110,000 By Purchases 1,520,000
1,660,000 1,660,000

© The Institute of Chartered Accountants of Nepal 565


CAP II Paper 1: Advanced Accounting

3. Gajani Limited operates a number of retail outlets to which goods are invoiced at wholesale
price which is cost plus 25%. These outlets sell the goods at the retail price which is
wholesale price plus 20%.

Following is the information regarding one of the outlets for the year ended 21-3-2012:

Particulars Rs.
Stock at the outlet 1-4-2011 30,000
Goods invoiced to the outlet during the year 324,000
Gross profit made by the outlet 60,000
Goods lost by fire ?
Expenses of the outlet for the year 20,000
Stock at the outlet 31-3-2012 36,000

You are required to prepare the following accounts in the books of Gajani Limited for the year
ended 31-3-2012: (CAP Dec. 2012 Q3a-10 Marks)
ii) Outlet Stock Account
iii) Outlet Profit & Loss Account
iv) Stock Reserve Account

Answer:
Dr. Outlet Stock Account Cr.
Particulars (Rs.) Particulars Rs
To Balance b/d 30,000 By Sales (Working Note 1) 360,000
To Goods sent to outlet 324,000 By Goods Lost by fire 18,000
To Gross Profit c/d 60,000 By Balance c/d 36,000
Total 414,000 Total 414,000

Outlet Profit & Loss


Particulars (Rs.) Particulars Rs
Expenses 20,000 Gross Profit
60,000
Goods Lost by fire 18,000
(Working Note 2)
Net Profit 22,000
Total 60,000 Total
60,000

Dr. Stock Reserve Account Cr.


Particulars (Rs.) Particulars
(Rs.)
To Profit/ Loss – Transfer 6,000 By Balance b/d
6,000

© The Institute of Chartered Accountants of Nepal 566


CAP II Paper 1: Advanced Accounting

To Balance c/d 7,200 By Head Office- Profit/Loss


(Working Note 3)
(Stock Reserve Required)
7,200
Total 13,200 Total
13,200

Working Notes:

(1) Wholesale Price 100+25 = 125


Retail Price 125+20% = 150
Gross Profit at the outlet
Wholesale Price – Retail Price (150-125) = 25
Retail Sales Value = 60,000 x 150/25 = 3,60,000

(2) Goods lost by fire


= Opening Stock + Goods Sent + Gross Profit – Sales – Closing Stock
= 30,000 + 3,24,000 + 60,000 – 3,60,000 – 36,000
= Rs. 18,000

(3) Stock Reserve


On Opening Stock = 30,000 x 25/125 = Rs. 6,000

On Closing Stock = 36,000 x 25/125 = Rs. 7,200

4. On 1st Shrawan 2068, M/s Krishna Traders had a Bad Debts Provision of Rs. 6,500. On
31st Ashadh 2069 the total debts amounted to Rs. 254,000 out of which Rs. 4,000 were bad
debts and had to be written off. The firm wants to maintain the Bad Debts Provision at 5%
of the debtors.
On 31st Ashadh 2070, total debts amounted to Rs. 103,200. Out of the debts, the position of
some of the parties was as follows:
Sundar Rs. 3,200 Bad to be written off.
Ramesh Rs. 8,000 Expected to realize only 70%
Basu Rs. 6,000 Expected to realize only 60%
Indra Rs. 4,000 Financial condition very poor and no recovery is likely.
A suitable provision is supposed to be maintained in respect of doubtful debts.
You are required to show the Bad debts Account and the Bad Debts Provision Account for the
years ended 31st Ashadh 2069 and 31st Ashadh 2070. (CAP Dec. 2013 Q3a-8 Marks)

Answer
Note: It can be seen that Rs. 3,200 is to be written off, the extent of the amount expected to be
unrealizable is Rs.8,800 as shown below:
Rs.

© The Institute of Chartered Accountants of Nepal 567


CAP II Paper 1: Advanced Accounting

Ramesh 30% of Rs.8,000 2,400


Basu 40% of Rs.6,000 2,400
Indra 100% of Rs.4,000 4,000
Total 8,800
The provision for doubtful debts required to be maintained as on 31st Ashadh 2070 is Rs. 8,800.
Bad Debts Account
Date Particulars Amount Date Particulars Amount
2069 2069
31/3 To Sundry Debtors (2) 4,000 31/3 By Provision for Bad debts (3) 4,000

2070 To Sundry Debtors (6) 3,200 2070 By Provision for Bad debts (7) 3,200
31/3 31/3

Provision for Bad Debts Account


Date Particulars Amount Date Particulars Amount
2069 2068
31/3 To Bad Debts A/c (3) 4,000 1/4 By Balance c/d (1) 6,500
To balance c/d 12,500 2069
31/3 By Impairment loss A/c (4) 10,000
16,500 16,500
2070 2069
31/3 To Bad Debts A/c (7) 3,200 1/4 By balance c/d 12,500
To Reversal of allowance (8) 500
To balance c/d (9) 8,800
12,500 12,500
2070
1/4 By balance c/d 8,800

Note:
1. The numerical at the end of each entry denote successive steps.
2. The balance to be carried forward is 5% of debtors after writing off bad debts on 31 st
Ashadh 2069 i.e. 5% of Rs. 2,50,000. The amounts to be transferred to Profit or Loss is the
shortfall in Provision for Bad Debts Account after leaving the desired balance of Rs.
12,500. On 31st Ashadh 2070 the required balance of Provision for Bad Debts Account is
8,800

5. Chemical Limited delivers goods to customers in drums (which are valued in books at Rs.
30) charging them out at Rs. 60 each. Customers are credited with Rs. 40 if the drums are
returned within three months. The following information is supplied to you:

Drums (No.)
Stock in hand in the beginning of the year 2,000
Drums with customers in the beginning of the year 4,200
(Returnable at that time)

Transactions during the year:

© The Institute of Chartered Accountants of Nepal 568


CAP II Paper 1: Advanced Accounting

Purchases at Rs. 50 each 3,000


Drums sent out to customers 6,400
Drums returned by the customers 4,600
Drums destroyed by accident 60
Drums sold as scrap for Rs. 10 each 100

Repairs carried out to drums for Rs. 2,500

At the end of the year, 3,800 drums were with the customers and could be returned within the
following month. Show the relevant accounts in the books of the Chemical Limited.
(INTER Jun. 2008 2a - 10 Marks)

Answer:
Books of Chemical Limited
Drums Trading Account
Dr. Cr.
Amoun Amoun
No. t No. t
By Customers (6,400 at
384,000
To Balance b/d Rs. 60)

2,000 60,000 By Profit & Loss A/c 60 1,800


- In hand (Accidental Loss)
4,200 126,000 100 1,000
- With Customers By Cash (Scrap Value)
By Drums Provision a/c
3,000 150,000 (Drums no longer 2,200 88,000
To Bank (Purchase) returnable at Rs. 40)
To Drums Provision
Account (6,400 at Rs. 40 256,000
each) By Balance c/d.
To Cash/Bank (Repairs) 2,500 In Hand - Old 40 1,200
To Profit & Loss (Profit) 145,500 In Hand - New 3,000 150,000
With customers 3,800 114,000
9,200 740,000 9,200 740,000
Drums Provision Account

To Customers 4,600 184,000 By Balance b/d 4,200 168,000


To Drums Trading By Drums Trading
Account 2,200 88,000 Account 6,400 256,000
To Balance C/d. 3,800 152,000
Total 10,600 424,000 Total 10,600 424,000

© The Institute of Chartered Accountants of Nepal 569


CAP II Paper 1: Advanced Accounting

Working Note 1:
Calculation of drums not returned by customer:
Drum with customer at the beginning of the year 4,200
Add: Drums sent to customer during the year 6,400
Less: Drums returned by customers 4,600
Drums with customers at year end
returnable by customer 3,800 8,400
Drums not returned by customer and no
provision required 2,200

6. The following is an extract from the Trial Balance of Golden Bank Ltd. as at 31st March,
2011:
Rebate on bills discounted as on 1-4-2010 341,295 (Cr.)
Discount received 850,780 (Cr.)
Analysis of the bills discounted reveals as follows:
Amount (Rs.) Due date
14,00,000 June 1,2011
43,60,000 June 8,2011
28,20,000 June 21,2011
40,60,000 July 1,2011
30,00,000 July 5,2011
You are required to find out the amount of discount to be credited to Profit or loss for the year
ending 31st March, 2011 and pass Journal Entries. The rate of discount may be taken at 10%
per annum. (CAP Jun. 2011 Q3a ii- 8 Marks)

Answer
The amount of rebate on bills discounted as on 31st March, 2011 the period which has not
been expired up to that day will be calculated as follows:
Discount on Rs.14,00,000 for 62 days @ 10% 23780
Discount on Rs.43,60,000 for 69 days @ 10% 82,420
Discount on Rs.28,20,000 for 82 days @ 10% 63,355
Discount on Rs.40,60,000 for 92 days @ 10% 102,335
Discount on Rs.30,00,000 for 96 days @ 10% 78,905
Total 350,795

The amount of discount to be credited to the profit and loss account will be:
Rs.
Transfer from rebate on bills discounted as on 31.03.2010 341,295
Add: Discount received during the year 850,780
1,192,075
Less: Rebate on bills discounted as on 31.03.2011 (as above) 350,795
841,280

© The Institute of Chartered Accountants of Nepal 570


CAP II Paper 1: Advanced Accounting

Journal Entries
Rs. Rs.
Rebate on bills discounted A/c Dr. 341,295
To Discount on bills A/c 341,295
(Transfer of unexpired discount on 31.03.2010)
Discount on bills A/c Dr. 350,795
To Rebate on bills discounted 350,795
(Unexpired discount on 31.03.2011 taken into account)
Discount on Bills A/c Dr. 841,280
To Discounting Income A/c 841,280
(Discount earned in the year recognized)

7. M/s Icon Limited gives you the following information to find out total sales and total
purchases:
Rs.
Debtors as on 1.4.2069 157,500
Creditors as on 1.4.2069 182,250
Bills Receivables received during the year 105,750
Bills Payable issued during the year 119,250
Cash received from customers 351,000
Cash paid to suppliers 387,000
Bad debts recovered 36,000
Bills receivable endorsed to creditors 60,750
Bills receivable dishonored by customers 11,250
Discount allowed by suppliers 15,750
Discount allowed to customers 20,250
Endorsed Bills Receivables dishonored 6,750
Sales Return 24,750
Bills Receivable discounted 18,000
Discounted Bills Receivable dishonored 4,500
Cash sales 379,125
Cash Purchased 445,050
Debtors as on 31.3.2070 184,500
Creditors as on 31.3.2070 213,750
(CAP Jun. 2014 Q2a- 10 Marks)

Answer
1. Total sales = Cash sales + Credit sales(W.N.1)
= Rs. 3,79,125+ Rs. 5,06,250
= Rs. 8,85,375
2. Purchases = Cash Purchases + Credit Purchases
= Rs.4,45,050 + 6,07,500 (W.N.2)
= Rs. 10,52,550

© The Institute of Chartered Accountants of Nepal 571


CAP II Paper 1: Advanced Accounting

Working Notes:

1 Debtors Account
Particulars Rs. Particulars Rs.
To Balance B/d 157500 By Bills Receivable 105750
To Bills Receivable dishonored 11250 By Cash 351000
To Bills Receivable dishonored (endorsed) 6750 By Discount allowed 20250
To Bills Receivable dishonored (discounted) 4500 By sales return 24750
To Credit sales (bal.fig) 506250 By Balance C/d 184500
686250 686250

2 Creditors Account
Particulars Rs. Particulars Rs.
To Bills Payable 119250 By Balance B/d 182250
To Cash 387000 By Bill Receivable dishonored
To Discount Received 15750 (endorsed) 6750
To Bills Receivable endorsed 60750 By Credit Purchases (bal.fig) 607500
To Balance C/d 213750 ______
796500 796500

8. Three companies have the capital structures as shown below:


X Ltd. Y Ltd. Z Ltd.
Capital supplied by:
Equity shareholders (Rs.) 6,00,000 4,00,000 50,000
12% Debenture holders (Rs.) - 2,00,000 5,50,000

The return on capital employed was 20% for each company in 2064/65 and 10% in 2065/66.
Corporate income tax in both years was assumed to be 25%, and debenture interest is an
allowable expense against corporate income tax.

Calculate the percentage return on the shareholders’ capital for each company for both years.
Assume that all profits are distributed. Use your answer to explain the merits and the demerits
of high gearing. (Inter Jun. 2010 Q4- 10 Marks)

Answer
X Ltd. Y Ltd. Z Ld.
Capital Employed
Equity Share Capital 600,000 400,000 50,000
12% Debentures - 200,000 550,000
600,000 600,000 600,000
FY 2064/65
Return on capital Employed @20% 120,000 120,000 120,000
Less: Interest (net of tax) - 18,000 49,500

© The Institute of Chartered Accountants of Nepal 572


CAP II Paper 1: Advanced Accounting

Earnings available to equity shareholders 120,000 102,000 70,500


% Return on Equity 20% 26% 141%

FY 2065/66
Return on capital Employed @10% 60,000 60,000 60,000
Less: Interest (net of tax) - 18,000 49,500
Earnings available to equity shareholders 60,000 42,000 10,500
% Return on Equity 10% 11% 21%

Note: Alternate Solution


In the above solution, Return on Capital Employed has been taken as
[ Net profit + (Interest – Tax saving on Interest)] ÷ Capital Employed × 100

However,
Return on Capital Employed may be taken as
Net profit before interest and tax ÷ Capital Employed × 100; in which case, the answer may
vary.

High gearing accentuates the rate of return to the ordinary shareholders. In the zero-geared
position of X Ltd. the return to the shareholders reflects the change in profits earned on trading.
In the high-geared position of Z Ltd. the return to the shareholders decreases from 141% to
21%, i.e. a reduction of 85.1% whereas profit reduces by only 50%. Y Ltd. reflects an
intermediate position with a relatively moderate level of gearing.

High gearing increases the risk to shareholders for two reasons. First, if the profits earned are
not sufficiently high to meet interest charges then the company may find itself failing since the
lenders may seek a winding up order. Second, the risk is increased simply because of
exaggerated fluctuations in the returns which are accentuated in the high-geared situation.
However, it can be seen that if profits earned are higher than the interest rate, this will produce
a significantly higher return to the shareholder in a high-geared company. The fact that interest
is allowed as a deduction for tax purposes indicates that gearing may give an overall advantage.
The market’s assessment of the risk position will counter this, and will be based on the nature
of the business and management.

9. The following data relating to transactions of National Bank Limited have been furnished
to you. You are required to write up the necessary journal entries as well as their adjustment
in the Profit and Loss Account and the Balance Sheet for the year ended on December 31,
2002
Details on the bills discount at 5%.
Discount on Amount Rs. Due date inclusive of 3
days of grace
28-12-2002 1,50,000 31-1-2003

© The Institute of Chartered Accountants of Nepal 573


CAP II Paper 1: Advanced Accounting

29-7-2002 1,00,000 30-11-2002


29-10-2002 4,00,000 30-4-2003
31-12-2002 2,30,000 3-3-2003

(Inter Jun. 2003 Q7 -8 Marks)


Answer
Journal Entry
Discount Account Dr. 9,164
To Rebate on Bills Discount Account Cr. 9,164

Rebate on bills discounted, being discount unearned, will appear on the Liability side of
balance sheet. Discount to be shown on the credit side of profit and loss account will be reduced
by Rs. 9,164, as shown below.

Date of Bills 2002 Date of Maturity 2002-03 No. of days after


December 31st, 2002
28-12-2002 31-1-2003 31days
29-7-2002 30-11-2002 ------
29-10-2002 30-4-2003 120 days
31-12-2002 3-3-2003 62 days

Amount Rs. Rate Total annual Proportionate amount of


discount Rs. discount Rs.
1,50,000 5% 7,500 6361
1,00,000 5% 5,000 -----
4,00,000 5% 20,000 6,5752
2,30,000 5% 11,500 1,9533
Total 9,164
7500 x 31
(1) = Rs. 636
365

20000 x 120
(2) = Rs. 6,575
365

11500 x 62
(3) = Rs. 1,953
365

10. Ramesh runs a stationery shop but does not maintain proper books of account. Following
details are given:
Purchase made during the year Rs. 85,000
Closing stock Rs. 15,000
Opening cash, debtors & noncurrent assets Rs. 1,15,000
Opening creditors & capital Rs. 1,29,000

© The Institute of Chartered Accountants of Nepal 574


CAP II Paper 1: Advanced Accounting

The past record of business shows that the rate of gross profit is 20% on sales. Calculate
sales made during the year
(CAP Dec. 2009 3b-7 Marks)
Answer
The problem is silent on opening stock also. However, this can be easily derived from the
relevant given items of opening statement of affairs. Since the given Rs. 1,29,000 opening
creditors and capital exceeded by Rs. 14,000 over the specified opening cash, debtors and
fixed assets, this excess is taken to represent the opening stock. Now, the sales can be
reckoned by deriving cost of sales and adding there to the gross profit. But note that the
expressed 20% gross profit is on sales which are to be derived. Hence, this 20% on sales
be converted in to on cost which would be 25% ( 20*100/80)

Calculation of sales figure:


Cost of Goods sold = Opening Stock + Purchases – Closing Stock
= 14000+85000-15000 = 84000
Gross Profit = 25% on cost = 25% of 84000 = 21000
Sales = COGS + Gross Profit = 105,0000

© The Institute of Chartered Accountants of Nepal 575

Das könnte Ihnen auch gefallen